Barbri - Practice Set Questions - MBE
The plaintiff sued the defendant on a breach of contract theory. A witness testified for the plaintiff. On cross-examination, which of the following questions is the trial judge most likely to rule improper? A "Weren't you convicted last year of forgery?" B "Isn't it true that you and the plaintiff have been best friends for many years?" C "Isn't it true that you are known in the community as an alcoholic?" D "Didn't you cheat your business partner out of a large amount of money last month?"
C "Isn't it true that you are known in the community as an alcoholic?"
An entrepreneur entered into a written lease-option to purchase an office building. The option to purchase the building could be exercised at any time during the five-year term of the lease by giving the building's owner a written 30-day notice of the intent to exercise the option. A few months later, the entrepreneur assigned his lease-option to a dentist looking for office space by written agreement. If the dentist fully performs under the lease, can the dentist exercise the option to purchase that was given to the entrepreneur? A Yes, because both the burden and benefit of the covenant to convey run with the land. B No, because the covenant to convey does not touch or concern the land. C No, because the option to purchase was personal to the entrepreneur. D No, because the burden of the covenant to convey given in a lease does not run with the land.
A Yes, because both the burden and benefit of the covenant to convey run with the land.
A plaintiff brought a personal injury action against a defendant, the owner of a small fishing resort, for injuries he suffered when a dockside chair he was sitting on collapsed. At trial, the plaintiff testified that he had reported to the defendant the previous day that one of the chairs had a loose leg, whereupon the defendant tightened the screws holding the leg to the chair body, but that the next day the repaired leg of the chair collapsed while the plaintiff was fishing from it, injuring him. The plaintiff now wishes to offer evidence showing that the defendant had attached a new chair leg after the accident. Should the defendant's objection to that evidence be sustained? A No, because it tends to prove the defendant's negligence. B No, because it is relevant to the defendant's state of mind. C Yes, because it constitutes assertive conduct. D Yes, for public policy reasons.
D Yes, for public policy reasons.
A landowner conveyed his land "to my son for life, then to my son's widow for her life, then to my son's children." At the time of the conveyance, the son was 20 years old and unmarried. The son eventually married and had two children, the landowner's grandson and granddaughter. Many years later, the landowner and the grandson were involved in a train accident. The landowner was killed instantly. The grandson died a short time later of his injuries. The landowner left his entire estate by will to his friend. The grandson's will devised his entire estate to the city zoo. The son's wife was so grief-stricken that she became ill and died the next year, leaving her entire estate to her husband. Eventually the son met and married a 21-year-old. Ten years later, the son died, leaving everything to his second wife. When the second wife moved onto the land, the granddaughter filed suit to quiet title to the land, joining all of the appropriate parties. If the jurisdiction recognizes the common law Rule Against Perpetuities, unmodified by statute, in whom will the court most likely find that title to the land is held? A One-half in the granddaughter and one-half in the city zoo, subject to the second wife's life estate. B One-half in the granddaughter and one-half in the second wife, because the second wife took the son's interest. C Entirely in the friend, subject to the second wife's life estate, because the gift to the son's children violates the Rule Against Perpetuities. D Entirely in the granddaughter, subject to the second wife's life estate, because the grandson did not survive the son.
The granddaughter and the city zoo each own one-half of the land, subject to the second wife's life estate. At the time of the conveyance by the landowner, the son had a life estate, the son's widow had a contingent interest (because the son's "widow" cannot be ascertained until the son's death), and the son's children had a contingent remainder (because they have not yet been born). When the grandson and the granddaughter were born, however, their interests became vested subject to open (i.e., if the son had more children). Thus, when the grandson died, he had a vested remainder subject to open that he was free to devise by will; the city zoo took his vested remainder subject to open. At the son's death, the class of his "children" closed (because the son could not have any more children), and the granddaughter's and the zoo's vested remainders subject to open became indefeasibly vested. Also at the son's death, his widow was ascertained and her interest vested in possession. Because the second wife was the son's widow, she is entitled to the valid life estate. Thus, the granddaughter and the city zoo hold one-half interests, subject to the second wife's life estate. (B) is wrong because the son had no interest in the land when he died. He merely had a life estate, which ended at his death. He did not inherit any interest in the property from anyone else. The only person he inherited from in these facts was his first wife, and she had no interest in the land. Furthermore, this choice overlooks the city zoo's interest, which was inherited from the grandson. (C) is wrong because the son's children's interest does not violate the Rule Against Perpetuities. To be valid under the Rule, an interest must vest if at all within a life in being at its creation plus 21 years. The son is a life in being. At the son's death, his children's interest is certain to vest or fail: If the son had any children, at his death, the children's interest would become indefeasibly vested (i.e., the class would close and the children's interest would no longer be subject to open). Note that the children need not come into possession within the perpetuities period; the only requirement is that their interests vest within the period. Likewise, if the son had no children, the gift to them was certain to fail at his death. Thus, the children's interest does not violate the Rule. Because the son had children and their interest was valid, there was no interest to revert to the landowner and to be devised to the friend. Note that the unborn widow aspect of this question is a red herring. The fact would be relevant only if the children's gift were conditioned on their surviving the widow, in which case the takers would remain unascertained and their interest would remain contingent until that time. But because the children's interest vested at the son's death, it is irrelevant that the son's "widow" was not a life in being at the creation of the interest. (D) is wrong because the gift to the son's children was not conditioned on their survival of the son. The law does not imply such a condition. The grandson's interest was vested subject to open and could be disposed of by his will.
A credit card company filed a civil action against a consumer in federal district court seeking to recover the unpaid balance on the consumer's account. The credit card company properly served process on the consumer, but the consumer failed to file or serve a timely answer to the complaint. The clerk of court on motion of the credit card company made an entry of default, but default judgment has not been entered. On which grounds may the court set aside the entry of default? A For the same limited grounds for which any court judgment may be set aside. B Only if the court finds that the consumer could not have filed and served a timely answer despite using reasonable diligence. C If the consumer demonstrates that there was good cause for his failure to file and serve a timely answer and that he has a viable defense. D On any grounds that the court, in its discretion, finds just.
C If the consumer demonstrates that there was good cause for his failure to file and serve a timely answer and that he has a viable defense.
A homeowner sued a contractor in federal court for damages resulting from an unsuccessful roof repair. The court dismissed the case for lack of personal jurisdiction over the contractor. The homeowner then filed her complaint in a federal court that could properly assert jurisdiction over the contractor. In his answer the contractor asserts the affirmative defense of claim preclusion. Should the court dismiss the case? A Yes, because the first case involved the same claimant against the same defendant. B Yes, because there was a valid, final judgment on the merits in the first case. C No, because a dismissal based on lack of personal jurisdiction is not on the merits. D No, because the defense of claim preclusion should be raised at trial.
C No, because a dismissal based on lack of personal jurisdiction is not on the merits.
A wholesaler persuaded a retailer to order a line of dolls for the Christmas season, even though the retailer was skeptical of the dolls' marketability. The contract provided that the retailer would pay $1,500 for its order of 100 dolls if they sold during the Christmas season. Some dolls did sell, but on February 12, the retailer had 80 of them in inventory. He sent the wholesaler notice that he would be returning the 80 dolls. The wholesaler replied that it did not want the dolls back, that the retailer should continue to try to sell them. Despite this reply, the retailer sent the wholesaler a check for $300 and shipped the dolls to the wholesaler, who refused to accept them but did accept the check. Thereafter, the retailer held the dolls at his warehouse. The wholesaler brought an action to recover the $1,200 balance. Will the wholesaler likely recover? A Yes, because the retailer still has the dolls in his possession. B Yes, because it was not a condition precedent that the dolls be sold during the Christmas season, but merely a convenient time for payment. C No, because sale during the Christmas season was a condition precedent to payment. D No, because accepting the $300 constituted a waiver of any rights that the wholesaler may have had to enforce the contract.
C No, because sale during the Christmas season was a condition precedent to payment.
A plaintiff sued a defendant for negligence, seeking $100,000 in the federal court of State A, after the two were involved in a car accident while driving through State B. The defendant's defense was that the plaintiff was contributorily negligent; contributory negligence is a complete defense under the applicable state law. The plaintiff won after a full trial. May the defendant now bring a negligence claim against the plaintiff in a new suit for injuries from the accident that was previously litigated? A Yes, because the defendant's claim is not the same as that of the plaintiff. B Yes, because claim preclusion (res judicata) applies only if the claims are brought by the same parties. C No, because the counterclaim is barred as an unasserted compulsory counterclaim. D No, because the defendant's claim merged with the prior judgment and is now barred.
C No, because the counterclaim is barred as an unasserted compulsory counterclaim.
A company that was the leading supplier of home water filtration systems had a network of sales promoters who were under contract for two- or three-year terms and were compensated solely by commissions earned from sales and by occasional bonuses. Veteran promoters also earned commissions by recruiting other promoters for the company. One of the company's veteran promoters was contacted by a former top sales representative for another manufacturer who was looking for similar sales opportunities in the region. The sales rep knew that the promoter might be able to get her a position with his company, which was looking for additional promoters. At the time he met with the sales rep, the promoter's contract with the company had one more month to run. When the promoter's contract with the company expired, he announced that he was forming his own business to market a different line of water filtration systems manufactured by a competitor of the company, and that the sales rep would be in charge of his promotional network. The company brought an action against the promoter for interference with business relations for hiring the sales rep. At a preliminary hearing, the parties stipulated to the above facts and that the promoter was an independent contractor rather than an employee of the company. The promoter then filed a motion for a summary judgment in his favor. Should the court grant the promoter's motion? A Yes, because the sales rep had no business relationship with the company at the time the promoter's alleged interference occurred. B Yes, because the promoter was an independent contractor rather than an employee of the company. C No, because the jury could find that the means the promoter used to obtain the sales rep were not privileged. D No, because the jury could find that the promoter breached his contract with the company by meeting with the sales rep.
C No, because the jury could find that the means the promoter used to obtain the sales rep were not privileged.
A plaintiff was injured in an automobile accident caused by the defendant. The plaintiff sued the defendant for his injuries. In preparation for trial, the plaintiff's attorney hired a doctor to examine the plaintiff. At trial, the defense attorney attempts to call the doctor as a witness to testify about statements the plaintiff made in confidence to the doctor about his injuries, which the doctor then communicated to the plaintiff's attorney. The state recognizes only the common law privileges. Should this testimony be admitted? A Yes, because the plaintiff's statements are the statements of a party-opponent. B Yes, because the plaintiff waived the physician-patient privilege by placing his physical condition in issue. C No, because the plaintiff's statements are protected by the attorney-client privilege. D No, because the plaintiff's statements are protected by the physician-patient privilege.
C No, because the plaintiff's statements are protected by the attorney-client privilege. The testimony should be excluded because the attorney-client privilege applies to the examination done in preparation for trial. The communication between the doctor and the attorney's client is necessary to help the client convey his condition to the attorney. (A) is incorrect because admissions by party-opponents, while not hearsay under the Federal Rules, are still subject to potential privilege assertions. (B) is a true statement; the physician-patient privilege does not apply to any proceeding in which the condition of the patient has been put in issue by the patient. This is the case in the plaintiff's suit, so (D) is incorrect. However, (B) is incorrect because when a client is examined by a doctor at the attorney's request, the communications involved between the client and doctor (and the doctor and attorney) are not covered by the physician-patient privilege because no treatment is contemplated. Moreover, the physician-patient privilege is a statutory privilege, and this jurisdiction recognizes only the common law privileges (e.g., the attorney-client privilege).
An article in a newspaper reported that the city's professional basketball franchise announced that financial difficulties have forced them to sell the franchise to a group of investors who will probably move the team to another state. The article stated that, according to inside sources, the main reason for the financial difficulties is that the general manager of the team has been siphoning off proceeds from ticket sales to support his gambling habits. The general manager, who is well-known in the community, brought an action against the newspaper for defamation. If the newspaper stipulates at trial that the statement regarding the general manager is false, what additional facts does he have to prove to recover? A That the newspaper was at least negligent in verifying the story. B That the general manager suffered pecuniary damages from publication of the story. C That the newspaper acted with actual malice in publishing the story. D That the general manager suffered actual injury as a result of the story.
C That the newspaper acted with actual malice in publishing the story.
An owner and a builder executed a contract providing that the builder was to construct a residence on a specified lot according to plans and specifications. The total contract price was $800,000. No date was included in the contract for completion of the home. After the builder completed 60% of the residence, a flash flood from a nearby river partially eroded the lot but left the construction undamaged. The builder determined that it would cost an additional $1.7 million to repair the lot so that the residence can be constructed according to the plans. Without the additional lot repair work, the residence cannot be constructed at all. Which of the following states the probable legal consequences of the lot erosion? A The builder may avoid the contract if the increased costs of construction would bankrupt him. B The contract is void because of mutual mistake. C The builder is discharged from his duties under the contract because of impracticability. D The builder remains obligated to perform under the contract, but he may bring an action against the owner for the increased costs of construction.
C The builder is discharged from his duties under the contract because of impracticability. The builder will be discharged from his duties under the contract. Modern courts recognize that impracticability due to excessive and unreasonable difficulty or expense is a defense to breach of contract for nonperformance. Since the cost to the builder to perform under the original contract would exceed more than double what he would be paid, he likely would be excused from performance by commercial impracticability. Unlike destruction of the building itself before completion, which will not discharge a contractor's duty, the erosion of the lot, which destroys the means of performing the contract, will generally not be one of the risks that a builder will be deemed to have assumed. (A) is wrong because the builder's bankruptcy is irrelevant for this purpose. (B) is wrong because there is no mutual mistake here.
An athlete lent his swim fins to a friend. The friend promised to return the fins in a week, but two months had passed and, despite many reminders, the friend still failed to return the fins. The athlete went to the friend's house one night to retrieve them. After knocking on his friend's door several times with no response, the athlete tried the door and found it unlocked. He opened the door and entered the house. He looked around for 20 minutes, but he could not locate the swim fins. He concluded that his friend was probably using them and left the house, taking nothing from it and closing the door behind him. If the athlete is later charged and tried for burglary, his best defense to the charge is which of the following? A He left the house without doing anything. B He took nothing from the house. C The swim fins belonged to him. D His friend's door was unlocked.
C The swim fins belonged to him.
A city council and park board announced joint plans to tear down some old buildings and erect a park. Before the contracts were made, in order to garner the greatest political benefit from such projects, the city council adopted an ordinance requiring that 35% of the work force of contractors working on city-funded projects be residents of the city. One of the contractors working on the park project employed several people from the city, but he and most of his employees came from a town in a neighboring state that was a few miles west of the city. When the city projects inspector discovered that the contractor did not employ the required 35%, he told the contractor that if he did not hire a sufficient number of city workers within 20 days the contractor would forfeit the opportunity to work on the project. The contractor immediately filed an action in federal court seeking to have the employment requirement declared unconstitutional. Of which party should the court should rule in favor? A The city, because it is acting as a "market participant" here. B The city, because there is a rational basis for favoring city residents here. C The contractor, because the requirement interferes with his rights under the Privileges and Immunities Clause of Article IV. D The contractor, because the requirement interferes with his Contract Clause rights.
C The contractor, because the requirement interferes with his rights under the Privileges and Immunities Clause of Article IV.
A landowner and his friend owned a tract of land as joint tenants with right of survivorship. The landowner executed a deed conveying his interest in the land to his grandson. The landowner gave the deed to his attorney with instructions to deliver it to the grandson upon the landowner's death. The grandson first learned of the deed at the landowner's funeral the following year. The next day, the grandson recorded the deed. Who owns the land? A The friend and the grandson, as joint tenants. B The friend and the grandson, as tenants in common. C The friend. D The grandson.
C The friend.
A homeowner from State A hired a contractor from State B to build a vacation home for her in State C. The parties signed the contract in State A. The contractor breached the contract, and the homeowner sued the contractor in a court of State A, seeking damages of $100,000. The contractor removed the case to the federal court for State A. The homeowner promptly moved to remand the case to state court, arguing that venue was improper. Which of the following facts is most relevant to the court's decision on the homeowner's motion? A The contract was signed in State A. B The contractor resides in State B. C The homeowner commenced the action in a State A court. D The vacation home was to be built in State C.
C The homeowner commenced the action in a State A court. (C) The commencement of the action in State A is most relevant. When a case is removed from state court to federal court, venue is set in the federal district court that embraces the state court in which the action was pending, making the federal district court of State A the only appropriate venue choice. Thus, (C) is the correct answer because the most (indeed, only) relevant fact is the fact that the action was commenced in a court of State A. Unlike cases commenced in federal court, in cases removed to federal court, the residence of the defendant contractor (answer choice (B)) and the place where a substantial part of events relating to the claim occurred (answer choices (A) and (D)) are irrelevant.QUESTION ID: MJ233
As it approached the runway at an airport, a plane struck electrical wires and crashed. An injured passenger filed an action in federal district court against the Federal Aviation Administration ("FAA") under the Federal Tort Claims Act, alleging that the FAA was negligent in operating runway lights and performing its air traffic control functions. The passenger seeks to join in that action a state law negligence claim against the utility company that erected the power lines, alleging negligent placement of the lines. Both claims seek $1 million for the passenger's injuries. The passenger is a citizen of State A. The utility company is a State A corporation and all its offices and operations are in State A. Does the federal court have subject matter jurisdiction over the passenger's claim against the utility company? A No, because the claim is a state law claim between two citizens of the same state. B No, because, while the two claims arise from the same event and same injuries, supplemental jurisdiction does not extend to claims against a different party. C Yes, because the court has supplemental jurisdiction over the passenger's claim against the utility company. D Yes, because a plaintiff may join in an action all claims that he has against a particular defendant.
C Yes, because the court has supplemental jurisdiction over the passenger's claim against the utility company.
An investor owned a 100-acre parcel that contained several natural asphalt lakes. A construction company was erecting highways for the state in the vicinity of the investor's land and needed a supply of asphalt. The investor executed a document that, in return for a payment of $1 per barrel, gave the company the right to enter on the land and take asphalt in whatever quantities the company desired. The investor reserved the right to remove asphalt herself and to grant this right to others. Last year, the state commenced an action in eminent domain to take the investor's land for a public park. Is the construction company entitled to compensation? A No, because the nonexclusive nature of the company's right makes it a license, which is not an interest in property. B No, because a nonexclusive profit, although an interest in property, has no value separate and apart from the land itself. C Yes, because the company has a nonexclusive profit, which is a property right for which it is entitled to compensation. D Yes, because the company has a license coupled with an interest, which is a property right for which it is entitled to compensation.
C Yes, because the company has a nonexclusive profit, which is a property right for which it is entitled to compensation. The construction company is entitled to compensation because it has a property right to enter and remove minerals. Like an easement, a profit is a nonpossessory interest in land. The holder of the profit is entitled to enter on the servient tenement and take the soil or the substance of the soil (e.g., minerals, timber, oil, or game). When an owner grants the sole right to take a resource from her land, the grantee takes an exclusive profit and is solely entitled to the resources, even to the exclusion of the owner of the servient estate. By contrast, when a profit is nonexclusive, the owner of the servient estate may grant similar rights to others or take the resources herself. Although here the profit is nonexclusive, it is nevertheless an interest in property for which the company is entitled to compensation in any condemnation proceeding. (A) is incorrect because a license is merely revocable permission to enter on another's land. Unlike a profit, a license is not an interest in land; it is merely a privilege, ordinarily terminable at the will of the licensor. (B) is incorrect because a profit is the right to take something from another person's land; it has a value apart from the land itself and is alienable. (D) is incorrect because a license coupled with an interest has the effect of making the license irrevocable, but it does not convert the license into an interest in land for which compensation is required.
A state court is LEAST likely to take judicial notice of which of the following? A The blood type that occurs with greatest frequency in the population is O-positive. B Main Street, upon which the courthouse is situated, runs north and south. C The sun rose at 6:52 a.m. on Friday, December 12, of last year. D In Australian law, there is no private action for environmental issues.
D In Australian law, there is no private action for environmental issues.
After months of bilateral talks, the President entered into a treaty with a foreign nation previously designated by the President as a terrorist state. Under the treaty, the foreign nation agreed to curtail its nuclear testing program and the United States agreed to lift trading sanctions against the foreign country. The treaty was approved by a vote of more than two-thirds of the Senate. Subsequently, it was revealed that the government of the foreign nation had been sponsoring the copying and black market trade of products patented in the United States and protected by international law. Outraged, Congress approved a bill purporting to repeal the treaty. When the bill was presented to the President, he vetoed it, citing national security interests. Both houses of Congress then repassed the bill by a more than two-thirds vote. As a result of the foregoing, which of the following statements is correct? A The treaty is still valid, because it was both negotiated by the President and passed by Congress. B The treaty is still valid, because the President still supports the treaty and the President's power over foreign affairs is paramount. C The treaty was effectively repealed, because acts of Congress are the supreme law of the land, and any United States treaty in conflict with a congressional act is invalid. D The treaty was effectively repealed, because the repeal was approved over the President's veto after the treaty was made.
D The treaty was effectively repealed, because the repeal was approved over the President's veto after the treaty was made. The treaty was effectively repealed because the bill was passed over the President's veto. Valid treaties are on a "supremacy parity" with acts of Congress, meaning that they are both considered to be the "supreme law of the land." If a conflict exists between them, it is resolved by order of adoption—the last in time prevails. Here, the bill to repeal the treaty was approved by Congress over the President's veto by a more than two-thirds vote. Thus, the President's veto was overridden, and because the bill was passed after the treaty, the bill prevails. (A) is incorrect because, as stated above, acts of Congress and treaties are on a supremacy parity—the fact that a treaty is approved by both the President and the Senate does not make it supreme over conflicting legislation that is validly enacted at a later point. (B) is incorrect because of the parity rule discussed above. It is true that the President's power over foreign affairs is paramount, and the President might be able to enter into an executive agreement to get around the repeal of the treaty, but such a possibility does not change the status of the treaty in question. (C) is incorrect because, although it states the correct result, its rationale is too broad. Acts of Congress are on parity with treaties; they do not automatically invalidate all treaties (e.g., treaties that are entered into after the act of Congress is approved).
In a tort case involving personal injury, a hospital orderly is called to the stand. There is some dispute as to whether the plaintiff ever lost consciousness. The plaintiff's attorney wishes to have the orderly, who was working in the hospital emergency room when the plaintiff was brought in, testify that the plaintiff was unconscious at the time she entered the emergency room. Would such testimony be admissible over the defendant's objection? A No, because the orderly is not an expert witness. B No, because it impermissibly intrudes upon the province of the jury. C Yes, because it is the best evidence. D Yes, because it is proper opinion testimony by a lay witness.
D Yes, because it is proper opinion testimony by a lay witness.
A man and a woman were in a two-car traffic accident. Immediately after the accident, the man was treated by a physician at a nearby hospital's emergency room. The man later filed a negligence action against the woman in federal district court, seeking compensatory damages. The woman now seeks discovery regarding the emergency room physician's observations, opinions, and treatment of the man. Is the woman entitled to discovery regarding that information? A No, because such discovery is not relevant to the claim or defense of a party. B No, because the woman is not entitled to obtain discovery from persons who are not parties to the action. C Yes as to the physician's observation and treatment, but the physician's opinions are discoverable only if the man intends to call the physician as an expert witness at trial. D Yes, because the physician observed and treated the man and developed opinions about the man's injuries for purposes other than litigation or trial.
D Yes, because the physician observed and treated the man and developed opinions about the man's injuries for purposes other than litigation or trial.
A boy mowing his lawn noticed a strong vibration from the engine but continued to mow. The engine housing suddenly broke apart and pieces flew off the lawnmower. One piece struck the boy in the head, seriously injuring him. The boy's mother was inside the house and heard yelling from the backyard. She went to the window and saw her son lying on the ground by the lawnmower and a friend of his kneeling over him. She became very upset and fainted. Subsequent investigation showed that a negligent repair by a local mechanic caused the engine housing to shatter. The mother brought a lawsuit against the mechanic, seeking recovery for her son's injury and the emotional distress she suffered. Can the mother recover damages for her emotional distress? A No, because her son's continuing to mow after noticing the vibration was a superseding cause of the harm. B No, because the mother was not within the zone of danger from the mechanic's negligence. C Yes, because the mother was closely related to someone in the zone of danger from the mechanic's negligence. D Yes, because her son was injured by the mechanic's negligence.
B No, because the mother was not within the zone of danger from the mechanic's negligence. The mother cannot recover damages for her emotional distress. A duty to avoid negligent infliction of emotional distress may be breached when the defendant creates a foreseeable risk of physical injury to the plaintiff. The mother might try to assert two theories in support of her emotional distress claim, but she is unlikely to prevail on either one. First, she could claim distress flowing from fear for her own safety, but she may prevail only if the defendant's negligence placed her in a zone of danger. Here, because she was safely inside her home and quite distant from the mower when it exploded, she was not in a zone of danger. Alternatively, she could claim distress flowing from her anguish at seeing her son injured. However, for a bystander who is outside the zone of danger from the risk of physical injury but who suffers emotional distress from seeing the defendant negligently injure another, most states allow recovery only if: (i) the plaintiff and the person injured by the defendant are closely related; (ii) the plaintiff was present at the scene of the injury; and (iii) the plaintiff personally observed or perceived the event. Here, while she is related to her son, who was injured by the mechanic's negligence, she was not present at the scene of the injury and did not personally observe or perceive the event. Hence, she cannot recover damages for negligent infliction of emotional distress. (A) is incorrect. Under proximate cause rules, a third party's negligence that contributes to the plaintiff's harm will not be considered a superseding force where it is within the foreseeable risk created by the defendant's negligence. While the son may have been negligent in continuing to use the mower, this conduct would not cut off the defendant's liability to the mother if she otherwise could recover for her distress. (C) is incorrect because the plaintiff's close relationship with the injured person is only one of the requirements for a plaintiff outside the zone of danger to recover emotional distress damages. Because the mother was not present at the scene and did not observe the event, she cannot recover. (D) is incorrect. While the mother can recover on her son's behalf for his injury, she cannot recover for her emotional distress, as discussed above.
A plaintiff sued a defendant for damages suffered when a load of bricks fell off the defendant's truck directly in front of the plaintiff while she was driving on a highway. The plaintiff charged that the defendant was negligent in supplying his truck with a defective load chain clamp, which helped tie the load to the bed of the truck, and in failing to secure the load properly on the truck. The plaintiff calls a witness who testifies that he was formerly employed as a truck driver and is an acquaintance of the defendant. The witness further testifies that immediately prior to the accident he had coffee with the defendant at a cafe, and mentioned to the defendant that the tie chains holding the load of bricks looked kind of loose. Assuming proper objection by the defendant's attorney, how should the court rule on the admissibility of such testimony? A Admissible under an exception to the hearsay rule. B Admissible nonhearsay. C Inadmissible hearsay. D Inadmissible opinion evidence.
B Admissible nonhearsay. The witness's testimony is admissible nonhearsay. The statement by the witness is not being offered to prove the truth of the matter asserted therein and thus is not hearsay. Hearsay is a statement made out of court by the declarant, offered in evidence to prove the truth of the matter asserted. [Fed. R. Evid. 801(c)] Although hearsay is inadmissible (unless an exception to the hearsay rule is applicable), a statement that would be inadmissible hearsay to prove the truth thereof may be admitted to show the statement's effect on the listener or reader. Thus, in a negligence case, where knowledge of a danger is at issue, a person's warning statement is admissible for the limited purpose of showing knowledge or notice on the part of a listener. Here, one of the theories of recovery underlying the plaintiff's lawsuit is that the defendant negligently failed to secure the load. Therefore, the plaintiff must show that the defendant either knew or should have known that the load was not properly secured. Consequently, the witness's statement that the chains looked loose is admissible to show that the defendant had notice of the possible danger. If this same out-of-court statement were offered to show that its contents were true (i.e., that the chains were in fact loose), then it would constitute hearsay, but because the statement is offered to show notice to the defendant of a possible danger, it is nonhearsay and (C) is incorrect. (A) is incorrect because the admissibility of the statement arises from its status as nonhearsay. If a statement is nonhearsay, then there is no need to refer to hearsay exceptions in determining the statement's admissibility. (D) is incorrect for two reasons: First, the statement is not being offered to show the witness's opinion that the chains were loose. Rather, the statement is offered to show that the defendant had notice of a possible danger involving the chains. Because the testimony simply relates this statement made to the defendant, such testimony cannot be characterized as opinion testimony. Second, (D) incorrectly implies that opinion evidence is inadmissible. Even opinions of lay witnesses are admissible when they are: (i) rationally based on the perception of the witness; (ii) helpful to a clear understanding of the witness's testimony or to the determination of a fact in issue; and (iii) not based on scientific, technical, or other specialized knowledge. [Fed. R. Evid. 701] Certainly, the witness would be permitted to testify that the chains looked loose at the time he observed them, because such an opinion would be based on personal observation, would be helpful to the determination of a fact in issue (i.e., whether the load was properly secured), and would not be based on technical knowledge.
A city council passed an ordinance providing: "No person may contribute more than $100 annually to any group organized for the specific purpose of supporting or opposing referenda to be voted on by the city electorate or regularly engaging in such activities." If the ordinance is challenged in federal court, how should the court rule on the constitutionality of this ordinance? A Strike it down, because it violates First Amendment rights of free speech and freedom of association. B Strike it down as a violation of due process, because no hearing mechanism has been provided for. C Uphold it, because the city council has a legitimate interest in controlling such contributions. D Dismiss the case, because it involves a political question and is thus a nonjusticiable matter.
Strike it down, because it violates First Amendment rights of free speech and freedom of association.
In a contract suit by the plaintiff against the defendant, the plaintiff offers into evidence a document purporting to have the defendant's signature. Which of the following will the court NOT accept as a method of authenticating the defendant's signature? A A nonexpert who, in preparation for trial, has familiarized himself with the defendant's usual signature testifies that, in his opinion, the questioned signature is genuine. B The jury, without the assistance of an expert, compares the questioned signature with an admittedly authentic sample of the defendant's handwriting. C A witness offers proof that the signature is on a document that has been in existence for at least 20 years, that was in a place where it would likely be if it was authentic, and that has no suspicious circumstances surrounding it. D A witness testifies that the defendant admitted that the signature was his.
A A nonexpert who, in preparation for trial, has familiarized himself with the defendant's usual signature testifies that, in his opinion, the questioned signature is genuine.
An owner of land gave his friend a deed for a specified parcel of property. After the owner's death, the friend discovered that the owner had sold part of the property. The purchaser had been given an easement to cross over the owner's property to get to the property she had purchased, although there is no evidence that the purchaser had ever used the "right of way." If the purchaser does have the right of way over the property the owner gave to his friend, the owner's estate would be liable for breach of which covenant? A Against encumbrances. B Of quiet enjoyment, if such a right of way exists whether or not the purchaser is using it. C Of right to convey, because the owner, by not mentioning the right of way in the friend's deed, implied that there were no easements. D Of right to convey, because the existence of the right of way by the purchaser is inconsistent with the owner's alleged title.
A Against encumbrances.
A restaurant owner properly sued a food supplier in federal district court for breach of contract and timely demanded a jury trial. The complaint asserted both legal and equitable claims. Which of the following statements correctly states the proper order for trying both claims? A All legal claims should be tried first by the jury. B All equitable claims should be tried first by the court. C Legal and equitable claims may not be tried together, so the order does not matter since there will be separate trials. D It is up to the federal district court judge's discretion which claim will be tried first.
A All legal claims should be tried first by the jury.
A yoga instructor entered into a valid written contract with a builder to construct a large yoga studio on some land she owned outside of town. She agreed to pay the builder $150,000 upon completion of the job. As work progressed, and due to substantial increased building costs, the yoga instructor and the builder orally agreed that the builder may omit installation of the koi pond planned for the atrium (saving the builder $1,000), and that the contract price would be reduced to $149,500. The builder completed the job (minus the koi pond) in reliance thereon. What would most courts likely hold this subsequent oral agreement to be? A An enforceable contract. B Unenforceable under the Statute of Frauds. C Unenforceable, because a contract in writing cannot be modified orally. D Unenforceable under the parol evidence rule.
A An enforceable contract.
A man intending to play a practical joke on his friend broke into his house one night when he knew that he was out of town on a business trip. He proceeded to nail or otherwise attach his friend's furniture to the ceiling of his living room and glued knickknacks, books, and magazines to the surfaces of the upsidedown tables. He then got a beer from the refrigerator, drank it, and then drank several more beers, intending to replace them the next time he came to visit the friend. As he was staggering out of the kitchen on his way out of the house, the man noticed a framed, autographed photo of the coach of his friend's college football team. Being an alumnus of a rival college, the man impulsively took out his cigarette lighter and set the photo aflame as it hung on the wall. The ensuing blaze completely destroyed the friend's house. The jurisdiction has not altered the common law crimes in any way. If the man is prosecuted for arson and larceny, of which crime should he be convicted? A Arson only. B Both arson and larceny. C Larceny only. D Neither arson nor larceny.
A Arson only.
A homeowner borrowed $50,000 from a bank, secured by a mortgage on his home. Shortly thereafter, the homeowner sold his home to a buyer for $70,000 by a deed containing a recital signed by both parties that title passed "subject to" the bank's mortgage, "which obligation grantee expressly assumes." The buyer paid the homeowner $20,000, took possession of the house, and began making monthly payments of principal and interest to the bank. A few years later, a chemical manufacturing firm built a huge sulfur processing plant just down the road from the home, which caused the house to immediately decline in value to $35,000. Subsequently, the buyer stopped making the monthly payments to the bank. The bank exercised its contractual right of nonjudicial foreclosure and sold the house at a public auction for $34,000. The bank then brought suit against the homeowner and the buyer for $14,000, the difference between the proceeds of the foreclosure sale and the $48,000 principal remaining due on the original loan to the homeowner. The jurisdiction does not bar deficiency judgments. Against whom should the bank be granted a judgment for $14,000? A Both the homeowner and the buyer. B Only the homeowner. C Only the buyer. D No one.
A Both the homeowner and the buyer. Both the homeowner and the buyer are liable for the deficiency. If a sale of foreclosed property does not bring enough to satisfy the mortgage debt, the mortgagee/lender can bring a personal action against the mortgagor/debtor for the deficiency (as long as the jurisdiction does not bar deficiency judgments). When the mortgagor sells the mortgaged property and gives a deed, the grantee takes subject to the mortgage, which remains on the land. If the grantee does not sign an agreement to assume the mortgage, he does not become personally liable on the loan, and the original mortgagor remains personally liable. If the grantee does sign an assumption agreement, however, the lender is considered a third-party beneficiary of the agreement, and hence may recover from the assuming grantee, who is primarily liable, or the original mortgagor, who is secondarily liable. Here, the buyer signed the recital providing for the assumption, so she will be personally liable on the loan. Therefore, (A) is correct, and (B) is incorrect. (C) is incorrect because the homeowner, the original mortgagor, did not extinguish his own personal liability on the loan by obtaining the assumption agreement from the buyer. He remains secondarily liable as a surety. Thus, the bank may sue the homeowner on the original mortgage agreement. (Note that while the bank may obtain a judgment against both of them, its maximum recovery will be the $14,000 deficiency.) (D) is incorrect because the facts indicate that the jurisdiction does not bar deficiency judgments.
A driver was operating his vehicle along a residential street at the posted speed limit when he saw a ball roll across the street. The driver did not slow down because the ball cleared his path before he reached it. A few seconds later, a child darted out into the street after the ball without looking for cars. The driver's car struck the child, and the child was injured. The child's parents brought an action on the child's behalf against the driver in a jurisdiction that follows traditional contributory negligence rules. At trial, the above facts were established. At the close of the evidence, the driver moved for a directed verdict. What should the court do? A Deny the driver's motion, because it will be up to the jury to determine whether he should have slowed down to below the speed limit. B Deny the driver's motion, because it will be up to the jury to determine whether the child's parents were negligent in supervising the child. C Grant the driver's motion, because the child was negligent in darting into the street without looking out for traffic. D Grant the driver's motion, because he was going no faster than the posted speed limit.
A Deny the driver's motion, because it will be up to the jury to determine whether he should have slowed down to below the speed limit.
A utility company working underground installed a guardrail around its access hole for safety. Although the guardrail completely surrounded the hole, there was an opening in one part of the rail to make it easier to pass down tools to those working below. The owner of a show dog living across the street from the utility access hole frequently walked his dog in his front yard without a leash. One afternoon, the dog unexpectedly chased a squirrel out of the dog owner's yard and ran through the opening of the guardrail, falling into the open hole and suffering broken bones and internal injuries. Although expert veterinary care saved the dog's life, the dog was no longer of "show quality" after the injuries. The dog owner brought a negligence claim against the utility company to recover his economic losses resulting from the injuries to his dog. At trial, the dog owner presented the above facts. The utility company presented uncontested evidence that the guardrail used by the company meets typical industry standards, and that the opening in the guardrail was not large enough for a person to have fallen through. At the close of the evidence, the utility company moved for a directed verdict. What should the court do? A Deny the verdict, because the jury could find that the company failed to exercise reasonable care in making the dangerous condition safe. B Grant the verdict, because the guardrail used by the company meets typical industry standards. C Grant the verdict, because the owner's negligence claim does not support recovery of solely economic loss. D Grant the verdict, because a reasonably careful person would not have been injured in the same manner.
A Deny the verdict, because the jury could find that the company failed to exercise reasonable care in making the dangerous condition safe.
A landlord who owned a strip mall entered into a written five-year lease of one of the units with a discount retail perfumery. The lease provided for a monthly rent of $1,000, payable on or before the first day of each month. The perfumery dutifully paid its rent on time for two years and three months. At that time, with the oral permission of the landlord, the perfumery transferred its interest in the remainder of the lease to a dry cleaner in writing, and added a clause requiring the dry cleaner to get permission from the perfumery for any subsequent assignments. The dry cleaner promptly paid rent to the landlord for 14 months, and then asked the landlord to approve a transfer of its interest in the lease to a video rental store. The landlord gave her oral assent. To obtain the perfumery's approval of the transfer to the video store, the dry cleaner wrote a letter to the perfumery, promising that if any problems arose and anyone tried to go after the perfumery for money, the dry cleaner would "make it good." After the perfumery sent a letter back to the dry cleaner agreeing to the transfer, the dry cleaner executed a written transfer of its interest to the video store. The video store promptly paid rent for three months. Having failed to make any profits, the video store ceased paying any rent to the landlord and cannot be located. The landlord has been unable to find anyone interested in the unit. Given that any judgment against the video store would be worthless, from whom can the landlord collect the unpaid rent owed on the lease? A Either the perfumery or the dry cleaner. B The perfumery only, but the perfumery may recover in turn from the dry cleaner. C The perfumery only, and the perfumery has no recourse against the dry cleaner. D Neither the perfumery nor the dry cleaner.
A Either the perfumery or the dry cleaner.
A landowner owned a beachfront lot and home in a subdivision occupying several hundred acres near a lake. The recorded subdivision plan grants to each owner in the subdivision an easement to use the private roads therein for personal ingress and egress. Following seismic activity in the area, the level of the lake dropped substantially, exposing a considerable amount of land between the new shoreline and the old beachfront. It was judicially determined that this "new" land belonged to the county, which put portions of it up for sale. The landowner purchased the land extending from her old property line to the new shoreline, and constructed a boat launching ramp on the new property. She then permitted persons who did not own land in the subdivision to drive through her old property to reach the boat launching ramp on her new property, and thus to utilize the lake, for a small fee. The homeowners' association brought suit against the landowner, seeking to enjoin her from using or permitting nonresidents of the subdivision from traveling its streets to reach the boat launching ramp. How should the court rule? A For the homeowners, because the scope of the easement granted to the landowner as an owner in the subdivision does not extend to the use that she is making of the new property. B For the landowner, because she has an express easement over the streets of the subdivision. C For the landowner, because she has an easement by necessity as to the new property over the streets of the subdivision. D For the landowner, because she has an implied easement over the streets of the subdivision benefiting the new property since it abuts her old property.
A For the homeowners, because the scope of the easement granted to the landowner as an owner in the subdivision does not extend to the use that she is making of the new property.
A mapping service on the Internet that provided maps and satellite images of urban areas developed a "tourist view" option that offered street level views on its website of many downtown locations in major cities. In one of the street view images posted by the mapping service, a pedestrian could be seen on the steps outside the doors of a business while smoking a cigarette. He was recognized on the website by his supervisor, who was surprised to see him smoking because he had obtained an employee health insurance discount by affirming that he was a nonsmoker. If the pedestrian sues the mapping service for invasion of privacy, how should the court rule? A For the mapping service, because the disclosure was of someone in a public place. B For the mapping service, unless the pedestrian's employer canceled his health insurance discount. C For the pedestrian, because there was widespread public disclosure by the mapping service of a private fact. D For the pedestrian, because he was on the premises of a business rather than a public sidewalk when the image was taken.
A For the mapping service, because the disclosure was of someone in a public place. The mapping service should prevail because the pedestrian was in a public place when the photo was taken. To establish a prima facie case for invasion of privacy involving public disclosure of private facts about the plaintiff, the plaintiff must show that the publication was of private information about the plaintiff and that its public disclosure would be highly offensive to a reasonable person. There is no liability for publication of matters occurring in a public place. Here, the pedestrian was in public when he was photographed smoking. Even though he might not have wished that to be publicized, he was in a public place at the time, so he cannot claim invasion of privacy. (B) is incorrect because the fact that the pedestrian suffered economic damages due to the cancellation of his health insurance discount is irrelevant. He cannot prevail because he cannot establish the prima facie case; if he were able to establish the prima facie case, he could prevail even without proof of economic damages. (C) is incorrect because the fact that the disclosure was widespread does not matter if it is not a private fact; a photograph of someone in a public place is not actionable no matter how widely it is disseminated. (D) is incorrect because a "public place" is not limited to property owned by public authorities; it includes all areas open to the public, including the entrance to a business's premises.
A small cruise ship struck a whale swimming underwater, causing the ship to suddenly lurch sideways. A passenger on the ship who was walking down a corridor lost his balance and bumped his head on the edge of a doorway. Because of a previously existing medical condition that made him susceptible to bleeding on the brain, he suffered a cerebral hemorrhage and permanent mental impairment, despite prompt medical attention on the ship. The passenger brought suit against the cruise ship owner for his damages. At trial, the passenger presented evidence of how he was injured as he walked down the hallway, his previous medical condition, and his medical expenses and other damages. The cruise ship owner presented evidence that the cruise ship was following its approved route and that the whale could not have been detected before impact, and that the bump would not have injured someone in ordinary health. At the close of the evidence, the cruise ship owner moved for a directed verdict. How should the court rule? A Grant the motion, because there is no evidence that the crew operated the ship negligently. B Grant the motion, because the cruise ship owner introduced uncontroverted evidence that a person in normal health would not have been injured by the bump. C Deny the motion, because the jury could find that the cruise ship owner, as a common carrier and innkeeper, breached its high duty of care to the passenger. D Deny the motion, because the fact that the severity of the passenger's injuries was not foreseeable does not cut off the cruise ship owner's liability.
A Grant the motion, because there is no evidence that the crew operated the ship negligently.
A man is subpoenaed to appear before the House of Representatives Armed Services Committee and answer certain questions. When he appears, he refuses to answer and is cited for contempt of Congress. Which of the following is the man's best defense to the charge of contempt of Congress? A He demonstrates that the questions asked him did not relate to any matter on which Congress could legislate. B He establishes that he is an employee of the Department of Defense and may not be questioned relating to his duties as an officer of the executive branch of the federal government. C He establishes that he holds an office by appointment of the President and may not be questioned as to his duties except by the Senate. D He demonstrates that the questions asked him did not relate to any matter as to which funds appropriated by the House were expended.
A He demonstrates that the questions asked him did not relate to any matter on which Congress could legislate.
A high school teacher shot and killed one of the students in his class on the spur of the moment. Psychiatric examinations indicated that the teacher believed that the student was trying to ridicule him in front of other students in the class and that he had to do something to stop him. The examinations also indicated that the teacher did not comprehend that killing was condemned by society when he shot his student. If the teacher pleads not guilty by reason of insanity in a jurisdiction that applies the "M'Naghten test," what would be his best argument? A He did not know that the act of shooting the student was wrong. B He lacked the substantial capacity to appreciate the criminality of his act. C He did not know the nature and quality of his act. D His act was the result of an irresistible impulse.
A He did not know that the act of shooting the student was wrong.
An employee of the United States Department of Labor was instructed by his superior to solicit subscriptions to the Department's bulletin on a door-to-door basis in the city in which he worked. While doing so, the employee was arrested for violation of a city ordinance that prohibited commercial solicitation of private residences. What is the employee's best defense? A Intergovernmental immunity. B The First Amendment freedom of expression as it applies to the states through the Fourteenth Amendment. C The Equal Protection Clause as it applies to the states through the Fourteenth Amendment. D The city ordinance effectively restricts interstate commerce.
A Intergovernmental immunity.
The owner of a small computer consulting firm was attending the annual trade meeting of the computer industry and spoke with the owner of a second consulting firm about doing joint projects. The owner of the second firm replied by rejecting the idea immediately, stating that she believed that the first owner was incompetent. A sales representative of a computer supply firm overheard the remark. The owner of the first firm sued the owner of the second firm for defamation. If the first owner does not prevail in this lawsuit, what will be the likely reason? A It was not reasonably foreseeable that the second owner's remark would be overheard. B The second owner did not know that her remark would be overheard. C There was no publication. D The sales representative was not a party to the conversation.
A It was not reasonably foreseeable that the second owner's remark would be overheard.
On April 10, the owner of a small farm mailed a letter to a new resident of the area who had expressed an interest in buying the farm. In this letter, the farm owner offered to sell the farm to the resident for $100,000. The offer expressly stated that the offer expires on June 1, "if acceptance by the offeree has not been received by the offeror on or before that date." On the morning of June 1, the resident sent a written acceptance to the farm owner by messenger. However, through negligence of the messenger company, the acceptance was not delivered to the farm owner until June 2. On June 4, the farm owner entered into a contract to sell the farm to another buyer for more money but did not inform the resident of the transaction. When the resident followed up by phone on June 10, the farm owner told him that he had sold the farm to another buyer. Which of the following is the most correct statement? A No contract between the farm owner and the resident arose on June 2. B An enforceable contract arose on June 1. C The farm owner's silence constituted an acceptance of the resident's message on June 2. D A voidable contract arose on June 1.
A No contract between the farm owner and the resident arose on June 2.
A state statute provided for criminal penalties for "knowingly selling alcoholic beverages in violation of the regulations of the State Liquor Commission to any person under the age of 18." One of the State Liquor Commission regulations provided that "before an alcoholic beverage is sold to any person between the ages of 17 and 24, the seller must demand some form of photo identification to determine the buyer's age." A minor who looked much older than his age of 17 walked into a tavern located in the state and asked the bartender for a beer. The bartender never asked the minor for any form of identification, as he thought that he was at least 25 years old. Had the bartender asked for identification, the minor would have shown him a fake identification card showing that he was 21 years old. The bartender served the beer to the minor, who consumed it on the premises. The bartender was subsequently charged under the state statute for selling the beer to the minor. Is the bartender guilty? A No, because he reasonably believed that the minor was older than 25 years. B No, because the minor had fake identification with which he could have obtained the beer. C Yes, because he sold an alcoholic beverage to a minor, a strict liability crime. D Yes, because he failed to ask for identification, and the regulation does not provide for a mens rea requirement.
A No, because he reasonably believed that the minor was older than 25 years.
A woman was late for an appointment with her doctor across town. Because of this, she was driving recklessly through traffic at a high speed and ran through a red light. There were a number of people crossing the street at the time, and the woman accidentally hit one of them. The person she hit was seriously injured and was rushed to the hospital, but recovered. The woman was arrested and charged with attempted murder. Will the woman likely be convicted? A No, because she did not intend to hit anyone with her car. B No, because she had not gone far enough in her actions to constitute attempt. C Yes, because a person is presumed to intend the natural and probable consequences of her act. D Yes, because from her recklessness, the intent to inflict serious bodily harm will be presumed.
A No, because she did not intend to hit anyone with her car.
A wholesaler sued a retailer in a federal court in State A. The retailer timely filed and served a motion to dismiss for lack of subject matter jurisdiction. The court denied this motion. Thereafter, the retailer filed and served his answer. Ten days after serving his answer, the retailer filed an amended answer, raising, for the first time, the defense of lack of personal jurisdiction, which was available when the motion mentioned above was filed. Should the court consider the personal jurisdiction objection? A No, because that defense has been waived. B No, because objections to personal jurisdiction may only be made by making a motion to dismiss before filing an answer. C Yes, because the retailer may serve an amended answer as of right within 21 days after serving his original answer. D Yes, because the amendment relates back to the original answer, thus preserving his right to raise the objection.
A No, because that defense has been waived. (A) The court should not consider the retailer's objection because the retailer has waived the defense of lack of personal jurisdiction. A defendant may object to personal jurisdiction in two ways: (i) by raising it in a pre-answer motion to dismiss under Rule 12(b); or (ii) if he has not moved under Rule 12(b), by raising the defense in his answer. Thus, the retailer has waived the defense of lack of personal jurisdiction by making a pre-answer motion to dismiss for lack of subject matter jurisdiction under Rule 12(b) and failing to raise in that motion the defense of lack of personal jurisdiction. (B) is incorrect because a defendant may preserve the defense of lack of personal jurisdiction in his answer, provided he has not made a Rule 12(b) motion. (C) is also incorrect. If a defendant does not make a Rule 12(b) motion, he may preserve the defense by raising it in his answer or in any amendment as of right. However, the retailer has made a Rule 12(b) motion to dismiss; therefore, he has not preserved by putting it in his amended answer. (D) is incorrect because, as previously discussed, the retailer waived the defense by making a Rule 12(b) motion. Moreover, the concept of "relation back" is irrelevant here. It is used to determine when a claim asserted in an amended pleading relates back to the date of the original pleading for statute of limitations purposes.QUESTION ID: MJ114
A man committed a particularly brutal series of crimes that, because of their interstate character, were violations of a federal criminal statute. The man was convicted in federal court and sentenced to life imprisonment. Six months after the man was incarcerated, the President pardoned him. There was a great public outcry, amid charges that the President issued the pardon because the man's uncle had made a large contribution to the President's reelection campaign fund. Responding to public opinion, Congress passed a bill limiting the President's power to pardon persons convicted under the specific statute that the man had violated. The President vetoed the bill, but three-quarters of the members of each house voted to override the veto. Is the legislation constitutional? A No, because the power to pardon for federal crimes is expressly granted to the President in the Constitution and is an unqualified power (except as to impeachment). B No, because the President has the duty to enforce the laws, and therefore has plenary powers. C Yes, under Article I, Section 1. D Yes, because Congress wrote the federal criminal statutes and has the right to determine who should be convicted under such statutes.
A No, because the power to pardon for federal crimes is expressly granted to the President in the Constitution and is an unqualified power (except as to impeachment).
The defendant discovered that his friend had hit and killed a pedestrian while driving that afternoon, and that he had fled from the scene of the crime before the police arrived. To keep his friend out of trouble, the defendant fixed all the dents in the car caused by the collision and had the vehicle painted a different color. The friend, distraught about hitting and killing someone, eventually turned himself in and told the police what he had done and what the defendant had done for him. The defendant was charged as an accomplice to vehicular manslaughter in a state that follows the modern trend regarding accomplice liability. How should the defendant be found? A Not guilty, because he only helped his friend after the crime was already committed. B Not guilty, because he had no affirmative duty to the victim. C Guilty, because he aided his friend in the crime. D Guilty, because as a party to the crime, he is criminally responsible for all crimes committed by his co-felons.
A Not guilty, because he only helped his friend after the crime was already committed.
An elderly woman regularly corresponded with her only niece (her sister's daughter), who lived out of town. One day she sent her niece a letter telling her that she planned to leave everything she owned to the niece upon her death. When the woman died, her will left her entire estate valued at $200,000 to her nephew (her niece's only brother). The nephew wrote his sister a letter telling her that he felt bad about being the only person named in their aunt's will, and added, "I'm going to share her estate with you. We can discuss the details at the funeral." The niece spent $800 on a round-trip ticket to attend her aunt's funeral. After the funeral, she spoke with her brother, who told her that he had changed his mind about sharing their aunt's estate with her. He went on to say that he would be willing to share the estate with her if she were willing to share their mother's estate with him when their mother passed on. The niece responded by telling him that their mother had already signed over all the property to her. He replied that, given her attitude, he would keep their aunt's estate for himself. Later, after the two had returned to their respective homes, no longer on speaking terms, the niece sued the nephew for a 50% share of their aunt's estate. What amount, if any, should the niece realize from her suit? A Nothing, because the aunt's will left everything to the nephew, and the nephew's letter is an insufficient basis to compel him to share. B $800, because this represents the niece's actual expenses incurred in reliance on the nephew's letter. C $100,000 (half of the aunt's estate), because the nephew promised her that in a signed writing. D $100,000, but only if she shares their mother's estate with the nephew.
A Nothing, because the aunt's will left everything to the nephew, and the nephew's letter is an insufficient basis to compel him to share.
The owner of a pet shop received a flyer from a professional bird breeder that included the following: "Lovebirds! $120 a pair! Delivery in 4-6 weeks. Terms of sale, cash within 30 days of delivery." The shop owner e-mailed the bird breeder on April 26, ordering one pair of lovebirds for $120. After looking over the order the next day, the bird breeder sent an e-mail to the shop owner indicating that there was a mistake in the advertisement; it should have read "$120 per bird." The bird breeder went on to state that she would ship the pair of birds to the shop owner if she would pay the additional $120. In her e-mailed reply, the shop owner authorized the breeder to ship the pair of lovebirds and agreed to pay the additional $120, but she noted that they must be delivered on or before May 21. The bird breeder immediately e-mailed the shop owner agreeing to deliver the birds to her by May 21. However, the birds did not arrive at her shop until June 1, and the shop owner refused to accept them. In an action by the bird breeder against the shop owner for breach of contract, which of the following awards is most likely? A Nothing, because the birds did not arrive at the shop owner's store by May 21. B $120, because the promise to pay the additional amount with a new delivery date was only a counteroffer. C $120, because the bird breeder had a preexisting legal obligation to ship the birds to the shop owner. D $240, because consideration is not required to modify a contract for the sale of goods.
A Nothing, because the birds did not arrive at the shop owner's store by May 21.
The criminal statutes of the state define manslaughter and murder as they were defined at common law. As to insanity, the state has the following provision: "Under the defense of insanity a defendant may be entitled to acquittal if, because of mental illness, the defendant was unable to control his or her actions or to conform his or her conduct to the law." The defendant was put on trial in the state for the murder of his wife and her co-worker. The evidence at trial established that the defendant's wife was having an affair with the co-worker, and that the defendant learned of it and killed the pair. The defendant did not take the stand in his own defense. In his closing statement to the jury, the defendant's attorney made a statement, "Ladies and gentlemen, you must consider that there are some things that would provoke any one of us to kill, and there are things that make one unable to control one's actions." The defendant's attorney requested that the judge give the jury instructions on manslaughter and on insanity, and the judge agreed to do so. The judge also issued the following instructions: "INSTRUCTION #6: In order to mitigate an intentional killing to voluntary manslaughter, the burden of proof is on the defendant to establish that adequate provocation existed." "INSTRUCTION #8: Insanity is an affirmative defense and the burden of proof is on the defendant to establish that such insanity existed at the time of the killing." The jury found the defendant guilty of murder, and he appealed. He asserts that the jury instructions violated his rights under the federal Constitution. How should the appeals court rule? A Reverse the defendant's conviction, because Instruction #6 was improper. B Reverse the defendant's conviction, because Instruction #8 was improper. C Reverse the defendant's conviction, because both Instructions #6 and #8 were improper. D Uphold the defendant's conviction, because neither Instruction #6 nor Instruction #8 was improper.
A Reverse the defendant's conviction, because Instruction #6 was improper. The court should reverse the defendant's conviction because Instruction #6 requires the defendant to disprove one of the elements of murder. Due process requires in criminal cases that the state prove guilt beyond a reasonable doubt. The prosecution has the burden of proving all of the elements of the crime charged. Thus, if malice aforethought is an element of murder and voluntary manslaughter is distinguished from murder by the existence of adequate provocation, the defendant cannot be required to prove that he committed the homicide in the heat of passion (i.e., with adequate provocation). Such a requirement would impose on the defendant the burden of disproving the element of malice aforethought, because "heat of passion" negates malice. Although the defendant can be given the burden of going forward with some evidence on the provocation issue, once he has done so, the prosecution bears the burden of proving that the killing was not done in the heat of passion. In the case at issue, Instruction #6 requires a defendant to prove that he committed the intentional killing under adequate provocation. At common law, and consequently in the state, malice aforethought is an element of murder. Therefore, this instruction in effect requires the defendant to disprove the element of malice aforethought, thereby relieving the state of its burden of proving all elements of the crime. As discussed above, such an instruction cannot pass constitutional muster. On the other hand, for an affirmative defense such as insanity, it is permissible to impose the burden of proof on the defendant. Thus, Instruction #8 does not affect the state's obligation to prove all elements of the crime, and is permissible under the general principles mentioned above. Thus, (B) and (C) incorrectly state that this instruction is improper. (D) is incorrect because it states that Instruction #6 is proper. As explained above, this is not an accurate statement of the law.
A pedestrian brought suit against the driver of a car that struck him and caused him serious injuries and amnesia. During case preparation, the pedestrian's attorney was frustrated by his client's inability to remember any of the facts surrounding the accident. He therefore hired a qualified hypnotist to question the pedestrian. While hypnotized, the pedestrian described details of the accident, stating that the car that struck him ran a red light, and the car's license plate number was XYZ356. It so happens that XYZ356 is the license number of the driver's vehicle. At trial, the driver stipulated that his car struck the pedestrian, but asserts a defense that he was not negligent and that the light was green when he entered the intersection. Because the pedestrian's amnesia persists at the time of trial, the pedestrian's attorney wishes to call the hypnotist to the stand to testify as to the statements made by the pedestrian under hypnosis. The driver's attorney objects. How should the court rule on the objection? A Sustained, because the hypnotist's testimony would be hearsay not within any recognized exception to the hearsay rule. B Sustained, because the jury was not given a limiting instruction on the weight that should be given to testimony under hypnosis. C Overruled, because the accuracy of the license plate number establishes that the testimony is reliable. D Overruled, because the testimony falls within the catch-all exception to the hearsay rule.
A Sustained, because the hypnotist's testimony would be hearsay not within any recognized exception to the hearsay rule. The hypnotist's proposed testimony is hearsay evidence. It would be recounting an out-of-court statement (the pedestrian's hypnotically induced recollection of the accident) for the purpose of establishing the truth of the matter asserted in the statement. Therefore, the issue in deciding upon the admissibility of the hypnotist's testimony is to ascertain whether it fits within a hearsay exception. There is no hearsay exception for out-of-court statements given under hypnosis. The only hearsay exception that seems possibly applicable is the residual or "catch-all" exception, Federal Rule 803(24). Before evidence can be admitted under this exception, however, an elaborate pretrial notice procedure must be followed to assure that the other party has an opportunity to prepare to meet the hearsay evidence. No such notice was given in this case, and so (D) is incorrect. (D) is also incorrect because the catch-all exception was not meant to be a broad exception that could be invoked whenever some semblance of reliability could be attached to a hearsay statement. It was intended to be a narrow exception. It was not meant to revolutionize the law of evidence. To admit the hypnotist's testimony would have that effect. Because hypnotically induced testimony is generally inadmissible, to allow the admission of hypnotically induced hearsay statements would indeed be a radical change in the law of evidence. Because the hypnotist's testimony would be hearsay evidence not within any recognized exception to the hearsay rule, (A) is the correct answer. Both (B) and (C) are incorrect because they fail to give proper effect to the hearsay rule. (B) implies that the hypnotist's testimony would be admissible if the jury is given a limiting instruction on the weight that should be given to testimony under hypnosis. However, no instruction to the jury can cure the hearsay defect in the hypnotist's testimony; an instruction cannot magically transform inadmissible hearsay evidence into admissible evidence. (C) asserts that the testimony is admissible because the pedestrian's accurate recollection, under hypnosis, of the driver's license plate number establishes that the testimony is reliable. That assumption is highly questionable. It does not necessarily follow that, because a person can accurately recall specific numbers (e.g., a phone number), he can also accurately recall related events (e.g., who placed the phone call and what was said in the phone conversation). More fundamentally, the hearsay rule cannot be avoided simply by showing that hearsay evidence might be generally reliable. The hearsay rule can be avoided only if the hearsay evidence fits within a specific exception. Otherwise, the evidence is inadmissible. Because the hearsay testimony offered by the hypnotist does not fit within an exception, it is inadmissible.
A mother died, bequeathing all of her property to a trustee "to pay the income to my husband for life, and to distribute the principal to my son and daughter if they graduate from college. If they do not graduate from college, then the principal shall be distributed to charity." Subsequently, the son and the daughter graduated from college. Upon their graduation from college, how would the interests of the son and the daughter in the trust principal be classified? A Tenants in common to a vested remainder. B Joint tenants to a vested remainder. C Tenants in common to a fee simple absolute. D Tenants by the entirety to a fee simple absolute.
A Tenants in common to a vested remainder.
A landowner owned two adjoining parcels of land. The landowner sold the western parcel to a buyer by a deed that contained the following clause: "Grantee promises for himself, his heirs, successors, and assigns to not erect a structure over two stories on the land." The buyer recorded the deed and built a two-story house on the property and lived there for 30 years, after which he sold the land to a movie star by a deed that did not contain the structure height restriction. The movie star decided to tear down the existing house on the land and erect a three-story house. Her neighbor, who purchased the eastern parcel from the landowner 10 years earlier, discovers that the movie star's house will be larger than his and files suit to enforce the covenant. Who will prevail? A The neighbor, because the restrictive covenant runs with the land. B The neighbor, because privity is not required to enforce an equitable servitude. C The movie star, because her deed did not contain the restrictive covenant. D The movie star, because she had no notice of the structure height restriction.
A The neighbor, because the restrictive covenant runs with the land.
A federal law requiring that all automobiles driven on United States military bases be equipped with air bags would most probably be justified by which of the following? A The Property Clause of Article IV, Section 3. B The General Welfare Clause of Article I, Section 8. C The Supremacy Clause of Article VI, Section 2. D The Commerce Clause of Article I, Section 8.
A The Property Clause of Article IV, Section 3. Under Article IV, Section 3, Congress has the power "to make all needful rules and regulations respecting the territory or other property belonging to the United States." This power would encompass a regulation such as the air bag statute. Therefore, (A) is correct. (B) is incorrect because the General Welfare Clause is part of Congress's taxing and spending power; it does not authorize nonspending provisions that directly regulate an activity, such as the air bag provision. (C) is incorrect because the Supremacy Clause merely makes federal laws supreme over conflicting state laws. Here, there is no indication of any conflicting state law. (D) could be correct, but it is not the best answer. Congress does have very broad power over interstate commerce, and it could be argued that the regulation here affects interstate commerce, especially because transportation is involved. Nevertheless, (A) presents a more direct justification for the law because of its focus on federal property.
An attorney was employed by the United States Department of Health and Human Services in a regional office located in a tobacco-growing state. A labor contract between the agency and the clerical workers union contained a policy providing for termination of union employees only for certain specified grounds. The attorney, however, was not a member of the union and not covered by such a policy. The attorney was angered by the regional director's refusal to adopt a no-smoking policy for employees and visitors in the office. She posted a notice in the employee cafeteria ridiculing what she called the hypocrisy of an agency promoting health issues and nonsmoking programs while refusing to provide its employees with those same opportunities. The notice prompted a great deal of debate among the employees and was brought to the attention of the regional director, who was very displeased. Which of the following statements is most accurate regarding the director's right to dismiss the attorney? A The attorney has a liberty interest in the exercise of her First Amendment rights that entitles her to a hearing to contest the grounds of her dismissal. B The attorney has a property interest as a public employee that precludes her from being fired without notice and an opportunity to respond. C The attorney has no right to a hearing because her statements were not an expression of views on public issues. D The attorney has both a liberty interest and a property interest that entitles her to a pre-termination evidentiary hearing.
A The attorney has a liberty interest in the exercise of her First Amendment rights that entitles her to a hearing to contest the grounds of her dismissal. If the attorney is fired, she has a right to a hearing to determine whether her First Amendment rights were violated by her dismissal. Under the Due Process Clause of the Fifth Amendment, a person has a liberty interest in the exercise of specific rights provided by the Constitution, including freedom of speech. If a government employer seeks to fire an employee for speech-related conduct when the speech involved a matter of public concern but is not made pursuant to her official duties, the courts must carefully balance the employee's rights as a citizen to comment on a matter of public concern against the government's interest as an employer in the efficient performance of public service. Under the Court's expansive interpretation of what is a public issue in this context [see Rankin v. McPherson (1987)], the attorney's statement would probably qualify. At the very least, she can make a sufficient showing that her termination violates her free speech rights to be entitled to a hearing on the issue under procedural due process principles. [See Givhan v. Western Line Consolidated School District (1979)] (B) is wrong because the attorney does not appear to have a property interest in her job. A public employee who is subject to removal only for "cause" has a property interest in her job and generally must be given notice of the charges against her that are to be the basis for her job termination, and a pre-termination opportunity to respond to those charges. Here, however, the attorney did not have a property interest in her job; she could have been dismissed for no reason at all. She was not covered by the labor contract between the agency and its clerical workers, and there appears to be no other basis for her to claim an entitlement to continued employment. (C) is wrong because the attorney is entitled to a hearing as long as she can raise a prima facie claim that her speech, which was regarding an important health issue and the perception of her agency, was on a public issue and therefore protected by the First Amendment. (D) is wrong for two reasons. As discussed above, the attorney does not have a property interest in her job. Also, due process does not necessarily entitle her to a pre-termination evidentiary hearing; a post-termination evidentiary hearing is probably sufficient. [See Cleveland Board of Education v. Loudermill (1985)]
A plaintiff sued a defendant over a claimed debt. At the trial, the plaintiff established the existence of the debt and testified that he never received payment. In response, the defendant presents evidence sufficient to establish that she took her check to the post office and sent it to the plaintiff's proper address by certified mail. The defendant offers a certified mail receipt with an illegible signature, which she claims is the plaintiff's signature. The defendant also presents evidence that her basement flooded on March 28, and she claims that she cannot produce a canceled check because her box of canceled checks was destroyed from the water damage. Evidence is also presented that, due to a computer glitch, the defendant's bank cannot reproduce her checking account records for the months of February and March. After the defendant's testimony, which of the following is correct? A The burden of persuasion and the burden of going forward with the evidence are on the plaintiff. B The burden of persuasion is on the plaintiff, but he has no burden of going forward with the evidence. C The plaintiff has satisfied his burden of persuasion, but he has a burden of going forward with the evidence. D The plaintiff has satisfied both his burden of persuasion and his burden of going forward with the evidence.
A The burden of persuasion and the burden of going forward with the evidence are on the plaintiff. The burden of persuasion and the burden of going forward with the evidence are on the plaintiff because the defendant's testimony raises a rebuttable presumption that the check had been delivered in the mail. The burden of persuasion is the burden of a party to persuade the jury to decide an issue in its favor. If, after all the proof is in, the issue is equally balanced in the mind of the jury, then the party with the burden of persuasion must lose. The burden of persuasion does not shift from party to party during the course of a trial. Because the plaintiff sued the defendant for the debt, the plaintiff has the burden of persuasion when the time for the jury to make a decision arrives. The burden of going forward with the evidence is the burden of producing sufficient evidence to create a fact question of the issue involved. If a plaintiff makes out a prima facie case, he has met his burden of going forward with the evidence and the burden shifts to the defendant. Here, when the plaintiff made out a prima facie case of the defendant's debt, the burden of going forward with the evidence shifted to the defendant. The defendant met this burden through the use of a presumption. Federal Rule 301 provides that a presumption imposes on the party against whom it was directed the burden of going forward with the evidence to rebut the presumption. The defendant's evidence regarding the proper posting of the check raises a rebuttable presumption that the check was delivered to the plaintiff because a letter shown to have been properly addressed, stamped, and mailed is presumed to have been delivered in the due course of mail. Therefore, the burden of going forward with the evidence has shifted back again to the plaintiff, who must now produce evidence to rebut the presumption (i.e., evidence that he did not receive the check). (B) is incorrect because, as discussed above, the defendant's testimony raised a rebuttable presumption that the check was delivered in the mail, which shifted the burden of going forward with the evidence to the plaintiff. The fact that the plaintiff met his burden of going forward with the evidence of the debt once, when he made out his prima facie case, does not mean the burden cannot shift back to him. (C) is incorrect because the plaintiff has not satisfied his burden of persuasion. As discussed above, the burden of persuasion does not shift from party to party and is only a crucial factor when all the evidence is in. This burden is satisfied when the jury finds a party has been more persuasive in arguing his side of the issue than the other party. Because the defendant's testimony raises a rebuttable presumption that the check was delivered to the plaintiff, the plaintiff's burden of persuasion cannot be met until he offers evidence to prove that the check was not received (a necessary element of his case). (D) is incorrect because, as discussed above, the defendant's testimony raised a rebuttable presumption of delivery of the check in the mail to the plaintiff, which shifted the burden of going forward with evidence of nondelivery back to the plaintiff. The plaintiff's burden of persuasion cannot be satisfied until he comes forward with this evidence because a necessary element of his case is that the defendant never paid him.
A child was severely injured at an amusement park when she was ejected from a ride that went slightly off its track. The ride malfunctioned as a result of a manufacturer's defect, but had the child been properly secured in the ride's seatbelt by one of the ride operators, she would not have been injured. The child was unable to identify which ride operator improperly buckled her in. In the child's suit against the amusement park, who will win? A The child will win, because a ride operator failed to use reasonable care in securing the seatbelt. B The child will win, because operators of the ride did not discover the ride's defect through a reasonable inspection. C The amusement park will win, because the child cannot specifically identify which employee was negligent. D The amusement park will win, because the child's injury was caused by a defect that was present in the ride at the time of its purchase from the manufacturer.
A The child will win, because a ride operator failed to use reasonable care in securing the seatbelt.
A drug dealer is being tried in federal court for criminal conspiracy with a friend to violate a federal narcotics law. At trial, the prosecutor calls the drug dealer's new wife and asks her to testify about a meeting between the drug dealer and the friend that she observed before she married the drug dealer. Which of the following is the most accurate statement of the applicable rule concerning whether the wife may testify? A The choice is the wife's. B The choice is the drug dealer's. C The wife is permitted to testify only if both the wife and the drug dealer agree. D The wife may be compelled to testify even if both the wife and the drug dealer object.
A The choice is the wife's.
A brother and a sister held record title to a home as joint tenants with right of survivorship. The brother moved out of the home shortly after conveying his interest in the home to his friend by quitclaim deed. The friend did not record his deed. Several years later, the sister died, leaving her adopted daughter as her sole heir. Shortly after the sister died, the brother asked his friend to return his deed and give up his interest in the home. The friend agreed and returned the deed, which the brother destroyed. Who has title to the home? A The friend and the daughter as co-tenants. B The brother and the daughter as co-tenants. C The brother as sole owner. D The friend as sole owner.
A The friend and the daughter as co-tenants. The friend and the daughter own the home as co-tenants. A joint tenancy is an estate between two or more co-tenants who have a right of survivorship—when one joint tenant dies, the property is freed from her concurrent interest and the survivor or survivors retain an undivided right in the property. An inter vivos conveyance by one joint tenant of his undivided interest destroys the joint tenancy so that the transferee takes the interest as a tenant in common and not as a joint tenant. Here, when the brother conveyed his interest to the friend, the joint tenancy between the brother and the sister was severed. At that point, the friend and the sister held title to the home as tenants in common. The adopted daughter then inherited the sister's interest upon the sister's death. Because delivery of a deed cannot be canceled, the friend's return and subsequent destruction of his deed has no effect. Thus, (A) is correct and (C) is incorrect. (B) is incorrect because the brother has transferred his interest to the friend. As stated above, the destruction of the friend's deed has no effect on his interest. For the brother to have his interest back, the friend would have to reconvey by deed to him. (D) is incorrect because when the brother conveyed his interest to the friend, the joint tenancy was severed and the interest became a tenancy in common. Because a tenancy in common has no right of survivorship, when the sister died, her interest passed to her daughter as her heir.
A criminal suspect filed an action in federal district court against the police officer who arrested him and the city that employed the officer, claiming that the officer beat him even though he was not resisting arrest. The suspect's action asserted a battery claim against the police officer and a federal statutory claim against the city for violation of the suspect's civil rights. The suspect and the police officer are citizens of State A, and the city in which the arrest took place is in State A. The action seeks $50,000 each from the police officer and the city. Does the federal district court have subject matter jurisdiction over either of the claims? A The court has federal question jurisdiction over the federal statutory claim against the city and supplemental jurisdiction over the tort claim against the police officer. B The court has federal question jurisdiction over the federal statutory claim against the city, but it does not have subject matter jurisdiction over the tort claim against the police officer because the amount of that claim is too small. C The court has federal question jurisdiction over the federal statutory claim against the city, but it lacks subject matter jurisdiction over the tort claim against the police officer because it is a state law claim between two citizens of the same state. D The court does not have subject matter jurisdiction over either claim because the amount of each claim is too small.
A The court has federal question jurisdiction over the federal statutory claim against the city and supplemental jurisdiction over the tort claim against the police officer.
An acquaintance asked the defendant to give him a lift downtown because he did not have bus fare. While riding on the defendant's motorcycle, the acquaintance asked to stop at a convenience store to get a bottle of wine, showing the defendant a tire iron in his backpack that he was going to use. The defendant nodded in acknowledgment of what the acquaintance was planning to do. The defendant stopped at the store and waited in the parking lot while the acquaintance went in. He demanded money from the clerk, brandishing the tire iron. The clerk tried to grab a gun under the counter while he was filling a bag with money, and a struggle ensued. The gun discharged, killing the clerk. The defendant heard the gunshot and raced off, but was eventually apprehended. The jurisdiction's criminal code provides that a death caused during the commission of certain felonies, including robbery, is first degree felony murder, for which the death penalty is permitted. The code also permits cumulative penalties for first degree felony murder and for the underlying felony. The defendant was charged and convicted of both robbery and felony murder. After appropriate consideration of all relevant circumstances, the jury imposed the death penalty. On appeal, the defendant challenged both the convictions and the sentence. Assuming that the above facts were properly admitted into evidence, how should the appellate court rule? A The defendant's conviction for both offenses should be upheld, but imposition of the death penalty was not proper. B The defendant's conviction for both offenses should be upheld, and imposition of the death penalty was proper. C The defendant's conviction should be overturned under double jeopardy principles because robbery is a lesser included offense of felony murder. D The defendant's conviction for felony murder should be overturned because the circumstances do not establish the necessary degree of culpability.
A The defendant's conviction for both offenses should be upheld, but imposition of the death penalty was not proper. The defendant can be found guilty of robbery and felony murder, but the death penalty cannot be imposed. The defendant can be found guilty of robbery as an accomplice. The Supreme Court has held that, under the Eighth Amendment, the death penalty may not be imposed for felony murder where the defendant, as an accomplice, did not take or attempt or intend to take life, or intend that lethal force be employed. [Enmund v. Florida (1982)] Here, because the defendant's involvement in the crime was only to provide transportation, it cannot be said that he participated in such a major way that he acted with reckless indifference to human life; hence, the death penalty cannot constitutionally be imposed against him. (B) is therefore incorrect. (C) is incorrect because the defendant's conviction of both robbery and felony murder does not raise double jeopardy problems under these facts. Under the rule that lesser included offenses "merge" into greater offenses, a person may not be convicted of both the greater offense and a lesser included offense. While the Supreme Court has held that a subsequent prosecution for robbery is not permitted against a defendant who has been tried for felony murder where the robbery is the underlying felony, this situation is different. Imposition of cumulative punishments for two statutorily defined offenses arising from the same transaction and constituting the same crime does not violate double jeopardy when the punishments are imposed at a single trial, as long as the two offenses were specifically intended by the legislature to carry separate punishments. [Missouri v. Hunter (1983)] Here, the legislature did specifically provide for cumulative penalties for first degree felony murder and for the underlying felony. Thus, the defendant can be convicted of both robbery and felony murder. (D) is incorrect because the jury could properly find the defendant guilty of felony murder. When the felony murder rule is combined with accomplice liability rules, the scope of liability becomes very broad. The felony murder rule provides that a killing—even an accidental one—committed during the course of a felony is murder. All parties to the felony are liable for the murder as long as (i) it was committed during the commission of the felony or in fleeing from the scene, and (ii) it was a foreseeable result of commission of the felony. Courts have been willing to find most deaths committed during a felony to be foreseeable. Here, the jury could reasonably find the shooting death of a store clerk by the acquaintance during a struggle for a gun to be a foreseeable result of the commission of a robbery and impose felony murder liability on the defendant as an accomplice because he knew that the acquaintance was going to commit a robbery.
A delivery company employed several messengers to deliver packages by car to nearby towns. The company also allowed some employees to use company cars for personal use from time to time. A clerical employee had her car in a body shop because she had run a red light and been broadsided by another vehicle. This was the second time she had run a light and been hit. She borrowed a company car for the weekend and was using it to do some grocery shopping. The employee negligently went through a red light and crossed the path of a rented van. The man driving the van swerved to avoid the employee and struck a light post and several parked cars, severely damaging the van. At the time of the accident, the driver of the van was exceeding the posted speed limit; he would have been able to avoid hitting the light post and the cars had he been going the proper speed. The leasing company that had rented the van to the driver brought a lawsuit against the delivery company employee and the delivery company. The jurisdiction retains traditional contributory negligence rules. If the delivery company prevails in the lawsuit, what is the most likely reason? A The delivery company had no reason to know that its employee had a poor driving record. B The driver of the van had the last clear chance to avoid the accident. C The driver of the van exceeded the posted speed limit. D The employee had agreed to assume all liability when she borrowed the delivery company's car.
A The delivery company had no reason to know that its employee had a poor driving record.
A plaintiff sued a defendant in a contract dispute. The plaintiff calls a witness to testify as to his personal knowledge of the agreement. The plaintiff now wants a second witness to testify as to her knowledge of the first witness's honesty. The defendant objects and the court sustains the objection. Why is the testimony of the second witness inadmissible? A The first witness's credibility has not been questioned. B It would be inadmissible under the hearsay rule. C The second witness may not testify as to an opinion. D Character evidence is generally inadmissible in civil cases.
A The first witness's credibility has not been questioned.
An eight-year-old girl was playing catch on the sidewalk with her friend when her friend made an errant throw over the girl's head. The ball hit a pedestrian, who was walking on the sidewalk in the other direction. The pedestrian angrily threw the ball into the street. The girl ran out into the street to retrieve it and was hit and seriously injured by a car. If the girl's guardian considers legal action on her behalf against the pedestrian and the driver of the car, which of the following best states the pedestrian's liability? A The girl may have a personal injury claim against the pedestrian for negligence. B The girl has no claim against the pedestrian, but the driver may obtain contribution from the pedestrian. C The girl has no claim against the pedestrian, but the driver may obtain indemnity from the pedestrian. D The girl has no claim against the pedestrian, and the driver will not be able to obtain contribution or indemnity from the pedestrian.
A The girl may have a personal injury claim against the pedestrian for negligence. The girl may have a personal injury claim against the pedestrian for negligence because throwing the ball into the road would expose the children to an unreasonable risk of harm. A prima facie case for negligence consists of: (i) a duty on the part of the defendant to conform to a specific standard of conduct for the protection of the plaintiff against an unreasonable risk of injury; (ii) a breach of that duty by the defendant; (iii) the breach of duty was the actual and proximate cause of the plaintiff's injury; and (iv) damage to the plaintiff's person or property. A person is under a legal duty to act as an ordinary, prudent, reasonable person. It is presumed that an ordinary, prudent, reasonable person will take precautions against creating unreasonable risks of injury to other persons. Thus, if the defendant's conduct creates an unreasonable risk of injury to persons in the position of the plaintiff, then the general duty of care extends from the defendant to the plaintiff. Here, throwing the ball into the road created an unreasonable risk of injury to the girl and her friend. The pedestrian knew, or in the exercise of reasonable care should have known, that one or both of the children would run after the ball, exposing them to the danger of being hit by a car. Thus, the general duty of ordinary, reasonable care extended from the pedestrian to the girl, and he breached this duty by throwing the ball into the road. This breach actually caused the girl's injury because the girl would not have been in the road but for the pedestrian throwing the ball there. The breach also proximately caused the girl's injury, despite the fact that the driver was possibly negligent in not watching the road. The driver's conduct was an independent intervening force; however, it was a foreseeable intervening force that brought about a foreseeable result, because the pedestrian's act of throwing the ball into the road created a foreseeable risk that the girl would be hit by a car when chasing after the ball. Thus, the driver's conduct will not cut off the liability of the pedestrian. The final element of a prima facie case for negligence is made out by the damage to the girl's person. (B) and (C) are both incorrect for the reason that one from whom contribution is sought or against whom indemnity is sought must be originally liable to the plaintiff. Contribution allows a defendant who is required to pay more than his share of damages to have a claim against other jointly liable parties for the excess. Indemnity involves shifting the entire loss between or among tortfeasors. If, as (B) and (C) state, the girl has no claim against the pedestrian, then the pedestrian cannot be considered a joint tortfeasor with the driver. Consequently, the driver would have no right of contribution against the pedestrian, nor would he be entitled to indemnification from the pedestrian. (D) is incorrect because, as discussed above, the girl may have a personal injury claim against the pedestrian.
A defendant is on trial for violating a statute forbidding possession of a concealed weapon within 100 yards of a government building. The prosecution presents evidence that the defendant was arrested on a street corner with a handgun in his pocket. The building housing the local city hall occupies the entire block on the north and east sides of the two streets where the defendant was apprehended. Which of the following statements is most accurate regarding judicial notice of the location of the city hall? A The judge may take judicial notice of this fact without resort to a map, and should instruct the jury that it may, but need not, accept this fact as evidence of an element of the offense. B The judge may take judicial notice of this fact only upon reference to an official street map of the city. C The judge may not take judicial notice of this type of fact in a criminal case without a request by the prosecution. D If the judge properly takes judicial notice of this fact, a presumption is created that shifts the burden of persuasion to the defendant to disprove this fact.
A The judge may take judicial notice of this fact without resort to a map, and should instruct the jury that it may, but need not, accept this fact as evidence of an element of the offense.
A landowner and her neighbor owned large adjoining properties. The boundary line between the properties was never clearly marked. Twenty-five years ago, the landowner dug a water well on a section of the property that she thought was hers, but in fact was the neighbor's. The landowner has continued to use the water and to maintain the well on a regular basis ever since. The neighbor was adjudicated mentally incompetent 15 years ago. He died recently, and his executor has filed suit to eject the landowner and quiet title. The jurisdiction's statute of limitations for adverse possession is 20 years. With respect to the land on which the water well was dug, which of the following is correct? A The landowner has acquired title by adverse possession. B The landowner cannot claim title as an adverse possessor because she did not enter with hostile intent. C The landowner cannot acquire title because the neighbor was adjudicated incompetent. D The landowner has an implied easement in the land.
A The landowner has acquired title by adverse possession. The landowner has acquired title to the land by adverse possession. She has been in possession of the land on which the well was dug exclusively, openly, hostilely, and continuously for a period in excess of the statutory limitations period for adverse possession. Such title results from the running of the statute of limitations for trespass to real property. If an owner of real property fails to take legal action within the statutory period to eject a possessor who claims adversely to the owner, title to the property vests in the possessor, and the owner is barred from suing for ejectment. Adverse possession must be actual and exclusive (i.e., the possessor is not sharing with the true owner or the public at large). Also, the possession must be open and notorious (i.e., such as the usual owner would make of the land and sufficiently apparent to put the true owner on notice that a trespass is occurring). The possessor must occupy the property and enter without the owner's permission. The possessor need not believe that she has a right to possession. Finally, the possession must also be continuous throughout the statutory period. Here, the landowner has possessed the subject property openly and notoriously by digging a well. This is something that the usual owner would do on the land, and it is sufficiently apparent to put the neighbor on notice that a trespass is occurring. The landowner's possession has also been exclusive because she has not shared it with the neighbor or the public. This possession has been hostile because the landowner has entered the land without the neighbor's permission and has acted as would an owner. Finally, possession has been continuous for 25 years, which is longer than the limitations period of 20 years. Thus, the landowner has satisfied all of the elements required to obtain title by adverse possession. (B) is incorrect because hostile intent does not require that the possessor realize that the land is not her own. For purposes of adverse possession, the landowner's possession was hostile by virtue of the fact that it was without permission and in derogation of the neighbor's rights. (C) is incorrect because the statute of limitations for adverse possession does not begin to run if the owner is under a disability to sue at the time his cause of action accrues (when the claimant begins the adverse possession). Here the neighbor was not adjudicated to be mentally incompetent until 10 years after the landowner began her possession of the well property. Thus, the landowner can acquire title because the statute of limitations began to run before the neighbor's disability arose. (D) is incorrect because the landowner has acquired title to the land by adverse possession. The holder of an easement has only the right to use the land, but has no right to possess and enjoy the land. Moreover, implied easements typically arise when a parcel is divided and (i) there was an existing use prior to the severance, or (ii) the severance deprives one lot of access to a public road or utility line. Here, there is no evidence that the landowner's and neighbor's properties were severed from a unified parcel.
An owner of a parcel of land instructed his lawyer to draw up an instrument deeding the land to his friend's "nieces." The owner acknowledged the deed before a notary and signed it. As directed by the owner, the lawyer recorded the deed and then returned it to the owner. The owner put the deed in the drawer of his desk, intending to present it to the friend's nieces when they came to visit him next month. The following week, however, the owner died, leaving his daughter as his sole heir at law. The daughter discovered the deed to the land in the owner's desk. She filed an appropriate action to quiet title in the land, naming the friend's only two nieces as defendants. The only evidence presented at the trial was the deed itself, the evidence of recordation, and the lawyer's testimony regarding the owner's intent. Who should the court rule owns the land? A The nieces, because recordation of a notarized deed is prima facie evidence of delivery. B The nieces, because a deed is prima facie valid absent evidence to the contrary. C The daughter, because the evidence is insufficient to support a valid delivery. D The daughter, because the grantees in the deed are too indefinite.
A The nieces, because recordation of a notarized deed is prima facie evidence of delivery. The nieces own the land because recordation is prima facie evidence of delivery. To be valid, a deed must be "delivered," which means that the grantor must have taken some action (not necessarily a manual handing over of the deed) with the intent that it operate to pass title immediately. Recording a deed that has been acknowledged before a notary is such an action and is presumed to carry with it the requisite intent. Even without the knowledge of the grantee, delivery to the recorder's office will satisfy the delivery requirement. If the grantor intends the recording of the document to be the final act in vesting title in the grantee, then such recording constitutes delivery. (B) is wrong because a deed alone is not prima facie valid absent delivery. There must be evidence of delivery. (C) is wrong because recordation can constitute valid delivery, and there is sufficient evidence that the deed was recorded. Note that a rebuttable presumption of no delivery may arise from the grantor's retention of the deed. However, this presumption is rebutted by the recording of the deed. (D) is wrong because a description of the grantees in a deed is sufficient if it describes the grantees with sufficient particularity that it can be determined who is to take the property. The grantee need not actually be named. Because the friend has a finite number of nieces and they are easy to locate and identify, the deed from the owner satisfies this requirement.
The owner of a large screen television agreed to sell it for $2,500. The buyer made a down payment of $700, took possession of the television and agreed to pay the outstanding balance in 18 equal $100 installments, beginning on June 5, with subsequent installments due on the fifth of each month until the balance was paid in full. The buyer's friend owed her $2,000. On May 20, the buyer and her friend entered into an oral agreement whereby the friend agreed to make the 18 $100 installment payments to the seller in exchange for the buyer's promise to forgive the friend's $2,000 debt. On June 7, the seller called the buyer to ask her where his first $100 installment payment was, and she told him at that time of her agreement with her friend. The friend has made none of the installment payments. If the seller files suit against the friend demanding payment, who will prevail? A The seller, because he was a third-party beneficiary of the agreement between the buyer and her friend. B The seller, because he is an assignee of the buyer's rights against her friend. C The friend, because there was no consideration for her promise to the buyer. D The friend, because the surety provision of the Statute of Frauds prevents the seller from enforcing the friend's promise.
A The seller, because he was a third-party beneficiary of the agreement between the buyer and her friend. The buyer has delegated her duties under the agreement with the seller to her friend, and the friend has agreed to assume the duties by agreeing to make the 18 installment payments to the seller. Where a delegate's promise to perform the delegated duty is supported by consideration, there results a third-party beneficiary situation, so that the nondelegating party to the contract can compel performance or bring suit for nonperformance. The friend's promise to make the payments to the seller, totaling $1,800, was given in exchange for the buyer's promise to forgive the $2,000 debt owed by the friend to her. The buyer thus relinquished her right to take action against her friend for the full amount owed, thereby incurring legal detriment. Consequently, the promise of the friend was supported by consideration, and a situation arose in which the seller became a third-party beneficiary of the agreement between the buyer and her friend, and able to enforce performance of the friend's promise to pay. (C) is incorrect because, as explained above, the friend's promise to the buyer was supported by consideration. (B) is incorrect because there was no assignment of the buyer's rights as against the friend; i.e., the buyer did not manifest an intent to transfer to the seller her rights against her friend. Rather, the buyer transferred to her friend the duties that she owed to the seller. (D) is incorrect because the surety provision of the Statute of Frauds requires only that a promise to answer for the debt or default of another be in writing. Such a promise must be collateral to another person's promise to pay rather than a primary promise (a promise to pay directly for the benefits given to another). Here, the friend did not promise the seller that if he sold the entertainment system to the buyer and the buyer did not pay, she (the friend) would pay. Instead, the friend promised the buyer that she would directly perform the buyer's obligation to pay the seller. Thus, this is not the type of promise required by the Statute of Frauds to be in writing.
A man who belonged to an ancient religion whose rituals require the use of bald eagle feathers traveled to an area where bald eagles were known to roost. After searching the area, he found a fallen eagle feather and returned home. A few weeks later, the man showed the feather to an acquaintance, who happened to be a state park ranger, and explained how the feather was obtained. The ranger informed the man that a state anti-poaching law makes any possession of a bald eagle feather without a special permit a crime. The ranger then cited the man for possession of the feather and confiscated it. At the man's trial for violating the state bald eagle feather possession statute, which of the following constitutional arguments is most appropriate for the prosecution to make? A The statute is a neutral law that only incidentally burdens the man's rights under the First Amendment. B The Free Exercise Clause applies only to belief and not to conduct. C The government has a substantial and important interest in protecting bald eagles and there is no other feasible way to achieve the legislative purpose. D Making an exception for the man on religious grounds would violate the Establishment Clause of the First Amendment.
A The statute is a neutral law that only incidentally burdens the man's rights under the First Amendment. The best argument for the prosecution is that the Free Exercise Clause does not afford a right to a religious exemption from a neutral law that happens to impose a substantial burden on a religious practice, if the law is otherwise constitutionally applied to persons who engage (or fail to engage) in the particular conduct for nonreligious reasons. Here, the state law interferes with the man's religious beliefs. However, the statute prohibits any possession of a bald eagle feather without a permit. Thus, the state should argue that the law was enacted to protect eagles and not merely to interfere with the religious beliefs of people such as the man here. (B) is incorrect because it is too broad. Conduct is protected (although the protection is limited). For example, the government cannot punish conduct merely because it is religious (although if the law affects both religious and nonreligious conduct, it is generally valid). [See, e.g., Employment Division v. Smith (1990)] (C) is incorrect because it states the former rule in these cases. In the past, the Court used a balancing test to determine whether a religious exemption had to be granted from a law with a secular purpose that happened to burden religious practices or beliefs. The Court would consider the severity of the burden, the strength of the state's interest, and the existence of alternative means. Now, however, the Court no longer uses a balancing test in actions involving state laws; the state need not establish a strong interest or a lack of alternative means if the challenged statute is neutral. (D) could be successfully argued, but its chances for success are not as certain as for the argument in (A). The Establishment Clause prohibits laws respecting the establishment of religion. If a law includes a preference for one religious sect over another, the law will be held invalid unless it is narrowly tailored to promote a compelling interest. If there is no sect preference, the law is valid if: (i) it has a secular purpose; (ii) its primary effect neither advances nor inhibits religion; and (iii) it does not produce excessive government entanglement. Here, no sect preference appears, because nothing indicates that an exception would apply only to members of the man's religion. It could be argued, however, that the only purpose for an exemption here is to favor religious believers over nonbelievers. If that is the purpose, the exemption would not have a secular purpose and would fail the secular purpose test above. On the other hand, the state could argue that free exercise of religion is also protected, and an exemption protects sincerely held religious beliefs. [See, e.g., Wisconsin v. Yoder (1979)] Thus, the outcome of the argument in (D) is uncertain, and (A) is the state's most appropriate argument.
While at a party, a wife came up behind a younger woman, grabbed her by her arm, and accused her of having an affair with the wife's husband. The wife knew that her accusation was not true. Of the following facts, which would be most helpful to the younger woman in a suit against the wife for intentional infliction of emotional distress? A The wife knew that the younger woman is very religious, and her religious beliefs strongly condemn adultery. B When the wife grabbed her arm, it caused the younger woman great pain and she has suffered an upset stomach from the trauma of it. C Other people at the party overheard the wife's accusation. D The younger woman's employer heard of the accusation and did not give her a promotion.
A The wife knew that the younger woman is very religious, and her religious beliefs strongly condemn adultery. The wife's knowledge of the younger woman's religious beliefs would be most helpful in the woman's suit. The tort of intentional infliction of emotional distress requires: (i) an act by defendant amounting to extreme and outrageous conduct; (ii) intent on the part of the defendant to cause the plaintiff to suffer severe emotional distress, or a reckless disregard that the conduct would cause emotional distress; (iii) causation; and (iv) damages—severe emotional distress. The statement in (A) is most helpful to establish reckless disregard because it shows that the defendant knew of plaintiff's peculiar susceptibility to such an accusation. Given her strong religious beliefs, the woman may have suffered severe distress (even though the accusation was not true) because the wife gave her every reason to think that she believed it to be true. (B) is incorrect because, while one of the damages in an action for battery may be for emotional suffering caused by the battery, the younger woman is suing for intentional infliction of emotional distress, which requires severe emotional distress from the outrageous nature of the conduct. (C) is not the best answer, because while evidence that it happened in front of other people may show the "outrageousness" of the conduct, an act is not outrageous just because it occurs in the presence of others. Furthermore, nothing in choice (C) establishes the wife's intent. (D) is incorrect because this tort does not require proof of economic damages.
A young woman went to her local shoe shop and selected a pair of shoes. She gave the salesperson cash for the shoes. As the salesperson was putting the shoes into a bag, a robber brandishing a gun entered the store, forced the salesperson to put all of the money in the register into the bag with the shoes, and fled with the bag, the money, and the shoes. After the police had come, the young woman asked the salesperson to get her another pair of shoes. He told the young woman that she would have to pay for them again. The young woman refused. If the young woman sues the shoe shop for another pair of shoes, who will prevail? A The young woman, because she did not yet have possession of the shoes. B The young woman, because the purpose of the contract had been made impossible by an unforeseen event. C The shoe shop, because title to the shoes had already passed to the young woman. D The shoe shop, because the contract goods had already been identified.
A The young woman, because she did not yet have possession of the shoes.
A plaintiff sued a defendant for defamation, asserting in her complaint that the defendant had called the plaintiff a thief in front of a number of business associates. The plaintiff calls two witnesses to the stand, both of whom testify that they heard the defendant refer to the plaintiff as a thief in front of the business associates. The plaintiff does not take the stand herself. The defendant pleads truth of the statement as an affirmative defense and calls a witness to the stand. The defense witness is prepared to testify that he was a co-worker of the plaintiff when the plaintiff supplemented her income by tending bar three nights a week. The witness will testify that he saw the plaintiff take a $20 bill from the tavern's cash register and secrete the money in her pocket. The plaintiff's attorney objects. May the defense witness's testimony be allowed? A Yes, as substantive evidence that the plaintiff is, in fact, a thief. B Yes, because theft is a crime indicating dishonesty. C No, because specific bad acts may not be used to show bad character. D No, because the plaintiff never took the stand.
A Yes, as substantive evidence that the plaintiff is, in fact, a thief. The defense witness's testimony is admissible character evidence because the plaintiff's character is directly in issue in the case. As a general rule, evidence of character to prove the conduct of a person in the litigated event is not admissible in a civil case. However, when proof of a person's character, as a matter of substantive law, is an essential element of a claim or defense in a civil action, character evidence is admissible because it is the best method of proving the issue. Under the Federal Rules, any of the types of evidence—reputation, opinion, or specific acts—may be used. Here, character is an issue in the plaintiff's defamation action because the defendant has pleaded as an affirmative defense that his statement claiming that the plaintiff is a thief is the truth. The defense witness's testimony that he saw the plaintiff take the money from the cash register is relevant because it tends to show that the defendant spoke the truth. Hence, it should be allowed. (B) is incorrect because the fact that the theft here could be considered a crime of dishonesty would be relevant only if the plaintiff's credibility were being impeached, and only then if proof of an actual conviction were provided. Here, the testimony is admissible because it is being offered as substantive evidence of an aspect of the plaintiff's character that is an essential element of a defense in the case. (C) is incorrect. One of the few cases where testimony as to specific acts of a person may be used to show that person's character is when character itself is one of the essential issues in the case, as it is here. (D) is incorrect because the fact that the plaintiff never took the stand only means that she has not placed her credibility in issue and become subject to impeachment. Here, however, the plaintiff's character is in issue and the testimony is being offered as substantive evidence of her character rather than to impeach her credibility.
In an action by the plaintiff against the defendant, one of the issues is whether the defendant is a licensed physical therapist. Normally, the names of all licensed physical therapists are registered with the office of the state Department of Professional Registrations. The plaintiff wishes to introduce a certified document, signed by the chief registrar of the department (who cannot be located), stating that an examination of the department's rolls does not disclose the defendant's name. Should the document be admitted? A Yes, because a statement of absence from public record is admissible. B Yes, because the chief registrar is unavailable. C No, because the document is hearsay not within an exception. D No, because the document is not self-authenticating.
A Yes, because a statement of absence from public record is admissible.
A defendant was involved in an accident in which her car struck the rear end of the car driven by the plaintiff. The police issued tickets to the defendant, charging her with reckless driving and speeding. When the defendant's case came before the traffic court, her attorney entered into a plea bargain with the prosecutor. Under the plea bargain, the defendant agreed to plead guilty to speeding and to pay a fine of $100, and the prosecution agreed to drop the reckless driving charge. Accordingly, the defendant pleaded guilty and the court fined her $100. In the later civil suit, where the plaintiff is seeking damages from the defendant for personal injuries, is the guilty plea before the traffic court admissible? A Yes, because it is a statement by an opposing party. B Yes, because it is a statement against interest. C No, because there is a public policy in favor of plea bargaining to promote court efficiency. D No, because no felony was involved.
A Yes, because it is a statement by an opposing party.
On an icy day, a vehicle driven by the defendant struck the plaintiff's car in the rear, smashing a taillight and denting the plaintiff's bumper. Before the plaintiff could say anything, the defendant rushed out of his car and told the plaintiff, "Look, if you'll take $500 for the damage, I'm sure my insurance company will pay for it." The plaintiff refused and sued the defendant for damage to his car and minor personal injuries. The plaintiff wishes to testify as to the defendant's statement at the time of the accident. The defendant objects. Should the court allow the defendant's statement to be admitted? A Yes, because it is a statement by an opposing party. B Yes, because it is hearsay within the statement against interest exception. C No, because the statement took the form of a settlement negotiation. D No, because the statement is hearsay not within any exception.
A Yes, because it is a statement by an opposing party.
A plaintiff is suing a defendant in federal court for personal injuries arising out of an automobile accident. The defendant's automobile insurance policy covers the accident. Must the defendant disclose the existence of the insurance policy? A Yes, because it is required under the Federal Rules. B Yes, unless the discovery of the insurance coverage would not lead to other discoverable evidence. C No, unless the plaintiff submits an interrogatory. D No, because insurance coverage cannot be mentioned at trial.
A Yes, because it is required under the Federal Rules. The defendant must disclose the existence of the insurance policy under the Federal Rules, which expressly permit discovery of insurance agreements as an initial disclosure. (B) is incorrect. There is no need to show that the discovery of insurance coverage would lead to other admissible evidence. (C) is incorrect. Initial disclosures must be made regardless of whether the opposing party submits a request for the information. (D) is incorrect. Even though insurance coverage is not relevant to the case, insurance coverage is nonetheless discoverable.
Based on recommendations of a state commission studying the effect of pornographic films on violent criminal activity, a state adopted legislation banning films intended for commercial distribution that appealed as a whole to the prurient interest in sex of the average person in the community, portrayed sex in a patently offensive way to citizens of the state, and which a reasonable person in the United States would find had no serious literary, artistic, political, or scientific value. In ruling on a constitutional challenge to the legislation from a film distributor in the state who was convicted of distributing films in violation of the legislation, will the federal court likely find the legislation to be constitutional? A Yes, because it uses a national "reasonable person" standard for determining the social value of the work. B Yes, because it uses a statewide standard rather than a community standard for determining whether the material is patently offensive. C No, because it uses a statewide standard rather than a national standard for determining whether the material is patently offensive. D No, unless the court finds that the legislation is necessary to advance the state's compelling interest in reducing violent criminal activity.
A Yes, because it uses a national "reasonable person" standard for determining the social value of the work. The court will likely find the legislation to be a constitutional regulation of obscenity. Obscenity, which is not protected speech under the First Amendment, is defined by the Supreme Court as a description or depiction of sexual conduct that, taken as a whole, by the average person, applying contemporary community standards, appeals to the prurient interest in sex, portrays sex in a patently offensive way, and—using a national reasonable person standard—does not have serious literary, artistic, political, or scientific value. Thus, the legislation here is constitutional because it uses a reasonable person standard, rather than a community standard, for determining the value of the work. (B) is incorrect because while a statewide standard for determining whether the material is patently offensive is permissible, it is not mandatory. A state may use a "community standard" for making this determination. (C) is incorrect because, again, a statewide standard for determining whether the material is patently offensive is permissible. Only the "social value" element of the obscenity test requires a national standard. (D) is incorrect because the legislation is valid regardless of whether it is necessary to achieve the state's compelling interest in reducing violent crime. Speech that falls within the definition of obscenity is unprotected speech; the government does not need a specific compelling interest to ban it.
Thirty years ago, a power company constructed a power dam on a river. At the time the dam was constructed, the power company solicited and received express easements from all of the landowners in the river valley, including a farmer. The power company paid fair value for the easements, which would allow the company to release water from the dam at certain times of the year, resulting in flooding of the land in the river valley. In the 30 years since the dam was constructed, the farmer's property has never been flooded, and the farmer has been using his land in the same way as he did 30 years ago. Now, however, the power company wants to substantially increase power production from the dam. All landowners in the valley were notified by the company that henceforth all 200,000 acres (including the farmer's 200 acres) would be flooded in accordance with the company's rights under the easement. The farmer reviewed the easement for his property and discovered that it lacked the requisite grantor's acknowledgment and thus was improperly recorded. The state's adverse possession statute requires hostile occupation for a period of 20 years. May the power company properly flood the farmer's land under the terms of the easement? A Yes, because such flooding is within the terms of the easement. B No, because the state's adverse possession statute requires hostile occupation for a period of only 20 years. C No, because the company has failed to exercise its rights under the easement for 30 years, and the easement has lapsed. D No, because the easement was not properly acknowledged and recorded.
A Yes, because such flooding is within the terms of the easement.
A bicycle manufacturer manufactured a bicycle that it sold to a retail dealer. The bicycle had a serious manufacturing defect in its brakes, but the dealer did not discover the defect before putting it on the sales floor despite a careful inspection of the bicycle. The retail dealer sold the bicycle to a bicycle messenger. Shortly thereafter, while the messenger was riding the bicycle along a busy city street, he saw a traffic light facing him turn from green to yellow. He sped up, hoping to cross the intersection before the light turned red. However, the messenger quickly realized that he could not do so and applied the brake, which failed and caused him to crash. The messenger sustained injuries. Assume that the jurisdiction follows traditional contributory negligence rules. If the messenger asserts a claim against the retail dealer based on strict liability in tort, will the messenger prevail? A Yes, because the brake failed because of a dangerous defect present when the bicycle left the factory of the manufacturer. B Yes, because the brake failed while the messenger was riding the bicycle. C No, because the messenger contributed to his own injury by speeding up. D No, because the retail dealer carefully inspected the bicycle before selling it.
A Yes, because the brake failed because of a dangerous defect present when the bicycle left the factory of the manufacturer. The messenger will prevail. In a strict liability action, the plaintiff must prove that a product was so defective that it is unreasonably dangerous. The defect causing the harm must have existed when the product left the defendant's control. The defendant must be a commercial supplier of the product in question. Brake failure on a bicycle is an unreasonably dangerous defect. If this defect existed when the bicycle left the factory of the manufacturer, then the messenger has a viable cause of action sounding in strict liability against the retail dealer, a supplier in the distributive chain. Thus, (A) is correct. (B) is wrong because it implies absolute liability, not strict liability; i.e., the retail dealer is not liable simply because the brakes failed. It must be established that the brakes were defective when placed in commerce. (C) is wrong because, in jurisdictions retaining traditional contributory negligence rules, ordinary contributory negligence does not bar recovery in strict liability cases where the plaintiff fails to discover the defect or to take steps to guard against its existence. (D) is wrong because a careful inspection would be relevant to a negligence action, but not to one based on strict liability.
A buyer for a chain of shoe stores ordered 1,000 pairs of shoes from a shoe manufacturer. The shoes cost $50 per pair, so the total contract price was $50,000. It happened that the manufacturer owed $50,000 to a trucking company. The manufacturer assigned, in writing, "all proceeds from the contract with the buyer" to the trucking company. The manufacturer notified the buyer that he had assigned the proceeds of the contract to the trucking company and then shipped the 1,000 pairs of shoes to the buyer. Upon receipt of the shoes, the buyer discovered that 10% of the shoes were defective. He sent a check for 90% of the contract price ($45,000) to the manufacturer, who deposited the check. Shortly thereafter, the manufacturer closed down its business and disappeared without a trace. The trucking company, meanwhile, demanded payment from the buyer, to no avail. If the trucking company sues the buyer for the $45,000 that the buyer paid on the contract, will the trucking company prevail? A Yes, because the buyer had notice from the manufacturer that the contract had been assigned to the trucking company. B No, because the manufacturer wrongfully took the money that was assigned to the trucking company and is solely liable to the trucking company. C No, because the buyer fulfilled his obligations under the contract by paying the manufacturer. D No, because the trucking company could not have performed the other side of the contract by furnishing the shoes.
A Yes, because the buyer had notice from the manufacturer that the contract had been assigned to the trucking company.
A woman was stopped at a police roadblock to check for drunk drivers. The police were stopping every third vehicle that came through the checkpoint, and the woman's car turned out to be a third vehicle. After failing a field sobriety test, the woman was arrested and charged with driving while intoxicated. Was the stop of the woman's car legal? A Yes, because the car was stopped at a fixed checkpoint to check for drunk drivers. B Yes, because temporarily stopping a car does not constitute a seizure of the automobile. C No, because not every car was being stopped. D No, because there was no probable cause to stop the vehicle.
A Yes, because the car was stopped at a fixed checkpoint to check for drunk drivers.
A sportscaster on a local television show interviewed the parent of a child on a high school football team. The interviewee told the sportscaster that the head football coach "openly condones the use of steroids by team members." The coach, who had always conducted a strong anti-drug program for his football players, watched and recorded the show daily. He was outraged when he saw the live broadcast, and filed suit for defamation against the interviewee, the sportscaster, and the television station. At the trial of the suit, the coach wishes to testify as to what the interviewee said on the television show. The defense objects. Should such testimony be admitted? A Yes, because the coach saw the live television broadcast. B Yes, because the matter goes to the ultimate issue of the case and is thus highly relevant. C No, because a videotape of the broadcast is available. D No, because such testimony would be hearsay, not within any recognized exception to the hearsay rule.
A Yes, because the coach saw the live television broadcast.
On February 3, a property owner mailed an offer to a married couple who had expressed an interest in buying his property at 337 Green Street. The offer asked for $200,000, "terms $60,000 cash, with the balance secured by a first mortgage." The offer reached the couple on February 5. On February 8, the couple replied by e-mail that the offer had been received and was being considered, and added, "We would much prefer a straight cash deal. Would you consider an immediate purchase for $180,000 cash?" On February 10, the property owner responded with a one-word e-mail: "No." After reading the property owner's response, on February 11 the couple e-mailed: "E-mail received. We accept your offer of February 3. Tender the deed c/o our agent, The First National Bank and Trust." The property owner now refuses to sell the property. If the couple sues for specific performance, are they likely to succeed? A Yes, because the couple's February 8 e-mail was an inquiry. B Yes, because the couple's response of February 8 operated as an acceptance. C No, because the couple's response of February 8 operated to terminate the property owner's offer. D No, because the communications fail to establish the terms of the proposed agreement with sufficient definiteness to be enforced through specific performance.
A Yes, because the couple's February 8 e-mail was an inquiry.
A lawyer sued a client for his fee, based on an agreed hourly rate. The client subpoenaed the lawyer's time records for the days on which he purported to have worked for the client to show that the lawyer had billed an impossible number of hours to the client and others on those days. The client's subpoena provided that any information concerning the matters handled for other clients be deleted or masked. The lawyer moved to quash the subpoena on the ground of attorney-client privilege. Should the subpoena be upheld? A Yes, because the information about hours billed is not within the privilege. B Yes, because a lawyer has no right to invoke his client's privilege without instructions from the client. C No, because a lawyer is entitled to a right of privacy for the work product in his files. D No, because no permission was obtained from the other clients to divulge information from their files.
A Yes, because the information about hours billed is not within the privilege
A driver was traveling through an area plagued with a high incidence of burglaries and assaults. Acting pursuant to a police department plan to combat crime by the random stopping of automobiles in the area between midnight and 6 a.m., a police officer stopped the driver and asked him for identification. As the driver handed the officer his license, the officer directed a flashlight into the automobile and saw what appeared to be the barrel of a shotgun protruding from under the front seat on the passenger side of the car. The officer ordered the driver from the car, searched him, and discovered marijuana cigarettes and a shotgun. At the driver's trial for unlawful possession of narcotics, should his motion to suppress the use of the marijuana as evidence be sustained? A Yes, because the marijuana was discovered as a result of the unlawful stopping of the driver's automobile. B Yes, because the use of the flashlight constituted a search of the interior of the driver's automobile without probable cause. C No, because the officer's conduct was consistent with the established police plan. D No, because the discovery of the gun in plain view created the reasonable suspicion necessary to justify the arrest and search of the driver.
A Yes, because the marijuana was discovered as a result of the unlawful stopping of the driver's automobile.
A local entertainment section of a newspaper published a story on the town's business district, accompanied by photos of various businesses in the district. A minister who happened to be walking on the sidewalk in front of an adult bookstore when a photo was taken for the story became very upset when he saw it in the newspaper, because the camera angle made it appear that he was exiting the bookstore. If the minister sues the newspaper for invasion of privacy and establishes the above facts, is he likely to prevail? A Yes, because the photo made it appear as if he was exiting an adult bookstore. B Yes, because the newspaper made a public disclosure of a private fact. C No, because he was on a public sidewalk when the photo was taken. D No, because he has not alleged any economic or pecuniary damages.
A Yes, because the photo made it appear as if he was exiting an adult bookstore. The minister likely will prevail because unauthorized use of his picture that falsely makes him appear to be exiting the adult bookstore would be highly offensive to a reasonable person under the circumstances and constitute a false light invasion of privacy. To establish a prima facie case for invasion of privacy based on publication by defendant of facts placing plaintiff in a false light, the following elements must be proved: (i) publication of facts about plaintiff by defendant placing plaintiff in a false light in the public eye; and (ii) the "false light" must be something that would be highly offensive to a reasonable person under the circumstances. Here, the photo created the false impression that the minister was exiting an adult bookstore. Publication of the photo conveying this false impression of the minister's conduct would be highly offensive to a reasonable person under the circumstances. (B) cannot be correct because it is couched in terms of disclosure of private facts about the plaintiff. To establish a prima facie case for invasion of privacy based on public disclosure of private facts about plaintiff, the following elements must be proved: (i) publication or public disclosure by defendant of private information about the plaintiff; and (ii) the matter made public is such that its disclosure would be highly offensive to a reasonable person of ordinary sensibilities. Here, no private facts were disclosed, and therefore an action based on public disclosure of private facts will not succeed. (C) is incorrect. Because the minister was placed in a false light, it makes no difference that the picture was taken on a public sidewalk. An invasion of privacy based on false light can occur on public property as well as private property. (D) is incorrect. The absence of economic harm to the minister has no bearing on a privacy action. In an action for invasion of right to privacy, the plaintiff need not plead and prove special damages, providing the elements of a prima facie case are present. In other words, emotional distress and mental anguish are sufficient damages.
A plaintiff corporation filed an antitrust action against two defendant corporations in federal district court. The action seeks damages based on the defendants' allegedly conspiring to fix prices in violation of federal antitrust statutes. The plaintiff has in its possession three internal memoranda written by key officers of the defendants that indicate that the defendants in fact were conspiring to fix prices. Absent an applicable discovery request from the defendants, must the plaintiff disclose to the defendants the plaintiff's knowledge and possession of the memoranda? A Yes, because the plaintiff must produce without any request all documents in its possession that it may use as evidence to support its claim. B Yes, because any party must produce without request all documents in its possession that are relevant to a claim or defense in the action. C No, because parties must produce documents only in response to appropriate requests for production of documents. D No, because they were written by the defendants and the defendants should thus be aware of their existence.
A Yes, because the plaintiff must produce without any request all documents in its possession that it may use as evidence to support its claim.
A plaintiff sues her employer for sexual harassment. During the trial, the plaintiff attempts to introduce into evidence company records that include written complaints from other employees alleging that they too were sexually harassed by the employer. The defense objects to the admission of the records on the basis of hearsay. Should the objection be sustained? A Yes, because the records are hearsay not within any recognized exception. B No, because the records qualify under the business records exception. C No, because the records qualify as a statement against interest. D No, because the records are not hearsay.
A Yes, because the records are hearsay not within any recognized exception.
Two law students ranked high in their class were competing for one opening at a prestigious law firm. During the interview with the hiring partner, one student was asked what he thought of the other's work as an editor of the law review. The student responded that there was a rumor around the school that the editor got outside help on her law review comment. Based in large part on his statement, that student was chosen over the law review editor, who later accepted a less lucrative position with another firm. If the law review editor brings a slander action against the other student and establishes the above facts, will she prevail? A Yes, because the student's statement to the hiring partner was defamatory. B Yes, because the law review editor suffered special damages. C No, because the hiring partner asked the student for his opinion. D No, because the law review editor did not establish that the student made the statement with at least negligence.
A Yes, because the student's statement to the hiring partner was defamatory. The student's statement constitutes slander per se and therefore the student will be liable. To establish a prima facie case for defamation, the following elements must be proved: (i) defamatory language on the part of the defendant; (ii) the defamatory language must be "of or concerning" the plaintiff (i.e., it must identify the plaintiff to a reasonable reader, listener, or viewer); (iii) publication of the defamatory language by the defendant to a third person; and (iv) damages to the reputation of the plaintiff. Here, the student's suggestion that the law review editor received outside help on an article she authored impeaches her integrity and legal skills. The defamatory language directly related to the editor. The publication requirement is satisfied because the student made the statement to the hiring partner. To recover damages for slander, special damages must be pleaded and proved unless the spoken defamation falls within one of four categories, characterized as slander per se. Hence, a defamatory statement adversely reflecting on the plaintiff's abilities in his business, trade, or profession is actionable without pleading or proof of special damages. The student's statement adversely reflected on the law review editor's honesty and capability in her profession, and as such is slander per se. (B) is incorrect because, as noted above, the student's defamatory statement adversely reflecting on the editor's abilities in her profession is actionable without proof of special damages. (C) is incorrect because the mere fact that the interviewer asked the student his opinion does not justify a defamatory response. The student did not have a common law qualified privilege to make the statements because he was not a former employer of the law review editor and was not yet a member of the hiring partner's firm (negating any common interest privilege). Furthermore, once publication is established, it is no defense that the defendant had no idea that the publication was defamatory. It is the intent to publish, not the intent to defame, that is the requisite intent. Thus, even if the student thought that his comments did not constitute defamation because they were in response to the interviewer's question, he could still be found liable for defamation assuming all other elements of the tort were satisfied. (D) is incorrect because it states a standard for private persons suing on matters of public concern. Private plaintiffs must show that the defendant was at least negligent as to truth or falsity in making the statement, but here no matter of public concern is involved, so fault need not be shown. Also, since a matter of public concern is not involved, the plaintiff does not need to establish that the statement was false; a defamatory statement is presumed false at common law.
In a criminal trial, the prosecutor called a witness to the stand to authenticate the voice in a tape recording as the defendant's. The only other time the witness had heard the defendant's voice was after his arrest. Assuming a proper foundation has been laid, may the witness properly authenticate the defendant's voice? A Yes, because the witness is now familiar with the defendant's voice. B Yes, because the prosecutor can qualify the witness as an expert on the defendant's voice. C No, because the witness's testimony would be inadmissible hearsay. D No, because the witness did not hear the defendant's voice until after he was arrested.
A Yes, because the witness is now familiar with the defendant's voice. The witness may properly authenticate the defendant's voice because she is now familiar with his voice. Any person familiar with an alleged speaker's voice may authenticate a recording of the voice by giving an opinion as to its identity. Thus, because the witness is now familiar with the defendant's voice, she may give her opinion as to whether it is his voice on the tape. (B) is incorrect because the witness does not need to be qualified as an expert—lay opinion testimony is sufficient to identify a voice (assuming the lay witness is familiar with that voice). Expert testimony is appropriate only when the subject matter is one where scientific, technical, or other specialized knowledge would assist the trier of fact in understanding the evidence or determining a fact in issue. [Fed. R. Evid. 702] Here, identification of the defendant's voice does not require such specialized knowledge; rather, all that is required is familiarity with his voice. (C) is incorrect because the witness's testimony would not be hearsay at all. Hearsay is a statement, other than one made by the declarant while testifying at the trial or hearing, offered in evidence to prove the truth of the matter asserted. [Fed. R. Evid. 801(c)] The witness is not going to be testifying to an out-of-court statement. Rather, she will testify that, being familiar with the voice of the defendant by virtue of having heard that voice before, she can now identify the voice on the tape as being that of the defendant. Because the witness will not be testifying as to any particular statement made by the defendant, there is no hearsay problem. (D) is incorrect because, as long as the witness is familiar with the voice of the defendant, it makes no difference that she acquired such familiarity only after he was arrested. Thus, the witness may properly authenticate the voice.
A worker who missed his ride home because he was working late walked across the street to a tavern to get a drink. He chatted with a patron of the tavern and discovered that he lived only a short distance from the patron. The patron offered to give the worker a ride home. Although he knew the patron was probably too drunk to drive, the worker reluctantly agreed. On the way home, the patron, driving in a dangerous manner, was involved in a collision with another car, whose driver was also driving negligently, and the worker was injured. If the worker sues the patron to recover for his injuries and the above facts are established at trial, will the worker's recovery be reduced? A Yes, because the worker knew that the patron was drunk. B Yes, because the other driver involved in the collision was also negligent. C No, because the negligence of the patron and the other driver were the proximate causes of the accident. D No, as long as the worker joins the other driver in his lawsuit.
A Yes, because the worker knew that the patron was drunk.
A three-car accident occurred in which the drivers were a citizen of State A, a citizen of State B, and a citizen of State C. The State A citizen filed a negligence action against the other two in federal district court and lost his case. After judgment, may the State C citizen assert and maintain a negligence action against the State B citizen seeking damages for the injuries the State C citizen sustained in the same accident? A Yes, because, while the State C citizen could have asserted the claim as a cross-claim in the prior action, he may wait and assert it as an independent action. B Yes, because the State C citizen could not have asserted the claim in the prior action and thus may assert it independently. C No, because the State C citizen's claim was a compulsory cross-claim in the prior action and, since it was not asserted as a cross-claim in that action, it is now barred. D No, because the State C citizen's claim is barred by claim preclusion.
A Yes, because, while the State C citizen could have asserted the claim as a cross-claim in the prior action, he may wait and assert it as an independent action.
Congress enacted a statute that purported to ban all discrimination against African-Americans in any commercial transaction taking place within the United States. Would the statute most likely be held constitutional? A Yes, under Thirteenth Amendment provisions barring badges or incidents of slavery. B Yes, because the federal government has an important interest in furthering the equal protection provisions of the Fourteenth Amendment. C No, because Congress's powers under the Commerce Clause do not extend so far as the statute would require. D No, because commercial transactions are not among the privileges or immunities of national citizenship.
A Yes, under Thirteenth Amendment provisions barring badges or incidents of slavery. The statute is constitutional as a legitimate exercise of congressional enforcement powers under the Enabling Clause of the Thirteenth Amendment. The Thirteenth Amendment prohibits slavery. The Enabling Clause of the amendment has been held to confer on Congress the authority to proscribe almost any private racially discriminatory action that can be characterized as a badge or incident of slavery. Because the statute at issue bans all discrimination against African-Americans in commercial transactions, it necessarily reaches private conduct. Such congressional action is constitutionally permissible pursuant to the Thirteenth Amendment. (B) is incorrect. Application of the Fourteenth Amendment has been limited to cases involving state action. [See United States v. Morrison (2000)] The statute here reaches private action, and so the Thirteenth Amendment is the correct source for the law, since that amendment addresses private action. (C) is incorrect because, even if Congress's power over interstate commerce would not reach every commercial transaction, the statute would be enforceable under the Thirteenth Amendment, as discussed above. (D) is incorrect because it is irrelevant. While it is true that the commercial transactions here are not among the privileges or immunities of citizenship (which include rights such as the right to petition Congress for redress and the right to interstate travel), the law can be based on the Commerce Clause or the Thirteenth Amendment, and thus is constitutional.
A woman was crossing the street at a crosswalk, but did not look both ways. The woman was hit by a truck, and immediately afterwards, she was struck by a car. As a result of these collisions with the vehicles, the woman suffered severe injuries. Although it was impossible to determine which portion of the woman's injuries was caused by the driver of the car and which was caused by the truck driver, at the trial of the woman's suit, the jury determined that the driver of the car was 20% negligent, that the truck driver was 40% negligent, and that the woman was 40% negligent. It was further determined that the woman had suffered $100,000 in damages. The woman had already received $10,000 from her group medical insurance plan. The driver of the car had a $500,000 auto liability insurance policy, and the truck driver is now insolvent. How much will the woman recover in damages from the driver of the car? A $90,000. B $60,000. C $50,000. D $20,000.
B $60,000
At trial, questions have been raised as to whether the proposed testimony of the witness is relevant and whether it falls within the present sense impression exception to the hearsay rule. How should a preliminary determination of the admissibility of the witness's testimony be made? A A judge should determine whether the proposed testimony falls within the exception before it is heard by the jury, and in making that determination she is limited by the rules of evidence. B A judge should decide whether the testimony falls within the present sense impression exception, but in making that determination she is not limited by the rules of evidence other than privilege. C The jury, after being instructed on the rules of evidence by a judge, should determine whether the testimony falls within the scope of the present sense impression exception. D The jury should determine whether the testimony falls within the scope of the exception and the judge should then instruct the jury on the appropriate uses for that evidence.
B A judge should decide whether the testimony falls within the present sense impression exception, but in making that determination she is not limited by the rules of evidence other than privilege.
After picking up a load of hazardous chemical waste, a truck driver for a waste management company set out on the road to his next stop. However, he had failed to secure the latch on the back panel of the truck. Consequently, the panel opened while the truck was on the road, and a metal canister full of chemical waste fell onto the road. A car struck the canister, causing the car to veer off the road and injure the driver. The driver filed suit against the company for his injuries. The jurisdiction in which the above events took place has adopted a rule of partial comparative negligence. At trial, the driver of the car admitted that he had momentarily taken his eyes off the road to look at his speedometer. When he had looked up again, the canister was there and he could not stop in time. The jury found that the company, through its truck driver, had acted willfully and wantonly and was 90% at fault, while the driver of the car was 10% at fault. The driver filed a motion for judgment notwithstanding the verdict, seeking recovery for 100% of his damages. If the judge grants the motion, what is the most likely reason? A A plaintiff's comparative negligence is not taken into account in cases of willful and wanton conduct by the defendant. B A state ordinance mandating motorists to stay within the posted speed limit requires as a matter of law an occasional glance at the speedometer. C The company was more than 50% at fault. D The company was engaged in an abnormally dangerous activity.
B A state ordinance mandating motorists to stay within the posted speed limit requires as a matter of law an occasional glance at the speedometer.
In an attempt to induce her niece to improve her grades, an aunt told her niece that she would give her $50 for every "A" she earned in law school. Inspired by the opportunity to earn extra spending money, the niece studied like never before and ended up earning three "A's" the following semester. Upon contacting her aunt with the news, the niece learned that her aunt was "only kidding" about paying her for every "A" earned in law school. If the aunt's offer to her niece was held to be valid, it would be an offer for what type of contract? A A bilateral contract. B A unilateral contract. C A bilateral contract or a unilateral contract, according to the offeree's intentions. D A unilateral contract that became a bilateral contract when the niece began to perform.
B A unilateral contract.
The plaintiff instituted suit against a development firm and its employee for injuries suffered by the plaintiff when a basement stair collapsed as he was being shown one of the firm's buildings that it was leasing. At trial, the employee for the firm claims that he told the plaintiff about the defective stair. The plaintiff then offers evidence that after his fall, the CEO of the development firm called him at home and offered to pay all of the plaintiff's medical expenses, saying, "I guess I owe you that much after our employee didn't warn you about the broken stair." The statement regarding the lack of warning is: A Admissible, as a statement against interest. B Admissible, as an admission by the CEO of the development firm that the employee had not warned the plaintiff about the defective stair. C Inadmissible, as hearsay not within any exception. D Inadmissible, on public policy grounds.
B Admissible, as an admission by the CEO of the development firm that the employee had not warned the plaintiff about the defective stair. The offer to pay medical expenses in itself is not admissible as an admission. Here, however, there is a statement that follows such an offer. This statement is admissible as an admission by the CEO of the development firm that the employee had not warned the plaintiff about the dangerous condition of the stairs. Because this is an admission, it is not hearsay under the Federal Rules. (A) is an exception to the hearsay rule and is thus unnecessary. Furthermore, it is an exception requiring the unavailability of the declarant, and there is no indication in the facts that the employee is now unavailable. (C) is wrong because, as mentioned, the statement is not hearsay under the Federal Rules. (D) is a misstatement of the law; only the offer to pay medical expenses is inadmissible on public policy grounds.
A passenger in a vehicle that was struck by another car sued the other car's driver, claiming that the collision severely injured his right leg. The defendant claimed that the plaintiff's leg injury resulted from an earlier, unrelated industrial accident. At trial, after having testified to his pain from the injury allegedly caused by the defendant, the plaintiff called as a witness the physician who treated him. The physician offers to testify that the plaintiff told him that his earlier leg problems had completely cleared up before the alleged injury caused by the defendant. If the defendant objects to the admission of this testimony, how should the court proceed? A Admit it, as a statement of a party. B Admit it, as a statement for purposes of diagnosis and treatment. C Exclude it, because of the physician-patient privilege. D Exclude it, because the statement related to a past physical condition.
B Admit it, as a statement for purposes of diagnosis and treatment.
A motorist was driving his car down the street when he struck a 10-year-old boy who had darted into the road to retrieve a bouncing ball. After the accident, the boy's mother refused to take the boy for treatment on religious grounds. As a result, the boy's injuries were more severe than they would otherwise have been. What argument provides the boy with his best chance to recover for all of his injuries? A The doctrine of avoidable consequences at most bars recovery for the aggravation of his injuries, but not for the original injuries themselves. B Any negligence on the mother's part is not to be imputed to her child. C Victims have no duty to take steps for their own safety after the accident. D Defendants must take their victims as they find them, including their mothers' attitudes toward medical treatment.
B Any negligence on the mother's part is not to be imputed to her child.
A consumer purchased a new television set from an electronics store. When he got home, he opened the box and found an owner's manual that contained operation instructions, warnings regarding the danger of electricity, and a warranty that stated: "The store expressly warrants that this set shall be free of manufacturing defects for 30 days. If a set is defective, the store's liability shall be limited to the cost of repair or replacement of defective parts. The store "HEREBY DISCLAIMS ANY AND ALL OTHER WARRANTIES, EXPRESS OR IMPLIED, INCLUDING THE WARRANTY OF FITNESS FOR PARTICULAR PURPOSE AND THE WARRANTY OF MERCHANTABILITY."" Five weeks later, after the set was properly installed, the consumer turned on the set, heard a crackling noise, and watched as his television exploded and was destroyed. Under which of the following theories will the consumer most likely recover? A Breach of express warranty. B Breach of the implied warranty of merchantability. C Breach of the implied warranty of fitness for a particular purpose. D Breach of the warranty of reasonable workmanship.
B Breach of the implied warranty of merchantability.
The driver of a tanker truck was transporting radioactive waste from a nuclear power plant to a permanent storage facility in a remote western region of the United States. After driving all night, the driver fell asleep at the wheel and the truck crossed over the center line, off the road, and onto a homeowner's property, coming to rest after crashing into several glass cases containing the homeowner's collection of poisonous snakes, the keeping of which was permitted by local ordinance. When the driver exited the truck, he was bitten on the leg by one of the poisonous snakes and became seriously ill. The driver brought an action against the homeowner for his injuries. The parties stipulated to the above facts, and that the driver violated a state statute by driving off of the road. Both parties moved for judgment as a matter of law on the liability issue. How should the court rule? A Grant the driver's motion and deny the homeowner's motion, because the homeowner is strictly liable for the injury caused by the snake. B Deny the driver's motion and grant the homeowner's motion, because the driver was a trespasser on the homeowner's property. C Deny the driver's motion and grant the homeowner's motion, because the driver's violation of the state statute establishes contributory negligence as a matter of law. D Deny both parties' motions, because both parties were engaged in an activity for which strict liability is imposed.
B Deny the driver's motion and grant the homeowner's motion, because the driver was a trespasser on the homeowner's property. The court should grant the homeowner's motion for judgment as a matter of law because the driver has not established a prima facie case against the homeowner. An owner of wild (dangerous) animals is strictly liable for injuries caused by those animals as long as the person injured did nothing, voluntarily or consciously, to bring about the injury. However, strict liability generally is not imposed in favor of undiscovered trespassers against landowners in the absence of negligence, such as when the landowner knows that the trespassers are on the land and fails to warn them of the animal. Here, despite the fact that the driver did not intend to enter the homeowner's land (and thus would not be liable for the intentional tort of trespass), his status on the homeowner's land is that of a trespasser rather than a licensee or invitee. The driver has presented no evidence of negligence on the homeowner's part and therefore has not established a prima facie case against the homeowner. (A) is wrong because, as discussed above, the homeowner is not strictly liable to the driver because the driver was a trespasser. (C) is incorrect because the driver will not prevail regardless of whether he was contributorily negligent, because he cannot establish a prima facie case against the homeowner in either negligence or strict liability. (D) is incorrect for several reasons: While the driver's transport of radioactive waste may have been an abnormally dangerous activity, that danger had nothing to do with the accident that occurred. Furthermore, the fact that the driver may have been engaged in an abnormally dangerous activity would not prevent him from recovering damages from another tortfeasor if he established the requisite prima facie case. Finally, the fact that the parties were engaged in activities potentially creating strict liability has nothing to do with whether issues of fact regarding liability still exist that would require denying both motions and going to trial.
A seller entered into a written contract to sell a tract of land to a buyer. The buyer was to pay $1,500 per month for five years, at which time the seller would deliver a warranty deed. The contract was silent as to the quality of title to be conveyed. After making 12 payments, the buyer discovered that a neighbor had an easement of way over the land, which was not discussed at the time the seller and buyer entered into the contract. The neighbor had not used the easement over the previous year because she had been out of the country. On the basis of the easement, the buyer wishes to cancel the contract. Which party is more likely to prevail? A The seller, because the neighbor's easement has been extinguished. B The seller, because the buyer has no basis on which to rescind the contract. C The buyer, because the obligation to convey marketable title is implied. D The buyer, because the seller has breached the covenant against encumbrances.
B The seller, because the buyer has no basis on which to rescind the contract. The seller is more likely to prevail because the buyer has no basis on which to rescind the contract. Absent a provision to the contrary, a contract for the sale of land contains an implied promise by the seller that she will deliver to the buyer a marketable title at the time of closing. This promise imposes on the seller an obligation to deliver a title that is free from reasonable doubt; i.e., free from questions that might present an unreasonable risk of litigation. Title is marketable if a reasonably prudent buyer would accept it in the exercise of ordinary prudence. An easement that reduces the value of the property (e.g., an easement of way for the benefit of a neighbor) generally renders title unmarketable. If the buyer determines, prior to closing, that the seller's title is unmarketable, he must notify the seller and allow a reasonable time to cure the defect. If the seller is unable to acquire title before closing, so that title remains unmarketable, the buyer can rescind, sue for damages caused by the breach, or obtain specific performance with an abatement of the purchase price. However, the buyer cannot rescind prior to closing on grounds that the seller's title is unmarketable. Where an installment land contract is used, the seller's obligation is to furnish marketable title when delivery is to occur, e.g., when the buyer has made his final payment. Thus, a buyer cannot withhold payments or seek other remedies on grounds that the seller's title is unmarketable prior to the date of promised delivery. Here, there is a valid easement on the property (see below), but the seller has four years in which to cure this defect. Thus, the buyer cannot yet rescind on grounds that title is unmarketable. (A) is incorrect because the neighbor's easement has not been extinguished. An easement can be extinguished where the owner of the privilege demonstrates by physical action an intention to permanently abandon the easement. Mere nonuse is not sufficient to terminate an easement, unless the nonuse is combined with other evidence of intent to abandon it. Here, the fact that the neighbor did not use the easement for a year because she was out of the country does not establish her intent to abandon the easement. (C) is incorrect because, although the law implies in every land sale contract a covenant that title will be marketable, the seller has until the time of delivery to cure the defect. (D) is incorrect because the deed has not yet been delivered. The covenant against encumbrances is a covenant contained in a general warranty deed which assures that there are neither visible encumbrances (e.g., easements) nor invisible encumbrances (e.g., mortgages) against the title or interest conveyed. This covenant is breached, if at all, at the time of conveyance. Here, the deed has not yet been delivered, and thus this covenant has not yet been breached.
A police officer saw a car containing three teenagers driving slowly down the street at 1 a.m. She waited for it to go by her and, after it was far enough ahead, started to follow it. Several blocks later, the car rolled through a stop sign. The officer immediately pulled the car over and requested the driver's license. A license check showed that the driver had five outstanding parking tickets. A statute in the jurisdiction permits an arrest to be made if a driver has four or more outstanding parking or traffic violations. The officer decided to take the driver in on the tickets. She informed the driver that he was under arrest and asked him to step out of the car. When the driver got out, the officer patted him down and found a gun in his waistband. Calling for backup, she decided to haul all three teenagers to jail. Subsequent testing showed that the gun had been used in a recent homicide during a store robbery by three young men. One of the passengers made a motion to prevent the introduction of the gun at his trial for murder and robbery. How should the judge rule? A Deny the motion, because the gun was found after the driver had been arrested. B Deny the motion, because the officer lawfully stopped the car. C Grant the motion, because the officer had no valid reason to be following the automobile. D Grant the motion, because the officer had not arrested the driver for suspicion of robbing the store or committing the homicide.
B Deny the motion, because the officer lawfully stopped the car.
An uncle's will devised his lakefront estate "to my butler for life, remainder to my niece." The 40-acre estate includes a mansion, a 20-acre orchard, a beach, and gardens. At the time of the uncle's death, the butler was 40 years old and of modest means. The niece was 18 years old and quite wealthy. The estate was encumbered by a mortgage that was not entitled to exoneration. After the first year, the butler could no longer make the mortgage payments, so the niece paid them. Ten years after the uncle's death, the town in which the estate was located became a hot resort area. A major resort chain approached the butler with a multimillion-dollar offer for the easternmost 20 acres of the estate, which included the residence and beach. The resort chain planned to raze the mansion to erect a high-rise hotel. The butler approached the niece about the offer. He proposed to give her most of the money from the sale and offered to build any house she desired on the remaining land. The niece refused to go along with the plan. The butler decided to proceed with the sale, and the niece brought a suit to enjoin the butler's proposed actions. Which of the following is the niece's best argument? A The eventual use of the property by the remainderman will be as a residence. B Destruction of the mansion constitutes waste. C Because the niece paid the mortgage payments, the butler is subrogated to her rights. D The butler has no right to transfer his life estate.
B Destruction of the mansion constitutes waste. The niece's best argument is that destruction of the residence constitutes waste. The other choices do not present arguments giving her a chance of success. A life tenant is entitled to all ordinary uses and profits of the land, but he cannot lawfully do any act that would injure the interests of the remainderman. A grantor intends that the life tenant have the general use of the land in a reasonable manner, but that the land pass to the owner of the remainder, as nearly as practicable, unimpaired in its nature, character, and improvements. Even ameliorative waste, which actually increases the value of the land, is actionable if there is no reasonable justification for the change. A life tenant can substantially alter or even demolish existing buildings if (i) the market value of the future interests is not diminished and either (ii) the remainderman does not object, or (iii) a substantial and permanent change in the neighborhood conditions has deprived the property in its current form of reasonable productivity or usefulness. Here, the market value of the property would not be diminished. The remainderman (the niece), however, is objecting, making option (ii) unavailable. Furthermore, although the neighboring properties have been sold for hotels and resorts, it does not necessarily follow that the conditions have changed to such a degree that the estate should be similarly converted. The property is large enough to be somewhat isolated from the changes in the surrounding areas; thus, despite the surrounding hotels, an owner could still enjoy the land as a private residence, orchard, and beach. Therefore, the property is still useful and option (iii) is also unavailable. In this case, the life tenant's desire to raze the mansion is not because the changes in the neighborhood have made the mansion uneconomical or impractical. The life tenant can make more money by tearing the mansion down, but its usefulness and value are apparently unaffected by the changes in the neighborhood. Thus, the niece will be able to enjoin the butler from allowing the resort chain to raze the mansion and build a hotel. (A) is wrong because the fact that the niece intended to use the property as a residence is irrelevant. Even if the niece intended to change the use of the property, she is still entitled to receive the land in the condition in which it passed to the butler. (C) is wrong because the fact that the niece made mortgage payments does not affect the butler's rights. The niece had to make the payments to protect her remainder interest. She will be entitled to reimbursement of those payments, but the butler is not subrogated to her rights. (D) is wrong because life estates generally are alienable. The transferee merely takes the same interest as the life tenant. In this case, the butler may convey his interest in the property. Of course, anyone taking the butler's interest would have only an estate for the butler's life, i.e., a life estate pur autre vie.
A storeowner properly filed a complaint for breach of contract against a food distributor in federal district court. After the food distributor timely answered the complaint, the parties proceeded through discovery, which lasted over 14 months and cost the parties over $200,000 in attorneys' fees and related costs. At the final pretrial conference, the presiding judge indicated that he did not think much of the merits of the storeowner's claims. As a result, the storeowner wants to dismiss this case and refile in a different federal court to get a more sympathetic judge. How may the storeowner try to achieve this goal? A File a motion for voluntary dismissal without prejudice, and it will likely be granted on the grounds that the judge is biased against the storeowner's case. B File a motion for voluntary dismissal without prejudice, but it will likely be denied due to the time and money already invested in the case by the parties and the court. C File a notice of dismissal with the federal district court, and it will likely be granted because the Federal Rules of Civil Procedure give a plaintiff the right to unilaterally dismiss an action once without prejudice. D File a motion for voluntary dismissal without prejudice, and it will likely be granted because courts are required to freely grant such motions to further judicial economy.
B File a motion for voluntary dismissal without prejudice, but it will likely be denied due to the time and money already invested in the case by the parties and the court.
A feed store owner agreed to purchase several tons of grain products at a specified price from a large supplier of cattle feeds. The supplier later failed to deliver the promised grains, and the owner was forced to cover by purchasing from local producers at a higher price. The owner contacted a large law firm in the city and obtained their agreement to represent him in connection with his possible claims against the supplier. Due to error, the applicable statute of limitations period passed without the filing of any action on the owner's behalf. The owner retained another lawyer and sued the large law firm for malpractice. The jurisdiction retains traditional contributory negligence. In addition to the firm's negligence, what else does the owner have to establish as part of his prima facie case? A He had a good faith claim against the supplier that was lost by the law firm's dilatoriness. B He would have recovered from the supplier if an action had been timely filed. C He did not contribute to the failure to timely file an action through his own negligence. D The losses resulting from breach of the sales agreement by the supplier severely harmed his financial situation.
B He would have recovered from the supplier if an action had been timely filed.
A plaintiff has brought an action for personal injuries against a store, in connection with an incident in which he slipped and fell after the store's linoleum floors had been mopped. A major issue at trial is the degree of moisture that remained on the floor, because it had been mopped 45 minutes before the plaintiff walked on it. The store offers the testimony of an expert, who will testify about an experiment he conducted measuring the amount of time necessary for a linoleum floor to dry completely after having been mopped. Under what condition should the court admit this testimony? A A representative of the plaintiff was present when the experiment was conducted. B It is shown that the conditions of the expert's experiment were substantially similar to the conditions of the store's floor when the plaintiff slipped. C The plaintiff was given an opportunity to conduct his own experiment with the same type of linoleum used by the expert. D It is shown that the expert is not an employee or otherwise related in interest to the store.
B It is shown that the conditions of the expert's experiment were substantially similar to the conditions of the store's floor when the plaintiff slipped.
A college student from State A hit another car driven by a resident from State B when the college student was traveling through State B. The State B driver brought an action in State B state court against the State A college student, who has limited financial resources. The college student filed a motion to dismiss, claiming the State B court lacks personal jurisdiction. What is the best argument to support the college student's motion to dismiss? A The college student did not take actions to purposely avail herself of State B simply by driving through State B. B It is unfair, inconvenient, and highly unreasonable to require a college student with limited financial resources to defend the action in State B for financial reasons. C The interstate judicial system's interest in obtaining the most efficient resolution of the controversy is violated by requiring the college student to defend the action in the forum state. D State B is not the proper venue to file this claim.
B It is unfair, inconvenient, and highly unreasonable to require a college student with limited financial resources to defend the action in State B for financial reasons.
A driver and his passenger were involved in an automobile accident when the driver ran a red light and crashed into another car. Due to a manufacturing defect in the automobile's airbag system, the passenger side airbag did not deploy. The passenger was killed on impact. The passenger's estate brought suit against the driver and the airbag's manufacturer. At trial it is established that the driver was negligent in running the red light. What effect would such proof have on the claim of the passenger's estate against the airbag manufacturer? A It would reduce recovery by the estate if the action against the manufacturer is based on negligence. B It would bar recovery by the estate if the trier of fact finds that the driver was the sole legal cause of the passenger's death. C It would bar recovery by the estate if it is shown that the driver is the sole legal heir of the passenger's estate. D It would have no effect on recovery by the estate as long as the action against the manufacturer is based on strict liability.
B It would bar recovery by the estate if the trier of fact finds that the driver was the sole legal cause of the passenger's death. The driver's negligence would bar recovery if it was the sole legal cause of the passenger's death. Regardless of the theory that the plaintiff is using in a products liability action, actual and proximate cause must be established. If the driver's negligence is the sole legal or proximate cause of the passenger's death, it would preclude the estate's suit against the airbag manufacturer because the defect was not a legal cause of the passenger's death. (A) is incorrect because the driver's contributory negligence will not be imputed to the passenger; hence, it will not reduce the estate's recovery under comparative negligence rules. (C) is incorrect. A potential beneficiary who was negligent will be subject to the jurisdiction's fault rules. Hence, his negligence will reduce his recovery under pure comparative negligence rules but will not bar it, even if he is the sole heir. (D) is incorrect. In most pure comparative negligence jurisdictions, the same comparative fault rules will apply whether the action against the manufacturer is based on negligence or strict liability. As discussed above, whether the estate can recover, and the extent of its recovery, depends on causation issues and the driver's status as a beneficiary of the estate.
In lieu of a snack vending machine, a small company put candy bars into a box in the office kitchen. Using the honor system, employees were asked to put a dollar into an envelope in the box if they took out a candy bar. The office manager would periodically take the money collected in the envelope and use it to purchase new candy. One night while cleaning the kitchen, the company's night janitor impulsively took all the money from the envelope. His actions were recorded on the company's newly installed security camera. The janitor later admitted that he took the money to fund illegal dogfights. Which of the following crimes did the janitor commit? A Embezzlement, because the janitor took money that rightfully belonged to his employer. B Larceny, because he did not intend to purchase replacement candy with the money. C Robbery, because the theft was motivated by an underlying felony. D No crime, because the money in the envelope was voluntarily abandoned by his co-workers.
B Larceny, because he did not intend to purchase replacement candy with the money.
Congress enacted legislation prescribing the specifications for various types of containers that all suppliers of nonprescription drugs were required to use in marketing their products. From where does Congress's power to pass this legislation derive? A The Necessary and Proper Clause. B The Commerce Clause. C The General Welfare Clause. D The Tenth Amendment.
B The Commerce Clause. Congress's power is derived from the Commerce Clause. The Supreme Court has construed the scope of the Commerce Clause very broadly, so that it covers the regulation of drug packaging, which has a substantial economic effect on interstate commerce. (A) is wrong because this type of regulation definitely falls under the Commerce Clause, and there is no need to imply a power (under the Necessary and Proper Clause) when an enumerated power controls. Moreover, the Necessary and Proper Clause, by itself, does not grant Congress the power to do anything. (C) is wrong because the General Welfare Clause presents a limitation on Congress's spending and taxing powers, which are not relevant here. (D) is wrong because the Tenth Amendment is not a grant of power to Congress; rather, it reserves the powers not granted to Congress to the states.
A defendant was convicted in federal court of possession of one kilogram of heroin with intent to distribute. She was sentenced to a prison term. Subsequently, the defendant was indicted by a federal grand jury for conspiracy to distribute the same kilogram of heroin. She moved to dismiss the indictment. Should her motion be granted? A No, because the Double Jeopardy Clause does not apply when the second prosecution is for violation of a separate statute. B No, because each prosecution requires proof of an element that the other does not. C Yes, because the Double Jeopardy Clause protects her against a second prosecution for the same criminal conduct. D Yes, because the Due Process Clause protects her against double punishment for the same criminal conduct.
B No, because each prosecution requires proof of an element that the other does not. The defendant's motion should be denied because a prosecution for conspiracy is distinct from a prosecution for any substantive offense involving the same conduct as the conspiracy. The Fifth Amendment provides that no person shall be twice put in jeopardy for the same offense. The general rule is that two crimes do not constitute the same offense if each crime requires proof of an additional element that the other crime does not require, even though some of the same facts may be necessary to prove both crimes. [Blockburger v. United States (1932)] Furthermore, a prosecution for conspiracy is not barred merely because some of the alleged overt acts of that conspiracy have already been prosecuted. [United States v. Felix (1992)] Here, both the conspiracy charge and the possession charge require proof of an element that the other charge does not; hence, there is no double jeopardy problem with the indictment. (A) is incorrect because it is too broad a statement. The fact that separate statutes are involved does not establish that these are not the "same offense" for purposes of double jeopardy. (C) is incorrect because the "same conduct" test is not currently used by the Supreme Court to evaluate a double jeopardy claim. (D) is incorrect because the question involves the defendant's motion to quash an indictment and not her ultimate punishment.
A landowner validly conveyed a parcel of land to a veterinarian "for so long as the property is used as a veterinary practice, but if the property is used for any other purpose, it is to go to the American Cancer Society." Two years later, the landowner died, validly devising all of his property to his friend. The landowner's only heir is his daughter. Although this jurisdiction is a common law jurisdiction with respect to all real property considerations, the state's probate laws provide that future interests or estates in real property may be passed by will or descent in the same manner as present or possessory interests. Last month, the veterinarian approached the daughter and asked her to join with him to sell the parcel of land, which he had been using as an animal shelter, in fee simple absolute to a developer. The veterinarian and the daughter entered into a contract of sale with the developer. However, after consultation with an attorney, the veterinarian decided against the sale. The developer sued the daughter and the veterinarian for specific performance. Will the requested relief likely be granted? A No, because the American Cancer Society did not join in the contract of sale. B No, because the friend did not join in the contract of sale. C Yes, because the veterinarian had the power to sell his interest. D Yes, because together, the daughter's and the veterinarian's interests would merge and they would have a fee simple estate.
B No, because the friend did not join in the contract of sale. The requested relief will be denied because the friend did not join in the contract of sale. The conveyance purported to create a fee simple determinable in the veterinarian subject to an executory interest in the American Cancer Society. A fee simple determinable subject to an executory interest is an estate that, on the happening of a stated event, is automatically divested in favor of a third person, who holds the executory interest. However, the executory interest in the American Cancer Society is void under the Rule Against Perpetuities because it might vest beyond lives in being plus 21 years. The charity-to-charity exception to the Rule does not apply because the veterinarian is not a charitable organization. Because any interest that violates the Rule is void and stricken from the instrument, what is left is a fee simple determinable in the veterinarian and a possibility of reverter in the landowner. On the landowner's death, the possibility of reverter passed to the friend. (A) is incorrect because the American Cancer Society does not have any interest in the land. (C) is incorrect because, although the veterinarian had the power to sell his interest, he did not own a fee simple absolute. (D) is incorrect because the daughter does not have any interest in the land. Even if she did, the merger doctrine—which provides that whenever the same person acquires all of the existing interests in land, present and future, a merger occurs and that person holds a fee simple absolute—would not apply because the friend also has an interest in the land.
A homeowner contracted with a local heating company to install two baseboard heaters in an addition to his home for a total cost of $3,500. This figure included the heaters and labor costs for installation. Upon completion of the installation, the heating company sent an invoice to the homeowner for the $3,500. The homeowner did not immediately pay the bill because the heaters were too noisy. The heating company sent a repair worker to the home to service the heaters, but after several attempts to fix the problem, the heaters were still too loud. The homeowner contacted a qualified repairman to find out how to fix the problem and was told it would cost an additional $300 for new blowers and $150 in labor costs to replace the faulty blowers. The homeowner mailed the heating company a copy of the repair estimate and a check for $3,050—the contract price less the cost of new blowers and labor to install them—and wrote prominently on the check "Payment in full for installation of two baseboard heaters." The heating company cashed the check upon receipt. The heating company then sued the homeowner for $450, the difference between the agreed contract price and the amount paid. Is the heating company likely to prevail in its suit seeking the $450 from the homeowner? A No, because the heating company failed to fix the problem after several attempts. B No, because the heating company cashed the check. C Yes, because the check did not represent payment in full (i.e., the contract price) for the work done. D Yes, because the heating company should have been given more time to cure the defect.
B No, because the heating company cashed the check.
A State A citizen and a State B citizen were in a car accident in State A. The State A citizen filed a negligence action in a State A state court seeking $200,000 in damages. The State B citizen filed an answer and four months later filed a notice of removal, removing the action to federal court. Two months after the State B citizen filed and served the notice of removal, the State A citizen filed in the federal court a motion to remand the case back to state court. Should the federal court remand the action to state court? A No, because the State B citizen was not a citizen of the forum state. B No, because the motion to remand the action to state court was untimely. C Yes, because the notice of removal was untimely. D Yes, because federal courts do not have subject matter jurisdiction over state law negligence actions.
B No, because the motion to remand the action to state court was untimely.
During the trial of a personal injury case, the plaintiff calls a witness to testify that he saw the defendant spill a slippery substance in the roadway. Following the testimony of the witness, the defendant calls the witness's neighbor, who testifies that the witness has a poor reputation for truthfulness in the community. The plaintiff's attorney then cross-examines the neighbor, asking her, in good faith, if she committed the crime of false pretenses last year. Last year, the neighbor had in fact been charged with and convicted of the crime of false pretenses. The defendant's attorney objects to this question. Should the objection be sustained? A No, because the neighbor was convicted of the crime of false pretenses. B No, because the plaintiff's attorney asked the question in good faith. C Yes, because an impeaching witness cannot be impeached on collateral matters. D Yes, because such an inquiry is not proper on cross-examination.
B No, because the plaintiff's attorney asked the question in good faith. The question by the plaintiff's attorney should be allowed because he was acting in good faith. A witness may be impeached by means of being interrogated upon cross-examination, in the discretion of the court, with respect to any act of misconduct that is probative of truthfulness (i.e., an act of deceit or lying). The cross-examiner must act in good faith with some reasonable basis for believing that the witness may have committed the bad act inquired about, but it is not required that the witness have been convicted of a crime. Here, the plaintiff's attorney is attempting to cast an adverse reflection on the truthfulness of the neighbor. The commission of the crime of false pretenses involves the making of a false representation and is therefore an act of misconduct that is probative of the actor's truthfulness. Thus, because the plaintiff's attorney inquired as to this matter in good faith, his question is a permissible method of impeachment, and the objection of the defendant's attorney should be overruled. (A) is incorrect because it implies that the objection could be sustained if the neighbor was not convicted of the crime. As noted above, such an inquiry can be conducted regardless of whether the witness was convicted. Therefore, the objection to the plaintiff's attorney's good faith inquiry would be overruled even if the neighbor was not convicted of false pretenses. (C) is incorrect. Although impeaching witnesses who testify to a witness's reputation for truth and veracity are often impeached by asking the "Have you heard" and "Do you know" questions, that is not the only method of impeachment available. Any witness who takes the stand puts her character for honesty and veracity in issue and may be impeached by evidence that might show her to be unworthy of belief. Instances of misconduct may properly be inquired into only if they are probative of truthfulness. By taking the stand, the neighbor has put her character for honesty in issue. The crime of false pretenses is probative of truthfulness and is a proper subject for impeachment. (D) is incorrect because a specific act of misconduct offered to attack the witness's character for truthfulness can be elicited only on cross-examination of the witness. Extrinsic evidence is not permitted. Thus, (D) states the opposite of the correct rule.
A pedestrian was struck and seriously injured by a car driven by an intoxicated driver. The driver had been served several alcoholic drinks by a bartender at a local bar. The pedestrian sued the bartender in a jurisdiction that does not have a dramshop act. Is the bartender vicariously liable for the pedestrian's injuries? A No, because the driver acted recklessly by driving while intoxicated. B No, because there is no dramshop act in the jurisdiction to impose liability. C Yes, because there is no dramshop act in the jurisdiction to limit liability. D Yes, because the intoxicated driver caused the pedestrian to suffer personal injuries.
B No, because there is no dramshop act in the jurisdiction to impose liability.
Pursuant to a contract, a landscaper performed $30,000 of landscape work for a homeowner. By coincidence, the homeowner and the landscaper were involved in an automobile accident that was unrelated to the landscape work. The homeowner was injured in the accident and sued the landscaper in federal district court for negligence, seeking $100,000 in damages. The homeowner and the landscaper are citizens of different states. May the landscaper assert and maintain a counterclaim against the homeowner for breach of contract, seeking the $30,000 due under the landscape contract? A No, because the Federal Rules of Civil Procedure permit a counterclaim only if the counterclaim arises from the same transaction or occurrence as the plaintiff's initial claim. B No, because, while the court has diversity of citizenship jurisdiction over the homeowner's negligence claim and the Federal Rules of Civil Procedure permit the counterclaim, the federal court does not have subject matter jurisdiction over the landscaper's contract claim. C Yes, because the Federal Rules of Civil Procedure permit the counterclaim, and the court has diversity of citizenship jurisdiction over both claims. D Yes, because the court has diversity of citizenship jurisdiction over the homeowner's negligence claim, and it has supplemental jurisdiction over the landscaper's contract claim because it is a compulsory counterclaim.
B No, because, while the court has diversity of citizenship jurisdiction over the homeowner's negligence claim and the Federal Rules of Civil Procedure permit the counterclaim, the federal court does not have subject matter jurisdiction over the landscaper's contract claim.
Late one evening, a cook at a diner coming off his shift was grabbed in the parking lot by a large man wearing a ski mask. The man threatened to kill the cook and demanded his wallet. The man then pulled a knife from his pocket and lunged at the cook. The cook, having taken several self-defense courses, was able to fend off the man's attack. After being struck by the cook several times, the man dropped the knife and fell to the ground. The cook, angry at the assault, took the knife and stabbed the man, killing him instantly. Should the cook be convicted of murder? A No, because he was acting in self-defense. B No, but he may be convicted of manslaughter. C Yes, because the killing was committed during the course of a felony. D Yes, because the killing was not committed while acting in self-defense.
B No, but he may be convicted of manslaughter.
A brother and a sister each hold an undivided one-half interest in a tract of land. By the terms of their agreement, each has the right to possess all portions of the property and neither has the right to exclusive possession of any part. The brother wrongfully ousts the sister from the property. What can the sister recover in an action against the brother? A The fair rental value of the property for the time excluded. B One-half of the fair rental value of the property for the time excluded. C One-fourth of the fair rental value of the property for the time excluded. D Nothing, because each co-tenant has the right to possess all portions of the property and neither has the right to exclusive possession of any part.
B One-half of the fair rental value of the property for the time excluded.
A homeowner contracted for construction of a custom-built, elevated deck in his backyard. The deck's designer supervised the construction, which was carried out by several employees of a local building company. The homeowner was pleased with the appearance of the deck, but the first time he stepped on it, a support on one side of the deck gave way, causing the homeowner to fall and be injured. The homeowner brought an action joining the building company and the deck's designer as defendants, alleging negligence. In his complaint, he alleged that he does not know which of the defendants is responsible for the damages. Which of the following doctrines would be most helpful against the designer? A Respondeat superior. B Res ipsa loquitur. C Contribution. D Indemnity.
B Res ipsa loquitur. Res ipsa loquitur will be most helpful against the designer. Res ipsa loquitur means the thing speaks for itself. It is appropriate in situations where an injury does not usually occur unless someone was negligent and the plaintiff does not know which of the defendants caused the injury. While res ipsa loquitur is not generally available where more than one person may have been in control of the instrumentality causing the injury, it is available in a case where a particular defendant had the power of control over the site of the injury. Even if the homeowner does not know why the deck collapsed, the deck's designer would be responsible because he designed the deck and was supervising the construction. Hence, res ipsa loquitur likely could be used. (A) is not correct because the doctrine of respondeat superior imposes vicarious liability on an employer for the tortious conduct of its employee. That doctrine would be helpful against the building company for any negligence by its employees but not against the designer, because the workers were not the designer's employees. (C) and (D) are incorrect because the doctrines of contribution and indemnity pertain to how the responsibility of the loss is apportioned or shifted among the defendants after the plaintiff has recovered his judgment. They are not relevant to the homeowner's right to recover.
A state legislature enacted a statute providing for loaning certain textbooks on secular subjects to students in all public and private schools. In accordance with the statute, the state board of education distributed textbooks to a private school that offered religious instruction and admitted only Caucasian students. Which of the following is the strongest argument against the constitutionality of free distribution of textbooks to the students at the private school? A A state may not constitutionally aid private schools through distribution of textbooks. B Segregation is furthered by the distribution of textbooks to these students. C The distribution of textbooks advances religion because it is impossible to separate their secular and religious uses. D The distribution of textbooks fosters excessive government entanglement with religion.
B Segregation is furthered by the distribution of textbooks to these students.
A comprehensive federal health-care reform statute created a Federal Health Policy Board, which was directed to monitor the fees charged for various medical procedures covered by insurance. The board also had the power to subpoena records to determine whether fee increases were a true reflection of cost increases. Nothing in the statute provided for caps on fee increases. Because of the continuing escalation of health-care costs while the statute was being debated, several states had passed health-care legislation on their own. One state passed legislation that prohibited most fee increases of 10% or more per year for specified health-care services covered by insurance, and created a health-care review board to regulate these costs and impose monetary penalties on health-care providers or insurers that tried to circumvent the cap. Which of the following would be the best basis for finding the state provision unconstitutional? A The federal legislation was passed after the state legislation and therefore supersedes it. B The Federal Health Policy Board was constituted with many of the same powers as the state board but was not given the power to impose sanctions. C The state provision impairs existing contracts between health-care providers and insurers in violation of the Contract Clause. D Health-care fee caps create an undue burden on interstate commerce even in the absence of federal regulation.
B The Federal Health Policy Board was constituted with many of the same powers as the state board but was not given the power to impose sanctions.
After the dictator of a Caribbean island country was deposed, the President of the United States extended official recognition to the country's new government. As ambassador to the newly recognized government, the President nominated an aging industrialist who was a close personal friend of the deposed dictator. Unable to muster enough votes to block approval of the new ambassador, the President's political opponents in the Senate caused a resolution to be passed requiring that all consular staff below the rank of ambassador be selected from a list of "approved" candidates who have been certified as acceptable by the new country's government. The President refused to consider any of the Senate's list of approved candidates. Which of the following is the President's strongest constitutional basis for refusing to obey the Senate resolution? A The Senate could have effectuated its policies by a less intrusive method, such as refusing to appropriate funds to staff the new embassy if the President's selections were inappropriate. B The President has the authority to nominate and appoint the diplomatic representatives of the United States. C The President has exclusive authority, as commander in chief, to protect American interests abroad. D The President's control over the foreign policy of the United States may not be limited by other branches of government.
B The President has the authority to nominate and appoint the diplomatic representatives of the United States.
Congress enacted legislation intended to protect children from unsafe car seats. The act established a commission to supervise the manufacturing and sale of car seats, and empowered the commission to promulgate car seat safety regulations. The commission members were also required to investigate safe and sound methods of installing child car seats. The commission's chairperson was designated as an Undersecretary of Health and Safety; the President appointed two commissioners from child safety groups; and the three major car seat manufacturers chose one commissioner each, who were then appointed by Congress to the commission. For its violation of the commission's rules with regard to car seat manufacturing, a car seat manufacturer was fined $5,000, to be paid immediately without a trial on the merits. The manufacturer files suit in the federal court to enjoin the commission's enforcement of this rule. Which of these is the manufacturer's best argument in support of its contention that the rule was illegal? A Regulations concerning criminal conduct cannot be made by agency rules, but must be made by federal statute. B The appointment of the commissioners was illegal; therefore, the rules promulgated by the commission are invalid. C Because the fine was potentially $5,000 for violation of the rule, the manufacturer had a right to a trial by jury. D The presumptive fine violated the manufacturer's rights of equal protection as guaranteed by the Fourteenth Amendment.
B The appointment of the commissioners was illegal; therefore, the rules promulgated by the commission are invalid. The appointment of the commissioners was illegal. The Appointment Clause of the Constitution permits Congress to vest appointments of inferior officers only in the President, the courts, or the heads of departments. Enforcement is an executive act; therefore, Congress cannot appoint its own members to the commission to exercise enforcement powers. A duly appointed commission does have the power to make rules and regulations governing the subject matter for which it is appointed. Those rules are not "criminal" statutes in this case. Thus, (A) is wrong. (C) is wrong because Congress may establish new public rights and actions that may be adjudicated by agencies, without juries. (D) is wrong because there is no actionable discrimination.
As a result of a personal injury lawsuit, a victim obtained a judgment against a tortfeasor for $100,000. The tortfeasor, who had few assets, did not pay the judgment. On April 1 of the following year, the tortfeasor inherited a parcel of land from her uncle. On May 1, the tortfeasor entered into a contract with a buyer to sell the land for $120,000. The contract was not recorded. The buyer immediately applied to a bank for a loan of $100,000. The bank approved the buyer's loan, and on May 15, a closing was held. The tortfeasor deeded the land to the buyer, and the buyer executed a mortgage for $100,000 to the bank. Due to an error by the title company, the deed from the tortfeasor to the buyer was not recorded, although the mortgage to the bank was recorded. Neither the buyer nor the bank had any knowledge of the victim's judgment. On May 20, the victim recorded his judgment in the county recorder's office where the land was located. At that time, he had no knowledge of the buyer's or the bank's rights. When he learned about them, he immediately brought a proceeding to foreclose his judgment lien, naming the tortfeasor, the buyer, and the bank as parties. The jurisdiction has a typical grantor/grantee recording index, and has enacted the following statute: "Any judgment properly filed in the county recorder's office shall, for 10 years from filing, be a lien on the real property then owned or subsequently acquired by any person against whom the judgment is rendered. No conveyance or mortgage of real property shall be good against subsequent bona fide purchasers for value and without notice unless the same be recorded according to law." As between the victim and the bank, which party's interest in the land will be given priority? A The bank, because the bank recorded its mortgage before the victim recorded his judgment lien. B The bank, because the victim is not protected by the recording statute. C The victim, because the victim's judgment was filed in the recorder's office before the buyer's deed was recorded. D The victim, because the judgment lien extends to after-acquired property.
B The bank, because the victim is not protected by the recording statute.
A bookie testified before a grand jury regarding allegedly illegal gambling activities. As a result, the bookie was indicted and a warrant was issued for the bookie's arrest, along with a search warrant for the bookie's home. The police went to the bookie's home, informed him of the charges against him, and placed him in handcuffs. The officers then conducted a search of the bookie's home and found a desk calendar, which had possibly incriminating information written on it relating to appointments. They seized the desk calendar and one of the officers asked the bookie what he had to say about their find. The bookie made an incriminating statement in response. Before trial, the prosecutor obtained an exemplar of the bookie's handwriting to compare it with the handwriting on the calendar. If introduced at trial, which of the following would most clearly violate the bookie's Fifth Amendment self-incrimination rights? A The grand jury testimony. B The bookie's response to the police officer. C The bookie's handwriting exemplar. D The bookie's desk calendar.
B The bookie's response to the police officer.
A pedestrian walking along an unpaved road on his way to work saw a school bus coming in the opposite direction suddenly begin to careen toward him. The bus driver had momentarily lost control of the bus while attempting to light a cigarette. To avoid being hit by the bus, the pedestrian jumped off the road into a landowner's yard. Unfortunately, he landed in a bed of prize-winning zinnias and damaged them extensively. In a suit by the landowner against the bus driver for the damages to her zinnias, what is the likely result? A The bus driver is liable for trespass because his driving caused the pedestrian to enter the landowner's yard and damage her zinnias. B The bus driver is liable on the theory of negligence. C The bus driver is not liable because the landowner's zinnias were not within the scope of any duty he owed in operating a bus on a public road. D If the bus driver is held liable on any theory, he is entitled to indemnity from the pedestrian, who did the damage.
B The bus driver is liable on the theory of negligence.
A seller, who owned a home with a very leaky roof, was told that she would need to replace the roof before she could sell the home. Instead, the seller painted the shingles that could be seen from the street to make them appear new, and covered the watermarks on the interior. A buyer entered into a written contract with the seller that included a clause making the buyer's obligation to tender the purchase price at the closing date "subject to approval by an inspector of the buyer's choosing." Prior to closing, the buyer inspected the property with his friend, a local tradesman with a good reputation. Neither the buyer nor the friend climbed onto the roof, but the friend mentioned that the roof looked fairly new. The friend also looked for signs of water damage to the ceilings and walls on the interior, but found none. At closing, the seller conveyed the property to the buyer by warranty deed. Three months after the buyer moved into the home, a major rainstorm occurred. The roof leaked like a sieve and much of the buyer's personal property was damaged. The buyer replaced the roof at a cost of $8,000. The buyer's homeowner's insurance covered the cost of the water damage to his floors and personal property, but would not reimburse his expenses incurred in installing the new roof, which the insurance carrier deemed "normal maintenance and repair." If the buyer sues the seller for the $8,000 cost of installing a new roof, who is likely to prevail? A The buyer, because the seller breached an implied warranty that the house was constructed in a reasonably workmanlike manner. B The buyer, because the seller concealed the defects in the roof from the buyer. C The seller, because the buyer had an ample opportunity to inspect the property before tendering the purchase price, and the seller had no duty to disclose defects to him. D The seller, because the property conveyed to the buyer was not a new house constructed by the seller.
B The buyer, because the seller concealed the defects in the roof from the buyer.
A car owner was having problems with her car's brakes. The brake shop to which she brought her car told her that the entire hydraulic brake system needed to be replaced at a cost of $1,800. The car owner agreed to have the work performed, and the shop replaced the brake system. When the car owner picked up the car, she insisted on testing the brakes before paying for the work. During a test drive, the brakes squeaked when compressed but otherwise worked perfectly. The mechanic told the car owner that the squeak would disappear on its own in a few days. The car owner stated that she would pay the shop when the squeak disappeared and left with her car. A month later, the brakes still squeaked and the car owner refused to pay for the work. The shop files suit to collect the $1,800 contract price. What is the likely outcome of the suit? A The car owner must pay the shop the $1,800, because she got the substantial benefit of her bargain. B The car owner must pay the shop the $1,800, but is entitled to set off the amount necessary to fix the squeak. C The car owner does not have to pay the shop anything, because the squeak amounted to a breach of contract that relieved the car owner of her duty to perform. D The shop must "cure" by providing the car owner with a new set of brakes that do not squeak, after which the car owner must pay the shop the $1,800.
B The car owner must pay the shop the $1,800, but is entitled to set off the amount necessary to fix the squeak.
The plaintiff was exiting from a parking garage owned and operated by the city when he discovered that the exit ramp was blocked by construction barricades and a pile of broken-up concrete. No workers or detour signs were around and the plaintiff was in a hurry, so he backed up and drove down an entrance ramp that was clearly marked as such. As he came around a corner, his car was broadsided by a pickup truck. The plaintiff was seriously injured in the collision. A statute in the jurisdiction requires drivers to obey all traffic directional markings in both public and private parking lots and garages. The jurisdiction retains governmental immunity for municipalities. If the plaintiff brings a lawsuit against the city to recover for his injuries, which of the following facts will be LEAST helpful in the city's defense? A The plaintiff was aware that another exit on the other side of the garage was open. B The construction workers responsible for blocking off the exit ramp were employees of an independent contractor rather than the city. C The city does not collect fees or make a profit in the operation of the garage. D The pickup truck driver could have avoided the plaintiff but recognized him as an old enemy and deliberately ran into him.
B The construction workers responsible for blocking off the exit ramp were employees of an independent contractor rather than the city. The fact least helpful to the city's defense of the plaintiff's lawsuit is the identity of the workers who blocked the exit ramp. Under vicarious liability rules, a principal will be liable for the tortious acts of an independent contractor if the duty is nondelegable on public policy grounds; included is the duty of a possessor of land to keep its premises safe for its invitees. If the workers were negligent in leaving the ramp blocked without providing another means of exiting, the fact that they were not city employees would not absolve the city of liability; hence, their identity would be of no help to the city's defense. (A) is incorrect because if the plaintiff was aware of an alternate route, he may have been contributorily negligent in exiting down the entrance ramp. A plaintiff's contributory negligence may be established by violation of an applicable statute. However, as with a statutory duty imposed on a defendant, the plaintiff's violation of the statute may be excused if compliance was beyond the plaintiff's control. If no other means of exiting the garage were known to the plaintiff, he may be excused for violating the traffic statute; however, if he knew of an alternative exit, the city will probably be able to establish contributory negligence on his part by his violation of the statute, reducing his potential recovery. (C) is incorrect because whether the city collects fees and makes a profit in operation of the garage will be considered by the court in determining whether the jurisdiction's governmental immunity applies. Where municipal immunity still exists, courts have limited its scope by differentiating between "governmental" and "proprietary" functions of the municipality. If the municipality is performing a function that might as well have been provided by a private corporation, the function may be construed as a proprietary one and no immunity will attach. The inference that a function is proprietary will be strengthened where the city collects revenues by virtue of providing the service. Hence, the fact that the city is not collecting revenues or making a profit in operating the garage will make it less likely that the function will be deemed to be proprietary and more likely that it will be deemed to be governmental and thus immune; in other words, it will be more helpful rather than less helpful in the city's defense. (D) is incorrect because the pickup truck driver's conduct under these circumstances would be deemed a superseding force that breaks the causal connection between any negligence on the part of the city and the plaintiff's injury. Assuming that the city workers were negligent, the fact that an independent intervening force caused the injury generally would not cut off the city's liability, because its negligence created a foreseeable risk of that harm occurring. However, where this foreseeable harm is caused by an unforeseeable crime or intentional tort of a third party, most courts would not hold the city liable, treating the crime or tort as a superseding force. Here, while blocking the exit ramp created a foreseeable risk that someone might collide with the plaintiff, it was not foreseeable that his enemy would take that opportunity to commit an intentional tort against him. Because the pickup truck driver's conduct was unforeseeable under the circumstances in choice (D), the city would be relieved of liability for any negligence in blocking the ramp.
Upon graduation from high school, a young man wanted to enroll in a nine-month program at a community college to study to be an electrician, but he could not afford tuition and the costs of being unemployed for that time period. His uncle told him that if he enrolled and participated in the program, he would pay his tuition and living expenses for the time involved, and that he would also pay him a $1,000 bonus for each "A" he earned as a final grade in a class. The young man told his uncle that he would enroll in the program. The next day, the young man's grandfather called and told him that he had learned of the uncle's offer and that if the uncle failed to pay the young man as promised, he (the grandfather) would. The young man attended the program and earned "A's" as final grades in three classes. Shortly thereafter, the uncle died, and the executor of the uncle's estate refused to pay the young man the bonus for each of the three "A's." When the young man told his grandfather that the uncle's estate refused to pay, his grandfather sympathized but said he no longer thought it was a good idea to pay for grades. He too refused to pay. If the young man brings suit against his grandfather for breach of contract, which of the following represents his grandfather's best defense? A The contract was illusory. B The contract was oral. C There was no consideration flowing to the grandfather. D The fact that the young man received nine months' worth of free education and living expenses was sufficient compensation for his efforts in earning the three "A's."
B The contract was oral. The young man will not succeed in trying to enforce his grandfather's promise because the promise was not in writing, as is required under the Statute of Frauds. Generally, contracts do not have to be in writing to be enforceable; however, under the Statute of Frauds, certain contracts will not be enforceable unless they are evidenced by a writing signed by the party to be charged. One such contract is to pay the debt of another, such as the grandfather's promise here to pay the uncle's debt if he does not pay. (A) is wrong because the contract clearly is not illusory. Each party is bound by his promise to the other party, so the requisite mutuality exists. The young man agreed to (and did) attend the nine-month program, and he worked hard and earned three "A's," by which he incurred a legal detriment because he was doing something he was under no legal obligation to do. The grandfather agreed to pay the young man's tuition and living expenses plus a $1,000 bonus for each "A" he earned as a final grade in a class if the uncle failed to pay the young man as promised. Detriment to the promisee in performing an act or making a promise is valid consideration. Thus, the contract was not illusory. (C) is wrong because, as discussed above, consideration flowed to the grandfather when the young man participated in the program and earned the three "A's," neither of which he was legally obligated to do. (D) is wrong because nine months' worth of free education and living expenses was not the full consideration for which the parties bargained.
A gang member threatened to kill the defendant unless he robbed a convenience store and gave the proceeds to the gang member. The gang member also demanded at gunpoint that the defendant kill the clerk to prevent identification. In abject fear of his life, the defendant did everything that the gang member requested. If the defendant is arrested and charged with murder and robbery in a common law jurisdiction, what result? A The defendant should be convicted of murder and robbery. B The defendant should be acquitted of the robbery and convicted of murder. C The defendant should be convicted of robbery, and the killing will be reduced to voluntary manslaughter. D The defendant should be acquitted of the robbery, and the killing should be reduced to voluntary manslaughter.
B The defendant should be acquitted of the robbery and convicted of murder.
A driver was driving his car negligently along a mountain road. He lost control of his car and careened over the side of a cliff. A jogger saw the driver's car go off the cliff and stopped to see if he could help. The jogger started to climb down the cliff to render aid to the driver. In doing so, the jogger slipped and broke his leg. The jogger sued the driver to recover damages for his broken leg. Regarding any defenses the driver might raise, which of the following statements is correct? A A rescuer acts at his own peril. B The excitement of the accident and the speedy response of the rescuer would be considered in a case such as this. C Assumption of the risk cannot be invoked against rescuers. D The driver would not have a valid defense.
B The excitement of the accident and the speedy response of the rescuer would be considered in a case such as this.
A fashion student at a prestigious fashion design school bought a new sewing machine for $1,000 so that she would be more than adequately equipped for her design assignments. One day, her roommate loaned the sewing machine to their neighbor, as she had done on several prior occasions. Unfortunately, the neighbor caused extensive damage to the machine. The cost to repair the sewing machine was $400. If the fashion student sues her roommate for the damage the neighbor caused to the sewing machine, what will be the result? A The fashion student will recover $1,000. B The fashion student will recover the fair market value of the sewing machine. C The fashion student will recover $400. D The fashion student will recover nothing, because her roommate did not damage the machine and the neighbor's conduct was not intentional.
B The fashion student will recover the fair market value of the sewing machine.
A park board in a large suburb announced that it was accepting bids for renovation work on its recreation center. A builder advertised for sub-bids for the electrical work, and a local electrician submitted to the builder by electronic bidding service a sub-bid of $130,000. However, due to the bidding service's negligence, the sub-bid that the builder received from the electrician read $30,000 instead of $130,000. Because this was the lowest sub-bid that the builder received for the electrical work, and $60,000 less than the next lowest sub-bid, the builder awarded the subcontract to the electrician. Based in part on the electrician's sub-bid, the builder came up with a bid for the job that beat out all of the competition and won the job. What is the electrician's best argument to successfully refuse to perform the resulting contract? A The contract would be unconscionable. B The great difference between the $30,000 figure and the next lowest bid should have alerted the builder to the existence of a mistake in the sub-bid. C The electrician was not responsible for the negligence of the bidding service. D The builder's own negligence in not checking out all sub-bids precludes enforcement of the contract.
B The great difference between the $30,000 figure and the next lowest bid should have alerted the builder to the existence of a mistake in the sub-bid.
In litigation over whether an uncle conveyed a parcel of land to his nephew, the nephew wishes to offer into evidence a tape recording of his uncle made by a well-known oral historian at the nearby state university. The voice on the tape is discussing various conveyances of the parcel of land and other property owned by the uncle. The nephew wishes to have the historian testify that the voice on the tape is the uncle's. If the court allows the historian to testify, it will be because: A The historian is testifying regarding an admission by a party-opponent. B The historian has heard the uncle speak before. C The historian became familiar with the uncle's voice before the dispute over the property arose. D The historian's experience as an oral historian qualifies him as an expert in voice recognition.
B The historian has heard the uncle speak before.
To better reflect the age range of its citizens, a city council passed an ordinance providing that no city employee could be hired or promoted unless that employee had reached the age of 55. A 25-year-old city mechanic in the lower pay classification had recently finished first on a promotional exam for the senior mechanic position, which would entitle him to a substantial increase in pay. His supervisor told him that, under instructions from the head of the city administrative office, a 56-year-old city mechanic who scored lower on the exam would be promoted to the position instead. If the mechanic brings an appropriate action in federal court to challenge the ordinance, which party would bear the burden of proof? A The mechanic, to prove that there is no compelling state interest furthered by the challenged ordinance. B The mechanic, to prove that the challenged ordinance is clearly arbitrary and irrational. C The city, to prove that its ordinance is necessary to further a compelling state interest. D The city, to prove that there was a rational basis for enacting the challenged ordinance.
B The mechanic, to prove that the challenged ordinance is clearly arbitrary and irrational.
A landowner owned a large tract of undeveloped land in fee simple. Although no excavation had been done on the land, it was believed to contain gold. The landowner therefore began to mine the land, financing his operation with a $100,000 mortgage to a bank. Subsequently, the landowner sold all of the interest in gold on the land to a miner. Shortly thereafter, the landowner conveyed his ownership in the land to a mining company. Realizing that none of their interests had been recorded, the bank recorded its mortgage first, the miner recorded her deed second, and the mining company recorded its deed third. None of the parties dealing with the landowner had any knowledge of the others at the time of their transactions. The jurisdiction in which the land is located has the following statute: "No conveyance or mortgage of an interest in land is valid against any subsequent purchaser for value without notice thereof, unless it is recorded." If the mining company brings an action to quiet title in the land, what is the most likely result? A Because the rights of the bank, the miner, and the mining company are different in nature, the court would most likely validate all of the interests, with the mining company having the ownership subject to the payment of the mortgage to the bank. B The mining company would be successful in quieting title to the land. C The bank's mortgage would be declared valid because it is first in time to all of the grants by the landowner concerning the land. D The bank's mortgage would take priority over the miner and the mining company because the bank had no notice at the time it recorded its mortgage.
B The mining company would be successful in quieting title to the land. The mining company would likely be successful. Under a notice statute, which the jurisdiction in this question has, a subsequent bona fide purchaser prevails over a prior grantee who fails to record. The important fact under a notice statute is that the subsequent purchaser had no actual or constructive notice at the time of the conveyance, not at the time of recording. When the property was conveyed to the mining company, it had neither actual nor constructive notice of the conveyances to the bank or the miner, whose interests were not recorded at that time. Therefore, the mining company was a bona fide purchaser and would be entitled to protection under the statute. (A) is incorrect because the recording statute applies to all conveyances and mortgages of an interest in land, including a conveyance of the mineral interests. Thus, both the bank's and the miner's interests are not enforceable against the mining company. (C) is incorrect because that would be the result in the absence of a recording statute—priority is given to the grantee who was first in time. The recording statute changes this result. (D) is wrong because, as discussed above, the mining company did not have notice of the bank's interest at the time of its conveyance from the landowner, so it takes free of that interest under the statute.
A defendant calls a secretary present at a meeting between the defendant and the plaintiff in which attempts were made to negotiate a settlement. The secretary offers to testify that the plaintiff said at the meeting, "Well, maybe it wasn't fraud—I'll settle for refund of the purchase price plus $50,000." The plaintiff objects. Is the offered testimony admissible? A Yes, as a statement by a party-opponent. B Yes, because statements made in the course of settlement negotiations are admissible where relevant, even though offers to compromise are excluded. C No, because of public policy. D No, because the statement was made in the office of the defendant, an attorney, and hence privileged.
C No, because of public policy.
The owner of an art gallery entered into a written contract with an avid art collector whereby the art collector agreed to buy and the gallery owner agreed to sell for $7,500 any painting in the gallery by artist Alpha. The contract was to be executed on July 6 according to its written terms. The art collector went to the gallery on July 6 with a certified check in the amount of $7,500. The art collector pointed out a painting by a different artist hanging on the wall, and told the gallery owner that that was the painting he wanted, and that he would also take its old-fashioned $250 gilt frame to go with it. The gallery owner responded that the painting was by the artist Beta, but that the art collector could have it with the frame if he was willing to pay $250 extra for it. This enraged the art collector, and he filed suit against the gallery owner, asserting in his pleading that he remains able and willing to tender $7,500 to the gallery owner. He also asserts that prior to signing the contract, the parties agreed orally that the art collector could have a painting by Beta for the same price in lieu of one by Alpha, and that the gallery owner would throw in the frame for whatever painting he chose. The gallery owner denied that any such conversation took place. There are no other witnesses. About which agreements should the court allow the art collector to testify? A The oral agreement for the painting, but not the oral agreement for the frame. B The oral agreement for the frame, but not the oral agreement for the painting. C Both the oral agreement for the painting and the oral agreement for the frame. D Neither the oral agreement for the painting nor the oral agreement for the frame.
B The oral agreement for the frame, but not the oral agreement for the painting.
A publisher entered into a contract with a paper manufacturer who used very fine materials, whereby the publisher was given the right to purchase all paper refined by the paper manufacturer for the next five years at a price set at 95% of the domestic market price at the time of delivery. The publisher agreed to purchase no less than 1,000 pounds of paper a week. At the time this contract was signed, the publisher gave written notice to the paper manufacturer that it intended to buy all paper produced by the paper manufacturer until further notice. The paper manufacturer then sold its business to a lumber-processing company. What is the effect of this sale on the paper manufacturer's obligation to the publisher? A The sale discharges the paper manufacturer's obligation to the publisher because there has been a full performance. B The paper manufacturer is liable for damages if the lumber processing plant fails to deliver paper to the publisher. C The paper manufacturer is excused from further performance because it no longer has a factory to produce paper. D The paper manufacturer breached its contract with the publisher.
B The paper manufacturer is liable for damages if the lumber processing plant fails to deliver paper to the publisher. The paper manufacturer is liable for damages if the lumber processing plant fails to deliver paper to the publisher. Because delivery of paper is not personal in nature, that duty can be delegated. The quantity will be measured by the paper manufacturer's original output. However, when a duty is delegated to a delegate, the delegator remains liable should the delegate fail to perform. (A) is incorrect because the contract was for five years, and five years have not yet elapsed. (C) is wrong because a delegator remains liable. (C) would be correct only if the paper manufacturer went out of business without delegating its duties to another by selling the other the business, not the case here. (D) is wrong because, as indicated above, such a delegation is proper.
The owner of a small fleet of taxicabs had his cabs serviced by a national chain of auto service centers. One of his cabs went through a stop sign when its brakes failed without warning. The ensuing collision seriously injured the passenger. An investigation revealed that brake repairs had been made on the cab a week before, but the service center's mechanic had used the wrong parts and had made numerous errors in reassembling the brakes. If the passenger sues the cab company owner for her injuries, who should prevail? A The passenger should prevail, unless the jury determines that the owner exercised a high degree of care in selecting the service center for maintenance of his cabs. B The passenger should prevail, because the owner breached his duty to her to provide a safe vehicle in which to ride. C The owner should prevail, because he had no reason to know that the service center's mechanic would be negligent. D The owner should prevail, because he is not vicariously liable for the negligence of an independent contractor.
B The passenger should prevail, because the owner breached his duty to her to provide a safe vehicle in which to ride.
A seller entered into a written contract to sell a tract of land to a buyer. The buyer was to pay $1,500 per month for five years, at which time the seller would deliver a warranty deed. The contract was silent as to the quality of title to be conveyed. After making 12 payments, the buyer discovered that a neighbor had an easement of way over the land, which was not discussed at the time the seller and buyer entered into the contract. The neighbor had not used the easement over the previous year because she had been out of the country. On the basis of the easement, the buyer wishes to cancel the contract. Which party is more likely to prevail? A The seller, because the neighbor's easement has been extinguished. B The seller, because the buyer has no basis on which to rescind the contract. C The buyer, because the obligation to convey marketable title is implied. D The buyer, because the seller has breached the covenant against encumbrances.
B The seller, because the buyer has no basis on which to rescind the contract.
Shortly before their wedding, a man and a woman bought a tract of land, taking title in both names. They had intended to build a summer cottage there, but many years after their marriage the land was still a vacant lot. The man decided that their introverted son would have more confidence if he were a landowner; thus, the man drew up a deed conveying a one-quarter interest in the land to him. Not wanting to show favoritism, two weeks later the man drew up a deed conveying a one-quarter interest in the same land to their daughter. Who owns the land? A The man and woman share ownership of the land with rights of survivorship, and the son and daughter have no interests. B The son has a one-quarter interest, the daughter has a one-quarter interest, and the woman has a one-half interest. C The son has a one-quarter interest, the daughter has a one-quarter interest, and the woman has a one-half interest, with rights of survivorship. D The woman owns the land.
B The son has a one-quarter interest, the daughter has a one-quarter interest, and the woman has a one-half interest. The son has a one-quarter interest, the daughter has a one-quarter interest, and the woman has a one-half interest. There is a presumption that the man and woman are tenants in common. For a joint tenancy to exist, there must be an express creation of such tenancy; thus, there is a presumption of tenancy in common unless the conveyance is to a husband and wife in a state that recognizes tenancy by the entirety. Here, the man and woman were not married when they took title to the land. Each tenant in common has an undivided interest, which may be conveyed by inter vivos transfer. The man started with an undivided one-half interest, one-half of which he conveyed to his son and the other half of which he conveyed to his daughter. The man has thus conveyed all of his interest in the land, and so (A) is incorrect. There is no right of survivorship in a tenancy in common; therefore, (C) is incorrect. (D) is not supported by the facts.
In a civil action for personal injuries, a plaintiff asserts that on the previous July 15 he was injured when the defendant negligently operated her motor vehicle and caused it to strike his vehicle when both cars were being driven along a principal north-south arterial street in a small town. Concerning the following facts that arise during the trial, to which is it most appropriate for the judge trying the case to apply the doctrine of judicial notice? A The pavement on the street was wet on July 15, based on the judge's recollection that it was raining heavily that day. B The street runs in a north-south direction, based on information generally known by residents of the town. C The brakes on the defendant's car were faulty, based on the uncontroverted testimony of her auto mechanic and an automotive engineer testifying as an expert for the plaintiff. D The defendant was exceeding the speed limit when the accident occurred, based on the testimony of the plaintiff, two credible eyewitnesses, and the police officer who examined the skid marks, but controverted by the testimony of the defendant and a passenger in her car.
B The street runs in a north-south direction, based on information generally known by residents of the town.
A state enacted a statute to provide financial aid for residents of the state who attend public or private colleges and universities in the state. Under this statute, eligible students receive varying amounts of money, depending on need. A student living in the state who has never paid taxes applied for a grant of funds under this statute to attend a private college in a different state. His application was denied because the college was outside of his home state. The student filed suit in federal court against the appropriate state official, challenging the constitutionality of the denial on equal protection grounds and to compel the granting of his application. Which of the following statements is most correct? A The suit is barred by the Eleventh Amendment. B The student has standing to maintain the action despite the fact that he has never paid taxes in the state. C The federal court will not grant the injunctive relief sought by the student in the absence of "extraordinary circumstances." D The doctrine of sovereign immunity bars the student's action.
B The student has standing to maintain the action despite the fact that he has never paid taxes in the state. The student has standing. A person challenging the constitutionality of a government action must have standing to raise the issue. To have standing, a person must show that he is injured by a government action (injury in fact) and that a favorable decision will eliminate the harm. Generally, a taxpayer does not have standing to challenge the way tax money is spent because any alleged injury is too remote. However, here the student is not bringing suit as a taxpayer; rather he is alleging that the state policy of providing financial aid only for residents who attend schools in the state injures him by depriving him of such aid solely on the basis of attending a college outside the state, thus violating his right to equal protection. A ruling in the student's favor will eliminate the harm to him. Therefore, the student has a concrete stake in the outcome of this controversy, entirely independent of whether he has ever paid taxes in the state. Regarding (A), the Eleventh Amendment prohibits a federal court suit against a state by a citizen of that state or by a citizen of another state. However, the Eleventh Amendment does not bar a suit against a state official acting pursuant to state law but allegedly in violation of the plaintiff's constitutional rights. Here, the student is seeking an order that a particular state official be compelled to act in conformity with the student's right to equal protection. The lawsuit is not brought against the state, nor does it seek a retroactive recovery from state funds. The prospective payment of state funds that the student seeks through the compelled granting of his application for aid is not prohibited by the Eleventh Amendment. Thus, (A) is incorrect. Regarding (D), the doctrine of sovereign immunity refers to the rule that a governmental entity may not be sued unless it consents to be sued (which consent is generally afforded by statute). Here, the facts do not state whether the state has consented to be sued. However, as detailed above, the student is not actually suing the state. He is suing a state official who is allegedly enforcing an unconstitutional enactment, and is seeking to compel the official to grant his application. Because the suit is not against the state, the doctrine of sovereign immunity is not applicable. (C) is incorrect because it is only with regard to state criminal statutes or prosecutions that a party seeking to enjoin such statutes or prosecutions must show irreparable injury or exceptional circumstances (i.e., a showing of significant harm that could not be avoided by state adjudication and appellate review of the proceedings). Here, there is no criminal statute or prosecution at issue. Thus, there is no need to show "extraordinary circumstances."
A state study indicated that an inordinately high percentage of homeless in the state were afflicted by alcoholism or addiction to illegal drugs. The legislature therefore decided to levy a special tax, with all proceeds marked for rehabilitative services for the homeless. However, the legislators determined that direct taxes on alcoholic beverages would be resented by the citizenry. Lobbyists from the state's growing wine industry also objected to anything that would retard the industry's development. There were no breweries or distilleries within the state. Thus, a tax was eventually passed requiring newspapers and magazines of general circulation published in the state to be taxed at a rate of 20% on all advertising space sold for beer or distilled spirits promotions. For certain historical reasons, a high proportion of the advertising revenue of a particular small newspaper within the state came from beer and wine ads. The publisher of the small paper filed suit to have the tax declared unconstitutional. A major wholesale beer and liquor distributor located within the state and several out-of-state brewers and distillers who sold and advertised their products in the state also joined in the suit as plaintiffs. If the tax is declared unconstitutional, what is the most likely reason? A The tax burdens interstate commerce by exempting advertisements for the local wine industry from the tax, while the ads of out-of-state brewers and distillers are subject to the tax. B The tax infringes on freedom of the press, which is guaranteed by the First and Fourteenth Amendments. C The tax is unconstitutional because it is not properly apportioned. D The tax violates the Equal Protection Clause of the Fourteenth Amendment, because it does not treat all alcoholic products equally.
B The tax infringes on freedom of the press, which is guaranteed by the First and Fourteenth Amendments.
A developer owned a large urban property, which she subdivided into 10 lots. The developer conveyed Lot 1 to an architect by a deed that contained a restriction banning commercial use of the property. The developer subsequently conveyed Lots 2 through 7 to six separate purchasers. Each of the deeds to these purchasers also contained the restriction on commercial use. The architect left Lot 1 undeveloped, but the purchasers of Lots 2 through 7 all used their lots for commercial purposes. The developer subsequently conveyed Lot 8 to a florist. The florist's deed contained the restriction banning commercial use of the lot, but he decided that he wished to use Lot 8 commercially. The developer retains ownership of Lots 9 and 10. The florist wants to bring suit to establish his rights to use Lot 8 for commercial purposes. Which of the following best describes the parties the florist should join in his lawsuit? A The developer only. B The developer and the architect only. C The other commercial users only. D All landowners in the subdivision.
D All landowners in the subdivision.
A landlord leased residential property to a tenant. The written lease was for a period of one year, with the monthly rent of $1,000 payable on or before the first of each month. The termination date set out in the lease was October 1. On August 10 of the first year of her tenancy, the tenant received a letter from the landlord along with a new lease form. The lease was for a period to terminate on October 1 of the following year, and the rent stated in the new lease was $1,200 per month. Both the rent increase and the notice given were in full compliance with relevant state statutes. An accompanying letter, signed by the landlord, asked the tenant to sign the lease on the line marked "tenant." On September 15, the tenant sent the lease back to the landlord unsigned. On September 20, the tenant sent a letter to the landlord by certified mail. The landlord signed the return receipt, which the post office duly sent to the tenant. Enclosed with the tenant's letter was a check for $1,000 for "next month's rent." The landlord deposited the check into his bank account. With the landlord's acquiescence, the tenant remained in possession after October 1. Which of the following statements is most accurate? A The tenant has a month-to-month tenancy at $1,000 monthly rent. B The tenant has a month-to-month tenancy at $1,200 monthly rent. C The tenant has an annual tenancy at $1,200 per month rent. D The tenant has a tenancy at will.
B The tenant has a month-to-month tenancy at $1,200 monthly rent.
A defendant was charged with murdering his boss. After obtaining a valid search warrant and executing a valid search of the defendant's office, an officer found a love letter from the defendant's wife to his boss describing their sexual relations. The letter stated, "I can no longer hide my love for you from my husband. I intend to tell him about us and leave him for you." At trial, the officer seeks to testify about the contents of the letter as proof of the defendant's motive for killing his boss. The defense counsel should object on which of the following grounds? A The letter was not authenticated as being from the defendant's wife. B The testimony violates the best evidence rule. C The testimony is hearsay not within any exception. D The probative value of the evidence is substantially outweighed by the danger of unfair prejudice.
B The testimony violates the best evidence rule.
A 13-year-old boy who lived on a farm with his parents in a rural area had learned to drive the family's tractor when he was 11. A state statute permitted persons without a driver's license to operate farm vehicles on public roads for short distances. One morning the boy took the tractor onto a public road to reach one of the outlying fields a few hundred yards away. As he neared the field he was distracted by a girl riding by on a bicycle, and cut in front of a milk delivery truck that was starting to pass him. The truck swerved off the road, injuring the driver. If the driver sues the boy to recover damages for his injuries, which of the following statements is most correct regarding the standard of care to be applied? A The state statute replaces the general common law standard of care with a statutory standard. B The trier of fact should take into account the boy's experience at driving a tractor when considering the applicable standard of care. C Persons 13 years of age or older are held to the same standard as adults. D An adult standard of care will not be applied because it is common in that region for children of that age to be operating tractors.
B The trier of fact should take into account the boy's experience at driving a tractor when considering the applicable standard of care.
A producer hired a violinist to play in an orchestra that was to leave on a 10-week tour. The violinist turned down another job opportunity in order to accept the producer's job offer. One week after the start of the tour, the violinist was hospitalized with a bad back and was unable to perform. The producer hired another musician to take the violinist's part in the orchestra. Four days later, the violinist recovered but the producer refused to allow her to rejoin the orchestra or to complete the tour. She then sued the producer for breach of contract. If the violinist brings an action against the producer for breach of contract, which of the following represents the violinist's best legal argument? A The violinist's reliance on the job offered by the producer by declining another job opportunity created an estoppel against the producer. B The violinist's failure to perform with the orchestra for four days was not so material as to discharge the producer's duty to perform. C The violinist's performance with the orchestra for the four-day period was physically impossible. D The violinist was never told that an injury might jeopardize her continued employment with the orchestra.
B The violinist's failure to perform with the orchestra for four days was not so material as to discharge the producer's duty to perform.
A manufacturer of high-speed computers entered into a written agreement with a distributor whereby the distributor would purchase a specified computer from the manufacturer for $50,000. The parties had orally agreed that the delivery date would be November 4. However, when the agreement was reduced to writing, a glitch in the word processor caused the printout to show the delivery date as "12/4" instead of "11/4." Both parties signed the paper without noticing the incorrect delivery date. Before reducing their agreement to writing, the parties had also orally agreed that the agreement would not become binding unless the distributor notified the manufacturer, in writing, by October 7, that it (the distributor) had obtained a buyer for the computer. On September 25, the distributor found a buyer who needed the computer for her business and who agreed to buy it from the distributor. However, the distributor did not inform the manufacturer that it had found a buyer until October 30. In the meantime, due to a strike at the manufacturer's leading competitor, the price of high-speed computers rose rapidly during the month of October. By the end of the month, the market value of the computer in question was $70,000. Because of the increase in the value of the computer, the manufacturer does not want to deliver the specified computer to the distributor for $50,000. Which of the following provides the manufacturer the best defense if the distributor sues to enforce the contract? A The increase in value of the computer makes the contract unconscionable. B There has been a failure of a condition precedent. C A case of mutual mistake exists because of the word processing error regarding the delivery date. D There has been a failure of a condition subsequent.
B There has been a failure of a condition precedent.
A new college graduate entered into an oral agreement with a freshman to lease the freshman her mini-refrigerator for a term of four years. The freshman was to pay the graduate $20 a month, of which $10 of the monthly charge was to be paid directly to the graduate's parents, in satisfaction of a debt the graduate owed her parents. While the graduate was putting the agreement into writing she accidentally failed to include the agreement to pay her parents directly. The freshman also failed to notice that the direct payment provision was missing before she signed the contract, which the graduate signed. If the parents bring an action against the freshman, which of the following will have the greatest effect on the outcome? A Whether the parents were a party to the agreement between the graduate and the student. B Whether the agreement between the graduate and the freshman was completely integrated. C Whether the graduate was negligent in not discovering that the agreement omitted mention of the payment of money directly to her parents. D Whether the freshman was negligent in not discovering that the agreement omitted mention of the payment of money directly to the parents.
B Whether the agreement between the graduate and the freshman was completely integrated. The most critical factor will be whether the agreement was completely integrated. The effect of a completely integrated agreement, meaning that the writing embodies the entire agreement of the parties, is that evidence could not be introduced to show a prior or collateral oral agreement. (A) is incorrect because the parents need not be a party to the contract; they have rights as third-party beneficiaries. (C) and (D) are incorrect because the negligence of either of the parties has no bearing on whether the collateral oral agreement can be proved or enforced.
The plaintiff brought a breach of contract suit against the defendant, alleging that the defendant paid for only 25 cases of baseball cards even though the order was for 50. The defendant claimed that only 25 cases were delivered to his store. The plaintiff then introduced a shipping bill from a freight company showing that pickup had been made from the plaintiff on 50 cases. May the defendant now compel the plaintiff to introduce the remainder of the record, showing that the freight company had only 25 cases of baseball cards on its truck? A Yes, after a proper foundation is laid. B Yes, because fairness dictates that it should be considered contemporaneously with what the plaintiff is offering. C No, because the information is irrelevant to the case. D No, because the record would constitute inadmissible hearsay.
B Yes, because fairness dictates that it should be considered contemporaneously with what the plaintiff is offering.
A boater taking his new powerboat out on a large lake ran out of gas because of a defective seal in the gas tank. The defect was not discoverable by an ordinary inspection. His frantic signaling alerted the captain of a sightseeing boat passing by. The captain pulled up alongside to assist and attempted to restart the boat. A spark ignited a pool of gas that had leaked from the gas tank and collected in the lower part of the boat, causing an explosion and fire. The captain was severely burned and died from his injuries. The captain's estate brought a wrongful death action based on strict liability against the powerboat dealer and the manufacturer. Evidence at trial established that the dealer had sold the manufacturer's boats for years without any problems reported by customers. Can the captain's estate recover any damages from the dealer? A Yes, unless the jury finds that the boater was negligent in failing to investigate where the gas had gone. B Yes, because harm to someone in the captain's position was a foreseeable result of the gas leak. C No, because the dealer had no reason to anticipate that the manufacturer assembled the gas tank improperly. D No, because the captain did not have a sufficient relationship to the boater to make the dealer liable for the captain's death.
B Yes, because harm to someone in the captain's position was a foreseeable result of the gas leak.
In answer to a radio advertisement, a teenager two months shy of his 18th birthday contracted to buy a late model car from a car dealership. The agreement required a $1,500 down payment with the remainder of the $7,200 price to be paid in monthly installments to a local finance company. The teenager's first eight payments were made regularly until his driver's license was suspended. He then informed the company that no further payments would be forthcoming. The finance company sued for the remaining payments. The age of majority in the teenager's state is 18 years. Would the teenager be liable for the balance of the payments? A Yes, because the car dealership was liable on the contract from the outset, notwithstanding his minority. B Yes, because he kept the car for six months after reaching the age of majority. C No, because he was a minor at the time of contracting, and the contract was voidable by him. D No, because he informed the finance company in a timely manner after his driver's license was suspended.
B Yes, because he kept the car for six months after reaching the age of majority.
For many years, a landowner owned a parcel of land bordered on the west by a public road, and his neighbor owned a parcel of land located immediately to the east of that parcel. The neighbor had an easement to cross the west parcel to enter the public road bordering it. Because the neighbor's east parcel is surrounded by swampland on the north, south, and east, the only route of ingress to and egress from that parcel over dry land passed through the west parcel. Subsequently, the neighbor sold the east parcel to the landowner, who proceeded to use both lots as a common tract. Last year, the landowner sold the east parcel to his friend. Does the friend have an easement over the landowner's west parcel? A Yes, she has an easement in gross. B Yes, because her only access to her parcel from the public road is across the west parcel. C No, because the easement was extinguished when the landowner purchased the east parcel. D No, because she has not used the property long enough to gain an easement by prescription.
B Yes, because her only access to her parcel from the public road is across the west parcel. The friend has an easement by necessity over the landowner's west parcel, because only by crossing over that parcel can she gain access to her parcel. When the owner of a tract of land sells a part of the tract and by this division deprives one lot of access to a public road, a right-of-way by absolute necessity is created by implied grant over the lot with access to the public road. The facts state that the east parcel is surrounded by swampland on the north, south, and east. Thus, when the landowner sold that parcel to the friend, there was an implied grant of an access easement across the landowner's parcel because it was clearly her only access to a public road. (A) is wrong because an easement in gross does not have a dominant tenement. The holder of an easement in gross has a right to use the servient tenement independent of her ownership or possession of another tract of land. Here, the easement over the west parcel arises solely as a consequence of the friend's ownership of the adjacent east parcel. Thus, the easement is appurtenant, not in gross. (C) is wrong because even though the neighbor's easement was extinguished, the friend has acquired a new easement by necessity. When the ownership of the easement and the servient tenement is in one person, the easement is extinguished. Thus, when the landowner bought the east parcel, the neighbor's easement was extinguished. After the easement was extinguished, however, a new easement was created by operation of law when the land was again subdivided into two lots and as a result of this subdivision one of the lot owners was deprived of access to a public road. (D) is wrong because the friend has an easement by necessity, which can arise any time the appropriate circumstances exist. The friend need not wait out the prescriptive period to gain the legal right to pass over the landowner's parcel.
A plant nursery sued a nearby factory in federal court, alleging that the factory was emitting toxic fumes that were harming the nursery's plants. The nursery sought an injunction and damages. The court granted a preliminary injunction against the factory, ordering it to stop operations until a final judgment was reached in the case. The damages issue has not yet been addressed. The factory files an appeal against the injunction. Can the court of appeals hear the factory's appeal? A Yes, because any ruling by a trial court can be appealed immediately. B Yes, because injunctions are reviewable on appeal before there is a final judgment. C No, because there has not been a final judgment as to all the claims. D No, because interlocutory orders are never reviewable.
B Yes, because injunctions are reviewable on appeal before there is a final judgment.
A plaintiff filed an action against a defendant in federal district court, seeking compensatory damages for negligence after a car accident. The defendant has a policy of automobile liability insurance under which its insurance company will pay for the defendant's defense and all or part of the defendant's liability in the action. Is the defendant's insurance policy subject to discovery by the plaintiff? A Yes, because it is relevant to the claims and defenses asserted by the parties. B Yes, because insurance policies generally are part of the defendant's required disclosures. C No, because the insurance policy is not relevant to the claim or defense of any party. D No, because the insurance policy is protected by privilege.
B Yes, because insurance policies generally are part of the defendant's required disclosures.
A landlord entered into a written lease of a bakery for a term of 25 years with a baker. The parties agreed to a right of first refusal if the bakery was offered for sale during the term of the lease. The lease also permitted assignments and subleases on notice to the landlord. Three years later, the baker retired and, after notifying the landlord, transferred the lease to a chocolatier. Twenty-one years later, the landlord entered into a contract with a buyer for the sale of the bakery for $100,000. The landlord had informed the buyer of the lease but had forgotten about the right of first refusal. When the chocolatier learned of the sale to the buyer, she informed both the landlord and the buyer that she wanted to exercise her option and was prepared to purchase the bakery for the contract price. The jurisdiction's Rule Against Perpetuities is unmodified by statute. Can the chocolatier enforce the option? A Yes, because an option held by a tenant on leased property cannot be separated from the leasehold interest. B Yes, because the option touches and concerns the leasehold estate. C No, because the transfer to the chocolatier made the option void under the Rule Against Perpetuities. D No, because the option was not specifically included when the lease was transferred to the chocolatier.
B Yes, because the option touches and concerns the leasehold estate.
The plaintiff sued the defendant in federal court for breach of contract. The case went to trial, and the jury found in favor of the plaintiff and awarded her $125,000. Judgment was entered on June 1. On June 10, the defendant filed a motion for a new trial. On June 18, the plaintiff files to enforce the judgment. The court has not issued any orders since the final judgment on June 1. May the plaintiff enforce the judgment? A Yes, because judgments are enforceable as soon as they have been entered. B Yes, because judgments are enforceable during pendency of post-trial motions unless the court otherwise orders. C No, because judgments are not enforceable until 28 days after entry. D No, because judgments cannot be enforced while a post-trial motion is pending.
B Yes, because judgments are enforceable during pendency of post-trial motions unless the court otherwise orders.
One of the employees of a department store informed the chief of security that he suspected a customer of shoplifting. The employee pointed out the suspected shoplifter, but the security chief thought that the employee was pointing at a different woman. He waited until the woman had left the store and then followed her outside. He went up to the woman, pointed a gun at her, and told her she needed to return with him to the department store's security office. The woman did so, even though she insisted that she had not done anything wrong. The woman was kept waiting in the office for a few minutes until the employee informed the security chief that he had arrested the wrong woman. If the woman sues the department store on the theory of false imprisonment, will she prevail? A Yes, unless the security chief's belief that she was the shoplifter is judged to be reasonable. B Yes, because she was intentionally detained. C No, because the period for which the woman was detained was the minimal period necessary to establish her identity. D No, because the woman suffered no harm.
B Yes, because she was intentionally detained.
A State A consumer was in a traffic accident with a State B driver. The State A consumer's car burst into flames, causing horrific injuries to the consumer. The State A consumer believes that his injuries were caused by both the State B driver's negligence and design defects in his own car. The State A consumer thus filed a tort action for damages against both the State B driver and the manufacturer of the car. The manufacturer is also a citizen of State B. The State B driver was also burned in the accident and believes that manufacturing and design defects in the State A consumer's car also caused his burns. Can the State B driver assert his tort claim against the State B manufacturer in the pending action asserted by the State A consumer? A Yes, because the State B driver and State B manufacturer are already parties to the State A consumer's action. B Yes, because the State B driver's claim is a proper cross-claim and is within the court's supplemental jurisdiction. C No, because the State B driver's tort claim against the State B manufacturer is not between citizens of different states. D No, because the State B driver's claim is not a proper cross-claim.
B Yes, because the State B driver's claim is a proper cross-claim and is within the court's supplemental jurisdiction.
The President of the United States and the leader of a bordering foreign nation agreed that each should appoint three members to a special joint commission to deal with a wildlife problem. The President of the United States, acting in concert with the foreign leader, named the joint commission as a permanent enforcement agency for the regulations that were adopted by both nations. Although the President received prior congressional authorization to enter into this agreement, the Senate did not ratify the agreement by a two-thirds vote. The President then entered into an executive agreement with the foreign leader whereby the joint commission was granted adjudicative as well as enforcement powers with respect to a particular issue. Is the executive agreement by the President valid? A Yes, because the President has unlimited powers in entering into executive agreements. B Yes, because the agreement is within the President's powers in the area of foreign affairs. C No, because the Senate did not ratify the executive agreement by a two-thirds vote. D No, because wildlife is not an area left solely to presidential discretion.
B Yes, because the agreement is within the President's powers in the area of foreign affairs.
The defendant is on trial for fraudulently signing a check for $10,000. The defendant has denied that she signed the check. The prosecutor calls the landlord of the apartment building in which the defendant has resided for three months before her arrest. The landlord intends to testify that it is the defendant's signature on the check, and he bases his opinion of the authenticity of her signature on the ground that he saw her sign the lease to his apartment. Should the trial court find this testimony admissible? A Yes, because there was only a short period of time between when the landlord saw her sign the lease and the time of trial. B Yes, because the landlord has previously seen the signature. C No, because the landlord has seen the signature only once and is not acting as a handwriting expert. D No, because the testimony is inherently unreliable.
B Yes, because the landlord has previously seen the signature.
While fleeing from an armed robbery he had just committed, a man struck a pedestrian with his car, seriously injuring the pedestrian. The robber was soon apprehended and charged with armed robbery and reckless driving, both felonies. Just prior to trial, the pedestrian died from his injuries. The trial on the robbery and driving charges proceeded, and the robber was convicted of the armed robbery charge and acquitted of the reckless driving charge. The robber was then indicted under the jurisdiction's felony murder statute for causing the death of the pedestrian during the course of committing an armed robbery. The robber moved to dismiss the indictment on the ground that a second trial would violate double jeopardy. Is the robber's claim correct? A Yes, because he was acquitted of the reckless driving charge. B Yes, because the pedestrian died before the robber's first trial had begun. C No, because he was convicted of the armed robbery charge. D No, because felony murder requires proof of an additional element not required by the felony itself.
B Yes, because the pedestrian died before the robber's first trial had begun.
A truck driver collided with a motorcyclist in a busy intersection. A police officer present at the scene cited the trucker for running a red light. At the preliminary hearing, the trucker initially pleaded guilty, but he withdrew his plea when the judge told him what she had in mind for a sentence. The judge let the trucker change his plea to not guilty. The trucker, however, had no success at his trial and was convicted. The motorcyclist is now suing the trucker in a civil action for the injuries he sustained in the accident. If the motorcyclist tries to introduce evidence of the trucker's original guilty plea, on proper motion, should this evidence be excluded? A Yes, because it is hearsay not within any exception. B Yes, because the plea was withdrawn. C No, because an admission is not hearsay. D No, because it described the trucker's state of mind.
B Yes, because the plea was withdrawn.
A truck driver is suing a car driver for injuries he suffered when their vehicles collided at an intersection controlled by stoplights. The truck driver called a witness to the accident to testify that he saw the driver of the car drive through a red light. On cross-examination, the car driver's attorney asks the witness, "Isn't it true that the car driver's ex-wife is paying $500 for your testimony today?" The truck driver's attorney objects. Should the objection be overruled? A Yes, because the question gives the witness an opportunity to explain or deny the allegation. B Yes, because the question is a proper form of impeachment. C No, because the question addresses a collateral issue. D No, because it is a leading question.
B Yes, because the question is a proper form of impeachment.
Congress created the National Agency for Burglar Alarms ("NABA"), giving it the power to regulate both burglar alarm hardware and installation personnel. NABA adopted a regulation requiring that all burglar alarm installation companies be licensed, and providing that anyone installing an alarm without a license could be fined. The regulation also provided that any company in the installation business on the day the regulation was adopted automatically would receive a license, but to obtain a license thereafter, an applicant would have to show that he has worked as an installer at a licensed company for at least three years. A man who has been installing alarm systems for eight years sold his installation business a few months before the NABA regulation was adopted and went to work for the purchaser servicing his old accounts. A few months after the NABA regulation was adopted, a representative from a national department store chain approached the man with an offer to hire him as an independent contractor to revamp the chain's alarm systems. The man quit his job and applied for a NABA installer's license. His application was denied because he was not in business on the day the NABA regulation was adopted and had worked for a licensed installer for only a few months. The man decided to install the alarm systems anyway. Can the man properly be fined for installing alarms? A Yes, because the NABA was established under Congress's power to legislate for the general welfare, and Congress may take whatever steps are necessary and proper to enforce its laws. B Yes, because the regulation falls within the scope of Congress's commerce power, and Congress may delegate its authority to regulate as it has done here. C No, because the regulation interferes with the man's fundamental right to earn a living without a substantial justification and so violates the Privileges and Immunities Clause of Article IV, Section 2. D No, because a government agency cannot itself levy fines for a violation of its regulations.
B Yes, because the regulation falls within the scope of Congress's commerce power, and Congress may delegate its authority to regulate as it has done here. The man can be fined. Congress has the power to regulate alarm installation companies under the Commerce Clause because the clause permits Congress to regulate any local or interstate activity that, either in itself or in combination with other activities, has an effect on interstate commerce. Burglar alarm companies use instrumentalities of interstate commerce such as phone lines and have a cumulative effect on interstate commerce even though some may only do business locally. Hence, their activities can be regulated by Congress. The delegation to the NABA is valid because Congress has broad discretion to delegate its legislative power; the Supreme Court will uphold almost any delegation of congressional power. Therefore, the man can be fined. (A) is incorrect because it improperly mixes two concepts. Congress does not have the power to legislate for the general welfare—there is no federal police power—but rather Congress has the power to spend for the general welfare. (C) is incorrect because, even assuming that the regulation here interferes with the man's right to make a living, it would not violate the interstate Privileges and Immunities Clause of Article IV because the clause restricts states, not the federal government. (D) is incorrect. Congress can provide that violation of an agency's regulations is a criminal offense that can be enforced through the imposition of fines. Furthermore, an agency has the power to impose civil fines and penalties for a violation of its regulations.
Congress passed a law imposing a 50% excise tax on each pack of cigarettes manufactured for sale in the United States. An amendment was successfully added to the original bill requiring that all proceeds from the tax be used for antismoking educational programs in the audio, video, and print media and elsewhere. The amendment also provided for the establishment of federal stop-smoking clinics funded through the excise tax. The various tobacco companies were required to pay the tax directly to the federal government. A tobacco company filed suit in the appropriate federal court, contending that the tax should be struck down as unconstitutional. Is the court likely to find the tax constitutional? A Yes, because the tax is severable from its purpose. B Yes, because the tax is a proper exercise of Congress's taxing and spending powers. C No, because it does not provide equal time for the tobacco companies to present their side of the smoking controversy. D No, because it abridges the First Amendment rights of tobacco manufacturers by forcing them to pay for messages with which they may not agree.
B Yes, because the tax is a proper exercise of Congress's taxing and spending powers.
A bookstore owner entered into an agreement with a building contractor to have a facade attached to the front of his bookstore. The contractor constructed the facade and attached it to the storefront, using plans prepared by himself and his own employees. After completing the work, the contractor was paid the contract price by the bookstore owner. A week later, a woman was walking past the front of the bookstore when the facade and a portion of the original building collapsed, striking and injuring her. The woman sued both the contractor and the bookstore owner for damages arising from her injuries. The parties stipulated that the attachment of the facade to the storefront caused the building to collapse and that the bookstore owner was not negligent in selecting or supervising the contractor. If the woman recovers against the bookstore owner, does the latter have any right of action against the contractor? A Yes, because the bookstore owner's conduct was not a cause in fact of the injuries to the woman. B Yes, because the woman recovered from the bookstore owner on the basis of vicarious liability. C No, because the bookstore owner selected the contractor to perform the work. D No, because payment for the work without reservation was acceptance by the bookstore owner.
B Yes, because the woman recovered from the bookstore owner on the basis of vicarious liability.
A homeowner and a carpenter entered into an oral agreement, under which the carpenter agreed to construct an addition to the homeowner's home, using materials supplied by the homeowner, in exchange for $20,000. After the work had been completed but before the homeowner had made any payment, the carpenter called the homeowner and instructed him to pay the $20,000 due on the addition work to a creditor of the carpenter. If the homeowner fails to pay the creditor, and the creditor thereafter brings an action against the homeowner for $20,000, is the creditor likely to prevail? A Yes, because the creditor was the intended beneficiary of the contract between the homeowner and the carpenter. B Yes, because there has been a proper assignment by the carpenter. C No, because personal service contracts are not assignable. D No, because the performance was not executory on both sides at the time of the assignment.
B Yes, because there has been a proper assignment by the carpenter. The creditor will prevail because the carpenter has made a valid assignment of his right to payment from the homeowner. The general rule is that all contractual rights can be assigned, and all the carpenter assigned here was his right to payment. (A) is wrong because it is contrary to the facts; nothing indicates that payment to the creditor was contemplated in the original contract. (C) is a misstatement of the law; the assignment of a right to receive payment is permitted, even in a personal service contract. (D) is irrelevant. There is no requirement that the performance of the contract be executory on both sides for a valid assignment. The carpenter could validly assign his right to payment even though he had completed his performance.
A utility company constructed a building costing approximately $2 million that encroached on a rancher's property. The rancher is suing the company in federal district court to force the company to remove the office building. In the same action, the rancher is asking for $200,000 in damages incurred because of the trespass. The court has diversity of citizenship jurisdiction. Is the company entitled to a jury trial? A Yes, for all issues. B Yes, but only for issues involving the trespass action. C Yes, but only for issues involving the action to require the company to move its building. D No, because the equitable issue predominates.
B Yes, but only for issues involving the trespass action.
An investor who owned several thriving shopping malls was negotiating to purchase a local mall from the company that currently owned it. A staff attorney for the state transportation department who shopped at the mall regularly learned of the negotiations and contacted the investor. The mall had deteriorated noticeably during the time the current company had owned it and the attorney believed that new ownership would revitalize the mall considerably. Although the attorney had no information to support this, she told the investor that the state was currently planning to construct a new interchange for the turnpike only three blocks from the mall. The investor went ahead with the purchase, believing that the new interchange would boost sales. In fact, no interchange was being considered by the state at that time, and nothing that the investor did after he purchased the mall could stem the decline in sales. He ended up selling the property at a substantial loss several years after the purchase. Does the investor have a cause of action against the attorney for his losses? A Yes, for negligent misrepresentation, because the owner made a business transaction in reliance on the attorney's statements. B Yes, for intentional misrepresentation, because the attorney was aware that she did not know whether the state was planning an interchange. C No, because the attorney's statement pertained to a future event that may not be justifiably relied upon. D No, because the attorney made her statement to the owner gratuitously.
B Yes, for intentional misrepresentation, because the attorney was aware that she did not know whether the state was planning an interchange. The attorney acted with scienter for purposes of an intentional misrepresentation action because she was aware that she did not know whether the state was planning an interchange. To establish a prima facie case of intentional misrepresentation or fraud, plaintiff must prove (i) misrepresentation by defendant, (ii) scienter, (iii) intent to induce plaintiff's reliance on the misrepresentation, (iv) causation (actual reliance on the misrepresentation), (v) justifiable reliance on the misrepresentation, and (vi) damages. The element of scienter, which involves defendant's state of mind, requires plaintiff to show that defendant made the statement knowing it to be false or made it with reckless disregard as to its truth or falsity. Because the attorney made her statement even though she had no information that the state was planning an interchange, she acted with scienter. The other elements of intentional misrepresentation are established by the facts. Thus, the investor has a cause of action against the attorney under the condition stated in (B). (A) is incorrect because an action for negligent misrepresentation is not supported by these facts. Negligent misrepresentation requires (i) a misrepresentation made by defendant in a business or professional capacity, (ii) breach of duty toward that particular plaintiff, (iii) causation, (iv) justifiable reliance, and (v) damages. Here, even though the investor was involved in a business transaction, the attorney was not. She was not acting in a business capacity but rather for her own personal interests. Hence, she is not liable for negligent misrepresentation. (C) is incorrect because the attorney's statement was a false representation of an existing fact—that an interchange was currently being planned. If the attorney had instead assured the owner simply that the interchange was going to be built in the future without any assertion of present facts, the investor could not justifiably rely on the statement because it is a statement of a future event over which the attorney did not have control. The statement here is actionable because an interchange was not even being planned. (D) is incorrect. The fact that the attorney made the statement to the owner gratuitously rather than in a commercial transaction absolves her from liability for negligent misrepresentation, but it has no relevance to her liability for intentional misrepresentation.
Suspecting criminal activity, a police officer acting without a warrant peeked through a small opening in the shutters of an apartment. The officer observed the apartment's tenant and the defendant making methamphetamine. The officer immediately entered the apartment and arrested the tenant and the defendant, and he confiscated the ingredients for the methamphetamine, the tools used for methamphetamine production, and any completed methamphetamine for evidence. The search is later ruled invalid at a suppression hearing. May the defendant now claim that her Fourth Amendment rights have been violated by the seizure of the ingredients, tools, and methamphetamine from the apartment? A Yes, because the items will be used in evidence against her. B Yes, if she was an overnight guest of the tenant. C No, because she was not the owner or occupier of the apartment. D No, unless she admits to ownership of the items.
B Yes, if she was an overnight guest of the tenant.
In a drug raid, police in a city searched 25 apartments selected at random in a 300-unit housing project. In a class action approved by the federal court, the 25 tenants sued the city for violation of their constitutional rights. The tenant named as class representative gave notice to all unnamed class members, including another tenant who decided not to opt out. The class action was then certified as a "common question" type. After negotiating with the class representative, the city police agreed to pay each tenant $500 and to conduct no further raids without proper warrants. The representative and the city signed a settlement agreement and a stipulation of dismissal of the class action. The other tenant objects to the amount of damages he is to receive and would rather opt out now and proceed on his own. May the tenant opt out now? A Yes, because a class member of a "common question" class action always has the right to opt out. B Yes, if the court allows the tenant a second opportunity to opt out. C No, because the tenant did not opt out after receiving notice of the class action. D No, if class certification was proper.
B Yes, if the court allows the tenant a second opportunity to opt out.
A man purchased a large flat screen television and decided to mount it on the ceiling over his bed. The manual that came with the product included detailed instructions and illustrations on how to mount the television on different types of walls, along with all the required hardware, but contained neither instructions nor warnings regarding mounting on the ceiling. The man carefully followed the wall-mounting instructions and was satisfied that it would hold. In fact, however, the mounting was not appropriate for ceilings. The next night, a woman who was the man's overnight guest was seriously injured when the television came loose and fell on the bed. Will the woman prevail in a suit against the company that manufactured the television? A Yes, because the manufacturer had a duty to include warnings for all potential placements of its product. B Yes, if the manufacturer knew that its television was sometimes mounted on ceilings rather than walls. C No, if the manufacturer's manual had all of the customary warnings for this type of product. D No, because the man was negligent in mounting the television on the ceiling.
B Yes, if the manufacturer knew that its television was sometimes mounted on ceilings rather than walls.
The plaintiff and defendant were involved in a car accident in which the plaintiff's vintage sports car was severely damaged. The plaintiff sued the defendant in federal court for $200,000 in damages, alleging that the defendant had run a red light and caused the accident. The defendant claimed that the light was green. The plaintiff files a motion for summary judgment supported by an authenticated surveillance video that clearly shows the defendant driving through the red light, as well as an affidavit from a vintage car expert who described the value of the plaintiff's car and the cost of repairing the damage. The defendant opposes the motion for summary judgment but offers no supporting evidence regarding the color of the light. He does, however, offer affidavits from two car experts who stated that the plaintiff's car was not nearly as valuable as he claimed and that only $50,000 worth of damage was done to it. May the court grant the motion for summary judgment? A Yes, the court can grant complete summary judgment. B Yes, the court can grant partial summary judgment on the matter of liability. C No, because there is a genuine dispute of material fact. D No, because federal courts do not have the power to grant partial summary judgments.
B Yes, the court can grant partial summary judgment on the matter of liability.
A shoe manufacturer entered into a contract to purchase all cowhide refined by a tannery for the next five years at a price set at 95% of the domestic market price at the time of delivery. The shoe manufacturer agreed to purchase no less than 500 pounds of cowhide a week. At the time this contract was signed, the shoe manufacturer gave written notice to the tannery that it intended to buy all cowhide produced by the tannery until further notice. For the first year, the shoe manufacturer continued to purchase all cowhide produced by the tannery. However, by the end of that year, the tannery doubled its production of cowhide. At a meeting between the tannery and the shoe manufacturer, the tannery's president noted that the shoe manufacturer was getting as much cowhide as it needed, and that the tannery intended to sell the extra cowhide it was producing on foreign markets at a higher price than the shoe manufacturer was paying. The shoe manufacturer agreed to maintain its purchases at the first year's level and signed an addendum to the original agreement reflecting this change. Is the modification of this contract made by the addendum enforceable? A Yes, as long as the shoe manufacturer's needs continue to be met by the first year's amount. B Yes, the modification is enforceable as agreed. C No, because there was no consideration for the shoe manufacturer's agreement to take only one-half of the cowhide produced. D No, because the contract did not state the amount of cowhide that the tannery would produce.
B Yes, the modification is enforceable as agreed.
An investor owned two adjacent lots in a downtown area, one fronting directly on a public street and the other behind the first. The investor ran a small dry cleaning business on the lot next to the street, and had built a café on the rear lot. Because the rear lot had no access to any public street, the investor used the parking lot of the dry cleaning business, which extended from the street all the way back to the rear lot, for access to the café. The café was only open during the tourist season, from May through September. After several years, the investor sold the rear lot to a chef by a deed that granted an easement over the dry cleaning business's parking lot, to be used as an accessway to the café for the chef and her customers. The chef promptly recorded the deed. Two years later, in February, the investor sold the dry cleaning lot to a sub sandwich franchise. The deed did not mention the easement previously granted to the chef. The franchise immediately demolished the small dry cleaning building and constructed its own restaurant. An outdoor patio area completely blocked access to the chef's rear lot. Because the chef's café was closed for the winter, nothing came of the franchise's construction until April, when the chef returned to open her café for the summer season. The shortest alternate route over other parcels from the chef's lot to a public street would have to pass through several buildings and lots. If the chef brings an action to compel the sub sandwich franchise to demolish the outdoor patio, how should the court rule? A For the franchise, because its deed contained no mention of the access easement. B For the franchise, because construction of the outdoor patio extinguished the chef's rights to the access easement. C For the chef, because ownership of the easement gives her the right to use it for access to her lot. D For the chef, because she has no other access to her lot.
C For the chef, because ownership of the easement gives her the right to use it for access to her lot.
In a trial for bank robbery, a teller has identified the defendant as the robber. Defense counsel offers into evidence a still frame from a video taken by the bank security camera the day after the robbery to show that a column obstructed that teller's view of the defendant. Is such evidence admissible? A Yes, upon testimony by the camera operator that the still frame was developed from film that was taken from that camera the day after the robbery. B Yes, upon testimony by a bank employee that the photo accurately portrays the scene of the crime. C No, not admissible into evidence but usable by a witness for explanatory purposes. D No, if a still frame can be obtained from a video taken at the time of the robbery.
B Yes, upon testimony by a bank employee that the photo accurately portrays the scene of the crime. The photo should be admitted into evidence upon testimony that it is an accurate representation of the location depicted. To be admissible, real or demonstrative evidence must not only be relevant but must also be authenticated, i.e., identified as being what the proponent claims it to be. For a photograph that is used as demonstrative evidence, authentication is by testimony that the photo is a faithful reproduction of the object or scene depicted. Here, testimony by a bank employee that the still frame from the video accurately portrays the setting where the robbery took place is sufficient for admissibility. (A) is incorrect because the frame from the video is not being offered as original evidence that played an actual role in the robbery itself, such as a gun used by the robber, which would require the "chain of custody" type of authentication in (A). Here, the still frame is only being used for demonstrative purposes; hence, authentication focuses on whether it is an accurate representation rather than how it was handled. (C) is incorrect. Charts and diagrams that are used solely to help explain a witness's testimony may be permitted at trial but not admitted into evidence where they are not offered as representations of a real object or scene but only as aids to testimony. Here, however, the photo is being offered as a faithful representation of the scene of the crime and should therefore be admissible into evidence. (D) is incorrect because it is a misapplication of the best evidence rule. The best evidence or original document rule, which is made applicable to photographs by the Federal Rules, generally requires that in proving the terms of a writing the original writing must be produced where the terms are material. The terms are material and the rule applies only when (i) the document is a legally operative or dispositive instrument, or (ii) the witness's knowledge results from having seen the fact in the document. Neither situation arises in this case. The location of the columns in the bank and the circumstances of the robbery are facts that exist independently of the document (the videotape on the day of the robbery), and thus may be proved by other evidence.
A young woman who recently graduated from college landed her dream job teaching kindergarten at the same elementary school she had attended as a child. The young woman's contract provided that she would be paid $40,000 for the school year, and that she could be fired for only just cause. Days before the young woman was to begin teaching, the school's principal fired her, without cause, so that he could hire his cousin for the job instead. The young woman submitted her resume to an employment agency, but was so depressed over the loss of her dream job that she turned down a nearby school's offer for a similar job paying a $30,000 salary, and instead spent the rest of the school year miserably unemployed. In an action by the young woman against the elementary school for damages, which of the following awards is most likely? A $40,000. B $30,000. C $10,000. D Nothing.
C $10,000.
A decedent died without having executed a will, leaving a substantial estate to be distributed by the probate court. The jurisdiction's applicable statute provides that where a decedent leaves neither issue nor spouse, nor parents, his estate goes to his brothers and sisters and their descendants. The decedent was never married, had no children, and both of his parents are dead. A woman whose birth certificate was destroyed by fire seeks to establish that she is the daughter of the decedent's only sibling, who is now also deceased. The woman offers into evidence a statement in a properly recorded trust instrument. The instrument was executed by the decedent's father and recited that certain specified real property conveyed by the decedent's father into the trust should be held for her benefit, as "my loving granddaughter." The document actually offered is an enlarged print photocopy of microfilm records, authenticated by an employee of the county. What should the trial court do? A Exclude the evidence, because it is not the best evidence. B Exclude the evidence, because it is inadmissible hearsay not within any recognized exception. C Admit the evidence, because it is a record of a document affecting an interest in property. D Admit the evidence, because it constitutes a past recollection recorded.
C Admit the evidence, because it is a record of a document affecting an interest in property.
A student started a small fire in a trash can in the men's room at his university. His plan was to set off the school's fire alarms so that he could break into the computer lab and steal a laptop computer while the building was being evacuated. The student was stopped after he had set the fire and was attempting to smash in the glass on the computer lab door. The fire was quickly extinguished and no serious damage was done to the building by the fire beyond some charring on the walls in one stall of the men's room. A statute in the jurisdiction extends the crime of arson to buildings other than dwellings. Which of the following best describes the crimes of which the student could be properly convicted? A Larceny only. B Larceny and attempted arson. C Attempted larceny and arson. D Arson only.
C Attempted larceny and arson.
To combat rising unemployment, a state offered a $25,000 prize to anyone who could devise a scheme to create at least 200 jobs within the state and demonstrate its viability. While hiking through a national park within the state, a geologist noticed rock containing titanium. Knowing that titanium was commonly used in military aircraft built within the state and that mining and refining titanium could provide the state with thousands of jobs, the geologist chipped out a sample of the ore and took it back to the state employment division. After reviewing the geologist's ideas, the state announced in a press release that he was the first recipient of the $25,000 prize. Within a few days, the federal ranger in charge of the valley from which the sample was taken had the geologist arrested for violating a federal law making it illegal to remove any "plants, animals, or minerals from federal lands." The geologist was convicted and fined $5,000. He appeals the conviction to the federal court of appeals, claiming that the fine is unconstitutional. How should the court rule? A For the geologist, because the state has a compelling interest in reducing unemployment and the federal statute unreasonably interferes with the state interest. B For the geologist, because removing the ore was a purely intrastate act and had no effect on interstate commerce. C For the government, because the federal statute providing for the fine is constitutional under the Property Clause of Article IV, Section 3 of the federal Constitution. D For the government, because the federal statute providing for the fine is constitutional under the Commerce Clause.
C For the government, because the federal statute providing for the fine is constitutional under the Property Clause of Article IV, Section 3 of the federal Constitution. The court should affirm the geologist's conviction. The fine is constitutional under the Property Clause, which gives Congress the power to "make all needful rules and regulations respecting the territory or other property belonging to the United States." This power permits Congress to acquire and dispose of all kinds of property, and to protect its property with a law such as the one here. (D) is not as good an answer as (C) because the Commerce Clause is not as directly applicable to regulation of acts on federal lands as is the Property Clause. Nevertheless, (B) is incorrect because the fine could probably be upheld under the Commerce Clause. Under the Clause, Congress may regulate any act that may itself or in combination with other activities have a substantial effect on interstate commerce, even intrastate activities. If everyone removed minerals from federal lands, the necessary substantial effect on interstate commerce would be present. (A) is incorrect because, notwithstanding the state's compelling interest, by virtue of the Supremacy Clause a valid act of Congress supersedes any state or local action that actually conflicts with the federal rule.
A thief sold some stolen goods to a dealer. Several weeks later, the police raided the dealer's store and arrested him. In this raid, the police seized the goods the thief sold to the dealer and a record book in which the dealer had recorded this transaction. However, at the dealer's subsequent trial for receiving stolen goods, the charges against him were dismissed when the court ruled that the search warrant had been improperly issued. The police were able to trace the stolen goods to the thief because of fingerprint identification and the information contained in the dealer's record book. At his trial, the thief made a motion to suppress the stolen goods and record book. What should the judge do? A Grant the motion, because the evidence is the fruit of the poisonous tree in that the search of the dealer's store was improper. B Grant the motion, because the trial court in the dealer's case has already ruled that the evidence was improper. C Deny the motion, because the thief has no standing to object to the search. D Deny the motion, because the thief's fingerprints on the stolen goods were what led to his identification.
C Deny the motion, because the thief has no standing to object to the search.
A customer slipped and fell in a store and was severly injured. As required by state regulation, the store's manager conducted an investigation into the cause of the accident and filed the report with the appropriate state agency. The customer subsequently retained an attorney, who filed a civil action against the store in federal district court. The complaint alleged that the store negligently left a spill on the floor of the store, which caused the customer's fall. The customer's attorney served on the store a request for production of documents, which included a request for all documents and reports prepared by the store that relate to the customer's fall and injury. If the store believes that the report prepared by the store manager is protected from discovery under the work product doctrine and the store wants to withhold disclosure of the report, what should the store do? A Produce all the documents that respond to the request that are not protected and simply not include the report in the documents produced. B File a motion for a protective order with the court before its response to the request for documents is due, to avoid waiving any protection from discovery that it might have had. C Describe the nature of the report so that the customer and her lawyer can assess the validity of the store's claim that the report is protected from discovery. D File the report with the court under seal so that the court may assess whether it is protected from discovery.
C Describe the nature of the report so that the customer and her lawyer can assess the validity of the store's claim that the report is protected from discovery.
The plaintiff was severely injured when a tire on her car blew out and caused her to lose control of the car. She filed an action in federal district court against the manufacturer of the tire, alleging that the tire was defective. The plaintiff intends to introduce remnants of the tire into evidence at trial as part of her proof that the tire was defective. In what way are the remnants subject to discovery? A Discovery is not available for such physical items, only for documents and electronically stored information. B The plaintiff does not have to disclose the existence or a description of the tire remnants unless the manufacturer asks about them in discovery requests. C Even without a request from the manufacturer, the plaintiff must provide the manufacturer with a description of the tire remnants. D The court will order the plaintiff to make the tire remnants available for the manufacturer to inspect only if the manufacturer shows good cause.
C Even without a request from the manufacturer, the plaintiff must provide the manufacturer with a description of the tire remnants.
A plaintiff sued a defendant for serious personal injuries he incurred when the defendant allegedly drove through a red light and collided with the plaintiff's car. Calling the defendant as an adverse witness, the plaintiff asked her if she had been drinking before the accident. The defendant refused to answer, asserting her privilege against self-incrimination. The plaintiff then offers in evidence a certified copy of a court record indicating that, eight years previously, the defendant had been convicted of reckless driving while intoxicated that caused serious personal injury, a felony. How should the trial court rule on the admissibility of the court record? A Admit the record as relevant character evidence because the plaintiff suffered serious personal injuries. B Admit the record as impeachment evidence. C Exclude the record as irrelevant because as yet the defendant has given no testimony to be impeached. D Exclude the record because the conviction is too remote and does not necessarily reflect on the defendant's credibility as a witness in the present proceedings.
C Exclude the record as irrelevant because as yet the defendant has given no testimony to be impeached.
A landowner devised her home "to my daughter for life, then to the eldest survivor of her two children, my grandson and granddaughter, for life, remainder to the eldest surviving offspring of the two grandchildren who is alive at the death of the last life tenant." After the landowner's death, the daughter lived in the family home for 15 years. Upon the daughter's death, both of her children were alive, so the home passed to the grandson, the eldest. He lived in the house for three years, and then conveyed it to the city historical society, which converted it into its headquarters and museum. Eight years later, the grandson died. At the time of his death, he was survived by his widow, his two sons, the granddaughter, and the granddaughter's daughter, who was the eldest of the niece and nephews. Four years after the grandson's death, the granddaughter's daughter brought an action for ejectment and to quiet title against the city historical society. The jurisdiction has a statutory period of adverse possession of 10 years, or five years if entry was made by the adverse possessor under color of title. How should the court rule in the granddaughter's daughter's action? A For the society, because it has occupied the home for the statutory period required for adverse possession. B For the society, because it purchased the home in fee simple absolute from the grandson. C For the granddaughter's daughter, because the society has not been in adverse possession for the requisite period. D For the granddaughter's daughter, because a purchaser of property from a life tenant cannot acquire a fee simple absolute through adverse possession.
C For the granddaughter's daughter, because the society has not been in adverse possession for the requisite period.
While executing a search warrant at the home of a suspected arsonist, the police heard a knock on the door. A plain clothes officer answered the door and found a young woman standing outside with a backpack in her hand. The woman asked for the owner of the home by name and was told that the owner was not available at the moment. The woman replied, "Give him this, and tell him thanks for the $8,000," and she gave the officer the backpack. The officer opened the backpack and found eight hi-tech delay timers. The officer then identified himself as a police officer and placed the woman under arrest. He performed a quick patdown of the woman's outer clothing. In her jacket pocket, the officer found a package that appeared to be (and later proved to be) heroin. In addition to arson-related charges, the woman was charged with possession of heroin. At trial, the woman's attorney moved to have the heroin excluded from evidence. How should the court rule? A For the woman, because the warrant only entitled the police to search the premises. B For the woman, because the heroin was seized during an unlawful arrest. C For the state, because the heroin was seized during a lawful arrest. D For the state, because searching the backpack was within the scope of the warrant.
C For the state, because the heroin was seized during a lawful arrest.
An employee sued her employer in federal court for sexual harassment. The jury returned a verdict in favor of the employer. Three months after the verdict, the employee's attorney received an anonymous letter stating that a key document presented at trial by the employer had been altered. The employee moved for relief from judgment, alleging that a document presented by her employer at trial had been altered. Assuming that the employee can show that the alteration was intentional, how should the court rule? A Deny the employee's motion, because, while fraud and misconduct of an adverse party are proper grounds for relief, such motions must be brought within 28 days of the final judgment. B Deny the employee's motion, because her claim fails to state a proper ground for relief from judgment. C Grant the employee's motion, because fraud and misconduct of an adverse party are proper grounds for relief, and she filed within the appropriate time frame. D Grant the employee's motion, because fraud and misconduct of an adverse party are proper grounds for relief, and there is no time limit on such actions.
C Grant the employee's motion, because fraud and misconduct of an adverse party are proper grounds for relief, and she filed within the appropriate time frame.
An employee sued her employer in federal court for sexual harassment. The jury returned a verdict in favor of the employer. Three months after the verdict, the employee's attorney received an anonymous letter stating that a key document presented at trial by the employer had been altered. The employee moved for relief from judgment, alleging that a document presented by her employer at trial had been altered. Assuming that the employee can show that the alteration was intentional, how should the court rule? A Deny the employee's motion, because, while fraud and misconduct of an adverse party are proper grounds for relief, such motions must be brought within 28 days of the final judgment. B Deny the employee's motion, because her claim fails to state a proper ground for relief from judgment. C Grant the employee's motion, because fraud and misconduct of an adverse party are proper grounds for relief, and she filed within the appropriate time frame. D Grant the employee's motion, because fraud and misconduct of an adverse party are proper grounds for relief, and there is no time limit on such actions.
C Grant the employee's motion, because fraud and misconduct of an adverse party are proper grounds for relief, and she filed within the appropriate time frame.
A homeowner discovered that the siding on his house was defective and had allowed water to enter the structure, causing damage to the wood framing. The homeowner tried for some time to negotiate a settlement with the corporation that the homeowner believed had manufactured the defective siding. When no settlement was forthcoming, the homeowner filed an action in federal district court against the corporation one week before the statute of limitations expired. Service of process was effected on the corporation several months later. After inspecting the home, the corporation filed and served its answer in which it denied manufacturing the siding used on the homeowner's house. Upon examining the corporation's evidence, the homeowner conceded that the siding was manufactured by another company. With leave of the court, the homeowner then filed an amended complaint substituting the actual manufacturer of the siding for the original incorrect defendant. The amended complaint was served on the manufacturer approximately seven months after the original complaint was filed and after the statute of limitations had expired. The manufacturer was unaware of the action until it was served with the amended complaint. The manufacturer filed a motion for summary judgment on the grounds that the homeowner's claim against it is barred by the statute of limitations. How should the court rule on the motion? A Deny the motion, because the homeowner is entitled to recover from the actual manufacturer the damages caused by the defective siding. B Deny the motion, because the amended complaint relates back to the time the original complaint was filed, which was before the statute of limitations expired. C Grant the motion, because the amended complaint was filed after the statute of limitations expired and the actual manufacturer did not receive timely notice of the action. D Grant the motion, because amendments to pleadings may not be used to substitute an entirely new defendant unrelated to the one originally named.
C Grant the motion, because the amended complaint was filed after the statute of limitations expired and the actual manufacturer did not receive timely notice of the action.
An elderly woman filed a complaint in federal district court, alleging that a salesperson "made fraudulent statements that induced the plaintiff to sign the contract to the plaintiff's detriment." The salesperson then filed a motion to dismiss the complaint for failure to state a claim upon which relief can be granted. How should the court rule? A Deny the motion, because the complaint stated a possible claim for relief. B Deny the motion, because the complaint stated a plausible claim for relief. C Grant the motion, because the complaint did not state the claim for relief with particularity. D Grant the motion, because the complaint did not state the claim for relief with probability.
C Grant the motion, because the complaint did not state the claim for relief with particularity. The court should grant the motion to dismiss for failure to state a claim upon which relief can be granted. The federal pleading rules generally require the pleader to assert short and plain statements in the complaint to put the other side on notice of the claim being asserted; detailed assertions of facts underlying the claim are generally not required. However, there are certain special pleading rules that require a party to state more detail under special circumstances, including claims that assert fraud or mistake. Under such special circumstances the federal rules specifically require that a plaintiff assert the claim for relief with particularity. Here, choice (C) is the best answer because it is the only choice to correctly state the rule. The salesperson's motion to dismiss should be granted in this case because the plaintiff's complaint fails to state the claim with particularity. Choices (A), (B), and (D) all misstate the special pleadings rule that applies to complaints asserting fraud and mistake.
Even though the gambling laws of the state prohibit gambling on professional sports games, the defendant placed a bet with a bookie on the outcome of a football game. There was a disputed call near the end of the game that resulted in the defendant losing the bet. However, later films showed that in fact the call should have been for the defendant's team, which would have changed the outcome. The bookie refused to pay the bet to the defendant. Later that night, the defendant broke into the bookie's home and took the amount he would have won. What is the defendant's best defense to a charge of common law burglary? A He was so enraged that he had an irresistible impulse to take the money. B Since the original gambling agreement was illegal, the two are in pari delicto and the court should not interfere. C He lacked the specific intent necessary for burglary because he believed that the bookie owed him the money. D He had a mistaken belief that the definition of burglary would not apply to a dispute over gambling winnings.
C He lacked the specific intent necessary for burglary because he believed that the bookie owed him the money.
A credit card company filed a civil action against a consumer in federal district court seeking to recover the unpaid balance on the consumer's account. The credit card company properly served process on the consumer, but the consumer failed to file or serve a timely answer to the complaint. The clerk of court on motion of the credit card company made an entry of default, but default judgment has not been entered. On which grounds may the court set aside the entry of default? A For the same limited grounds for which any court judgment may be set aside. B Only if the court finds that the consumer could not have filed and served a timely answer despite using reasonable diligence. C If the consumer demonstrates that there was good cause for his failure to file and serve a timely answer and that he has a viable defense. D On any grounds that the court, in its discretion, finds just.
C If the consumer demonstrates that there was good cause for his failure to file and serve a timely answer and that he has a viable defense.
A State A citizen and a State B citizen were in a car accident in State C. The State A citizen filed a negligence action against the State B citizen in a State C state court, seeking $500,000 in damages. If the State B citizen wishes to remove the action to federal district court, in which federal district should the State B citizen file a notice of removal? A In either the district in State C in which the accident took place or the district in State B in which the State B citizen resides. B In either the district in State C in which the accident took place or the district in State A in which the State A citizen resides. C In only the district in State C in which the State C state court is located. D In only the district in State B in which the State B citizen resides.
C In only the district in State C in which the State C state court is located. The notice of removal should be filed in the district in State C in which the State C court is located. The federal removal statute provides that the notice of removal should be filed in the federal district court for the district that geographically encompasses the state court from which the action is being removed. Thereafter, copies must be sent to the parties and filed with the applicable state court. Thus, (A), (B), and (D) are incorrect. (A) describes the general venue rule—i.e., an action may be filed in the judicial district in which any defendant resides (if they are all from the same state) or in which a substantial part of the events took place—for actions filed originally in federal court. (B) incorrectly states that venue may be affixed based on the residence of the plaintiff. This is incorrect even under the general venue rule. For federal venue purposes, the residence of the plaintiff is irrelevant. (D) incorrectly provides one-half of the general venue rule, not the venue rule for removed actions.
A man loaned $10,000 to his good friend who was heavily in debt. He told his friend that he could have until the following June to repay the loan and to make repayment directly to the man's grandson. The man knew that his grandson, who would be graduating from law school in June, was hoping to buy a new car upon graduation and could use the money as a down payment on the car. Neither the man nor his friend told the grandson of this agreement. Before the friend repaid the $10,000 loan, the grandson went out and bought his dream car, cashing some savings bonds to make the down payment. A week later, the grandfather suffered a reversal of fortune and told his friend to pay the $10,000 to him after all, which the friend did. A short time later, the grandson learned of the aborted plan to pay the $10,000 to him. If the grandson sues the friend for the $10,000, will he prevail? A Yes, because he was the intended beneficiary of the contract between his grandfather and the friend. B Yes, because he detrimentally relied on the promise between his grandfather and the friend by purchasing the car. C No, because his rights had not yet vested when the contract was modified. D No, because the man intended the repayment of the $10,000 to the grandson to be a gift.
C No, because his rights had not yet vested when the contract was modified.
A bolt of lightning struck a tree, causing it to fall on a farmer's fence which enclosed a pasture containing the farmer's large bull. The bull escaped through the broken fence and entered the neighbor's property. It gored a hiker who was crossing the neighbor's property without permission. In the hiker's action against the farmer based on strict liability, is the hiker likely to prevail? A Yes, because the bull caused harm while trespassing on another's property. B Yes, because bulls have known dangerous propensities. C No, because a bull is a domestic animal. D No, because the hiker was a trespasser.
C No, because a bull is a domestic animal. The hiker will not prevail because strict liability does not apply to a bull, which is a domestic animal. The owner of a domestic animal, including a farm animal, is not strictly liable for injuries it causes, as long as the owner has no knowledge that the animal has abnormally dangerous propensities (i.e., propensities more dangerous than normal for that species). A bull is a domestic animal, and nothing in the facts suggests that the bull was more dangerous than normal for that type of animal. Hence, strict liability will not apply. (A) is incorrect because the rule for trespassing animals does not apply. The owner of a trespassing animal is strictly liable for harm done by the trespass as long as it was reasonably foreseeable. Here, the bolt of lightning caused the fence to break and allowed the bull to escape. This unforeseeable intervening force was the cause of the trespass; hence, the strict liability rule for trespassing animals does not apply here. (B) is incorrect because, as discussed above, strict liability does not apply for domestic animals with normal dangerous propensities. Only domestic animals with propensities more dangerous than normal for the species may subject the owner to strict liability. (D) is incorrect because the hiker's status as a trespasser on the neighbor's land is irrelevant as to the farmer's liability. If the hiker were a trespasser on the farmer's land, strict liability would not apply even if the bull were abnormally dangerous, but the farmer's liability is not affected by the hiker's status as to the neighbor. Note that if strict liability applied for harm from an animal trespassing on a neighbor's property, the hiker's status as a trespasser might be relevant because strict liability applies only to injured persons who were rightfully on the property. However, as discussed above in (A), that liability is inapplicable here because the bull's trespass was unforeseeable.
A patient properly filed a medical malpractice claim against a doctor in federal district court. The complaint simply asserted negligence as the grounds for relief without any facts supporting the claim. The doctor filed a pretrial motion for a more definite statement, which the court denied. Immediately thereafter, and without submitting an answer, the doctor filed a motion to dismiss, asserting that the court lacked personal jurisdiction. Will the court grant this motion to dismiss? A Yes, because a defendant may file a motion to dismiss based on lack of personal jurisdiction at the time he files a motion or at the time he files his answer, whichever occurs later. B Yes, because the lack of personal jurisdiction defense can be raised at any time, even for the first time on appeal. C No, because a defendant must file a motion to dismiss based on lack of personal jurisdiction at the time he files a motion or at the time he files his answer, whichever occurs first. D No, because a lack of personal jurisdiction defense can only be raised in an answer or it is waived, and the defendant did not submit an answer.
C No, because a defendant must file a motion to dismiss based on lack of personal jurisdiction at the time he files a motion or at the time he files his answer, whichever occurs first.
A locksmith knew that his friend had been having marital troubles. The friend had told the locksmith that he suspected his wife was having an affair with his rival. One afternoon, the friend, visibly upset, asked to borrow some of the locksmith's tools, telling him that he knew that his rival was going to meet up with his wife later that day. The locksmith gave his friend the tools, advising him not to do anything that he would regret later. The friend stated that it would be others who would have regrets. The friend went to his rival's apartment and picked the door lock with the locksmith's tools. He found his wife and rival in bed together. The friend stabbed his rival, seriously wounding him. A few minutes later the locksmith called the apartment to try to warn the rival that his friend might come over. After the friend was arrested, he agreed to plead guilty to aggravated battery and attempted voluntary manslaughter in exchange for testifying against the locksmith, who was charged as an accomplice to attempted murder. Can the locksmith be convicted of that charge? A Yes, because he recklessly disregarded a substantial risk to human life and was not provoked. B Yes, because his failed attempt to neutralize his assistance did not prevent the crime from occurring and therefore did not constitute an adequate withdrawal. C No, because he did not have the requisite intent to be liable as an accomplice. D No, because an accomplice cannot be found guilty of a more serious offense than that for which the principal has been convicted.
C No, because he did not have the requisite intent to be liable as an accomplice. The locksmith cannot be convicted as an accomplice because he did not have the requisite intent for attempted murder. To be convicted as an accomplice under the prevailing rule, a person must have given aid, counsel, or encouragement with the intent to aid or encourage the principal and the intent that the principal commit the substantive offense. Mere knowledge that a crime would result from the aid provided is generally insufficient for accomplice liability. Here, the locksmith did not provide the tools to the friend with the intent that he kill the rival. His knowledge that the friend might be intending harm to the rival is not sufficient to establish the intent to kill required for attempted murder. (A) is incorrect because even if the locksmith's conduct constituted reckless disregard of high risk to human life, that state of mind is not sufficient for attempted murder. Unlike murder, attempted murder is a specific intent crime and requires the intent to kill. (B) is incorrect. Although the locksmith's attempt to neutralize his assistance would not have been enough to raise the defense of withdrawal if he had incurred liability as an accomplice, here he did not have the requisite intent for accomplice liability. (D) is an incorrect statement of law; the degree of liability of a principal is irrelevant to the potential liability of an accomplice. If the locksmith had had the intent to aid his friend in killing the rival, the fact that the friend could show adequate provocation to reduce his offense to attempted voluntary manslaughter would have no effect on the locksmith's liability for attempted murder.
The owner of a valuable painting hired professional movers to transport it to an auction house when she decided to sell it. As the movers were carrying it to their van, a window air conditioner that a tenant had been trying to install fell out of his second floor window and crashed through the painting and onto the ground. The owner had been watching from her apartment across the street and saw her painting destroyed. She became extremely upset and needed medical treatment for shock. If the owner brings a claim for negligent infliction of emotional distress against the tenant, is she likely to recover? A Yes, because she suffered physical symptoms from her distress. B Yes, because she was a foreseeable plaintiff. C No, because she was not within the zone of danger. D No, because she suffered no physical impact.
C No, because she was not within the zone of danger. Because the owner was across the street at the time of impact, she was not within the zone of danger, thus precluding her recovery for negligent infliction of emotional distress. A defendant breaches a duty to avoid negligent infliction of emotional distress when he creates a foreseeable risk of physical injury to the plaintiff through causing a threat of physical impact that leads to emotional distress. Damages generally are recoverable only if the defendant's conduct causes some physical injury, rather than purely emotional distress (although a severe shock to the nervous system that causes physical symptoms is sufficient). If plaintiff's distress is caused by threat of physical impact to her, she must have been within the zone of danger. Here, the owner witnessed the air conditioner striking her painting from across the street. This vantage point placed her outside the zone of danger from the falling air conditioner. Thus, the owner cannot recover for negligent infliction of emotional distress. It is true that, as implied by (A), there usually can be no recovery for this tort absent some accompanying physical consequences. However, even though the owner did suffer physical symptoms from her distress, she cannot recover because she was not within the zone of danger. Therefore, (A) is incorrect. (B) is incorrect because the owner's distance from the accident makes her an unforeseeable plaintiff, because it is unforeseeable that someone on the other side of the street would suffer physical impact or the threat thereof. (D) is incorrect because physical impact is not required for this tort; the threat of impact is enough. Therefore, even in the absence of impact, the owner could recover if she had been within the zone of danger from the tenant's negligence.
Two people agreed to steal a valuable painting that they knew was hanging in the victim's home. One would wait in the car with the engine running to ensure a quick getaway, while the other would break into the victim's home and steal the painting. The burglar broke into the home and reached the victim's library, where the painting was hanging. On the desk he noticed a large vial that appeared to contain cocaine. Thinking he could sell the cocaine and split the proceeds with the getaway driver, the burglar grabbed the vial and stuffed it in his pocket. He then took the painting off the wall and hurried back to the waiting car. The police arrived at that moment and apprehended the pair. A search incident to arrest turned up the vial of cocaine in the burglar's possession. The getaway driver is charged with being an accomplice to the unlawful possession of cocaine with intent to distribute. Will the driver likely be found guilty? A Yes, because she is liable for all crimes resulting from the conspiracy. B Yes, because the conspiracy was to steal items for resale. C No, because the conspiracy did not involve the possession or sale of cocaine. D No, because the burglar was the person who possessed the cocaine.
C No, because the conspiracy did not involve the possession or sale of cocaine.
Police officers were executing a search warrant at a home suspected of containing evidence of illegal gambling. No one was at home when the police arrived. After searching the first floor, the officers went upstairs. A friend of the owner then entered the house carrying a briefcase. He set the briefcase on the floor, opened it, and then heard the officers. He became frightened, left the briefcase sitting in the middle of the floor, and hid in a closet. The police officers returned to the first floor and immediately spotted the briefcase, which they knew was not there earlier. Because the briefcase was open, the officers saw its contents—betting slips—and seized them. Because they knew that someone had entered the house since they arrived, they re-searched the first floor. They found the friend and informed him that he was under arrest, clapped handcuffs on him, and read him his Miranda warnings. One of the officers patted the friend down to check for weapons. The officer noticed a bulge in the friend's pocket. Although the officer knew that the bulge was unlikely to be a weapon, he reached into the pocket anyway, and discovered a package that appeared to be (and later proved to be) heroin. The friend was charged with possession of narcotics. At a suppression hearing, will the court agree with the public defender's contention that the friend's arrest was unlawful? A Yes, because the police officer who searched the friend knew that he did not have a weapon in his pocket. B Yes, because the friend's mere presence in the house did not give the police probable cause to believe he had committed a crime, and they had no basis for searching him at all, because he did not act toward them in a threatening manner. C No, because the contents of the briefcase gave the police probable cause to arrest the man. D No, because the police had a right to search the friend for gambling slips, and the discovery of the heroin was merely incidental to a lawful search.
C No, because the contents of the briefcase gave the police probable cause to arrest the man.
A boy planned to rob the local currency exchange, but he needed a getaway car and somebody to be a lookout. He asked his sister, who immediately refused and warned him against carrying it out. The boy showed his sister a gun, threatening to shoot her if she did not go along with the plan. The sister consented to help, and the pair left for the currency exchange, with the sister driving. When they arrived, the sister left the car running so that she could get away and contact the police while her brother was inside, but the boy ordered her to go inside with him. During the robbery attempt, the clerk pulled out a gun, and the boy and the clerk shot and killed each other. If the sister is charged with the killing of the store clerk, should the jury find her criminally liable if it accepts the sister's version of the facts? A Yes, because duress is not a defense to murder. B Yes, because the clerk's death occurred in furtherance of and during the perpetration of a robbery in which the sister was participating. C No, because the sister lacked the requisite intent. D No, because the sister withdrew from the conspiracy to commit robbery and thus is not liable for any acts committed in furtherance of the conspiracy.
C No, because the sister lacked the requisite intent.
A foreign student who had entered the United States on a student visa four years ago was notified by federal immigration authorities that he was subject to being deported because his visa had expired. Federal law provided that an alien who is subject to being deported has the right to appear before an administrative officer appointed by the Attorney General's office for a hearing on whether he should be deported. This officer, appointed by the executive branch of the government, has the right under law to make a final order concerning whether the alien should be deported. After a hearing, the administrative officer entered an order allowing the student to remain in the United States as a permanent resident. However, a congressional rule permitted the House of Representatives, by resolution, to deport "undesirable aliens." After the administrative judge entered his order, the House passed a resolution that the student should be deported. The student petitioned the federal court to declare the legislative resolution invalid. Should the court find the resolution to be valid? A Yes, because Congress has plenary powers with regard to aliens and naturalization. B Yes, because aliens are not "citizens" within the meaning of the Fourteenth Amendment. C No, because the federal law removed congressional power with regard to aliens in this circumstance, and the resolution of the House violates the separation of powers doctrine. D No, because the student was denied due process when he was not given a hearing before the House of Representatives.
C No, because the federal law removed congressional power with regard to aliens in this circumstance, and the resolution of the House violates the separation of powers doctrine. The court should find the resolution invalid. While Congress has broad power to delegate, the separation of powers doctrine forbids Congress from trying to control the exercise of the power delegated in various ways, such as by overturning an executive agency action without bicameralism (i.e., passage by both houses of Congress). By enacting the federal law allowing the administrative law judge to enter a final order with regard to aliens, Congress has given up any control it may have had previously in these situations. The resolution by the House here is an unconstitutional legislative veto that violates the separation of powers doctrine. (A) is incorrect because, while Congress does have plenary power over aliens with regard to immigration and naturalization, here it has given up control over this area by enacting a law allowing an administrative officer appointed by the executive branch to make a final order concerning whether an alien should be deported. (B) is incorrect because the fact that aliens are not citizens has no bearing on whether the House resolution violated the Constitution. (D) is incorrect because, while resident aliens are entitled to notice and hearing before they can be deported, the student did receive a hearing before the administrative officer. There is no requirement that persons affected by legislative action have the right to be heard by the legislative body taking the action. Thus, the better argument as to why the resolution was invalid is based on separation of powers.
A builder from State A sued a homeowner from State B for breach of contract in federal court, alleging that the homeowner failed to pay the second half of the agreed-upon price for completion of construction on a house. The process server attempted to serve the summons and complaint on the homeowner at the house, but after two failed attempts, instead served it on an elderly next-door neighbor, who was out in the front yard and volunteered to accept it on the homeowner's behalf. The neighbor gave the homeowner the summons and complaint later that day. The homeowner then filed a motion to dismiss for insufficient process. Assume that the requirements for service of process in both states are identical to the requirements of the Federal Rules of Civil Procedure. Is the court likely to dismiss the action for insufficient process? A Yes, because the homeowner did not designate the neighbor as an authorized agent to receive service of process. B Yes, because the neighbor is not considered one residing at the homeowner's place of abode, even though the neighbor is of suitable age. C No, because the homeowner filed the wrong motion. D No, because the homeowner has no meaningful defense for insufficient service of process since she had actual notice.
C No, because the homeowner filed the wrong motion. The court will not likely dismiss the action. Insufficient process refers to defects in the documents and their content, whereas insufficient service of process refers to the manner in which the documents were presented, delivered and/or served to the defendant. The key to this answer is reading comprehension. The facts state that the homeowner filed a "motion to dismiss for insufficient process," not "insufficient service of process," which are two separate grounds for dismissal. [See Fed. R. Civ. P. 12(b)(4) and (5), respectively] (A), (B) and (D) are all wrong because they all relate to issues regarding insufficient service of process, which is not relevant to the call of the question. (C) is correct, because the homeowner filed the wrong motion. The homeowner should have filed a motion to dismiss based on insufficient service of process. Generally speaking, service of process is sufficient if it is made by: (i) personal service, (ii) service left at the defendant's usual place of abode with one of suitable age and discretion, or (iii) service upon an authorized agent of the defendant. Alternatively, service may be made under state rules or by mail under the waiver of service provision [see Fed. R. Civ. P. 4(d)].
A truck transporting explosives went out of control when a tire suddenly blew. The truck struck a motorist's car as it was waiting at a stoplight, seriously injuring the motorist. The area around the accident was immediately evacuated, but fortunately the explosives were not detonated. In an action alleging strict liability against the freight carrier that owned the truck, the motorist established the above facts and presented evidence of her injuries. The carrier presented evidence that the blowout was caused by a hidden defect in the tire that could not be detected by routine inspection. The tires were manufactured by the carrier's regular supplier and had not previously caused any problems. The carrier also presented evidence that the local authorities were supposed to restrict access to roads along the truck's route but had failed to do so. In this action, is the motorist likely to prevail? A Yes, because the tire was in a defective condition that made it unreasonably dangerous. B Yes, because the freight carrier was engaged in an abnormally dangerous activity. C No, because the injury did not arise from the dangerous propensity of the activity. D No, because the negligence of the local authorities in failing to restrict access to roads along the truck's route was a superseding cause of the motorist's injuries.
C No, because the injury did not arise from the dangerous propensity of the activity. The motorist is not likely to prevail in a strict liability action because her injury did not arise from the abnormally dangerous propensity of the freight carrier's activity. The carrier's transport of explosives likely qualifies as an abnormally dangerous activity because: (i) it creates a foreseeable risk of serious harm even when reasonable care is exercised by all actors; and (ii) the activity is not a matter of common usage in the community. However, the scope of liability extends only to the dangers that would be anticipated from the activity involved; strict liability does not apply to harms that were not caused by the normally dangerous propensity of the activity. Here, the carrier's activity is subject to strict liability because of the danger of explosion from the truck's cargo, but not from a crash by itself. Because the motorist's injuries were not caused by an explosion, strict liability does not apply. The carrier would be liable for the injuries from the crash only if the motorist established negligence. (A) is incorrect because it states the standard for strict liability for defective products, which requires that the defendant be a commercial supplier of the defective product, i.e., someone in the chain of distribution of the product. Here, the carrier is not a commercial supplier of the tire that failed; rather, it is the purchaser or consumer of the tire. (B) is incorrect because, as discussed above, strict liability does not apply to the harm that occurred here. (D) is incorrect. It is questionable whether the negligence by the local authorities could be considered an intervening force, which must come into play after the culpable conduct by the defendant. Even if it were an intervening force, it likely would not be considered so extraordinary as to be an unforeseeable intervening force. Hence, it would not constitute a superseding force that would break the causal connection between the motorist's injury and the carrier's actions.
Congress enacted a statute appropriating money to the states on condition that the states use the money to support "public performances of classical ballet open to the public." The statute provided that the money was not to be used to support any other type of dance, and that tickets to any performance paid for with these funds were to be distributed to the public on a first come, first served basis. A state that accepted a grant of $500,000 under the federal statute gave half of the grant to a state-sponsored ballet company. The company had been started 20 years earlier as part of a state effort to bring culture to poor, inner-city areas. By state law enacted when the company was formed, no less than 35% of the tickets to each performance of the ballet company must be distributed to the inner-city school systems to be given to minority school children. Is the state's method of distributing tickets to the state ballet company's performances constitutional? A Yes, because the state ballet company is state-operated and the doctrine of federalism prohibits the federal government from directly interfering with state operations. B Yes, because the state ballet ticket distribution system substantially conforms with the underlying purpose of the federal ticket distribution scheme. C No, because the state distribution system violates the Supremacy Clause. D No, because the state distribution system violates equal protection.
C No, because the state distribution system violates the Supremacy Clause. The state ticket distribution system is unconstitutional because of the Supremacy Clause. A valid act of Congress supersedes any state or local action that conflicts with it. The act here is valid because Congress has the power to spend for the general welfare, and in so doing may place conditions on grants as it sees fit. The state law directly conflicts with the federal law because the federal law requires that tickets be distributed on a first come, first served basis, and the state law requires that 35% of the tickets be given to minority school children. Because the state law conflicts with the federal law, it is invalid. (A), based on the Tenth Amendment, is incorrect because, even if Congress lacks the power to directly regulate the distribution of the tickets in question, the regulation here would still be valid as a spending power condition. The Supreme Court has held that Congress may condition grants under the spending power even where it cannot directly regulate, as long as the conditions are (i) clearly stated, (ii) related to the purpose of the program, and (iii) not unduly coercive. [See South Dakota v. Dole (1987)—conditioning federal highway grants on prohibiting minors from drinking] (B) is incorrect because the Supremacy Clause invalidates all conflicting state laws where there is a clash, no matter how complementary the state law may be viewed. (D) is incorrect because the state program probably is valid under the Equal Protection Clause. State programs that favor racial and ethnic minorities are subject to the same strict scrutiny standards as programs that discriminate against minorities: They must be narrowly tailored to promote a compelling government interest. There is a compelling government interest in remedying past discrimination, and the facts indicate that the ballet company was established to remedy the prior lack of cultural opportunities that existed in the inner city. The program also appears to be narrowly tailored, and so would likely survive an equal protection challenge.
A man on parole after being convicted of possession of cocaine was suspected of selling cocaine out of his home. His parole officer came to his house and rang the bell. As soon as the man opened the door to see who was there, the officer entered the home, despite the man's protests. After searching the home, the parole officer discovered several bags of marijuana in a drawer. The man was arrested and charged with possession of marijuana with intent to sell. A statute in the jurisdiction in which the search took place provides that, as a condition of parole, a parolee is on notice that his parole officer may conduct a search of the parolee's person or home, without probable cause, at any time of the day or night. The man moved to have evidence of the marijuana suppressed by the court, claiming that the state statute that authorized the search was unconstitutional under the Fourth Amendment prohibition of unreasonable searches and seizures. Will he prevail? A Yes, unless probable cause was established by the officer's tip in conjunction with other circumstances. B Yes, because a search warrant was not obtained and no exception to the warrant requirement applies. C No, because the man had a diminished expectation of privacy and the government has a heightened need to search parolees' homes. D No, because the search was incident to a lawful arrest.
C No, because the man had a diminished expectation of privacy and the government has a heightened need to search parolees' homes.
Acting with probable cause, the police arrested a man in connection with the armed robbery of a liquor store. After being given Miranda warnings, the man confessed to the robbery but denied his involvement with several other recent armed robberies of businesses in the area. He was formally charged with the one robbery and put into a cell with a paid informant working undercover for the police. The informant had been instructed to find out what he could about the other robberies but not to ask any questions. The informant began talking about a convenience store robbery in which a bystander was shot and seriously injured by the robber, and he deliberately misstated how it happened. The man, unaware that his cellmate was an informant, interrupted to correct him, bragging that he knew what really happened because he was there, and proceeded to make incriminating statements about the robbery. The man was subsequently charged with armed robbery and attempted murder in the convenience store robbery. At a motion-to-suppress hearing on that charge, if the man's attorney moves to exclude the statements made to the informant, should the motion be granted? A Yes, because the informant deliberately elicited incriminating statements in violation of the man's Sixth Amendment right to counsel. B Yes, because the informant's conduct constituted custodial interrogation in violation of the man's Fifth Amendment privilege against self-incrimination. C No, because the man had not yet been charged with the robbery of the convenience store when he made the statements to the informant. D No, because the informant's conduct did not constitute interrogation.
C No, because the man had not yet been charged with the robbery of the convenience store when he made the statements to the informant. The man's motion should be denied because neither his Fifth nor Sixth Amendment rights were violated by the informant's conduct. The Sixth Amendment right to counsel applies to all critical stages of a criminal prosecution after formal proceedings have begun, but does not apply in precharge custodial interrogations. Because this right is "offense specific," the fact that the right to counsel has attached for one charge does not bar questioning without counsel for an unrelated charge. Because the man has not been charged with the convenience store robbery, his Sixth Amendment right to counsel has not been violated. The Fifth Amendment privilege against self-incrimination requires Miranda warnings and a valid waiver before any statement made by the accused during custodial interrogation can be admitted. However, this requirement does not apply where interrogation is by an informant who the defendant does not know is working for the police, because the coercive atmosphere of police-dominated interrogation is not present. [Illinois v. Perkins (1990)] Because the man was not aware of the informant's status, the informant's conduct did not constitute a police interrogation. (A) is wrong despite the fact that the informant's conduct may have been deliberately designed to elicit incriminating remarks. As discussed above, the man's right to counsel did not attach for purposes of the convenience store robbery. (B) is incorrect because, as discussed above, the Miranda warnings need not be given before questioning by a cellmate working covertly for the police. (D) is incorrect because interrogation refers not only to express questioning, but also to any words or actions on the part of the police that the police should know are reasonably likely to elicit an incriminating response from the suspect. Here, the informant, working for the police, made statements about the convenience store robbery that were intended to, and reasonably likely to, prompt a response from his cellmate. Hence, it is not the absence of "interrogation" that avoids the Miranda problem, but the fact that the man did not know that his cellmate was working for the police.
A driver was operating her car on a city street when she was stopped by a police officer for speeding. As the police officer reached the driver's car, he saw her put something into her purse. The officer told the driver, "Ma'am, you were speeding; that's why I stopped you. I'd like your driver's license, and, by the way, what did you just put into your purse?" The driver responded, "It's just a marijuana cigarette, but don't worry, I've only had two and my driving judgment hasn't been impaired." The officer took her purse, removed the "joint," and charged the driver with possession of marijuana as well as speeding. At the driver's trial for marijuana possession, the prosecution seeks to introduce the marijuana cigarette into evidence. The driver's attorney moves to suppress the evidence. Should the defense motion be granted? A Yes, because the cigarette is fruit of the poisonous tree. B Yes, because the police officer did not have a valid search warrant. C No, because the police officer's asking about the contents of the driver's purse did not constitute custodial interrogation. D No, provided the police officer had a reasonable suspicion of criminal activity.
C No, because the police officer's asking about the contents of the driver's purse did not constitute custodial interrogation.
After a single vehicle accident, the passenger filed a negligence action in federal district court against the driver to recover for a whiplash injury allegedly suffered in the accident. On the advice of his attorney, the passenger consulted and retained five physicians in search of one who would serve as an expert witness on his behalf at trial. Four of the physicians determined that the passenger had suffered no injury. Obviously, the passenger does not intend to use those four physicians as witnesses at trial. May the driver obtain in discovery the opinions of the four physicians whom the passenger does not intend to have testify at trial? A Yes, the passenger must disclose in his required disclosures the identity of the four physicians and provide a written report concerning their qualifications and opinions. B Yes, the driver may obtain information concerning their opinions through interrogatories, but the driver may not depose the four physicians. C No, the driver will not be permitted to discover the facts known and opinions held by these four physicians unless the driver can demonstrate exceptional circumstances under which it is impracticable to obtain facts or opinions on the same subject by other means. D No, the parties may not under any circumstances obtain discovery of opinions held by experts retained by another party when the other party does not intend to have the expert testify at trial.
C No, the driver will not be permitted to discover the facts known and opinions held by these four physicians unless the driver can demonstrate exceptional circumstances under which it is impracticable to obtain facts or opinions on the same subject by other means.
At a products liability trial, a critical issue is whether the temperature was below freezing on January 16. A local man who works for a civil engineering firm is also an avid amateur meteorologist. One of the man's weather detection instruments in his backyard records temperature by markings from a stylus on a round barograph. The man's record of the day in question indicates that it was unseasonably warm and that the temperature never fell lower than 48 degrees Fahrenheit, 16 degrees above the freezing mark. The plaintiff offers into evidence the man's barograph record of the temperature on January 16. Is the barograph record admissible? A Yes, as past recollection recorded. B Yes, as a record of regularly conducted activity. C No, unless foundation testimony is given as to the accuracy and good working order of the man's instrument on the day in question. D No, because it is hearsay not within any recognized exception to the hearsay rule.
C No, unless foundation testimony is given as to the accuracy and good working order of the man's instrument on the day in question.
An antiques purchaser who did not speak English sued a dealer for breach of contract, alleging that he had agreed to sell her an antique chair for $15,000 but had refused to accept her certified check when she came to pick up the chair. At the trial, the purchaser, through an interpreter, testified that she asked her brother to communicate to the dealer her offer to purchase the chair. She wishes to testify that her brother told her, "The dealer has agreed to sell you the chair for $15,000." The agreement was not reduced to writing and the brother died a few days after that conversation. If the jurisdiction has a typical "Dead Man Act," what effect will the Act have upon the admissibility of the purchaser's conversation with her brother? A It will render the conversation inadmissible because a civil action is involved. B It will render the conversation inadmissible because the purchaser is an interested party. C None, because the dealer is not a protected party. D None, because a civil action is involved.
C None, because the dealer is not a protected party. The Dead Man Act will have no effect on the admissibility of the purchaser's conversation with her brother because the dealer is not a protected party. A typical Dead Man Act provides that a party or person interested in the event, or her predecessor in interest, is incompetent to testify to a personal transaction or communication with a deceased when such testimony is offered against the representative or successor in interest of the deceased. Such statutes are designed to protect those who claim directly under the decedent from perjured claims. Here, the dealer is not a representative or successor in interest of the brother, such as an executor, administrator, heir, legatee, or devisee. Therefore, the dealer is not a protected party for purposes of a Dead Man Act. Because the testimony of the purchaser is not being offered against a representative or successor in interest of the decedent (her brother), the Dead Man Act is inapplicable. Regarding (A), it is true that the bar to competency created by a Dead Man Act applies only to civil cases. However, the mere fact that a civil action is involved will not trigger applicability of a Dead Man Act. As explained above, the absence of someone who is deemed to be a protected party will prevent a Dead Man Act from having any effect. Thus, (A) is incorrect. Regarding (B), it is true that the purchaser is an interested party (i.e., she stands to gain or lose by the direct and immediate operation of a judgment in this case). Nevertheless, (B) is incorrect because a Dead Man Act requires not only an interested person but a protected party. As has been noted, the dealer is not a protected party. (D) is incorrect because it is based on the assumption that a Dead Man Act does not apply to civil cases. In fact, such statutes apply only to civil cases, and not to criminal cases.
A man was tried in state court for possession of heroin. The prosecution offered in evidence five rolled-up toy balloons containing heroin, which police officers had found on a table in the man's apartment. At a hearing on the defense's motion to suppress, testimony was presented that established that the police had put the apartment under surveillance and had watched a police informant go to the door of the apartment, hand four balloons of heroin to the man, and leave. The police had then knocked on the apartment door, identified themselves as police officers, and demanded entrance. Having heard nothing for 30 seconds, the police had then broken down the door and entered the apartment, discovering the heroin. The police had intended to arrest the man for the purchase of heroin, a felony. When they had gotten inside the apartment, they discovered that the man had left by a back exit. He was later arrested at the nearby newsstand. The trial court denied the motion to suppress, and the case is on appeal following the man's conviction for possession of heroin. How should the appellate court rule? A Affirm the conviction on the ground that the error, if any, in admitting the heroin was harmless error. B Affirm the conviction on the ground that the police complied with the "knock and announce rule" even though no one was there to admit them. C Reverse the conviction on the ground that the man's Fourth Amendment rights (as applied to the states by the Fourteenth Amendment) have been violated. D Reverse the conviction on the ground that the "knock and announce rule" was not satisfied when the police announced their presence and identity to an empty residence.
C Reverse the conviction on the ground that the man's Fourth Amendment rights (as applied to the states by the Fourteenth Amendment) have been violated.
To reduce deer overpopulation in state forests, state Blue adopted a statute allowing anyone with a valid deer hunting license from any state to hunt deer within state Blue. The act also imposed a $0.25 per pound tax on each deer killed within the state. Funds from the tax were earmarked to support state forest land. State Red is adjacent to state Blue and also has an overabundance of deer. To encourage hunting, state Red does not impose a tax on deer taken from its forests. A hunter who is a resident of state Red and who is licensed to hunt there earns his living by supplying wild game to several high-end restaurants in state Red. While legally hunting deer within state Red, the hunter inadvertently crossed the state line and killed a deer in state Blue. Upon hearing the hunter's shot, a state Blue game warden arrived at the scene, approximated the weight of the kill, and handed the hunter a tax bill based on the approximation. The bill provided a method for challenging the approximated weight of the deer, but the hunter refused to pay any tax on his kill. He instead filed suit in federal court to enjoin collection of the state Blue tax on constitutional grounds. Which of the following results is most likely? A The hunter will prevail because the tax is invalid under the Commerce Clause. B The hunter will prevail because the tax is invalid under the Interstate Privileges and Immunities Clause of Article IV, Section 2. C State Blue will prevail because the tax is valid under the Commerce Clause. D State Blue will prevail because the tax is valid under the Import-Export Clause.
C State Blue will prevail because the tax is valid under the Commerce Clause. State Blue will prevail because the tax is valid under the Commerce Clause. A tax is valid under the Commerce Clause if: (i) the tax does not discriminate against interstate commerce; (ii) there is a substantial nexus between the activity taxed and the taxing state; (iii) the tax is fairly apportioned; and (iv) the tax fairly relates to services or benefits provided by the state. The state of Blue tax is applicable equally to residents of Blue and nonresidents. Thus, there is no discrimination against interstate commerce. Because the taxed deer are taken from within the state, there is a substantial nexus between the activity taxed and the taxing state. There is fair apportionment if a tax is based on the extent of the taxable activity or property in the state. Here, the killing of a deer within state Blue obviously occurs entirely within the state. Thus, the state tax is fairly apportioned. Also, there is a fair relationship between the tax and any benefits provided by the taxing state, because the state is permitting those engaged in hunting to take deer from its forest lands, in return for a rather modest amount of $0.25 per pound. That revenue, in turn, is used to support state Blue forest land, which provides hunters with a place to hunt. Thus, the state tax meets all of the requirements for validity under the Commerce Clause. (A) incorrectly states that the tax is invalid under the Commerce Clause. (B) is incorrect because the Interstate Privileges and Immunities Clause prohibits discrimination by a state against nonresidents when such discrimination involves fundamental rights, such as those involving important commercial activities. Here, while the tax may affect the hunter's commercial activity (because he earns a living from hunting and selling meat), the tax treats residents and nonresidents equally. Thus, there is no constitutional violation under the Privileges and Immunities Clause. (D) is incorrect because the Import-Export Clause applies to the authority of a state to tax foreign commerce. This question here does not deal with imported or exported goods. Thus, the Import-Export Clause is inapplicable to these facts.
To provide jobs for its citizens, stimulate future tourism, and help the environment, a state legislature enacted a statute authorizing the state's department of parks and recreation to hire up to 5,000 persons to plant trees on land in the state that has been denuded of trees by overlogging. Among other things, the statute provides that resident aliens may be employed only if no United States citizens are available to fill the necessary positions. In a challenge to the constitutionality of that provision by a plaintiff with standing to raise the claim, which of the following constitutional provisions would be most helpful to the plaintiff? A The Privileges or Immunities Clause of the Fourteenth Amendment. B The reserved powers of the state under the Tenth Amendment. C The Equal Protection Clause of the Fourteenth Amendment. D The Fourteenth Amendment Due Process Clause.
C The Equal Protection Clause of the Fourteenth Amendment.
The United States was involved in a dispute with a small island nation over the ownership of an archipelago. On discovering that the archipelago was rich in oil, the President announced that he would appoint an ambassador to negotiate a treaty with the island nation to jointly exploit the oil reserve. A majority of Senators believed that the island clearly belonged to the United States and did not want to negotiate with the island nation. They passed a resolution requiring the President to include a Senator in his diplomatic mission to ensure that the Senate's view was presented in any negotiation with the island nation. What is the strongest constitutional ground for the President's refusal to do so? A As commander in chief, the President has the exclusive power to determine how to protect our national interest abroad. B The resolution is unreasonable because it includes a Senator and not any Representatives. C The President has the exclusive power to select diplomatic representatives of the United States. D The Senate, if it does not like the President's actions, can refuse to appropriate the necessary monies for the President to implement his policies.
C The President has the exclusive power to select diplomatic representatives of the United States. The President's strongest argument is that the power to select ambassadors is vested by the Constitution in the President, and the Senate's only power in this respect is to advise and give (or withhold) its consent. The Senate is not given the power to force ambassadors on the President. (A) is not a strong argument because the President's power as commander in chief is not involved here. That power involves the President's role as the supreme military leader, and military issues are not involved under the facts. (B) is not a strong argument because as far as foreign relations are concerned, the Senate does have more powers than the House. As stated above, ambassadors may be selected only with the advice and consent of the Senate, and the President's treaty power is also similarly limited. Thus, but for the fact that the resolution is not within the Senate's power to enforce, it would be appropriate to exclude the House from participating, because foreign affairs are involved. (D) may be a true statement, because Congress controls appropriations, but it is not a strong argument because it merely states that the Senate has another remedy (i.e., besides forcing an ambassador on the President), and the fact that the Senate has another method for achieving its goals has no bearing on whether its action here is permissible.
A snowmobiler, who lives in State A, hit a skier with a snowmobile while on vacation in the skier's home state of State B, causing damages in excess of $80,000. Although the skier has never been to State A, he sued the snowmobiler in State A federal court. Which of the following statements is true regarding the skier's State A lawsuit? A The State A federal court lacks personal jurisdiction over the parties because the skier has insufficient contacts with State A. B The State A federal court is an improper venue because the claim did not arise there. C The State A federal court may exercise personal jurisdiction over the parties because the skier consented to such personal jurisdiction. D The skier waived his claim against the snowmobiler by filing it in the wrong forum.
C The State A federal court may exercise personal jurisdiction over the parties because the skier consented to such personal jurisdiction.
A hardware store ordered 200 cans of wood stain in various shades. The written contract between the store and manufacturer provided that 100 cans of stain would be delivered on April 30, and the remaining 100 cans would be delivered on June 30. Payment would be due at the time of each delivery. The first shipment arrived on April 30. Sales of the stain were brisk, but 25 customers almost immediately returned their stain, complaining that it was not the color indicated on the can. The store owner called the manufacturer and informed it of the problem. The manufacturer truthfully told the owner that they had had a small problem with their labeling machine and a few cans in the store owner's lot must have been mislabeled before they caught the problem. The manufacturer offered to replace all 100 cans from the original order. The store owner refused the offer and told the manufacturer not to deliver the second lot, because he could no longer trust the manufacturer. The owner was very sensitive to the hardware store's good reputation, which he felt was harmed by this incident. If the manufacturer brings a claim of breach regarding the second shipment which was due on June 30, how will the court likely rule? A The buyer had the right to cancel the second shipment, because of legitimate fears that it would contain the same defects as the first shipment. B The buyer had the right to cancel the second shipment, because the first delivery was defective. C The buyer did not have the right to cancel the second shipment, because the defects in the first shipment did not substantially impair the value of the entire contract. D The buyer did not have the right to cancel the second shipment, because he failed to make a demand upon the manufacturer for adequate assurances that the second shipment would be free of defects.
C The buyer did not have the right to cancel the second shipment, because the defects in the first shipment did not substantially impair the value of the entire contract. The buyer did not have the right to cancel the second shipment, because the defects in the first shipment did not substantially impair the value of the entire contract. This case involves an installment contract, i.e., the contract authorizes or requires deliveries in separate lots, and the sale of goods, so Article 2 of the UCC applies. Under Article 2, a buyer may declare a total breach of an installment contract only if the defect substantially impairs the value of the entire contract. [UCC §2-612] The problem with the first shipment of the stain was discovered and corrected by the manufacturer. The manufacturer offered to cure the defect in the first shipment. In whole, the defect in the first shipment did not substantially impair the value of the entire contract. (A) is incorrect because legitimate fears, alone, are not enough to justify anticipatorily repudiating a contract, as the buyer did here. Anticipatory repudiation occurs when a promisor, prior to the time set for performance of his promise, indicates that he will not perform when the time comes. If, as here, the promisor is unsure of whether the other party will fulfill his contract obligations, the promisor may seek adequate assurances that performance will be forthcoming. In this case, the manufacturer already provided the buyer with such assurances, which fell on deaf ears. Because the manufacturer assured the buyer and, more importantly, had already corrected the problem at its factory before the second shipment was due, the store owner's fears would not justify canceling the second shipment. (B) is incorrect because, as discussed above, the defects in the first delivery would not warrant cancellation of the entire installment contract unless they substantially impaired the value of the entire contract, which they did not. (D) is incorrect because, as explained, the manufacturer had already given adequate assurances to the buyer.
A cyclist was injured when a driver ran a red light. The cyclist subsequently sued the driver to recover for her injuries, and obtained a money judgment of $50,000. The state where the cyclist and the driver reside has the following statute: "Any judgment properly filed shall, for 10 years from filing, be a lien on the real property then owned or subsequently acquired by any person against whom the judgment is rendered." The cyclist filed the judgment in the county where the driver owned a valuable ranch. Sometime later, the driver, who was also injured in the accident, undertook to remodel all the buildings on the ranch to make them wheelchair-accessible. The driver borrowed $30,000 from a bank for the improvements, securing the loan with a mortgage on the ranch. The bank properly recorded its mortgage. Before he paid any principal on the bank's loan, the driver decided to build a new barn. He borrowed $20,000 from a financing company for this purpose, also secured by a mortgage on the ranch. The financing company properly recorded its mortgage. The driver subsequently defaulted on the bank's mortgage, and the bank brought a foreclosure action, joining the financing company in the proceeding. The foreclosure sale resulted in $90,000 in proceeds after all expenses and fees were paid. The driver still owes the cyclist $50,000, the bank $30,000, and the financing company $20,000. How should the foreclosure proceeds be distributed? A The cyclist is entitled to $50,000, the bank is entitled to $30,000, and the financing company is entitled to the remaining $10,000. B The cyclist is entitled to $50,000, the bank is entitled to $30,000, and the driver is entitled to the remaining $10,000. C The bank is entitled to $30,000, the financing company is entitled to $20,000, and the driver is entitled to the remaining $40,000. D The bank is entitled to $30,000, and the driver is entitled to the remaining $60,000.
C The bank is entitled to $30,000, the financing company is entitled to $20,000, and the driver is entitled to the remaining $40,000. The bank is entitled to $30,000 of the foreclosure proceeds, the financing company is entitled to $20,000 of the proceeds, and the driver is entitled to the $40,000 balance. When an interest is foreclosed, after the expenses and fees are paid, the proceeds of the sale are first used to pay the principal and accrued interest on the loan that was foreclosed, next to pay off any junior liens, and finally any remaining proceeds are distributed to the mortgagor. Here, there are enough proceeds to satisfy the bank's (the foreclosing party's) $30,000 mortgage and the financing company's (the junior lienor's) $20,000 mortgage. The remaining balance ($40,000) is distributed to the driver (the mortgagor). (A) and (B) are wrong because the cyclist's interest, an interest senior to the bank's, is not affected by the foreclosure. Although foreclosure destroys all interests junior to the mortgage being foreclosed, it does not affect any senior interests. The buyer at the foreclosure sale takes subject to such interests. Without the cyclist foreclosing her lien, she is not entitled to a share of the proceeds, and her lien continues on the property in the buyer's hands. (B) and (D) are wrong because the financing company is entitled to have its mortgage fully discharged.
A grandfather told his granddaughter that she could have his house because he was moving to a retirement home, and entered into a valid contract to convey it to her. He promised her that he would have another wing added to the house in the back before turning it over to her, and entered into a written contract with a builder to construct the addition for his granddaughter. Before the grandfather had entered into the contract with the builder, the granddaughter had paid $5,000 for a 60-day option to purchase another house because she was not sure she would like the addition. However, when her grandfather showed her the plans for his house prepared by the builder, she liked it very much and decided to let her option to purchase the other house lapse. Shortly thereafter, the local zoning authority increased the minimum lot line setbacks, making it impracticable to put the addition on the back of the house. The builder offered to put an addition above the existing floor rather than in the back, and the grandfather agreed. After the granddaughter's option had lapsed, she discovered that the addition was now going up rather than in the back. She angrily demanded that the builder either build the addition according to the original specifications that she approved or pay her damages. The builder refused and the granddaughter filed suit. Who is more likely to prevail? A The granddaughter, because she was an intended beneficiary of the contract whose rights had vested. B The granddaughter, because the subsequent agreement between her grandfather and the builder to modify the construction was unsupported by consideration. C The builder, because he may raise all defenses that he had against the grandfather against the granddaughter. D The builder, because the granddaughter is merely an incidental beneficiary of the contract between the grandfather and the builder and, as such, has no power to enforce the contract against the builder.
C The builder, because he may raise all defenses that he had against the grandfather against the granddaughter. The builder will prevail because he may raise all defenses that he had against the grandfather against the granddaughter. The granddaughter is an intended third-party beneficiary of the contract between the grandfather and the builder. Generally, a third-party beneficiary has rights under the contract as soon as she does something to vest her rights (manifests assent to the promise, brings suit to enforce the promise, or materially changes position by justifiably relying on the promise). Here, the granddaughter materially changed her position by justifiably allowing her option on the other house to lapse. Generally, once the third-party beneficiary's rights have vested, the original contracting parties may not modify the contract without the assent of the third-party beneficiary. However, the third-party beneficiary is subject to any defenses that the promisor could have used against the original promisee, and here the builder could have used the defense of impracticability against the promisee. Therefore, he could use that defense against the granddaughter to avoid having to pay damages for not building the house as he originally agreed. (A) is incorrect because although it is true that the granddaughter's rights had vested, the answer fails to take into account the defenses available to the builder. (B) is incorrect because there was consideration for the modification. Because the original contract was impracticable to perform, the builder would have been discharged. By agreeing to build the addition above, he undertook something that he was not otherwise bound to do. Likewise, because of the impracticability, the grandfather would have been discharged from his original contract to pay. (D) is incorrect because the granddaughter is not an incidental beneficiary; rather, she is an intended beneficiary because she was specifically mentioned in the contract as the recipient of the house.
A photographer borrowed $100,000 from a bank, secured by a mortgage on his home, to build a studio and darkroom in the home. The bank properly recorded the mortgage. After completing this project, the photographer decided to remodel his kitchen and borrowed $25,000 from a lending company, also securing the loan with a mortgage on his home. The lending company did not record its mortgage. After the remodeling was complete, the photographer borrowed $15,000 from an investor, secured by a mortgage on his home, to redo his in-ground pool. Learning of this transaction, the lending company raced to the recording office and recorded its mortgage. The next day, the investor recorded its mortgage. A few months later, the photographer defaulted on all three mortgages, having not made any principal payments. The lending company brought a foreclosure action, joining the investor in the proceeding. The foreclosure sale resulted in $150,000 in proceeds after all expenses and fees were paid. A statute of the jurisdiction in which the photographer's home is located provides: "Any conveyance of an interest in land shall not be valid against any subsequent purchaser for value, without notice thereof, unless the conveyance is recorded." Which of the following statements is true? A The bank is entitled to $100,000 of the foreclosure proceeds, the lending company is entitled to $25,000 of the proceeds, the investor is entitled to $15,000 of the proceeds, and the buyer at the foreclosure sale is entitled to the remaining $10,000 in proceeds. B The buyer at the foreclosure sale will take the home subject to the bank's mortgage, the lending company is entitled to $25,000 of the proceeds, the investor is entitled to $15,000 of the proceeds, and the photographer is entitled to the remaining $110,000 in proceeds. C The buyer at the foreclosure sale will take the home subject to the bank's and the investor's mortgages, the lending company is entitled to $25,000 of the proceeds, and the photographer is entitled to the remaining $125,000 in proceeds. D The buyer at the foreclosure sale will take the home subject to the bank's and the investor's mortgages, the lending company is entitled to $25,000 of the proceeds, and the buyer at the foreclosure sale is entitled to the remaining $125,000 in proceeds.
C The buyer at the foreclosure sale will take the home subject to the bank's and the investor's mortgages, the lending company is entitled to $25,000 of the proceeds, and the photographer is entitled to the remaining $125,000 in proceeds. The buyer at the foreclosure sale will take the home subject to the bank's and the investor's mortgages, the lending company is entitled to $25,000 of the proceeds, and the photographer is entitled to the remaining $125,000 in proceeds. When an interest is foreclosed, after the expenses and fees are paid, the proceeds of the sale are first used to pay the principal and accrued interest on the loan that was foreclosed, next to pay off any junior liens, and finally any remaining proceeds are distributed to the mortgagor. (A) is wrong because the bank's interest, an interest senior to the lending company's, is not affected by the foreclosure. As a senior interest, the bank was not a necessary party to the foreclosure action and did not need to be named in the foreclosure action. Thus, the bank is not entitled to a share of the proceeds, and its lien continues on the property in the buyer's hands. (B) is wrong because the investor's mortgage is also senior to the lending company's, and as a result, the buyer also takes subject to the investor's mortgage. Under a notice statute, which this jurisdiction has, a subsequent bona fide purchaser prevails over a prior grantee who fails to record. The important fact under a notice statute is that the subsequent purchaser had no actual or constructive notice at the time of the conveyance (or mortgage), not at the time of recording. Mortgagees for value are treated as "purchasers." Here, when the mortgage on the property was granted to the investor, it had neither actual nor constructive notice of the mortgage given to the lending company. The fact that the lending company recorded its mortgage first is irrelevant. Thus, the investor was a bona fide purchaser and would be entitled to protection under the statute. (A) and (D) are wrong because the mortgagor (the photographer) rather than the buyer at the foreclosure sale is entitled to the surplus proceeds.
A landowner conveyed her parcel of land to a buyer. The buyer placed the deed in her safe deposit box but did not record the instrument before leaving town. Six months later, the landowner conveyed the same parcel of land to a farmer, who promptly recorded his deed. The farmer had heard that the landowner previously sold the land to a different buyer, but he was sure that the landowner would not sell him property she had already sold to someone else. Six months later, the buyer returned to the land and found the farmer there. A statute of the jurisdiction in which the land is located provides: "No conveyance or mortgage of an interest in land is valid against any subsequent purchaser for value without notice thereof whose conveyance is first recorded." The buyer now sues the farmer in ejectment. Who owns the land? A The farmer, because his recording cured any possible defect of his knowing of the earlier sale. B The farmer, because he recorded first. C The buyer, because the farmer is not protected by the recording act. D The buyer, because her deed from the landowner came earlier than the farmer's.
C The buyer, because the farmer is not protected by the recording act.
A citizen of State A sued a local city newspaper for defamation in state court. The newspaper's defense is that the statements related to the citizen were protected speech under the First Amendment to the United States Constitution. Which of the following statements regarding federal court subject matter jurisdiction is correct? A The case cannot be removed or appealed to federal court, including the Supreme Court, because the federal issue in the case is not part of the citizen's well-pleaded complaint. B The newspaper can remove the case to federal district court, because there is an embedded federal issue that must be determined for the citizen's claim to be resolved, and this creates federal question jurisdiction. C The case must be litigated through the state court system, but once a decision is rendered by the highest state appellate court, the United States Supreme Court would have subject matter jurisdiction over an appeal concerning the First Amendment issue. D The citizen can remove the action to the federal district court, because the newspaper's defense has raised a federal question that establishes federal court jurisdiction.
C The case must be litigated through the state court system, but once a decision is rendered by the highest state appellate court, the United States Supreme Court would have subject matter jurisdiction over an appeal concerning the First Amendment issue. The case must be litigated through the state court system. Subject matter jurisdiction in federal court based on a federal question must appear as part of the plaintiff's cause of action as set out in a well-pleaded complaint. The defendant's answer or defense is not relevant because the existence of a defense based on federal law will not give federal question jurisdiction. Supreme Court jurisdiction extends to final judgments rendered by the highest court of a state in which a decision could be had. These judgments may be reviewed by the Supreme Court by certiorari where a right is claimed under the federal Constitution. Here, choice (C) essentially reflects these rules because the defamation case is based on a state law claim, and the citizen did not assert a claim arising under federal question jurisdiction. Therefore, the case must be litigated through the state court system; however, the Supreme Court would have subject matter jurisdiction over an appeal from the highest state court in State A concerning the First Amendment issue. (A) is wrong because, as stated, a defendant can appeal the First Amendment issue to the Supreme Court once it has been litigated in the highest state court. In terms of removal jurisdiction, a defendant can only remove an action to federal court that could have been originally brought by the plaintiff in the federal courts. Here, although choice (A) correctly states that the case cannot be removed to federal court because the federal issue was not part of the citizen's well-pleaded complaint, it incorrectly states that the federal issue cannot be appealed to the Supreme Court. (B) is wrong because it is an incorrect statement of law. The mere existence of a defense based on federal law does not give rise to an action "arising under" federal law. (D) is wrong for the same reason.
The defendant was at a bus station on a stopover between two cities. An officer of the Federal Drug Enforcement Administration noticed that the defendant was tightly clutching a small bag and making numerous phone calls from a public telephone. The officer approached the defendant, identified himself, and began asking the defendant questions about his destination. The defendant agreed to follow the officer into an office for an inspection of his bag. In the office, the officer searched the defendant's bag and found nothing suspicious. He then proceeded to pat down the defendant and found a pouch around the defendant's stomach, which proved to contain cocaine. The defendant was charged with possession of cocaine. If the defendant seeks to have the cocaine excluded from evidence, how should the court rule? A The cocaine is admissible, because of the emergency conditions arising from the growing menace of drugs to the public. B The cocaine is admissible, because, in consenting to the search of his bag, the defendant impliedly consented to a body search. C The cocaine is inadmissible, because the officer had neither probable cause nor reasonable suspicion to search the defendant. D The cocaine is inadmissible, because the officer had no right to search the bag.
C The cocaine is inadmissible, because the officer had neither probable cause nor reasonable suspicion to search the defendant.
A grantor executed a valid deed conveying a tract of land to a city "for the purpose of constructing a planetarium thereon." The city held the property for a number of years, but decided on another site for the planetarium. When presented an offer to purchase the property by a privately owned garbage collection company, the city accepted and conveyed the land to the company. Which of the following statements about the title of the tract of land is true? A The grantor's conveyance to the city created a fee simple determinable in the city and a possibility of reverter in the grantor. B Upon conveyance of the land to the company, the property reverted back to the grantor. C The company owns the land in fee simple absolute. D The company owns the land, but it will revert to the grantor or his successors in interest if the property is used for anything other than a planetarium.
C The company owns the land in fee simple absolute.
A transportation company from State A sued a tourism company from State B in federal court for breach of contract seeking $76,000 in damages. The transportation company filed an appropriate demand for a jury trial, and both parties stipulated that the jury would consist of 10 people. Following the trial and deliberations, nine of the jurors voted in favor of the plaintiff for $29,000 in damages, but one juror voted for the defendant. The defendant appealed. On which of the following grounds would the defendant most likely prevail on appeal? A The court lacked subject matter jurisdiction because the verdict did not exceed $75,000. B The court lacked personal jurisdiction. C The ruling was defective because the jury's vote was not unanimous. D The ruling was defective because it was rendered by fewer than 12 jurors.
C The ruling was defective because the jury's vote was not unanimous.
A retailer and a wholesaler entered into an agreement under which the wholesaler would supply the retailer with goods and the retailer would make payment to one of the wholesaler's creditors. Before the creditor became aware of the agreement, the retailer and wholesaler agreed that the retailer instead would pay the wholesaler directly. The creditor sued to enforce the original agreement between the retailer and the wholesaler. Which of the following statements best describes the rights of the creditor? response - correct A The creditor is an incidental beneficiary only and has no rights under the agreement between the retailer and the wholesaler. B The creditor is an intended beneficiary and has rights against both parties under the agreement between the retailer and the wholesaler. C The creditor is an intended beneficiary of the agreement but has no rights against the retailer. D The creditor's rights as a third-party beneficiary vested when it brought suit to enforce the agreement between the retailer and the wholesaler.
C The creditor is an intended beneficiary of the agreement but has no rights against the retailer.
A cyclist was riding on a sidewalk when someone in a parked car suddenly opened the door of the car into her path. She swerved to avoid the car door and rode onto a landowner's property, damaging some plastic lawn ornaments of waterfowl placed in his front yard. In a suit by the landowner against the cyclist for the damage to his lawn ornaments, what is the likely result? A The cyclist is liable because she had no privilege to enter onto the landowner's property. B Whether the cyclist is liable depends on whether she was exercising due care. C The cyclist is liable for the damage to the lawn ornaments even though her entry was privileged. D The cyclist is not liable for the damage to the lawn ornaments because her entry was privileged.
C The cyclist is liable for the damage to the lawn ornaments even though her entry was privileged. The cyclist is liable for damage to the lawn ornaments even though she had a privilege to enter the landowner's yard. Pursuant to the privilege of necessity, a person may interfere with property of another where it is reasonably and apparently necessary to avoid threatened injury from a natural or other force and where the threatened injury is substantially more serious than the invasion that seeks to avert it. In cases of private necessity (where the act is solely to benefit a limited number of people rather than the public as a whole) the defense is qualified, so that the actor must pay for any injury she causes. The cyclist was faced with serious injury from being struck by the car door opening. Apparently the only way to avoid this injury was to swerve onto the landowner's yard. The threatened injury to the cyclist was substantially more serious than the cyclist's entry onto the landowner's yard. Thus, the cyclist was privileged to enter the yard. However, because this is a private necessity situation, she will be required to pay for the damage she caused to the lawn ornaments. (A) correctly states that the cyclist is liable for the damage, but incorrectly states that she was not privileged to enter upon the landowner's land. On the other hand, (D) is incorrect because it concludes that the cyclist's privilege absolves her of liability for the damage she caused, which is not true in private necessity cases. (B) is incorrect because the cyclist's exercise of due care is irrelevant. The landowner will be proceeding against the cyclist on a theory of intentional tort (either trespass or conversion). Due care is a concept that is applicable to a negligence action, but is not relevant to an action sounding in intentional tort. Therefore, the cyclist's liability is unaffected by whether she was exercising due care.
A buyer and seller entered into a written contract on March 31 for the sale of a beach house. Under the terms of the agreement, the buyer would purchase the house for $275,000, with 10% due at closing on May 1 and a 15-year mortgage. At the time the contract was entered into, the parties agreed orally that the written agreement would not become binding unless the buyer notified the homeowner, in writing, by the end of the day on April 15, that she had secured the proper financing. With the summer season approaching, the seller did not wish to risk any delay in selling the house if the buyer was not in a position to buy it. On the morning of April 15, the buyer's financing was approved. On April 16, the buyer telephoned the seller and told him that her financing had been approved. The buyer also told the seller that she was not able to get written confirmation to him by April 15 because of the postal workers' slowdown and because her fax machine just broke down. The seller assured the buyer that this was not a problem. However, before closing, the seller had a change of heart and decided not to sell the beach house after all. The buyer files an action for breach. Which of the following would be the basis for the buyer's best argument? A Statute of Frauds. B Parol evidence rule. C Waiver of condition. D Excuse of condition by hindrance.
C Waiver of condition. The buyer's best argument is that the seller's assurances that there was no problem with the buyer's failure to provide written notification by April 15 amounts to a waiver of the condition. The buyer's written notification by April 15 that he had obtained the proper financing was a condition precedent to the seller's absolute duty to perform under the contract. It is clear that the buyer did not provide the required notification by April 15; thus, the condition was not fulfilled. However, one having the benefit of a condition may indicate by words or conduct that he will not insist upon it. When a condition is broken, the beneficiary of the condition has an election: (i) he may terminate his liability; or (ii) he may continue under the contract. If a choice is made to continue under the contract, the person is deemed to have waived the condition. The seller was fully aware that the buyer had not satisfied the condition, yet, when speaking with the buyer on April 16, he stated unequivocally that it was not a problem. This is a definite indication that the seller elected to continue under the contract. Having so elected, the seller is deemed to have waived the condition. Therefore, the seller's duty of performance under the contract became absolute. Regarding (A), the only way the Statute of Frauds could bolster the buyer's position would be if the original oral agreement setting forth the condition were required by the Statute to be in writing. If that were the case, the buyer could argue that the condition is unenforceable because it is not in writing. However, the oral agreement is not of a type that falls within the purview of the Statute of Frauds. Therefore, the Statute of Frauds will provide no help to the buyer. Similarly, regarding (B), the parol evidence rule could help the buyer only if it could be used to preclude admissibility of the original oral agreement. Under the parol evidence rule, where the parties to a contract express their agreement in a writing with the intent that it embody the final expression of their bargain, any expression made prior to the writing and any oral expression contemporaneous with the writing is inadmissible to vary the terms of the writing. However, where it is asserted that there was an oral agreement that the written contract would not become effective until the occurrence of a condition, evidence of the oral agreement may be offered and received. Because the original oral agreement between the seller and buyer established a condition precedent to the effectiveness of the written agreement, the buyer will be unable to raise the parol evidence rule as a bar to the admissibility of evidence relating to the oral agreement. (D) is unsupported by the facts. If a party with a duty of performance that is subject to a condition prevents the condition from occurring, he no longer has the benefit of the condition. This is referred to as excuse of condition by hindrance. The seller did nothing to prevent the occurrence of the condition regarding written notification by April 15. Consequently, it cannot be said that the condition is excused by hindrance.
While the defendant was committing a robbery, he shot and killed the victim. The defendant is charged with first degree murder in a state that defines first degree murder as murders committed with premeditation or deliberation or during the commission of burglary, arson, rape, or robbery, and defines second degree murder as all other murders. The state also defines voluntary manslaughter as the unlawful killing of a human being with malice upon a sudden quarrel or heat of passion, and it defines involuntary manslaughter as the unlawful killing of a human being without malice in the commission of an unlawful act, not amounting to an enumerated felony, or in the commission of a lawful act that might produce death in an unlawful manner or without due caution and circumspection. Assuming evidence to support, what explanation for the shooting would best help the defendant in avoiding conviction for first degree murder? A In an act of resistance, the victim suddenly attacked the defendant and knocked him down, so the defendant pulled the trigger because he was afraid the victim was going to hit him again. B The defendant had the gun for many years, it was old and rusty, and he did not think it would fire. C The defendant had taken "angel dust" before the incident and does not remember getting a gun or holding up the victim. D When the defendant tried to hold up the victim, the victim said, "Get out of here, you dirty bum, or I'll kill you," and the defendant became so upset that he did not know what he was doing.
C The defendant had taken "angel dust" before the incident and does not remember getting a gun or holding up the victim.
A customer selected a new wallet at a local department store that the salesperson said was made of the finest calfskin and was stitched by hand. The customer bought the wallet and left the store. A few moments later, he took out the wallet to transfer his cash and credit cards into it. On close inspection, he noticed a small nick in the leather. He immediately went back to the department store and demanded a refund. The salesperson refused. If the customer sues for a refund, who will prevail? A The customer, because there was a breach of contract. B The customer, because he had a reasonable time after purchase in which to inspect. C The department store, because the customer accepted the goods. D The department store, because the customer did not give written notice of the breach.
C The department store, because the customer accepted the goods.
A proud grandfather who planned to take pictures of his grandson's graduation purchased a camera from a camera store. He used the camera on several occasions over the next few weeks without incident, but when he used it on the day before his grandson's graduation, it caught fire and exploded, burning him and destroying an expensive coat he was wearing. Although the grandfather was in a great deal of pain because of his injuries, he insisted on attending his grandson's graduation. However, because he no longer had a workable camera, the grandfather hired a professional photographer to take pictures of the special day. In a breach of warranty action, which of the following represents the most that the grandfather may recover? A The difference between the value of the camera accepted and its value if it had been as warranted. B The difference between the value of the camera accepted and its value if it had been as warranted, plus medical costs for treating the grandfather's burns. C The difference between the value of the camera accepted and its value if it had been as warranted, medical costs for treating the grandfather's burns, and the cost to replace the grandfather's coat. D The difference between the value of the camera accepted and its value if it had been as warranted, medical costs for treating the grandfather's burns, the cost to replace the grandfather's coat, and the cost of hiring the professional photographer.
C The difference between the value of the camera accepted and its value if it had been as warranted, medical costs for treating the grandfather's burns, and the cost to replace the grandfather's coat.
A driver purchased a new automobile from a car dealer. Within a few days of the purchase, the driver returned the car to the dealer for repairs because the car kept pulling to the left whenever the driver applied the brakes. The dealer's mechanic readjusted the brakes but did not detect any other problem with the brake system. The dealer's mechanic assured the driver that the brakes were fixed and, even if they did pull the car to the left again, the brakes would still allow the car to stop. The car worked fine for two days, but then the brakes started pulling to the left again. As the driver was driving the car back to the dealer's shop for further repair, he saw a pedestrian crossing the street. The driver pressed his foot down on the brake pedal, but the master cylinder failed, and the car would not stop. The driver's car struck the pedestrian, injuring him. If the pedestrian sues the driver for his injuries, who will prevail? A The pedestrian will prevail, because the driver knew that there was a problem with his brakes. B The pedestrian will prevail, because drivers have a duty to maintain their vehicles in safe working order. C The driver will prevail, because he had no reason to know that his brakes would not stop the car. D The driver will prevail, because he diligently had his brakes repaired.
C The driver will prevail, because he had no reason to know that his brakes would not stop the car.
A father was angry at his son's coach because the coach would never let the son into a game. In order to exact revenge, the father decided to plant an incendiary device on the coach's front porch. The father believed the device would start a fire that would destroy the coach's home and perhaps injure him as well. However, the father made a mistake while assembling the incendiary device, and it was impossible for the device to do any harm. When the device went off, it did nothing more than produce a foul odor. If the father is charged with attempted murder and attempted arson in a common law jurisdiction, which of the following decisions is most likely to be reached by the court? A The father is guilty of attempted murder and attempted arson. B The father is guilty of attempted murder, but he is not guilty of attempted arson. C The father is not guilty of attempted murder, but he is guilty of attempted arson. D The father is not guilty of attempted murder or attempted arson.
C The father is not guilty of attempted murder, but he is guilty of attempted arson. The father lacked the specific intent to kill that is required for attempted murder. However, the circumstances surrounding the "incendiary device" constitute factual impossibility and will not afford the father a defense to attempted arson. Criminal attempt is an act that, although done with the intention of committing a crime, falls short of completing that crime. To be guilty of attempt, the defendant must have the intent to perform an act and obtain a result that, if achieved, would constitute a crime. Regardless of the intent that would suffice for the completed offense, attempt always requires a specific intent to commit the target offense. Also, the defendant must have committed an act beyond mere preparation for the offense. Here, to be guilty of attempted murder, the father must have had the specific intent to kill his son's coach, even though the intent to inflict great bodily injury would be sufficient mens rea for murder. However, the facts indicate that the father intended at most only to injure the coach rather than kill him. Thus, the father cannot be guilty of attempted murder. However, the father did intend to burn the coach's home; therefore, he had the specific intent to commit arson by means of placing an incendiary device on the coach's porch, and his placing the device was an act beyond mere preparation for this crime. Although the device could not have actually burned the coach's house, it is no defense to attempt that it would have been impossible for the defendant to complete his plan. This is factual impossibility and is not a defense. Thus, the father is guilty of attempted arson. (A) and (B) are incorrect because the father did not have the specific intent to kill. (D) is incorrect because the father is guilty of attempted arson, as explained above.
An entrepreneur opened a specialized business on her land. After using up most of her capital to purchase inventory, however, the entrepreneur needed more funds and asked her friend for a $30,000 loan, to be secured by the business's inventory. The friend declined the loan. A desperate entrepreneur then told the friend she would convey the land, which had a fair market value of $100,000, to him if he would give her the loan at the current market rate of interest. The friend agreed, and the entrepreneur conveyed the land to the friend the next day. At that time, the friend gave the entrepreneur $30,000 in cash, and the parties orally agreed that the entrepreneur would pay the friend back at the rate of $1,000 per month, and that after the loan was paid in full, the friend would reconvey the land to the entrepreneur. The friend immediately recorded his deed to the land. The entrepreneur made three $1,000 payments to the friend and then paid no more. She continued to live on the land but, being very much in debt, could not repay the loan. The friend, meanwhile, had received an offer to buy the land for $100,000. Which of the following most accurately states the friend's right to sell the property? A The friend may sell the land and keep the entire proceeds. B The friend may sell the land, but he must give $73,000 of the proceeds to the entrepreneur. C The friend may sell the land only after formally foreclosing on the property. D The friend may not sell the land.
C The friend may sell the land only after formally foreclosing on the property. The friend may sell the land, but only after formally foreclosing on the property. If a deed is given for security purposes rather than as an outright transfer of the property, it will be treated as an "equitable" mortgage and the creditor will be required to foreclose it by judicial action like any other mortgage. In determining whether an absolute deed is really a mortgage, the court considers the following factors: (i) the existence of a debt or promise of payment by the deed's grantor; (ii) the grantee's promise to return the land if the debt is paid; (iii) the fact that the amount advanced to the grantor/debtor was much lower than the value of the property; (iv) the degree of the grantor's financial distress; and (v) the parties' prior negotiations. Here, the entrepreneur owed the friend a debt; the friend promised to return the property if the debt was paid; the amount advanced ($30,000) was much lower than the value of the property ($100,000); the entrepreneur was in great financial distress; and the parties' negotiations reveal that this transaction was intended as security for the loan. Thus, the friend must bring a judicial foreclosure proceeding before he can sell the land. (A) is wrong because a foreclosure is required. Furthermore, even in a foreclosure sale, the friend is not entitled to all of the proceeds. The proceeds are used to first pay the expenses of the sale, attorneys' fees, and court costs; then to pay the principal and accrued interest on the loan that was foreclosed; then to pay off junior interests. Any remaining proceeds are returned to the mortgagor. The friend is entitled only to his expenses and the amount still owing on the $30,000 loan, including accrued interest. Because the friend has a buyer willing to pay $100,000, the entrepreneur should get some money back. (B) is wrong for two reasons: (i) as explained above, the friend cannot sell the property without a judicial foreclosure; and (ii) the entrepreneur would not be entitled to $73,000. The friend is entitled to his expenses of sale and the principal amount owing, plus accrued interest. (D) is wrong because the friend can sell the land, provided he undertakes formal foreclosure proceedings.
A smoothie retailer and a fruit processor entered into an oral agreement that provided that the processor would deliver to the retailer 100 barrels of fruit each month at a price of $10 per barrel, with delivery on the first of the month and payment of the $1,000 to a creditor of the fruit processor on the 15th of each month. However, when the agreement was reduced to a writing, the fruit processor's manager inadvertently wrote $20 per barrel, and neither party noticed before signing. The creditor first learned of the agreement between the parties when he received a copy of it the day after it was signed, showing that he was to receive $2,000 per month. One day later, the retailer discovered the mistake and alerted the food processor. The parties prepared a revised writing reflecting the correct contract price of $10 per barrel, and also agreed in writing that the retailer would receive a $2 per barrel discount the first month because it discovered the mistake by the fruit processor. The first delivery under the contract was made two days late, on the third of the month. On the 15th of the month, the creditor demanded payment of $2,000 from the retailer according to the terms of the original writing. If the retailer contends that it is not liable to pay the full $2,000, which of the following would NOT be relevant to its defense? A The parties had modified the contract to provide for a $2 per barrel discount the first month. B The parties had originally agreed that the price per barrel would be $10, and neither party noticed before signing that the manager of the fruit processor had inadvertently written $20 in the contract. C The fruit processor owed its creditor only $1,800. D The fruit processor was late with its first delivery.
C The fruit processor owed its creditor only $1,800. Any defense that the fruit processor might have with respect to the money it owed to the creditor would not provide the retailer with a defense. If the promisor has made an absolute promise to pay the third-party beneficiary (and not simply a promise to pay whatever the promisee owed him), the promisor cannot assert the promisee's defenses. Hence, the fact that the processor owed only $1,800, even if it could be asserted as a defense by the processor against the creditor, cannot be asserted as a defense by the retailer. (A) is incorrect because the promisee and promisor in a third-party beneficiary contract are free to modify their contract until the third party's rights have vested. While the creditor learned of the agreement before the processor and the retailer modified it, he did not (i) manifest assent, (ii) bring suit, or (iii) materially rely on the agreement before it was modified; thus, his rights did not vest. Assuming the processor has performed and the retailer's duty to perform is now absolute, the retailer would be liable to the creditor for only $1,600 under the contract as modified. Therefore, (A) is a partial defense. (B) is also a partial defense. When a third-party beneficiary sues the promisor on the contract, the promisor may raise any defense he would have had against the promisee. Under the doctrine of reformation, either of the parties to the contract may ask a court in equity to modify the terms of the contract where the writing, through mistake or misrepresentation, does not incorporate the terms orally agreed upon. Here, the parties' mistake in memorializing the contract permits the retailer to have the contract reformed to show the parties' original agreement. This provides a partial defense that the retailer can use against the creditor to show that it is not liable for the full $2,000. (D) is incorrect for a similar reason. The processor's failure to perform according to the terms of the contract may be asserted as a defense by the retailer. The retailer's liability to the creditor would be offset by whatever lost sales the retailer incurred as a result of the fruit processor's late delivery.
A car owner lent her automobile to her girlfriend for the specific purpose of picking up a pizza that the owner and the girlfriend had ordered for dinner. The girlfriend drove to the shopping mall where the pizzeria was located and parked the owner's car there. Instead of going directly to the pizzeria, the girlfriend went into a bookstore, browsed, and eventually purchased a book. The girlfriend then went to the pizzeria and picked up the pizza, which had been ready for 15 minutes. Just as the girlfriend left the pizzeria to return to the car, another car struck the owner's parked car, causing extensive damage to the car. The owner did not carry collision insurance, and the car required $800 worth of body work. If the owner sues the girlfriend on a negligence theory for damage to the car, who will prevail? A The owner, because the girlfriend exceeded her authority when she went to the bookstore. B The owner, because but for the girlfriend's delay in getting the pizza, the owner's car would not have been damaged. C The girlfriend, because she did not create a foreseeable risk of damage to the owner's car. D The girlfriend, because the family car doctrine imputes any of the girlfriend's negligence to the owner.
C The girlfriend, because she did not create a foreseeable risk of damage to the owner's car. The girlfriend is not liable for the damage to the car because a reasonable person would not have foreseen damage arising from the delay in getting the pizza. A prima facie case for negligence consists of: (i) a duty on the part of the defendant to conform to a specific standard of conduct for the protection of the plaintiff against an unreasonable risk of injury; (ii) breach of such duty by the defendant; (iii) that such breach is the actual and proximate cause of the plaintiff's injury; and (iv) damage to the plaintiff's person or property. No duty is imposed upon a person to take precautions against events that cannot reasonably be foreseen. Here, it is true that, had the girlfriend gotten the pizza immediately, the owner's car would not have been at the location it was at the time it was struck. However, a mere delay in picking up a pizza while leaving a car properly parked does not create a foreseeable risk of damage to the car. Thus, there is no basis for holding the girlfriend liable in negligence. (B) is incorrect because, as noted above, the fact that the owner's car would not have been damaged if the girlfriend had not delayed in getting the pizza is not enough to establish liability for negligence. The fact that the damage to the owner's car was unforeseeable will preclude a finding that the girlfriend acted negligently by delaying in getting the pizza. Likewise, (A) is incorrect because the mere fact that the girlfriend stopped at the bookstore, when the owner loaned her the car specifically to pick up the pizza, will not mean that the girlfriend acted in a manner that created an unreasonable risk of damage to the car. The concept of "exceeding one's authority" might be relevant in the context of an employer trying to avoid vicarious liability for the tortious conduct of an employee by arguing that the employee acted outside the scope of her employment. However, in the context of this question, the girlfriend's having stopped at the bookstore will not subject her to liability. (D) misstates the family car doctrine, which holds an automobile owner liable for tortious conduct of immediate family or household members who are driving with the owner's permission. This doctrine would not be used to relieve the driver of liability for any negligence on her part by imputing such negligence to the owner; the driver is still liable. In any case, the girlfriend and the owner are not members of the same family or household, so the doctrine would not apply here.
A landowner's will left his ranch to a rancher, his heirs, and assigns, so long as the property was used exclusively for ranch purposes, then to the landowner's grandson. The remainder of the landowner's property passed through the residuary clause of his will to the grandson. Seven years after the landowner's death, the rancher began strip mining operations on the ranch. The grandson brought an action to quiet title to the ranch against the rancher, and the rancher counterclaimed on the same theory. Who should prevail? A The rancher, because the condition imposed on his interest under the will is void as violating the Rule Against Perpetuities. B The rancher, because the condition imposed is a restraint against alienation. C The grandson, pursuant to the residuary clause. D The grandson, because the condition imposed is valid and he takes according to the subsequent provision.
C The grandson, pursuant to the residuary clause.
A corporation was in the business of purchasing real property at below-market prices and reselling the properties to investors. The bylaws of the corporation authorized the chief executive officer ("CEO") and the director of the marketing division to enter into contracts on behalf of the corporation for the purchase or sale of properties. The corporation had recently purchased a large parcel of beachfront property for resale. The CEO secretly opened negotiations with an amusement park to sell the property. However, unknown to the CEO or anyone else in the corporation, the marketing director had already reached an agreement with a hotel for the sale of the property. On April 23, the marketing director and the hotel signed a written contract providing for sale of the property by the corporation to the hotel for $35 million. On April 25, the board of directors amended its bylaws, effectively depriving the marketing director of authorization to bind the corporation in purchase or sale transactions. This action was immediately publicized and became known to both the marketing director and the hotel. On April 26, the hotel duly recorded its contract. On May 1, the CEO, still unaware of the marketing director-hotel agreement, approved sale of the property to the amusement park for $39 million. The necessary documents of title were prepared and properly recorded by the amusement park on May 5. Two days later, the amusement park learned of the marketing director-hotel agreement. On May 10, the date scheduled for closing of the hotel's sale agreement, the CEO refused to accept the hotel's tender of $35 million and refused its demand for a deed to the property. The hotel subsequently brings action against the corporation and the amusement park for specific performance and to quiet title to the property. For whom will the court likely rule? A The defendants, because the board of directors had deprived the marketing director of authority to bind the corporation in the sale of real property. B The defendants, because the amusement park is the only purchaser who properly recorded a deed to the property. C The hotel, because the amusement park had constructive notice of the hotel's interests in the property when the agreement with the CEO was made. D The hotel, because the attempt to divest the marketing director of authority to approve sales of the corporation's property was invalid.
C The hotel, because the amusement park had constructive notice of the hotel's interests in the property when the agreement with the CEO was made.
A lawyer rented an office building for his law practice and subleased most of the building to three other tenants. The lawyer paid $2,000 per month to the owner and charged his subtenants $600 per month each. After having been in the building for three years, the lawyer and the owner orally agreed that the lawyer would purchase it for a price of $120,000, to be paid in monthly installments of $2,000 over a five-year period. It was further agreed that title would remain in the owner's name until $48,000 had been paid on the total price, whereupon the owner would deliver a deed to the lawyer. Shortly thereafter, the lawyer spent $4,000 redecorating his suite. During the course of the next two years, the lawyer hired an associate and placed her in one of the offices formerly occupied by one of the subtenants, and raised the monthly rental he charged the other two subtenants to $700. Two years after the agreement with the owner, the lawyer demanded that the owner convey the building by delivery of a deed. The owner refused, denying that any oral agreement for sale had ever existed. The lawyer brings an action for specific performance against the owner, who pleads the Statute of Frauds as a defense. If the owner wins, what is the likely reason? A The lawyer did not obtain the owner's approval before making the improvements to his offices. B The original violation of the Statute of Frauds was incurable. C The lawyer's actions in paying $2,000 per month and making improvements were as consistent with being a tenant as with the oral contract. D The owner received no unconscionable benefit entitling the lawyer to equitable relief.
C The lawyer's actions in paying $2,000 per month and making improvements were as consistent with being a tenant as with the oral contract.
A property owner owned a tract of commercial property that he conveyed in joint tenancy to his twin sons as a birthday present. Unfortunately, a few years after the conveyance, the property owner and his sons had a serious falling out over how to run the family business. The property owner no longer wished the sons to control valuable commercial property, and so he demanded that they return the deed with which he conveyed the property to them. The sons returned the deed, and the property owner destroyed it. A few months later, one of the twins learned that he was seriously ill and not likely to live much longer. He executed a quitclaim deed conveying "any interest I have in the commercial property conveyed to me and my brother from my father" to his daughter. The twin who conveyed the property subsequently died. Who owns the property? A The living twin. B The property owner. C The living twin and the deceased twin's daughter as tenants in common. D The living twin and the deceased twin's daughter as joint tenants.
C The living twin and the deceased twin's daughter as tenants in common. The living twin and the deceased twin's daughter own the property as tenants in common. A conveyance of a co-tenant's interest in joint tenancy property severs the joint tenancy, and that interest is subsequently held as a tenancy in common with the other co-tenants. Thus, (D) is incorrect. Because the joint tenancy with right of survivorship was severed before the deceased twin's death, (A) is incorrect. Returning the deed to the property owner did not return ownership of the property to him; that would require a reconveyance. Thus, (B) is incorrect.
It was common practice in a particular state for a security interest in land to be structured as a deed absolute, which gave a lender absolute title to the borrower's property as security for the loan. The lender would reconvey only on complete payment of the loan by the debtor party, and could dispose of the land immediately without a foreclosure sale on default. A new governor of the state whose campaign platform was built around abolishing the deed absolute mortgage encouraged the legislature to enact a bill that immediately outlawed use of the deed absolute, declaring that all such deeds would be considered mere liens against the secured property. The law applied not only to loans made in the future, but also to the thousands of such loans in existence at the time the legislation was passed. As soon as the governor signed the legislation, lending institutions and individuals who had loaned money secured through deeds absolute challenged the constitutionality of the new law. What is the strongest argument that the challengers can make? A As applied to loans outstanding at the time the bill was enacted, the law is an ex post facto law, and such laws are banned by the federal Constitution. B Lenders using the deed absolute have been singled out by the governor and his followers in the legislature as political scapegoats, and such discrimination against the lenders violates the Equal Protection Clause. C Lenders had property rights in the secured property and such rights were summarily abrogated by the new law, constituting an unconstitutional taking of property without due process of law. D As applied to loans outstanding at the time the bill was enacted, the law impairs the contract rights of the lenders and such rights are guaranteed by the Contracts Clause of the federal Constitution.
D As applied to loans outstanding at the time the bill was enacted, the law impairs the contract rights of the lenders and such rights are guaranteed by the Contracts Clause of the federal Constitution. The best argument against the statute is that it violates the Contracts Clause. The Contracts Clause prohibits states from retroactively and substantially impairing contract rights unless the governmental act serves an important and legitimate government interest and is a reasonable and narrowly tailored means of promoting that interest. Here, the legislation by its terms affects existing contracts. It is arguable that the effect is a substantial impairment, because deeds absolute have been turned into mortgages by the statute. While protecting debtors may be a legitimate government interest, it could be argued that the statute is not narrowly tailored to that interest. It is doubtful that the plaintiffs' attorneys would win on this argument, but it is their best approach. (A) is not a good argument because ex post facto laws are laws that retroactively alter criminal law, and here the law is not criminal, but rather civil. (B) is not a good argument because the law does not violate the Equal Protection Clause. The law does not involve a suspect or quasi-suspect class or a fundamental right, and so its validity would be tested under the rational basis standard. A law that has any legitimate rational basis will be upheld under this standard, and as established above, the law here serves the legitimate government purpose of protecting debtors. Thus, equal protection is not violated. (C) is not a good argument because the Due Process Clause does not prohibit laws that "summarily take" property from a class in general; no individual hearing or other process is required for laws of general applicability. Substantive due process is not violated either; since no fundamental right is involved, the law will be judged under the rational basis standard, and as stated above, will be upheld.
A minor entered a liquor store and asked the cashier to let him purchase a case of beer for a party even though he was underage. The cashier agreed as long as he paid double the retail price for the beer as compensation for the risks of the sale. As the cashier started to ring up the transaction, an undercover officer who overheard them intervened and arrested them. The state criminal code provides that it is a felony to knowingly provide alcohol to any person under the age of 21. If the state follows the unilateral theory of conspiracy, of what crimes can the minor and cashier be convicted? A The minor can be convicted of solicitation and the cashier can be convicted of conspiracy to violate the statute. B Both the minor and the cashier can be convicted of conspiracy to violate the statute. C The minor cannot be convicted of either solicitation or conspiracy, but the cashier can be convicted of conspiracy to violate the statute. D The minor cannot be convicted of either solicitation or conspiracy, and the cashier cannot be convicted of conspiracy.
C The minor cannot be convicted of either solicitation or conspiracy, but the cashier can be convicted of conspiracy to violate the statute. The cashier can be convicted of conspiracy in a unilateral jurisdiction even though the minor can be convicted of neither solicitation nor conspiracy. Under the unilateral approach adopted by the Model Penal Code, the crime of conspiracy requires that only one party have genuine criminal intent, and may be shown by proof that the defendant agreed with another to commit a crime, even if the other person does not share the commitment. Thus, the fact that no other party to the conspiracy could be found guilty does not prevent the defendant from being convicted of conspiracy. Here, the fact that the minor is a member of the class that the statute was designed to protect prevents him from being found guilty of conspiracy. (B) is therefore incorrect. However, this fact has no bearing on the cashier's liability for conspiracy under the unilateral approach. Thus, (C) is correct and (D) is incorrect. (A) is incorrect because the crime of solicitation is treated the same as conspiracy. If the solicitor could not be guilty of the completed crime because of legislative intent to exempt him, he cannot be guilty of solicitation of the crime. Because the minor is a member of the class intended to be protected by the statute, he cannot be found guilty of soliciting the cashier to complete it.
A salesman was employed by a florist, who owned a retail shop adjacent to a large wholesale nursery. The owner of the nursery liked to use a brand-name artificial fertilizer for her plants, although other effective fertilizers were available at comparable prices. She stored a large quantity of the fertilizer in a heap on the nursery's property, as did many other nursery owners without incident. The fertilizer gave off fumes that caused the salesman to suffer lung irritation. Occasionally, the salesman's irritations became so bad that he had to take off from work and seek medical attention. After losing a few hundred dollars in wages and amassing a few hundred dollars in medical expenses, the salesman sued the nursery owner for damages. The court is likely to rule in favor of: A The salesman, because the nursery owner had equally effective fertilizers available at comparable prices to the fertilizer used. B The salesman, because the nursery owner is strictly liable for injuries caused by emissions from her property. C The nursery owner, because the selection of the fertilizer was reasonable and it was stored in a reasonable manner. D The nursery owner, because the salesman is merely an employee of the florist and does not own the property on which the shop is located.
C The nursery owner, because the selection of the fertilizer was reasonable and it was stored in a reasonable manner.
After the release of various news stories about the President's possible violation of political campaign funding laws, a federal grand jury investigation and an investigation by a special Senate subcommittee were initiated. The Senate subcommittee subpoenaed documents and records from several top officers of the executive branch. Learning of the subpoenas, the President ordered all executive officials to refuse to turn over materials, claiming "executive privilege." Which of the following statements is most accurate? A The subpoena violates the constitutional principle of separation of powers. B The President's executive privilege is absolute, except in cases of impeachment. C The presidential papers are presumptively privileged, but the privilege must yield to a demonstrated specific need for evidence in a pending legislative proceeding. D The President's executive privilege applies to proceedings by Congress, but not to proceedings by the courts.
C The presidential papers are presumptively privileged, but the privilege must yield to a demonstrated specific need for evidence in a pending legislative proceeding. Executive privilege is an inherent privilege necessary to protect the confidentiality of presidential communications. Under this privilege, presidential documents and conversations are presumptively privileged, but this privilege must yield to a demonstrated need for such materials as evidence in a criminal case in which they are relevant and otherwise admissible. [United States v. Nixon (1974)] Although the Supreme Court has not expressly decided that the privilege must also yield to a demonstrated need for evidence in a pending legislative proceeding, such an extension of Nixon is likely, and none of the other alternatives is at all accurate. (A) is incorrect because it is too broad. In Nixon, supra, the Court decided that an evidentiary subpoena to the President in a criminal case does not violate the separation of powers principle. By extension, a subpoena issued by a Senate subcommittee, pursuant to the well-established implied power of Congress to investigate, would not be deemed to violate separation of powers. (B) is also incorrect because it is too broad. As stated above, although a presumptive privilege applies to presidential documents and conversations, that privilege must yield to a demonstrated need in criminal cases. Thus, executive privilege is not absolute. (D) is incorrect because executive privilege does apply to proceedings by the courts; in fact, the privilege is overridden only on a specific showing of need for specific information.
The United States Surgeon General was cited for contempt for refusing to answer questions as part of a Senate investigation regarding an issue in the Food and Drug Administration. His contempt citation will be dismissed if he can show which of the following? A As a member of the executive branch, he is immune from prosecution. B If he answered the questions, he could be subject to dismissal from his position as Surgeon General. C The questions do not relate to any matter concerning which the Senate may legislate. D The questions do not relate to any matter concerning current or planned legislation.
C The questions do not relate to any matter concerning which the Senate may legislate.
Congress provided by statute that any state that does not enact a ban on texting while driving on state highways within the next three years shall be denied 10% of federal highway construction funding. What is the best argument that can be made in support of the constitutionality of this federal statute? A The states ceded their authority over highways to the national government when the states accepted federal grants to help finance the highways. B The federal government can regulate the use of state highways without limitation because the federal government paid for most of their construction costs. C The requirement is not unduly coercive and is related to making travel on highways safer. D A recent public opinion survey demonstrates that 90% of the people in this country support the ban.
C The requirement is not unduly coercive and is related to making travel on highways safer.
A homeowner filed a federal diversity action against the manufacturer of a pressure cooker for a products liability action based on state law, alleging that she was severely burned because the top flew off the pressure cooker during operation. Under the state law, a jury verdict does not have to be unanimous. However, a specific rule under the Federal Rules of Civil Procedure directly conflicts with the state law and provides that jury verdicts in federal court must be unanimous. Which statement correctly presents what law the federal judge should apply regarding the jury verdict? A The state law will apply, because a federal district court with subject matter jurisdiction based on diversity of citizenship must apply state law when there is a conflict. B The state law will apply, because applying the specific Federal Rules of Civil Procedure may change the outcome of the case. C The specific Federal Rule of Civil Procedure will apply as long as both rules are consistent with the Rules Enabling Act and are not unconstitutional. D The specific Federal Rule of Civil Procedure will apply, because, on balance, the federal judicial system has the greater interest in having its rules applied.
C The specific Federal Rule of Civil Procedure will apply as long as both rules are consistent with the Rules Enabling Act and are not unconstitutional.
A state law required all automobile drivers to carry liability insurance; however, because of the high number of auto accidents in the state, the cost of insurance became prohibitive. A study sponsored by the state legislature showed that males under the age of 21 were four times more likely to get into automobile accidents than any other group, including females in the same age group. The study predicted that prohibiting males under the age of 21 from driving would result in a 15% reduction in all other persons' automobile insurance rates. Ultimately, the legislature raised the minimum age for obtaining a driver's license to age 21 for males. Females were still allowed to obtain licenses at age 16. An 18-year-old male living in the state when the limit was raised, and who worked as a pizza delivery driver, was fired from his job and replaced by a 17-year-old female. If the young man sues to have the law set aside and prevails, what is the most likely reason? A The state could not prove that the law was the least restrictive means of achieving a compelling government purpose. B The state could not prove that the law was rationally related to a legitimate government purpose. C The state could not prove that the law was substantially related to an important government interest. D The state could not prove that the law was necessary to achieve a compelling government purpose.
C The state could not prove that the law was substantially related to an important government interest.
A public school teacher was hired without a written contract. The school district handbook provides that all newly hired employees are at-will employees for their first year of service and may be dismissed for any reason or without reason. Six months after being hired, the teacher was dismissed without any prior notice or a hearing. Which of the following, if true, most strongly supports the teacher's argument that she should have been afforded notice and a hearing before dismissal? A She was the only newly hired teacher not to have survived the probationary period for the past three years. B There is no evidence that teachers with permanent status are any more competent than this teacher. C The teacher moved her household from out of state in reliance on an oral promise by the school board that the job would be permanent and that she could be dismissed only for cause. D She was the only teacher there over the age of 50.
C The teacher moved her household from out of state in reliance on an oral promise by the school board that the job would be permanent and that she could be dismissed only for cause. The teacher's argument is most strongly supported if she moved in reliance on an oral promise that she could be dismissed only for cause. Under the Due Process Clause, a person has a right to notice and a hearing if the government deprives the person of life, liberty, or property. The Supreme Court has held that a government employee will have a property interest in continued employment only if the employee has a legitimate claim to (as opposed to a mere expectancy of) continued employment. To have such a claim, there must be a contract, clear practice, mutual understanding, etc., that the employee can be terminated only for cause. If someone in a position of authority promised that the teacher could be dismissed only for cause and the teacher relied on this promise by moving, a case can be made that the teacher had a legitimate claim to continued employment despite the employee handbook. (A) would also support the contention of a legitimate claim to continued employment. If every teacher over the past three years has survived the probation period, an argument can be made that there was a policy of retaining all newly hired teachers. However, this argument is weaker, factually, than the contract argument and so is not as good an answer choice as (C). (B) and (D) are incorrect because they are irrelevant—whether other teachers are more or less competent or younger than the teacher involved here does not factor into whether the teacher here has a legitimate claim to continued employment. The teacher needs to point to a contract or policy providing for dismissal only for cause.
An uncle validly executed and notarized a deed conveying his beach house to his nephew, and then validly recorded the deed. When the nephew, who was experiencing financial difficulties, learned of the recordation of the deed, he immediately told his uncle that he did not want the beach house and could not accept such an expensive gift anyway. Later, the nephew filed for bankruptcy and the trustee in bankruptcy asserted an ownership interest in the beach house on behalf of the debtor's estate. The bankruptcy court ruled that the property belonged to the uncle and not to the nephew, and thus was not part of the debtor's estate subject to distribution. Which of the following is the strongest reason in support of the bankruptcy court's ruling? A There was no presumption of delivery created by recordation of the deed because the nephew did not know of the recordation. B The nephew's statements to the uncle were a constructive reconveyance of the property. C There was never an effective acceptance of delivery of the deed by the nephew. D The recordation of the deed was invalid because it was done without the nephew's permission.
C There was never an effective acceptance of delivery of the deed by the nephew.
The President of the United States entered into a bilateral agreement with another country regarding the probate of estates. One part of the agreement provided that, should a citizen of that country die owning property in the United States, the executor or administrator appointed in that country should have the power to deal with such property and a separate American administrator or executor need not be appointed. A citizen of that country died, owning property in the United States. His will appointed his brother, a citizen and resident of the country, as executor. A law in the state where his property was located requires that all property left by will within the state be probated by an executor or administrator who is a resident of the state. The brother desires to act as executor of the estate and properly distribute the property in accordance with the decedent's will, Should the brother be allowed to act as executor, notwithstanding the state statute? A No, because the probate of real property within the state is within the exclusive purview of the state. B No, because an executive agreement is not a treaty, and state law takes precedence over executive agreements. C Yes, because executive agreements supersede state laws. D Yes, because executive agreements are entitled to full faith and credit.
C Yes, because executive agreements supersede state laws.
A professional painter and his apprentice, in business as a partnership, were hired to paint a store. Midway through the job they ran out of paint, so the painter lent his truck to the apprentice to pick up more. On his way to pick up the paint, the apprentice stopped at a post office along the way to mail a personal letter. On pulling out of the post office parking lot, he negligently ran into a parked car, causing extensive damage. If the car owner brings a negligence action against the painter, will she prevail? A No, because the apprentice is an independent contractor. B No, because a bailor is not vicariously liable for the torts of his bailee. C Yes, because the apprentice's stop at the post office was not a frolic. D Yes, because the apprentice was using the painter's truck.
C Yes, because the apprentice's stop at the post office was not a frolic.
A developer owned a 240-acre parcel of land zoned for commercial and residential use. He prepared and recorded, after obtaining approval from all appropriate agencies, a subdivision plan that included a commercial center and a number of lots for single- and multi-family residences. The list of covenants, conditions, and restrictions recorded with the plan included provisions that required every building constructed in the subdivision to be of "simulated adobe style" architecture approved in advance by an association. A year later, the developer sold many of the lots in the commercial center, including several to a real estate firm. Each deed prepared by the developer contained a reference to the design restriction in the recorded plan. The developer also sold almost all of the residential lots, the deeds of which contained the same reference to the restriction. The following year, the real estate firm sold one of its lots to a burger franchise. The deed contained no reference to the design restriction. The franchise's prefabricated restaurant, complete with a giant burger logo mounted on the roof, was constructed over the weekend. A merchant, an original purchaser of one of the commercial lots, owned the lot next to the burger franchise. She did not learn of construction of the restaurant until she came in to work on Monday, and saw the giant burger logo. The merchant brings an action seeking a mandatory injunction compelling the burger franchise to demolish the restaurant. At trial, the merchant proves that the burger franchise did not seek or obtain approval of the association for its building. Should the court issue the injunction? A No, because destruction of the restaurant would be a tremendous waste of resources. B No, because the burger franchise's deed contained no restriction on the type of building that could be constructed on the lot. C Yes, because the restrictive covenant runs with the land. D Yes, unless the burger franchise can establish to the court's satisfaction that its restaurant design has at least as much aesthetic merit as any "simulated adobe style" design.
C Yes, because the restrictive covenant runs with the land.
A shoplifter set a small fire in a store's trash basket. The shoplifter knew that the store's automatic sprinkler system would promptly douse the fire, but his purpose was merely to create a distraction so he could walk out with an expensive watch, which would constitute felony theft. The shoplifter was stopped after he set the fire but before he could take any merchandise. No serious damage was done to the store by the fire, but the flame charred the wall next to the trash basket and blistered the paint on the surrounding area. A statute in the jurisdiction extends the crime of arson to buildings other than dwellings. If the shoplifter is tried for the crime of arson, should the court find him guilty? A No, because the shoplifter did not intend to burn the building. B No, because the shoplifter's act was sufficient only for attempted arson. C Yes, because the shoplifter's conduct demonstrated the requisite state of mind for the crime. D Yes, because the shoplifter started the fire during the act of perpetrating another felony.
C Yes, because the shoplifter's conduct demonstrated the requisite state of mind for the crime.
A man boarded a plane with his pet rattlesnake hidden in his carry-on bag. The man was not aware that the carry-on bag, which he put under the seat in front of him, had a defective zipper. The snake, which had no teeth or venom and was harmless, escaped from the bag and started slithering down the aisle while the man was using the lavatory. A woman who had just gotten up from her seat saw the snake heading towards her and tried to run in the other direction. She tripped over someone's foot and broke her ankle. If the woman sues the man on a theory of strict liability for her broken ankle, will she prevail? A No, because the snake was in fact a nondangerous animal. B No, because the injury she suffered was not caused by the dangerous propensity of a snake. C Yes, because the snake is a wild animal. D Yes, because it is not a common activity to bring snakes on a plane.
C Yes, because the snake is a wild animal. (C) The woman will prevail because the rattlesnake is classified as a wild animal. An owner of a wild (i.e., nondomestic) animal, even one kept as a pet, will be strictly liable for the damage caused by the animal. A rattlesnake, even a harmless one, will be classified as a wild animal. Therefore (C) is correct and (A) is wrong. (B) is wrong because the injury the woman suffered was within the "normal dangerous propensity" of the animal. Strict liability for wild animals includes liability for the harm that results when a person is attempting to flee from what is perceived to be a dangerous animal. (D) is wrong. The fact that the activity was uncommon in the locale would have some relevance if the lawsuit were based on a theory of strict liability for an abnormally dangerous activity. It has nothing to do with strict liability for damage caused by animals.
In an effort to protect the dwindling California condor population, Congress enacted the Condor Preservation Act, which made it illegal to take or sell any part of a California condor. The constitutionality of the Act is challenged by a seller of gifts and artifacts, including artifacts made out of California condor feathers. Is the statute valid? A No, the statute violates due process because the absolute prohibition on sale is an effective taking under the Fifth Amendment Due Process Clause without just compensation. B No, because the statute is discriminatory as applied. C Yes, because the statute is rationally related to interstate commerce. D Yes, because the statute is designed to protect a dwindling national resource.
C Yes, because the statute is rationally related to interstate commerce.
A driver in the local racing circuit brought his customized yellow stock car to a body shop to have it repainted before the new racing season began. When the driver returned to pick up the car, he was horrified to discover that it was repainted pink instead of yellow. The owner of the body shop apologized and offered to repaint the car, but the driver refused because the first race was in two days. The driver lost a couple of endorsements because the endorsers' ads did not work with the new color. He was also subjected to ridicule at the track, but he felt better after he drove the car to victory in the first race. If the driver sues the body shop for their treatment of his car, will he prevail? A No, because he won the race with the car. B No, unless he can prove that the body shop breached a duty of care owed to him. C Yes, because the value of his car was reduced. D Yes, because he suffered severe distress as a result of the conduct of the body shop.
C Yes, because the value of his car was reduced. The driver can recover for trespass to chattels because he can show that the value of his car has been reduced as a result of the conduct of the body shop. Trespass to chattels requires (i) an act of defendant that interferes with plaintiff's right of possession in the chattel, (ii) intent to perform the act bringing about the interference with plaintiff's right of possession, (iii) causation, and (iv) damages. The act of interference may be either dispossession of or damage to the chattel. Here, the body shop employees interfered with the driver's possession of his car by painting it contrary to his instructions, and they intended to do the act (painting) that caused the interference. The driver suffered damage because that conduct reduced the value of his car for advertising purposes. Hence, the driver will be able to satisfy the prima facie case for trespass to chattels. (A) is wrong because the fact that the driver won the race with the car does not establish absence of actual damages. Any loss in value of the chattels will suffice. (B) is wrong because it is not necessary for the driver to show negligence on the part of the body shop to recover. The driver can recover damages for trespass to chattels without proof of breach of duty. (D) is wrong because emotional distress alone is not sufficient to satisfy the actual damages requirement for the tort of trespass to chattels, and the facts do not establish the requisite extreme and outrageous conduct for an intentional infliction of distress action.
A trial judge presiding over a lawsuit plans to call a witness to the stand and question her. May the judge do so? A Yes, but only if the witness has already been called and examined by one of the parties. B Yes, but only if the witness is not testifying as an expert. C Yes, but the parties are entitled to cross-examine the witness. D No, because only parties may call and examine witnesses.
C Yes, but the parties are entitled to cross-examine the witness.
A seller entered into a written contract with a vintner on April 4, whereby the seller agreed to convey a vineyard to the vintner for $2 million. The terms of the contract set the closing date as June 1. At the time the seller entered into the agreement with the vintner, the seller had no interest in the vineyard. On April 15, the seller entered into a written agreement with a landowner, whom the seller believed to be the owner of the vineyard. According to the terms of the agreement, the landowner was to convey the vineyard to the seller on or before May 25. Another term of the agreement stated "time is of the essence." On May 24, the landowner conveyed his interest in the vineyard to the seller. When the seller went to record the deed, she discovered from records in the recorder's office that the landowner held clear title to only seven-eighths of the vineyard. It took some time for the seller to remove the cloud from the title and procure ownership in full of the vineyard. She finally did so on August 1, and on that day she tendered a warranty deed to the vineyard to the vintner. The vintner refused to tender $2 million or any other sum to the seller, asserting that the seller had broken her agreement by failing to close on June 1. The seller then sued the vintner for specific performance. If the vintner prevails, what is the likely reason? A Title was unmarketable, because the seller did not own the vineyard at the time she entered into the contract. B Title was unmarketable, because the seller only owned seven-eighths of the vineyard on the closing date. C Time was not of the essence in the seller-vintner contract. D A two-month delay in closing is determined to be unreasonable.
D A two-month delay in closing is determined to be unreasonable.
A victim was struck by a car in a hit-and-run accident. A police officer arrived half an hour after the accident. The victim was in shock and came in and out of consciousness. As the officer applied first aid, the victim muttered, "I know I'm going to die. Oh my, he ran the light!" The victim fell back into unconsciousness, but revived again and muttered, "Why didn't he stop?" The officer heard the comments clearly and made a note of them. Good police work by the officer and others led to the discovery of the driver of the car that struck the victim. The victim survived and filed a tort action against the driver. Before the case came to trial, the victim died of a heart attack. The causes of the heart attack were totally unrelated to the accident. The laws of the jurisdiction allow for survival of personal injury actions. Thus, the victim's estate is substituted for the victim as plaintiff. If the plaintiff's attorney seeks to have the officer testify to the victim's statements at the time of the accident, how will the court rule? A Inadmissible, because the victim did not die as a result of the accident. B Inadmissible, because this is a civil case and not a criminal matter. C Admissible, because the victim's statements were present sense impressions. D Admissible, because the statements were made at a time when the victim feared impending death.
D Admissible, because the statements were made at a time when the victim feared impending death. The officer's testimony as to the victim's statements is admissible because the statements were made when the victim feared impending death and so they qualify under the dying declaration exception to the hearsay rule. Hearsay is a statement, other than one made by the declarant while testifying at the trial or hearing, offered in evidence to prove the truth of the matter asserted. [Fed. R. Evid. 801(c)] Upon appropriate objection, a hearsay statement to which no exception is applicable must be excluded. Under the dying declaration exception to the hearsay rule, a statement made by a now-unavailable declarant while believing her death was imminent that concerns the cause or circumstances of what she believed to be her impending death is admissible. [Fed. R. Evid. 804(b)(2)] The declarant need not actually die as a result of the circumstances giving rise to her belief of imminent death. Here, testimony as to the victim's statements would be hearsay, because they are out-of-court declarations offered for the truth of the matter asserted; i.e., that the driver of the car that hit her ran a red light. However, these statements related to the circumstances of what the victim believed to be her impending death and the victim (who is now unavailable due to her death) made these statements under a fear of imminent death, as indicated by her condition and her statement "I know I'm going to die." Consequently, all of the elements of the dying declaration exception are present, and the officer's testimony as to the statements is admissible. (A) is incorrect because the declarant need not actually die as a result of the incident that gives rise to the statements. Indeed, the declarant need not die at all. All that is required is that the declarant be unavailable at the time the statements are offered. (B) is incorrect because it reflects the traditional view, which limited the admissibility of dying declarations to homicide cases, rather than the position of the Federal Rules, which allow such declarations in both civil cases and homicide prosecutions. (C) is incorrect because the statements do not qualify under the present sense impression exception to the hearsay rule. A present sense impression is a statement that describes or explains an event or condition, and is made while or immediately after the declarant perceives the event or condition. Here, the victim's statements were made at least one half-hour after the accident. This time lapse between the accident and the statements means that such statements were not made either at the time the victim received a sense impression or immediately thereafter; thus, the present sense impression exception is inapplicable to these facts.
A horse breeder offered to sell a colt to his neighbor and they agreed on a purchase price. The horse breeder subsequently received a letter from the neighbor thanking him for the sale and summarizing their agreement. The letter contained the neighbor's alleged signature. When the horse breeder attempted to set up transfer of the colt, the neighbor denied that she agreed to purchase it. In a breach of contract action against the neighbor, the horse breeder offers into evidence the letter. The horse breeder testifies that he is familiar with the neighbor's handwriting and recognizes the signature on the letter as being hers. Assuming appropriate objection by the neighbor, who claims that she did not sign the letter, how should the trial court rule on the admissibility of the letter? A Exclude the letter for lack of foundation because lay opinion testimony regarding handwriting identification is not admissible. B Exclude the letter unless its authenticity is established by a preponderance of the evidence. C Admit the letter as authentic and instruct the jury accordingly. D Admit the letter but instruct the jury that it is up to them to decide whether the letter is authentic.
D Admit the letter but instruct the jury that it is up to them to decide whether the letter is authentic. The court should admit the letter and instruct the jury that it is up to them to decide whether the letter is authentic. Before a writing may be received in evidence, it must be authenticated by proof showing that the writing is what the proponent claims it is. All that is necessary is proof sufficient to support a jury finding of genuineness. The authenticity of a document is a preliminary fact to be decided by the jury. Here, the horse breeder's testimony that he is familiar with the neighbor's handwriting and that he recognizes the signature on the letter to be that of the neighbor is sufficient to support a jury finding of genuineness. Thus, the letter should be admitted and authenticity should be left to the jury to decide. (A) is wrong because a lay witness who has personal knowledge of the handwriting of the supposed writer may state his opinion as to whether the document is in that person's handwriting. (B) is wrong because authentication of documentary evidence requires only enough evidence to support a jury finding that the matter is what its proponent claims it is. It is not required that the proponent establish its genuineness by a preponderance of the evidence. (C) is wrong because, as noted above, where there is a dispute as to the authenticity of a document, the issue of authenticity is a fact determination for the jury, not the judge, to decide.
A husband and a wife were arrested by federal agents and charged with distributing obscene materials through the United States mails. When called before a grand jury, the wife refused to say anything, invoking her Fifth Amendment right to be protected from compelled self-incrimination. The husband was terrified of the grand jury and readily admitted under questioning that he sent obscene matter through the mail. He also incriminated his wife in the illegal activity. The thought of a trial and a prison term drove the husband over the edge, and he committed suicide two days before his trial was to begin. A month later, the wife was put on trial in federal district court. The federal prosecutor seeks to introduce a transcript of the husband's grand jury testimony into evidence against the wife. The defense attorney objects. How should the court rule on the admissibility of the grand jury transcript? A Admissible, as a vicarious admission. B Admissible, as former testimony. C Inadmissible, because the wife can invoke the testimonial privilege, even though her husband is now deceased. D Inadmissible, because the husband's testimony was not subject to cross-examination.
D Inadmissible, because the husband's testimony was not subject to cross-examination. The grand jury transcript is not admissible because the husband's testimony was not subject to cross-examination. The husband's testimony was hearsay because it was an out-of-court statement offered to prove the truth of the matter asserted. [Fed. R. Evid. 801(c)] If a statement is hearsay, and no exception to the rule is applicable, the evidence is inadmissible. [Fed. R. Evid. 802] Under the former testimony exception to the hearsay rule, the testimony of a now unavailable witness given at another hearing is admissible in a subsequent trial as long as there is a sufficient similarity of parties and issues so that the opportunity to develop testimony or cross-examine at the prior hearing was meaningful. [Fed. R. Evid. 804(b)(1)] The party against whom the former testimony is offered must have had the opportunity to develop the testimony at the prior proceeding by direct, cross-, or redirect examination of the declarant. Thus, the grand jury testimony of an unavailable declarant is not admissible as former testimony against the accused at trial. This is because grand jury proceedings do not provide the opportunity for cross-examination. Therefore, because the husband's testimony was in front of the grand jury and was not subject to cross-examination, it is inadmissible as hearsay. (A) is incorrect because the husband's testimony cannot be considered a vicarious admission. A statement by an opposing party (commonly known as an admission) is not hearsay under the Federal Rules. [Fed. R. Evid. 801(d)(2)] An admission is a statement made by a party and offered against that party. An admission does not have to be the statement of the party against whom the statement is being offered at trial if it qualifies as a vicarious admission. For example, admissions of one conspirator, made to a third party in furtherance of a conspiracy to commit a crime, may be admissible against co-conspirators. Here, however, the husband's grand jury testimony was not made in furtherance of a conspiracy. Because he was not a party here, and his testimony does not otherwise qualify as a vicarious admission of the wife, it cannot be considered an admission of a party-opponent. (B) is incorrect because the husband's grand jury testimony was not subject to cross-examination. Federal Rule 804(b)(1) allows the former testimony of an unavailable witness to be admitted under circumstances where the opportunity to develop testimony or cross-examine at the prior hearing was meaningful. The husband was an unavailable declarant because he was unable to testify because of death. [Fed. R. Evid. 804(a)(4)] However, as discussed above, his grand jury testimony is not admissible as former testimony because grand jury proceedings do not provide the opportunity for cross-examination. (C) is incorrect because the testimonial privilege does not belong to the wife in federal court and because it may only be asserted while the marriage relationship exists. In federal courts, this privilege belongs to the witness-spouse. This means that one spouse may testify against the other in criminal cases, with or without the consent of the party-spouse. Thus, while the husband could not have been compelled to testify against his wife, he could not be foreclosed by her from testifying (except as to confidential communications).
A wealthy sportsman purchased a large old wooden sailing ship for $200,000. Although the boat was a classic, the sportsman wanted it to be modernized and made more comfortable. To that end, the sportsman entered into a written contract with a marine architect-engineer to draw up and then execute the modernization plans, for $7,500. At the time the parties entered into the agreement, the sportsman told the architect-engineer that his modernization plan would be subject to the approval of the sportsman's sister, that they would, in fact, have no deal unless the plans meet with her approval. The architect-engineer agreed to this. He finished his drawings and submitted them to the sportsman, who was enthusiastic about the designs. The sportsman's sister, a famous yachtswoman, was engaged in a trans-Pacific yacht race at the time and was not expected home for a number of weeks. Cheered by the sportsman's enthusiasm, the architect-engineer went ahead and modernized the ship according to his designs. When he finished the work, he submitted a bill to the sportsman, who refused to pay, pointing out that his sister had never approved the designs. If the architect-engineer sues the sportsman, which of the following issues of contract law is most likely to be decisive in determining the outcome of the case? A Statute of Frauds. B Parol evidence rule. C Rules of construction. D Conditions precedent.
D Conditions precedent.
A drug dealer was convicted in federal court of possession of 10 kilos of cocaine with intent to distribute. She was sentenced to a prison term. Subsequently, a federal grand jury indicted the dealer under a separate statute for conspiracy to distribute the same 10 kilos of cocaine. She moved to dismiss the indictment. How should the court rule on her motion? A Grant it, because the Double Jeopardy Clause protects her against a second prosecution for the same criminal conduct. B Grant it, because the Due Process Clause protects her against double punishment for the same criminal conduct. C Deny it, because the Double Jeopardy Clause does not apply when the second prosecution is for violation of a separate statute. D Deny it, because each prosecution requires proof of an element that the other does not.
D Deny it, because each prosecution requires proof of an element that the other does not.
The defendant was arrested, given Miranda warnings, and charged with burglary. At the police station, he telephoned his mother and asked her to come to the station to post bail. Instead, his mother immediately called the family attorney. In the meantime, the police had begun questioning the defendant. Although he never told the police to stop the questioning, his answers were at first vague or clearly unresponsive. During the course of the questioning, the family attorney phoned the station and told the police that she had been hired to represent the defendant and would be there in half an hour. The police did not inform the defendant of the attorney's call. Ten minutes later, the defendant admitted to committing the burglary, and signed a statement to that effect prepared by the police. The attorney arrived a few minutes later and advised the defendant to remain silent, but he told her that he had already signed a confession. How should the court rule on the attorney's pretrial motion to exclude the confession as evidence at trial? A Grant the motion, because the police had a duty to inform the defendant that an attorney was coming to represent him. B Grant the motion, because the defendant has been deprived of his Sixth Amendment right to counsel. C Deny the motion, because the defendant's statement admitting the crime was voluntary. D Deny the motion, because the defendant waived his Miranda rights.
D Deny the motion, because the defendant waived his Miranda rights.
A husband and wife were traveling in a car with the wife driving when they were in an accident with a truck. The accident occurred in a jurisdiction that followed the traditional rule as to joint and several liability among tortfeasors. The husband sued the truck driver in federal district court. The truck driver, contending that the wife was an indispensable party, filed a motion to dismiss the action because the husband did not join his wife as a party to the action. How should the court rule on the truck driver's motion? A Grant the motion, because the wife is an indispensable party. B Deny the motion and order service of process on the wife to make her a party, provided she is subject to the court's personal jurisdiction. C Deny the motion, because the wife and the truck driver cannot be joined as co-defendants in the action. D Deny the motion, because the wife is not "needed for just adjudication."
D Deny the motion, because the wife is not "needed for just adjudication."
The owner of a hotel in a resort town was approached by a seminar speaker who wanted to lease space in which to conduct a two-week seminar. The owner leased to the speaker the hotel's grand ballroom, the period of the lease being August 1 through August 14. To provide the proper atmosphere for the seminars, the speaker attached curtain rods to the walls of the ballroom, using lightweight screws to attach the rods. The speaker then strung light blue ring curtains through the rods. After the seminar, on August 16, the speaker arrived to remove the curtains and rods. The owner brought an action to enjoin the speaker from removing the curtains and the rods from the grand ballroom. How should the court rule? A In favor of the speaker, because he had a short-term lease and the curtains and rods were easily removable. B In favor of the speaker, because curtains and rods are trade fixtures. C In favor of the owner, because the curtain rods were attached by screws, and as such were fixtures, which became part of the realty. D In favor of the owner, because the speaker did not remove the curtains and rods before the lease expired.
D In favor of the owner, because the speaker did not remove the curtains and rods before the lease expired.
A mother's will left her farm to her son and daughter "jointly, as tenants in common." The son and the daughter, having had no interest in farming, had long since moved to a large city about 150 miles from the farm. However, after the mother's death the son decided to move back to the farm. The son rented various parts of the farm to sharecroppers and regularly sent half of any profits from the farm to the daughter. A few years later the daughter died, leaving a will devising all of her property to a friend. The son, however, refuses to pay any of the profits of the farm to the friend and claims an exclusive interest in the farm. If the friend sues the son, how will a court most likely rule? A For the son, because he actively managed the use of the farm and the daughter never showed any interest in it. B For the son, because he survived the daughter, the other joint tenant. C For the son, because the unities of time, title, and interest have been destroyed by the daughter's death. D For the friend, because he inherited the daughter's interest.
D For the friend, because he inherited the daughter's interest. The friend will prevail because he inherited the daughter's interest. Although the language in the mother's will uses the word "jointly," the grant also states "as tenants in common." Because no right of survivorship is mentioned, the court will most likely find that this language establishes a tenancy in common, rather than a joint tenancy. The daughter can pass her interest in the farm by will, and thus the friend now holds the farm as a tenant in common with the son. (A) is wrong because the son's management of the use of the farm does not entitle him to an exclusive interest in it. (B) is wrong because the interest created by the mother's will was a tenancy in common, not a joint tenancy. (C) is wrong because the unities only apply to a joint tenancy.
A three-car accident occurred in a city in the Northern District of State A. The cars were driven by a citizen of State B who resides in the Southern District of State B, a citizen of State A who resides in the Southern District of State A, and a citizen of State C who resides in the Northern District of State C. The State B citizen filed a negligence action against the other two drivers in the United States District Court for the Southern District of State A. Although the two defendants believed that venue was improper, neither filed a pre-answer motion objecting to venue. They instead proceeded to file their answers, responding to the merits of the claim. The State C defendant, however, included in her answer a motion to dismiss the action for improper venue. How should the court rule on the State C defendant's motion to dismiss for improper venue? A Deny the motion, because the defendants waived the defense of venue by not asserting it in a separate, pre-answer motion prior to filing their answers addressing the merits of the action. B Deny the motion, because all defendants must object to venue before the court may dismiss on venue grounds. C Deny the motion, because venue is proper. D Grant the motion, because the motion is timely and venue is improper.
D Grant the motion, because the motion is timely and venue is improper. The court should grant the motion. Objections to venue may be raised in a defendant's answer if the defendant did not assert a Rule 12(b) pre-answer motion, as is the case here. Venue here is improper because (i) none of the events giving rise to the claim occurred in the Southern District of State A, and (ii) all defendants do not reside in State A. (A) is incorrect because a defendant can object to venue in a pre-answer motion or in the answer. (B) is incorrect because all defendants do not have to object to venue before the court may dismiss the case on venue grounds. (C) is incorrect because venue is improper, as explained above.
In order to get some quick cash to pay off a gambling debt, an acquaintance of the defendant asked him to pretend to break into the acquaintance's home, take some silverware, and return the silverware to the acquaintance. The acquaintance believed that he could both collect the insurance proceeds for the "theft" of the silverware, and sell the silverware on the black market. The acquaintance provided the defendant with his address: "46 Maple Avenue." However, due to a strong windstorm, the house number "9" for "49 Maple Avenue" became detached and slid out of position, making it look like "46 Maple Avenue." Thinking that "49 Maple Avenue" was the acquaintance's home due to the mispositioned house number, the defendant slid open a window that was slightly ajar, entered, and took some silverware he found in the kitchen. The silent burglar alarm alerted the police, who arrived and arrested the defendant a short time later. Why would a court find the defendant not guilty of burglary? A He acted under a mistake of law. B There was no breaking. C There was no entry. D He reasonably thought that he was in the acquaintance's home.
D He reasonably thought that he was in the acquaintance's home.
A town with a population of 30,000 merged with a city of 60,000. To protect voting rights of the citizens of the former town, a proposal was made that for a period of 20 years, beginning at the date of the merger, the city council of the merged city would consist of six persons. Each formerly separate municipality would be divided into three council districts. Each district from the former town would have approximately 10,000 residents, and each district from the former city would have 20,000 residents. A mayor would be elected at large. Before this proposal was placed on the ballot, the state attorney general issued an advisory opinion stating that the proposal was not in violation of any state statutory or constitutional provision. The proposal was placed on the ballot and was carried by large majorities in both the town and the city, and the districts were carved out. Three taxpayers filed suit to enjoin the holding of an election with council districts of such disparate proportions. The suit reached the state supreme court, which ruled that the governmental formula was constitutional under both the state and United States Constitutions. The plaintiffs wish to take the case to the United States Supreme Court. How should the Supreme Court proceed? A Rely on the attorney general's opinion and not hear the case on its merits. B Not hear the case, because it was decided below on an independent state ground. C Not hear the case, but remand it to federal district court. D Hear the federal issues involved, but decline to rule on state issues.
D Hear the federal issues involved, but decline to rule on state issues.
A national fast food franchisor filed an action against one of its franchisees, alleging breach of contract. Both parties are corporations. The franchisor alleged that the franchisee had failed to pay the franchisor the requisite percentage of the franchisee's profits as required by the franchise contract. The franchisor timely served on the franchisee corporation a notice of deposition, which named the franchisee corporation as the deponent and which stated that the deposition would inquire into the accounting methods and accounting records of the franchisee corporation. How should the defendant franchisee corporation respond to the fact that the notice of deposition does not name a person to be deposed? A It may safely ignore the notice. B It should alert the franchisor to the flaw in the notice of deposition. C It should file a motion for a protective order. D It should designate a person who will testify on its behalf about its accounting methods and accounting records.
D It should designate a person who will testify on its behalf about its accounting methods and accounting records.
On January 2, a retiree borrowed $1,000 from a friend, agreeing in writing to repay the loan within a year. In September, it became clear to the retiree that he would have difficulty meeting the deadline, and so he approached an acquaintance who owned a print shop with the following proposition: He would perform 200 hours of work for the print shop owner within six months at the special rate of $5 per hour, if the print shop owner would agree to pay $1,000 for the entire 200 hours to the retiree's friend on January 1. The print shop owner agreed. By January 1, the retiree had worked only five hours for the print shop owner, and the print shop owner refused to pay the retiree's friend because the retiree had not worked the agreed-upon number of hours. The retiree told the print shop owner that it was "no problem," and to just hold on to the money until he worked the full 200 hours, and then pay his friend. The print shop owner agreed. Subsequently, the retiree's friend learned of the arrangements between the retiree and the print shop owner and sued the print shop owner for $1,000. What is the probable result of this action? A Judgment for the friend, because he was a third-party beneficiary to the original contract between the retiree and the print shop owner and there was no consideration for the modification of the contract. B Judgment for the friend, because he was a third-party beneficiary to the original agreement between the retiree and the print shop owner and he did not agree to the modification. C Judgment for the print shop owner, because his contract was with the retiree and therefore he cannot be liable to the friend. D Judgment for the print shop owner, because the original agreement had been modified before the friend knew of the original agreement.
D Judgment for the print shop owner, because the original agreement had been modified before the friend knew of the original agreement.
When a crowded city bus braked suddenly, the standing passengers were thrown together, and a woman wearing very high-heeled shoes began to stumble. A man who was unacquainted with her kept her from falling by reaching his arm around her waist. If the woman sues the man for battery, will she recover? A Yes, because the man intended to put his arm around her waist. B Yes, because the man touched her without her permission. C No, because the man prevented her from harm. D No, because his conduct was socially acceptable.
D No, because his conduct was socially acceptable. The woman will not recover in a suit for battery because the man's contact did not constitute a harmful or offensive contact. In order to establish a prima facie case for battery, the following elements must be proved: (i) an act by the defendant that brings about harmful or offensive contact to the plaintiff's person; (ii) intent on the part of the defendant to bring about harmful or offensive contact to the plaintiff's person; and (iii) causation. Judged by this standard, the man's conduct in trying to keep the woman from falling in a crowded bus would not be harmful or offensive. Contact is offensive if it would be considered offensive by a reasonable person of ordinary sensibilities. It will be deemed "offensive" if the plaintiff has not expressly or impliedly consented to it. Consent may be implied from custom, conduct, or words, or by law. Under these facts the consent would be inferred as a matter of usage or custom. A person is presumed to consent to the ordinary contacts of daily life, which would include contact resulting from assistance to a fellow passenger in a crowded bus. (A) and (B) are incorrect. Even though the man intended to put his arm around the woman's waist and touched the woman without her permission, the touching was not harmful or offensive and therefore the man cannot be deemed to have committed a battery. (C) is incorrect. The fact that the man prevented her injury is not the determining factor for him to prevail; rather, it is that he acted with implied consent.
A landowner executed a will, devising a parcel of land "to my sister for life, then to my brother for life, then to my nieces and nephews." When the landowner died, he was survived by the sister and the brother's son. The sister and the brother's son contracted to sell the land to a buyer for $225,000. At the time set for closing, the sister and the brother's son tendered a quitclaim deed to the buyer, who refused to complete the sale. The sister and the brother's son bring suit against the buyer for specific performance. The jurisdiction in which the land is located does not follow the Doctrine of Worthier Title. Will specific performance likely be granted? A Yes, because a quitclaim deed conveys whatever interest the grantors have in the property. B Yes, because the interests involved are freely alienable. C No, because the jurisdiction does not follow the Doctrine of Worthier Title. D No, because title is unmarketable.
D No, because title is unmarketable
A toy makers' union, angry about poor working conditions and low wages, staged a nationwide strike just weeks before the holiday season. Larger toy sellers immediately hired independent toy makers to fulfill the traditionally increased demand for toys during the holidays. Enraged, striking toy makers committed acts of violence against independent toy makers and attempted to destroy shipments of independently made toys as they were being loaded off trucks at toy sellers' receiving docks. In response to the increasing violence, Congress met in emergency session and enacted a measure directing the President to send military troops to the affected areas to preserve order and to ensure the continued flow of commerce. Is this enactment constitutional? A Yes, under Congress's power to raise and support the armed forces. B Yes, under Congress's power to regulate commerce. C No, because it infringes on the President's authority to faithfully execute the laws of the United States. D No, because it infringes on the President's authority as Commander in Chief of the armed forces.
D No, because it infringes on the President's authority as Commander in Chief of the armed forces. The enactment appears to be an unconstitutional infringement on the President's authority as Commander in Chief. The President's role as Commander in Chief of the armed forces includes extensive power to deploy the military against any enemy, foreign or domestic. Congress lacks such power. Therefore, this enactment directly infringes on the President's authority as Commander in Chief to make such orders as he deems proper with respect to the armed forces, and thus violates the doctrine of separation of powers. (A) is incorrect. Congress does have the power to raise and support an army, but the enactment here does not result in the appropriation of money to support the armed forces. Rather, it seeks to control their activities, and Congress has no such power. (B) is incorrect. Congress does have the power to regulate commerce, and when that power is combined with the Necessary and Proper Clause, Congress would have the power to enact legislation protecting the toy shipments at issue here. Nevertheless, the Commerce Clause, even when combined with the Necessary and Proper Clause, does not give Congress the power to violate other aspects of the Constitution. Ordering the President to send military troops violates the separation of powers doctrine because the Constitution gives the President the power as Commander in Chief of the armed forces. (C) states the correct result but is based on an incorrect rationale—the duty to execute the laws of the United States is an obligation, not a grant of authority.
The defendant was charged with aggravated assault. The defendant did not testify at trial; however, he sought to offer opinion evidence of his good character for truth and veracity. Should this testimony be admitted? A Yes, because a criminal defendant is entitled to offer evidence of his good character. B Yes, because a party's credibility is necessarily in issue. C No, because evidence of character is not admissible to prove conduct in conformity therewith. D No, because it is evidence of a trait not pertinent to the case.
D No, because it is evidence of a trait not pertinent to the case.
A state set up an intrastate message routing system to carry messages to and from the various state agency offices located throughout the state. This proved to be cheaper and more efficient than the United States Postal Service. The message service worked so well that the state offered the messenger service to its employees as a fringe benefit. Moreover, it expanded delivery options beyond state offices to any address in the state and permitted the employees to use the service for personal correspondence as well as for official business. Are the state's actions constitutional? A Yes, because the messenger service operates entirely within the state borders. B Yes, because the Commerce Clause does not prohibit states from acting as a market participant. C No, because the Equal Protection Clause prohibits this singling out of state employees for special benefits. D No, because it violates the federal postal monopoly.
D No, because it violates the federal postal monopoly. The legislation is unconstitutional because it violates the federal postal monopoly. Article I, Section 8, Clause 7 of the Constitution grants Congress the power to establish post offices and post roads. This power grants Congress a monopoly over the delivery of mail. No other system for delivering mail—public or private—can be established absent Congress's consent. Congress has delegated to the Postal Service the power to decide whether others may compete with it, and the Postal Service has carved out an exception to its monopoly for extremely urgent letters. However, this exception would not apply to the state messenger service here since the state service extends to every letter or package of an employee deliverable within the state. (A) is irrelevant because the postal monopoly applies even to wholly intrastate competing systems. The rationale is that the Postal Service must be protected from companies that would deliver only on profitable routes at a low cost, leaving the Postal Service only expensive, money-losing routes. (B) also is irrelevant. While it is true that there is a market participant exception to the Commerce Clause, the Commerce Clause is not the controlling law here; the controlling law is the federal postal power. (C) is incorrect because the Equal Protection Clause would not prohibit the special treatment here. Because no suspect class or fundamental right is involved, the program would be judged under the rational basis standard. Under this standard, a law is upheld if it is rationally related to any legitimate government interest. Here, the law would be upheld because there is a conceivable rational basis for the program (e.g., to make government employment more attractive), and the law is rationally related to that interest.
The plaintiff sued the defendant, who had constructed the plaintiff's house, for breach of warranty of habitability. At trial, in cross-examination of the plaintiff, the defendant's attorney asked whether the plaintiff had sued another contractor 30 years earlier, claiming similar defects in another house built for the plaintiff. The question was not objected to and the plaintiff answered that she had had some "water problems" with the first house she ever purchased, but no suit was filed. The defendant then called as a witness the contractor of 30 years earlier to testify that the plaintiff had brought suit against him for defects in the earlier house, many of which were like those now claimed to be found in the home the defendant built, but that the case was settled without trial. Should the trial court rule that the witness's offered testimony is admissible? A Yes, as proper impeachment because the plaintiff will have an opportunity to explain or deny the witness's statement. B Yes, because the plaintiff failed to object to the defendant's questions on cross-examination relative to the prior suit. C No, because the best evidence of the former suit is the court record. D No, because its probative value is substantially outweighed by the danger that it will confuse the issues and waste time.
D No, because its probative value is substantially outweighed by the danger that it will confuse the issues and waste time.
A plaintiff has brought suit in federal district court against the Social Security Administration because it denied her retirement benefits on the asserted ground that she had not reached the requisite age to qualify. At trial, the plaintiff introduced into evidence a family Bible given to her by her father in which is inscribed her date of birth, showing her to be 65 years old. The government introduced a certified copy of the plaintiff's birth certificate, which shows her age to be 55. The court admitted both items over objection of the nonpropounding party. Was this error? A Yes, as to the Bible only, because it contained inadmissible hearsay. B Yes, as to the birth certificate only, because it was not authenticated by the custodian of records. C Yes, as to both, for the reasons stated in the previous answers. D No, both records were admissible.
D No, both records were admissible.
A mother received a telephone call asking her to meet with the vice principal of her son's school because he was caught cheating. As she was waiting in the hallway to meet with the vice principal, the school's janitor, a big man who was an amateur body builder, introduced himself as the vice principal and asked the mother to step into his office, closing the door behind him. For several minutes, the janitor asked the mother a series of flirtatious personal questions, causing the mother to grow very uncomfortable. However, she did not leave the office out of concern for her son's academic standing. The actual vice principal eventually returned to his office and ended the questioning. If the mother sues the janitor for false imprisonment, is she likely to prevail? A Yes, because the janitor was a body builder who could have physically harmed her if she tried to leave the room. B Yes, because she would not have stayed in the room if the janitor had not pretended to be the vice principal. C No, because the janitor would have let her leave if she wanted. D No, because she had no reason to believe that she could not leave.
D No, because she had no reason to believe that she could not leave.
A man from a foreign country obtained a doctorate in political science from a state university and applied to teach there. The man was denied employment at the university under a state law requiring all teachers within the state to be United States citizens. Is the state's citizenship requirement constitutional as it applies to the man? A Yes, because states have the right to set minimal standards for state employees under the Tenth Amendment. B Yes, because a university political science teacher would exert a great deal of influence over the attitudes of students toward government, the political process, and citizenship. C No, because the citizenship requirement is not rationally related to a legitimate state interest. D No, because the citizenship requirement is not necessary to achieve a compelling state interest.
D No, because the citizenship requirement is not necessary to achieve a compelling state interest. A state generally may not discriminate against aliens absent a compelling state interest, and no compelling interest is served by prohibiting aliens from teaching at a state university. (A) is incorrect. The Tenth Amendment reserves to the states power not granted to the federal government. The Constitution vests the power to regulate aliens in Congress, and thus the states do not have power to control aliens under the Tenth Amendment. (B) is incorrect because it states the standard that the Supreme Court has applied to primary and secondary school teachers. The Supreme Court has upheld state statutes prohibiting aliens from teaching primary or secondary school on the rationale that teachers at the elementary and high school level have a great deal of influence over the attitudes of young students toward government, the political process, and citizenship. It is doubtful that the Court would extend this rationale to university teachers. (C) is incorrect because it states the wrong standard. If state discrimination against aliens relates to participation of aliens in the functioning of state government, the rational basis test applies. Merely teaching political science at a state university is not equivalent to participating in the political process.
A homeowner contracted with a local builder to build a wooden deck onto the back of her house. The contract called for half of the contract price of $20,000 to be paid to the contractor before he began work and the other half to be paid to him when the job was completed. The contractor began the work but, partway through the job, he got an offer for a rush job that paid better and abruptly quit. The homeowner sues the contractor for specific performance. Will she prevail? A Yes, because there has been a novation. B Yes, because the contract between the parties was valid and the contractor had no legal justification for abruptly quitting. C No, because by not paying the contractor for the second half of the job, the homeowner has not satisfied all of her conditions under the contract. D No, because the contract is for personal services.
D No, because the contract is for personal services.
After being notified by a doctor that her employment with his office was terminated, a nurse applied for a position with a hospital. In her application, the nurse listed her former employment with the doctor, with the understanding that the doctor might be contacted. The doctor, in response to a telephone inquiry from the hospital, stated that the nurse "lacked professional competence." Although the doctor reasonably believed that to be a fair assessment of the nurse, his adverse rating was based on an episode of malpractice for which he blamed the nurse but which in fact was chargeable to another doctor. Because of the doctor's adverse comment on her qualifications, the nurse was not employed by the hospital. If the nurse asserts a claim based on defamation against the doctor, will the nurse prevail? A Yes, because the doctor was mistaken in the facts on which he based his opinion of the nurse's competence. B Yes, because the doctor's statement reflected adversely on the nurse's professional competence. C No, because the nurse authorized the hospital to make inquiry of her former employer. D No, because the doctor had reasonable grounds for his belief that the nurse was not competent.
D No, because the doctor had reasonable grounds for his belief that the nurse was not competent. The nurse will not prevail because the doctor had reasonable grounds for his statement. As a former employer responding to queries of a prospective employer about a job applicant, the doctor has a qualified privilege. Such a privilege is not absolute; it exists only if exercised in a reasonable manner and for a proper purpose. The privilege may be lost if the speaker made a statement not within the scope of the privilege or if the speaker acted with actual malice (i.e., knowledge that the statement was untrue or with reckless disregard as to its truth or falsity). Because the doctor had reasonable grounds for his belief, he was not acting with actual malice. (A) is incorrect because of the reasons stated in the analysis above. A statement of opinion may be actionable if it appears to be based on specific facts which, if expressly stated, would be defamatory. However, because of the qualified privilege, the doctor will not be liable for his mistake as long as his belief was reasonable. (B) is incorrect because the fact that the statement was in a category that is slander per se (i.e., adversely reflecting on the nurse's abilities to practice her profession) goes to whether the nurse must plead special damages. It does not, however, undermine the qualified privilege. (C) is incorrect because permission to make inquiry is not tantamount to consent to be defamed.
A state law provides that all persons who have been residents of the state for more than three years shall be entitled to free tuition at the state's main university. It further provides that persons who have resided in the state for three years or less shall pay the nonresident tuition rate, which is significantly higher. A student at the state's university who had been a state resident for less than three years filed a class action in federal court on behalf of himself and other similarly situated university students, seeking a declaration that the state statute is unconstitutional. When the case came to trial, the student had been a resident of the state for more than three years and was no longer required to pay tuition. By that time, a number of amicus curiae briefs had been filed in the case, some supporting and some opposing the student's position. Nevertheless, the state moved to dismiss the case as moot. Should the state's motion to dismiss be granted? A Yes, because the student is now a three-year resident. B Yes, because the student lacks standing. C No, because amicus curiae briefs have been filed. D No, because there is a live controversy.
D No, because there is a live controversy.
A homeowner looked out his front window one day and saw a neighbor standing on a narrow ledge on the second story of the house across the street. He also saw a ladder lying on the ground beneath where the neighbor was stranded. The homeowner ran out and picked up the ladder and placed it against the side of the house. However, he set it atop a patch of ice. As the neighbor started down the ladder, a rotten rung broke and he fell to the ground and was injured. If the neighbor sues the homeowner for damages for his injuries, will he recover? A Yes, because the homeowner's action caused the injury to the neighbor. B Yes, because the homeowner assumed the duty of aiding the neighbor. C Yes, because it was foreseeable that the neighbor would be injured as a result of the homeowner's negligent conduct. D No, because the homeowner's negligence did not cause the injury to the neighbor.
D No, because the homeowner's negligence did not cause the injury to the neighbor. The neighbor will not recover from the homeowner because even if the homeowner acted negligently in setting the ladder atop a patch of ice, this negligence did not cause the injury to the neighbor. A person generally is under no duty to assist another. Therefore, the homeowner was under no duty to assist the neighbor. However, having gratuitously undertaken to do so, the homeowner came under a duty to act as an ordinary, reasonable person while rendering such assistance. He breached this duty by setting the ladder atop the patch of ice, thus creating an unreasonable risk that the ladder would slip while the neighbor was climbing down, causing him injury. However, the homeowner is not liable for the neighbor's injuries unless the homeowner's breach of duty caused those injuries. Before a defendant's conduct can be considered a proximate cause of the plaintiff's injury, it must first be a cause in fact (actual cause) of the injury. An act is the cause in fact of an injury when the injury would not have occurred but for the act. Here, the neighbor's fall and injuries would not have occurred but for the rotten rung. There is no indication that the homeowner's negligence in placing the ladder on the ice contributed in any manner to the injuries. If the homeowner had carefully placed the ladder on a solid, ice-free surface, the neighbor would have incurred the same injury by stepping on the rotten rung. Therefore, the homeowner's negligence was not a cause in fact of the neighbor's injuries. Because the element of causation is missing, the homeowner will not be liable for the injuries to the neighbor. (A) is wrong because the homeowner's negligent placement of the ladder was not the cause of the neighbor's injury. Rather, the neighbor fell as a result of stepping on the rotten rung. Thus, it cannot be said that the homeowner's negligent conduct caused the injury to the neighbor. (B) is wrong because the homeowner's assumption of the duty to aid the neighbor does not render him absolutely liable for all injuries incurred. As explained, the homeowner did assume the duty to act reasonably in aiding the neighbor, and he did breach his duty by placing the ladder on ice. Nevertheless, this breach of duty did not cause the injuries to the neighbor. Thus, if the homeowner's negligence did not cause the injury to the neighbor, he cannot be held liable, even if he was negligent. Regarding (C), while it is true that it was foreseeable that the neighbor would be injured as a result of the homeowner's negligent conduct (i.e., that the ladder would slip on the ice, causing the neighbor to fall and be injured), it is also true that the neighbor was not injured as a result of the homeowner's negligence. The homeowner cannot be held liable for something that was not in any way caused by his negligent conduct. Therefore, (C) is incorrect.
A landowner was declared insane and committed to a state mental hospital 30 years ago. Five years after that, a trespasser entered onto the landowner's 200-acre parcel of land, which was enclosed by a barbed wire fence. A solid wood fence ran through the middle of the land, separating the property into approximately equal east and west parcels. The trespasser began grazing cattle on the west parcel; no one else has been in possession of any part of the 200 acres. The period of time to acquire title by adverse possession is 15 years. Thirteen years ago, the landowner was declared competent and released from the hospital, but he did nothing until this year, when he brought an action to eject the trespasser. The trespasser counterclaimed to quiet title in him. In this action, will the trespasser likely prevail? A Yes, because he has acquired title to the 200 acres by adverse possession. B Yes, but only as to the west parcel, because that is the portion of the land he actually occupied during his adverse possession. C No, because the landowner can assert the defense of laches, as the trespasser did not bring an action to quiet his title within a reasonable time after the statute had run. D No, because the landowner was insane for 12 of the 25 years that the trespasser was in possession.
D No, because the landowner was insane for 12 of the 25 years that the trespasser was in possession.
While in a department store, a man picked up a sweater and slipped it under his shirt. The man then started for the door. A woman, who also was shopping in the store, saw the man take the sweater. The woman grabbed a baseball bat from the sporting goods aisle and chased the man into the parking lot. The woman began swinging the bat at the man's head, hoping to knock him out and thus prevent the theft. The man pulled a knife from his pocket and stabbed the woman, killing her. The man was arrested and charged with murder. At trial, will the man most likely be found guilty? response - correct A Yes, because the evidence shows that he provoked the assault on himself by his criminal misconduct. B Yes, because the evidence shows that the man intended to kill or cause serious bodily harm. C No, because the jury could find that the man acted recklessly and not with the intent to cause death or serious bodily harm. D No, because the man was acting in self-defense.
D No, because the man was acting in self-defense
Acting on information from reliable informants that drugs were being sold by residents at a certain fraternity house, the police obtained a search warrant that entitled them to search the entire premises for illegal narcotics. The police arrived at the house when a party was in progress and were admitted to the house by the fraternity president after showing the warrant. Officers proceeded to search the house. In an upstairs bedroom, they found a young woman who was a guest of a fraternity member sleeping on the bed. No one else was in the room. The police found a footlocker under the bed and opened it, finding a variety of illegal drugs. The police then awakened the woman and seized her purse from her. They found a small quantity of marijuana in the purse. The woman was charged with a drug possession offense. At her trial, the prosecution seeks to admit the marijuana seized from her purse over the objection of her attorney. Should the court admit the marijuana? A Yes, because the footlocker was within the woman's reach. B Yes, because the woman was present in a room where drugs were found. C No, because the woman had no possessory interest in the premises. D No, because the police had no reason to believe that the woman had drugs on her person.
D No, because the police had no reason to believe that the woman had drugs on her person.
A homeowner hired a computer repair technician to fix his computer. The technician discovered that she needed a different type of memory card than she had, so she drove to the local electronics store to get what she needed. On the way back, she stopped at a liquor store to get a bottle of wine for a dinner party she was attending that night. When she was backing out of the parking lot, she negligently knocked over a pedestrian who was walking on the sidewalk adjacent to the parking lot. In a negligence action by the pedestrian against the homeowner, is the pedestrian likely to prevail? A Yes, because a principal is vicariously liable for the negligence of his agent. B Yes, because the technician was on a detour and not a frolic. C No, because the technician was on a frolic and not a detour. D No, because the technician is an independent contractor.
D No, because the technician is an independent contractor.
An employee properly sued her employer for wrongful discharge in federal court. During discovery, the employee served the employer with a discovery request for information regarding all employment termination over the previous 15 years, regardless of the position. The employer objected, and the employee filed a motion to compel the requested discovery. The court denied the motion to compel, and the employee wants to file an immediate appeal to review this decision. Does the employee have a right to an immediate appeal? A Yes, because the trial court's order is a "collateral order" that is immediately appealable under the collateral order rule. B Yes, because the trial court's order is a "final judgment" that is immediately appealable. C Yes, because the plaintiff may obtain immediate appellate review of the trial court's order through a writ of mandamus. D No, because the trial court's order is an "interlocutory order" that can be reviewed prior to final judgment only if the trial court certifies it for immediate appeal and the appellate court, in its discretion, agrees to hear the appeal.
D No, because the trial court's order is an "interlocutory order" that can be reviewed prior to final judgment only if the trial court certifies it for immediate appeal and the appellate court, in its discretion, agrees to hear the appeal.
The driver of a tractor-trailer lost control of his vehicle after driving onto an icy bridge in excess of the speed limit and slid off the road. A state trooper responding to the accident got out of his squad car and walked toward the vehicle on the roadway. Before he could set up warning flares or safety cones, a delivery van approached the bridge in the oncoming lane. The van slid on the ice and spun out of control, striking and injuring the trooper. The trooper brought an action against the driver of the delivery van and the driver of the tractor-trailer. Will the trooper prevail as against the driver of the tractor-trailer? A Yes, because the trooper's injuries were a foreseeable result under the circumstances. B Yes, because the "firefighters' rule" does not apply in this case. C No, because the delivery van striking the trooper was the actual cause of his injuries. D No, because the trooper was injured during the course of his job.
D No, because the trooper was injured during the course of his job.
A plaintiff corporation incorporated in State A with its principal place of business therein entered into a contract with the defendant under which the defendant agreed to manufacture and sell equipment to the plaintiff. The plaintiff plans to sue the defendant for $1 million because the equipment does not conform to the contract specifications and does not operate properly. The defendant is a corporation that is also incorporated in State A but has its principal place of business in State B. Would a federal district court have subject matter jurisdiction over the plaintiff's action? A Yes, because the two corporations are citizens of different states based on their principal places of business. B Yes, because the transaction involves interstate commerce. C Yes, because both parties have sufficient contacts with State A. D No, because the two corporations are both citizens of State A based on their place of incorporation.
D No, because the two corporations are both citizens of State A based on their place of incorporation.
A woman decided to have a painting done of herself. She contracted in writing with an artist, who agreed to paint the woman for $10,000. The fee was payable on completion of the painting, provided that the painting was to the woman's "complete and utter satisfaction." On the same afternoon that the artist entered into the contract with the woman, he assigned the contract to his cousin. The artist then painted the woman's picture. After the job was done, the woman told him, "That's a very good likeness of me, but it shows my defects, so I'm not satisfied." She refused to accept the painting or to pay the artist or his cousin. Can the cousin recover from the woman? A Yes, because the condition in the agreement between the woman and the artist did not apply to his cousin. B Yes, because otherwise an unjust enrichment will occur. C No, because rights arising under personal services contracts are not assignable. D No, because the woman was not satisfied with the painting.
D No, because the woman was not satisfied with the painting.
The owner of an apartment building contracted with a painter to paint the porches of the apartments for $5,000. The contract was specifically made subject to the owner's good faith approval of the work. The painter finished painting the porches. The owner inspected the porches and believed in good faith that the painter had done a bad job. The painter demanded payment, but the owner told him that the paint job was poor and refused to pay. The painter pleaded that he was desperately in need of money. The owner told the painter that she would pay him $4,500, provided he repainted the porches. The painter reluctantly agreed, and the owner gave the painter a check in the amount of $4,500. The painter went to his bank, indorsed the check "under protest" and signed his name, then deposited the check in his account. He never returned to repaint the porches. The painter sues the owner for $500, which he believes is still owed to him on his contract to paint the porches. Will he prevail? A Yes, because he indorsed the check "under protest." B Yes, but only if he repaints the porches. C Yes, because he performed the contract by painting the porches the first time. D No, even if he repaints the porches.
D No, even if he repaints the porches.
A consultant operated a consulting firm from an office in his home. An employee asked if she could stay late one night to use one of the firm's computers. The consultant replied that she could consider the computer hers. The employee mistakenly believed that the consultant was giving her the computer. Late the next night, when the employee could borrow her roommate's car, she drove to the consultant's house to pick up the computer. She went to the door leading directly to the office, which was unlocked. She let herself in and took the computer. The next day, the consultant reported the computer as stolen, and the police arrested the employee. What crime has the employee committed? A Burglary. B Attempted burglary. C Larceny. D None of the listed crimes.
D None of the listed crimes.
A seller put her house and lot on the market for $200,000. After receiving several offers within $5,000 of her asking price, the seller entered into a contract to sell the house and lot to a buyer for $200,000. The contract provided that the buyer put up $4,000 in earnest money, which the seller could treat as liquidated damages unless: The seller fails to tender marketable title to the buyer by the agreed-upon closing date, the seller commits a material breach of this contract, or the buyer dies prior to the closing date, in which case the earnest money shall be reimbursed to the buyer's estate. The contract was signed on July 24, and the closing date was set for September 12. On August 5, the buyer was seriously injured in an accident. On September 10, the buyer was released from the hospital in a wheelchair. He determined that a ranch-style house would make his life much more bearable, but the seller's home was two stories. The buyer asked the seller to cancel the contract and to refund the $4,000 earnest money. The seller refused. The buyer did not appear on the closing date. On September 16, the seller contracted to sell the home to a purchaser for $198,000. The closing occurred as planned on October 20. The buyer files suit against the seller, praying for a refund of the $4,000 earnest money. How much is the buyer likely to recover? A The entire $4,000, because the buyer had a justified medical reason for his failure to perform. B $2,000, because the diminution in value of the property was only $2,000. C $2,000 less any of the seller's out-of-pocket costs involved in remarketing the home. D Nothing, because at the time the contract was entered into, $4,000 represented a reasonable estimate of damages in the event of breach.
D Nothing, because at the time the contract was entered into, $4,000 represented a reasonable estimate of damages in the event of breach.
A merchant sued a company for breach of contract, alleging that the products she purchased failed to conform to contract specifications. Shortly before the trial was to begin, the merchant suffered a stroke that left her paralyzed and virtually unable to communicate. Her guardian was properly substituted as the plaintiff in the lawsuit. At trial, following presentation of the plaintiff's case, the company calls as a witness a priest to question him about a conversation he had with the merchant at a church fundraiser. In this conversation, the merchant told the priest in confidence that the products she received were actually quite functional, but that she had become aware of a lower price being offered by another vendor, and thus wanted to get out of her contract with the company. The plaintiff's attorney immediately objects on the basis of clergy-penitent privilege. How should the court rule on the objection? A Sustained, because the merchant's statement was made to the priest in confidence. B Sustained, because this is not a criminal case. C Overruled, because the privilege can be invoked only by the person who made the confidential statement. D Overruled, because the circumstances under which the merchant made the statement take it outside the scope of the privilege.
D Overruled, because the circumstances under which the merchant made the statement take it outside the scope of the privilege.
A State A citizen filed a civil action against a State B citizen in a State B state court. The action arose from events that took place in State C. State C has only one federal district court (the District of State C). State B has two, the Northern District of State B and the Southern District of State B. The State B citizen resides in the Southern District of State B, while the state court action filed by the State A citizen is pending in a court located in the Northern District of State B. If the State B citizen wishes to remove the action to federal district court, in which federal district should the State B citizen file a notice of removal? A Either the Southern District of State B, the Northern District of State B, or the District of State C. B Either the Southern District of State B or the District of State C. C Either the Southern District of State B or the Northern District of State B. D The Northern District of State B only.
D The Northern District of State B only. The notice of removal should be filed in the Northern District of State B only. The federal removal statute provides that the notice of removal should be filed in the federal district court for the district that geographically encompasses the state court from which the action is being removed. Thus, (D) is correct. (A) incorrectly states that a notice of removal may be filed in the federal district in which a substantial part of a lawsuit took place or in any federal district of the state in which the defendant resides. (B) incorrectly states that a notice of removal may be filed in the federal district in which the events took place or in which the defendant resides. (C) incorrectly states that the notice of removal may be filed in any district of the state in which the defendant resides.
A small group of terrorists hijacked a foreign airliner. Ten prominent bankers from the United States were among the airline passengers. The terrorists demanded that three individuals imprisoned in a state prison be released. Although these individuals were properly convicted of violation of state laws, the terrorists contended that they were political prisoners and that their convictions were merely a subterfuge to prevent them from inciting the local populace. The President of the United States agreed to release the three prisoners in return for the freedom of the 10 bankers. He issued official pardons and ordered the state governor to release the prisoners. The governor refused to do so, contending that the President had overreached his constitutional powers and had violated the concept of federalism. What is the best argument for the governor's position? A The President had no power to make an agreement with the terrorists because his treaty power extended only to dealings with the legitimate governments of other nations. B The Constitution immunizes a governor acting pursuant to a state constitution from inconsistent orders by a federal official. C The President's duty to see that the laws are faithfully executed prevents him from ordering state officials to release prisoners. D The President's constitutional power to grant pardons extends only to prisoners who have violated federal laws.
D The President's constitutional power to grant pardons extends only to prisoners who have violated federal laws.
Congress created a seven-member safety commission to investigate and make recommendations to Congress for new fireworks safety laws, to make further rules for establishing safety and performance standards, and to prosecute violations of these safety standards. The chairman of the commission was appointed by the President. Three members were selected by the Speaker of the House of Representatives, and three members were selected by the President pro tempore of the Senate. An organization with proper standing seeks to enjoin enforcement of the commission's rules. Which of the following presents the strongest constitutional argument that the organization can make against the validity of the commission? A The commerce power does not extend to activities occurring solely within a state. B Legislative power may not be delegated by Congress to an agency in the absence of clear guidelines. C The organization is denied due process of law because it is not represented on the commission. D The commission lacks authority to enforce its standards because some of its members were appointed by Congress.
D The commission lacks authority to enforce its standards because some of its members were appointed by Congress.
A homeowner hired a contractor to make some improvements on his house. They entered into a written contract providing that the contractor would do the improvements for $5,000. Shortly after the contract was signed, the contractor told the homeowner to give the money to his (the contractor's) daughter when the job was finished, adding, "She is getting married soon and I want her to have a nice wedding present from me." The daughter was aware that her father made this statement to the homeowner. She married, but soon thereafter the contractor told the homeowner to pay him the $5,000, and not the daughter, because his son-in-law had a gambling problem and would probably use the money to bet at the racetrack. What is the best argument in favor of the daughter's being able to enforce a contract for $5,000 in her favor? A Statute of Frauds. B Parol evidence rule. C The daughter was an intended third-party beneficiary. D The daughter married in reliance on the promise.
D The daughter married in reliance on the promise.
A property owner conveyed commercial property in joint tenancy to his two daughters as a birthday present. The deed from the property owner to his daughters was never recorded. After a few years, the property owner no longer wished the daughters to control valuable commercial property, and so he demanded that they return the deed with which he conveyed the property to them. The daughters returned the deed, and the property owner destroyed it. The property owner then sold and conveyed the property to a third party. The jurisdiction's recording act states the following: "No conveyance or mortgage of an interest in land is valid against any subsequent purchaser for value without notice thereof whose conveyance is first recorded." If the third party brings a quiet title action and is successful, which of the following best explains this result? A As owner of the property, the property owner was entitled to convey it to the third party. B The daughters failed to record the deed they took from their father. C The daughters failed to record their deed, and the third party was unaware of their interest when she paid the property owner market value for the property. D The daughters failed to record their deed, the third party was unaware of their interest when she purchased the property, and the third party recorded her deed.
D The daughters failed to record their deed, the third party was unaware of their interest when she purchased the property, and the third party recorded her deed.
Which of the following will not negate a claim of strict liability for an abnormally dangerous activity? A There was an unforeseeable intervening force. B The type of harm that occurred was not foreseeable. C The plaintiff was not foreseeable. D The defendant did not foresee a risk of harm.
D The defendant did not foresee a risk of harm. The fact that the defendant did not foresee a risk of harm will not negate a claim of strict liability for an abnormally dangerous activity. To prevail, a plaintiff need only show that a reasonable person could have foreseen the risk of harm, regardless of whether the defendant did not. If the plaintiff was not foreseeable, the strict liability claim is not established. The defendant's liability for an abnormally dangerous activity extends only to foreseeable plaintiffs, who are persons to whom a reasonable person would have foreseen a risk of harm under the circumstances. Note, though, that the nature of the abnormally dangerous activity may create a large class of foreseeable plaintiffs. If the type of harm was not foreseeable, the plaintiff cannot establish a strict liability claim. The harm must result from the kind of danger to be anticipated from the abnormally dangerous activity; i.e., it must flow from the "normally dangerous propensity" of the condition or thing involved. An unforeseeable intervening force may allow a defendant to avoid liability in a strict liability action for an abnormally dangerous activity. The same rules govern causation for strict liability as they do for negligence, and thus a defendant's liability may be cut off by an unforeseeable intervening force that brings about the injury.
A lessee rented from the defendant a building containing a store on the ground floor and an apartment above. The lessee operated a jewelry store on the ground floor and lived in the apartment above. One night as the defendant happened by the store, he saw a light on in the shop and, finding the door unlocked, decided to investigate. While looking around the store, the defendant spotted an expensive watch. Recalling that the lessee was much behind in her rent, the defendant decided to take the watch and keep it until the lessee paid the rent. Just as the defendant was leaving the store, the lessee entered the store. Afraid of a confrontation, the defendant tossed the lessee the watch and ran out. What fact will prevent the defendant from being convicted of common law burglary? A The door was unlocked. B A different part of the building was used as a dwelling. C The defendant owns the building. D The defendant had no intent to commit any crime until he entered the shop.
D The defendant had no intent to commit any crime until he entered the shop
At a victory party after a hard-fought election, the campaign director consumed several drinks. A campaign worker who had also been drinking took the director to her hotel room for a nightcap. They later had intercourse. The worker filed a complaint with the police, claiming that the director had intercourse with her against her will, and the director was charged with rape. Which of the following is most likely to be admitted in the director's defense? A The worker has a reputation in the community as being sexually promiscuous. B Since the incident occurred, the worker has had sexual intercourse with two other campaign workers. C Two years ago during the candidate's previous campaign, the worker maintained a sexual relationship with the former campaign director. D The director and the worker had had consensual sex on two prior occasions.
D The director and the worker had had consensual sex on two prior occasions.
A doctor and a lawyer owned adjoining parcels of land. Ten years ago, the doctor installed a swimming pool on her land. The doctor obtained the lawyer's oral consent to run plumbing from the pool across part of the lawyer's land. Last year, the lawyer sold his land to a buyer. The buyer wants to plant a garden on the land under which the doctor's plumbing runs, and wants to eject the doctor and quiet title. The statute of limitations for ejectment is seven years. With respect to the land under which the plumbing was laid what, if anything, has the doctor acquired? A The doctor has acquired title by adverse possession. B The doctor has acquired a prescriptive easement. C The doctor has acquired both title by adverse possession and a prescriptive easement. D The doctor has acquired neither title by adverse possession nor a prescriptive easement.
D The doctor has acquired neither title by adverse possession nor a prescriptive easement. The doctor has acquired neither title by adverse possession nor a prescriptive easement in the land under which the plumbing was laid. To establish title by adverse possession, the possession must be (i) actual and exclusive, (ii) open and notorious, (iii) adverse (hostile), and (iv) continuous throughout the statutory period. Exclusive possession generally means not sharing possession with the true owner or the general public. Possession is open and notorious when it is such as the usual owner would make of the land and is sufficient to put the true owner on notice of the fact of possession. Possession is hostile when it is without the owner's consent. Continuous possession is possession that the average owner would make of the property under the circumstances. Acquiring a prescriptive easement is analogous to acquiring property by adverse possession; however, the use need not be exclusive. Here, although the doctor's use and possession of the lawyer's land was actual, exclusive, open, notorious, and continuous throughout the seven-year statutory period, the doctor used the land with the lawyer's consent, and there are no facts to indicate that the doctor in any way manifested an intention to claim the land as her own. Therefore, the "hostile" requirement is not met and (A), (B), and (C) are wrong.
A state's highway speed limits were 65 miles per hour in its flat land regions and 55 miles per hour its mountainous regions. To reduce traffic fatalities and combat the fact that most of the vehicles on state highways were exceeding posted speed limits, the state legislature proposed banning the use of radar detectors. Citizens in the mountainous regions of the state, where most of the state's highway fatalities occurred, generally supported the ban, but citizens in the flat regions of the state opposed the ban, so the legislature adopted a law banning use of radar detectors on any road with a speed limit below 60 miles per hour. A driver whose car was equipped with a radar detector lived in the mountainous region of the state but frequently drove to the state's flat region. While on a mountain highway with a posted speed limit of 55 miles per hour, the driver was pulled over by a state trooper for speeding. While approaching the driver's car, the state trooper noticed that the driver's radar detector was turned on. The trooper ticketed the driver for both speeding and illegal use of a radar detector. The driver challenges his ticket for use of the radar detector, arguing that it is unfair to allow people in the flat lands to use radar detectors while prohibiting residents of the mountainous region from using them. Which of the following statements is correct regarding the burden of proof in such a case? A The state will have to prove that the ban serves a compelling state interest. B The state will have to prove that the ban is rationally related to a legitimate state interest. C The driver will have to prove that the ban does not serve a compelling interest. D The driver will have to prove that the ban is not rationally related to a legitimate state interest.
D The driver will have to prove that the ban is not rationally related to a legitimate state interest.
A landowner owned a large parcel of land that he divided into two equal parcels. Thirty years ago, the landowner deeded the eastern parcel to a purchaser by warranty deed, including an easement over the south 25 feet of the western parcel for access to the navigable river that ran along the westerly boundary of the western parcel. The landowner acknowledged the deed and easement, and the purchaser recorded the document. The recording officer maintains an alphabetical grantor-grantee index, but no tract index. The purchaser made no use of the easement until five years ago, one year after her neighbor had purchased the western parcel from the landowner. The neighbor had paid at least market value for the western parcel and was not aware of the purchaser's easement. The neighbor objected to the purchaser's use of the easement shortly after she began using it, but the purchaser paid no attention. The neighbor sues the purchaser to quiet his title and to restrain the purchaser from using the easement over the western parcel. The purchaser has reasonable access to a public highway on the easterly boundary of the eastern parcel. If the purchaser is successful, what is the likely reason? A The absence of a tract index requires that the neighbor make inquiry regarding the riparian rights of owners abutting his property. B The neighbor and the purchaser trace their title to a common grantor, the landowner, whose covenants for title run with the land and estop the neighbor from denying the purchaser's title. C An easement is a legal and incorporeal interest that is not just attached to an estate in the land, but runs with the land itself and therefore binds successive owners of the servient estate regardless of notice. D The easement is a legal interest in the neighbor's chain of title even though there is no tract index.
D The easement is a legal interest in the neighbor's chain of title even though there is no tract index. If the purchaser prevails, it will be because the easement is a legal interest in the neighbor's chain of title. This is a recording act problem. Even though the neighbor had no actual or inquiry notice, the recorded easement by his grantor, the landowner, would give him constructive notice of the purchaser's interest in the western parcel, regardless of the absence of a tract index. (A) is a misstatement of the law. The absence of a tract index does not require a purchaser to inquire about the riparian rights of abutting landowners. (B) is incorrect because covenants for title are those contained in a general warranty deed. Although the future covenants for quiet enjoyment, warranty, and further assurances run to successive grantees, the present covenants of seisin, right to convey, and against encumbrances are breached, if at all, at the time of conveyance. (C) is incorrect because a bona fide purchaser of the servient parcel with no notice of the easement takes free of the easement.
The President of the United States issued an executive order banning conversation by all executive employees with members of the press unless prior permission had been obtained from a supervisor. Executive Department employees were subject to dismissal for violation of the order. An employee of the United States Department of Agriculture spoke to a reporter and told the reporter that the USDA sanctioned the approval of a dangerous pesticide because of payoffs made by lobbyists to a high-ranking USDA official. The newspaper printed the story and quoted the employee by name. After a hearing in line with civil service regulations, the employee was found to have violated the executive order barring unauthorized conversations with news media reporters, and the employee was fired. The employee brought suit in federal court for reinstatement, back pay, and other benefits and also prayed that the court should strike down the executive order as unconstitutional. If the employee's case reaches the United States Supreme Court, how should the Court rule? A The executive order is constitutional, because the President has plenary power to control Executive Department employees. B The executive order is constitutional, because government employment is a privilege and not a right. C The executive order is unconstitutional, because Congress, rather than the President, has authority to set the terms of federal employment. D The executive order is unconstitutional, because the President cannot broadly limit all executive employees' freedom of speech and association.
D The executive order is unconstitutional, because the President cannot broadly limit all executive employees' freedom of speech and association.
A wholesale seller sent a fax to a manufacturer with whom he had done business before: "Send 500 'Madewell' chairs at your usual price." The manufacturer responded, also by fax, that the line was being discontinued, but he would ship his last 500 chairs at $75 per chair, his usual price. The manufacturer immediately began the paperwork for processing the order and started preparing and packing the chairs for shipment. Before the chairs could be delivered, the wholesaler canceled his order, noting that the price was too high. The day after receiving the wholesaler's cancellation, the manufacturer sold the chairs to another buyer for $75 each. If the manufacturer sues the wholesaler for damages, how much should he recover? A Nothing, because this was a contract between merchants and the wholesaler canceled within a reasonable time. B Nothing, because the manufacturer was able to cover by selling the chairs at the same price he would have received from the wholesaler. C $37,500, the full contract price, because the wholesaler breached the contract and $75 per chair was a fair price. D The incidental costs of preparing the paperwork and other office costs connected with preparing and packing the chairs for shipment to the wholesaler.
D The incidental costs of preparing the paperwork and other office costs connected with preparing and packing the chairs for shipment to the wholesaler.
An insured purchased a life insurance policy on his life, naming his brother as beneficiary. Fifteen years ago, the insured traveled overseas on what was supposed to be a six-month trip, but has not been heard from since. The brother contacted the insurance company, which refused to pay the claim on the basis that there was no evidence that the insured was dead. The brother filed suit against the insurance company to collect the proceeds under the policy. The jurisdiction in which the action has commenced has a statute that states that a person is presumed dead if missing from the jurisdiction for seven years, and if no one in the jurisdiction has heard from the person in those seven years. Assume that no other evidence is admitted at the trial on the issue of the insured's death. Which of the following is the most accurate statement? A The jury will be permitted to find that the insured is alive. B The jury will be permitted to find that the insured is dead. C The judge must rule as a conclusive presumption that the insured is dead. D The jury must find that the insured is dead.
D The jury must find that the insured is dead. The jury must find that the insured is dead. Because the basic facts that support the presumption were proven at trial, and no other evidence was introduced, the jury must find in accordance with the presumption, because the other party did not meet its burden of going forward with rebuttal evidence. (A) and (B) are therefore wrong. (C) is wrong because the presumption regarding the insured's death is a rebuttable presumption. A rebuttable presumption will have no force or effect when sufficient contrary evidence is introduced. A conclusive presumption is really a rule of substantive law and cannot be rebutted by contrary evidence.
A manufacturer of down coats and jackets entered into a written agreement with a distributor, whereby the distributor agreed to distribute the manufacturer's products statewide for a one-year period to begin on June 1. Before the manufacturer signed the distribution contract with the distributor, the distributor told the manufacturer that their deal was exclusive, but nothing to that effect was in the written agreement. However, in the outerwear industry it has been a custom for many years for distributors to distribute only one brand of outerwear. On September 1, the distributor began distributing coats and jackets manufactured by one of the manufacturer's chief competitors. These coats and jackets were sewn with man-made fabrics, were as warm as the manufacturer's jackets, and were less bulky. The competitor's advertising campaign throughout the state emphasizes that "you don't have to look fat to stay warm." Seasonally adjusted sales figures showed that the manufacturer's sales in the state dropped 6% after its competitor's products were introduced. The manufacturer of the down coats and jackets complained to the distributor, demanding that it stop distributing the man-made coats and jackets made by the manufacturer's competitor. The distributor refused, and the manufacturer of the down coats and jackets brought suit against the distributor. Which of the following facts would provide a basis for the manufacturer's best case against the distributor? A The competitor's advertising campaign throughout the state alluding to the unattractive bulkiness of the manufacturer's coats and jackets. B The 6% drop in seasonally adjusted sales figures in the state after the competitor's products were introduced. C The distributor's oral statement to the manufacturer about their deal being exclusive. D The long-standing custom in the outerwear industry for distributors to distribute only one brand of outerwear.
D The long-standing custom in the outerwear industry for distributors to distribute only one brand of outerwear.
A landowner orally agreed to sell 100 acres of land to a buyer for $10,000. As a condition of the sale, the buyer agreed to pay $5,000 of the purchase price to the landowner's creditor. The buyer's attorney drafted the contract, which both the landowner and the buyer read before signing. The signed document made no reference to the payment to the landowner's creditor, and neither party noticed the oversight. In an action by the creditor against the buyer for $5,000, which of the following facts, if proved, would be most important? A The buyer was negligent in not having carefully read the written agreement. B The landowner-buyer agreement was completely integrated. C The terms of the signed document are unambiguous. D The omission of any reference to the creditor from the written document was accidental.
D The omission of any reference to the creditor from the written document was accidental. In making a determination as to who would prevail in an action by the creditor against the buyer, the most important fact, would be that the omission of any reference to the creditor in the written document was accidental. If there is an agreement between the parties, the agreement is put into writing, and there is a variance between the original agreement and the writing, the writing can be reformed to reflect the intent of the parties. A plaintiff who wants to obtain reformation of a contract must show that there was an antecedent agreement that is not correctly reflected in the writing (e.g., by mistake). These requirements are met here because the writing fails to include a provision that was included in the earlier oral agreement concerning payment of part of the purchase price to the landowner's creditor. (A) is wrong because the negligence of either the buyer or the landowner should not adversely affect the creditor's claim where the facts do not suggest that either party assumed the risk of the mistake. (B) is wrong because, even if the landowner-buyer agreement was completely integrated, the contract could still be reformed for mutual mistake if the omission of any reference to the creditor from the written document was accidental. Under the parol evidence rule, completely integrated writings cannot be contradicted or supplemented by either written or oral expressions made prior to the writing or oral expressions made contemporaneously with the writing, but the parol evidence rule does not apply if a party to a written agreement alleges facts that entitle him to reformation of the agreement. (C) is wrong because, while it is true that the terms of the writing are unambiguous, the answer turns on a mutual mistake involving omission of a material term and not on an ambiguity in the writing, and, as explained above, such mutual mistake allows for reformation of the contract.
Shortly after a professor at a state university completed her second year of teaching, she was informed that her contract was not being renewed for the following year. By state law, a professor does not acquire tenure until after she has completed three consecutive years of teaching. Before acquiring tenure, state law does not require either a statement of reasons or a hearing when a professor's contract is not renewed, and the university administration refused to give either to the professor. Which of the following, if established, sets forth the strongest constitutional argument that the professor could make to compel the university to furnish her a statement of reasons for the failure to rehire her and an opportunity for the hearing? A She purchased a home in anticipation of renewal of her contract, because most professors who had taught two years were rehired. B She had been voted the most popular professor on campus in each of her first two years of teaching. C She was the only teacher at the university whose contract was not renewed that year. D There is evidence to indicate that the decision not to rehire the professor was not based on her ability to teach.
D There is evidence to indicate that the decision not to rehire the professor was not based on her ability to teach.
A three-year-old girl attending nursery school punched a boy in the face because he was teasing her about wearing glasses. The blow knocked out the boy's newly acquired front teeth. If the boy's parents sue the girl's parents for the injury, what is their best defense? A The boy was the initial aggressor. B The girl is too young to be responsible for her actions. C A parent cannot be liable for damages due to the child's conduct. D The parents were unaware of any potentially violent behavior by the girl.
D The parents were unaware of any potentially violent behavior by the girl. The best defense of the girl's parents is that they were unaware of any potentially violent behavior by the girl. At common law, parents are not vicariously liable for the torts of their child. (Statutes in most states allow for limited liability for intentional torts, but there is no indication of such a statute here.) Parents can be liable, however, for their own negligence, i.e., in not exercising due care under the circumstances. Thus, if the parents know their child may be violent, they could be negligent if they do not take precautions to prevent that behavior or injury from that behavior. However, if the parents have no reason to know their child could be violent, they have no duty to protect against such behavior. Here, if the girl had never done anything like this before, and her parents had no idea that she would be violent, they were not negligent in allowing her to attend nursery school. (A) is wrong because, although the boy's teasing may have provoked the girl, he did not initiate the violence. He did nothing to allow the girl a right of self-defense, and so his actions would not provide the girl's parents with a good defense. (B) is wrong because there is no general tort immunity for children. As long as the child is old enough to intend the act, she can be held liable. Here it seems that the girl intended to cause a battery. She either intended or knew with substantial certainty that swinging her fist would strike the boy in the face, i.e., would cause a harmful or offensive contact. Thus, this choice does not present the best defense for the defendants. (C) is wrong because parents can be liable for damages due to their child's conduct. As explained above, although the parents are not vicariously liable at common law, they can be liable based on their own negligence (e.g., for negligent supervision).
A local physician who was prominent in the community and beloved by her patients died suddenly of a heart attack. A reporter with the local newspaper was assigned to write an obituary before the next day's edition went to press. The reporter talked briefly with the physician's widower, and then called the state medical school, from which the physician had always said she had graduated. As it was late in the afternoon, the reporter did not speak with any administrators but with a secretary in the office of the dean. When asked about the physician, the secretary replied that she did not think the physician ever graduated. The local newspaper printed this information in the next day's edition. On reading the obituary, the physician's widower became very angry, as the physician had, in fact, graduated with high honors. He demanded a retraction from the newspaper. The next day, on the front page, the newspaper admitted its error and stated that the physician graduated with high honors from the state's medical school. They also fired the reporter. Nonetheless, both the executor of the physician's estate and her widower sued the local newspaper for defamation. What is the local newspaper's best defense? A There was no malice on the part of the defendant. B The newspaper's retraction negated any harm. C The reporter got his information from a secretary at the medical school. D The physician is dead.
D The physician is dead.
A state statute provides: "Any merchant desiring to sell within this state any product or goods manufactured outside of the United States must (i) obtain a special license from the state for $50 and (ii) clearly mark the goods as to specify their country of origin." The statute makes it a misdemeanor for any merchant to willfully sell goods without complying with these statutory requirements. Which of the following statements is correct regarding the constitutionality of the statute? A The portion of the statute requiring the license fee can be sustained on the ground that reasonable inspection fees are proper; but the balance of the statute is invalid. B The portion of the statute requiring that the goods be labeled as to country of origin can be sustained because it only requires disclosure; but the balance of the statute is invalid. C The statute is constitutionally valid as long as the burden on foreign commerce is minimal and is justified by legitimate state interests. D The statute is unconstitutional in its entirety.
D The statute is unconstitutional in its entirety. The statute is an unconstitutional violation of the Commerce Clause. Regulation of foreign commerce is exclusively a federal power because of the need for the federal government to speak with one voice when regulating commercial relations with foreign governments. The existence of legitimate state interests underlying state legislation will not justify state regulation of foreign commerce. The state statute, in imposing requirements for a license costing $50 and for a clear marking of goods as being from a foreign country, clearly is an attempt by the state to restrict or even eliminate the flow of such goods in foreign commerce. Thus, the statute is unconstitutional. (A) is incorrect because even if the $50 fee represents a reasonable inspection fee, the fee would still constitute an interference with foreign commerce. In addition, the facts do not indicate that the license fee has anything to do with inspection, or that the amount of the fee bears any relation to legitimate inspection purposes. (B) is incorrect because the labeling requirement imposes a burden on goods that flow in the stream of foreign commerce. Although this burden may be relatively small, it is still impermissible in light of the exclusive power held in this area by the federal government. (C) is incorrect because it states factors that would be relevant in a matter involving regulation of interstate commerce, rather than foreign commerce. Congress's power over interstate commerce is shared with the states, so that a state law may regulate local aspects of interstate commerce if it does not discriminate against out-of-state competition to benefit local economic interests and its incidental burden on interstate commerce does not outweigh the legitimate local benefits arising therefrom. However, Congress's power over foreign commerce is exclusive, so that factors such as a minimal burden on foreign commerce and the presence of legitimate state interests will not save a state law from a challenge based on the power to regulate foreign commerce.
At a waterfront bar, a college student sought to provoke a fight with a merchant seaman by making insulting remarks. Eventually the seaman had had enough and threw a punch that connected to the student's jaw and sent him sprawling to the floor. The seaman then told the student that he wanted no further trouble. Getting up off the floor, the student pulled a knife out of his pocket and charged at the seaman. Three other students were standing between the seaman and the exit door. The seaman tried to dodge, but was cut on the forearm by the student's knife. The seaman immediately drew a gun and shot the student, killing him. The seaman was charged with murder. Which of the following points raised in the seaman's defense will not be helpful for his defense? A The student had no reason to fear serious bodily injury when he drew the knife. B The student's drawing of the knife constituted an escalation of the fight. C Three college students were standing between the seaman and the door, so there was no clear route of retreat. D The student's comments were motivated by a desire to provoke the seaman.
D The student's comments were motivated by a desire to provoke the seaman. Even though the student's words may have been intended to provoke the seaman, this fact alone would not justify the seaman's use of deadly force. A person may use deadly force in self-defense if he: (i) is without fault; (ii) is confronted with unlawful force; and (iii) reasonably believes that he is threatened with imminent death or great bodily harm. Generally, one who is at fault for starting a confrontation has no right to use force in his own defense during that confrontation. However, if the victim of the initial aggression suddenly escalates a relatively minor fight into one involving deadly force and does not give the aggressor a chance to withdraw or retreat, the aggressor may use deadly force in his own defense. Here, although the student instigated the hostile situation by repeatedly insulting the seaman, the seaman's throwing of a punch probably calls for his being characterized as the aggressor. The student, as the victim of the initial aggression, escalated matters by using a knife, especially because the seaman had said that he wanted no further trouble. This escalation (which is the point stated in choice (B)) entitled the seaman to employ deadly force in his own defense against the imminent threat of death or great bodily harm posed by the student's use of the knife. Thus, (B) presents a point that will be helpful to the seaman. (A) is incorrect because, if the student had no reason to fear serious bodily injury when he drew the knife, his use of the knife constitutes unlawful force, in response to which the seaman was entitled to use deadly force of his own. Consequently, (A) will also be of value in gaining the seaman an acquittal. Regarding (C), many courts hold that a person is not under a duty to retreat before using deadly force. Thus, even if the seaman could have safely retreated, he was still entitled to use deadly force in self-defense, so that he is not required to show why he did not retreat. Other courts, however, do require retreat before the use of deadly force, but only if the retreat can be made in complete safety. The fact that the seaman's route of retreat was blocked by other students would indicate that a retreat might not be able to be made in safety, and would be significant in a jurisdiction holding that there is a duty to retreat. Therefore, (C) also presents a point that can be helpful to the seaman. (D) is correct because the motive of the student in insulting the seaman is of no help to the defense. Even if the words did provoke him, the seaman would not be entitled to employ deadly force against the student on the basis of the student's desire for trouble. Use of such force would be justified only if the seaman held a reasonable belief that he was faced with imminent death or great bodily harm from one of the students if he did not respond with deadly force.
A state statute makes it a felony for anyone in the corridors or on the grounds of any building in which a court may be in session to make a speech or carry a sign intended to improperly influence judicial proceedings. During a murder trial, a friend of the defendant was arrested under the statute for carrying a sign on the steps of the courthouse bearing the message: "Free the defendant or the judge will die." Can the friend constitutionally be convicted under the statute? A No, because the statute could apply to others whose speech is constitutionally protected. B No, unless she personally intended to harm the judge. C Yes, if there was a clear and present danger that the judge would be influenced by the sign. D Yes, because the statute does not violate the freedom of expression guaranteed by the First Amendment.
D Yes, because the statute does not violate the freedom of expression guaranteed by the First Amendment.
Pursuant to an edict recently issued by the elders of their religion, a mother and father instructed their son who just turned age 14 to report to a community woodworking shop instead of school. A state law requires all children to attend school until the age of 16, and the woodshop does not qualify as a school under state law. Because the parents did not report their son's absence, a truant officer visited the family and warned them that parents who willfully refuse to comply with the mandatory attendance law are subject to a $500 fine and up to 30 days in jail for each day of noncompliance. The parents listened, but informed the officer that they could not comply with the state law because of their religious views. The following day, the 14-year-old again went to work in the community woodshop instead of to school. His parents were then arrested and charged with violating the state mandatory school attendance law. At the parents' criminal trial, which of the following may the court constitutionally consider in determining guilt or innocence? A Whether the tenets of the parents' religion are true. B Whether the parents' religion is a traditional, established one. C Whether the parents believe that the tenets of their religion are derived from a supreme being. D Whether the parents sincerely believe the tenets of their religion.
D Whether the parents sincerely believe the tenets of their religion.
Due to budget shortages and a critical need of funding to fight a war, Congress enacted a $25 tax on each person flying into an airport in the five most popular vacation destinations in the country, as determined by Congress. The tax was implemented, and officials in the five destinations were outraged, fearing that the number of vacationers to the taxed destinations would decrease due to the tax. If the tax is challenged in federal court by an official with standing, is the most likely result that the tax will be held constitutional? A No, because it makes it significantly more difficult for persons to travel between the states. B No, because the tax unfairly discriminates against certain vacation destinations by taxing them and not taxing other, similar vacation destinations. C Yes, because the tax is necessary to achieve a compelling government interest. D Yes, because Congress has plenary power to impose taxes to raise revenue.
D Yes, because Congress has plenary power to impose taxes to raise revenue. The destination tax will likely be held constitutional under Congress's taxing power. Congress has the power to lay taxes under Article I, Section 8, and a tax measure will usually be upheld if it bears some reasonable relationship to revenue production or if Congress has the power to regulate the taxed activity. Despite the protest from the officials of the affected locations, the tax here does appear to be related to revenue production and so will be upheld. (C) is incorrect because it is based on the wrong standard—the compelling interest test does not apply here. (A) is incorrect because the extent of the right to travel is not clearly defined. The Supreme Court has established that the right to travel from state to state is a fundamental right that may be violated by state laws designed to deter persons from moving into a state; however, the Court has not specifically applied this rule to the federal government or to the type of tax legislation present here. The state cases involved treating old vs. new residents differently for purposes of voting or some government benefit, which may have violated the Privileges or Immunities Clause of the Fourteenth Amendment, or treating outsiders differently from residents, which may have violated the Equal Protection Clause of the Fourteenth Amendment. Neither clause is applicable to the federal government, so neither analysis is appropriate. (B) also is incorrect. While the federal government is not subject to the Equal Protection Clause of the Fourteenth Amendment, it is prohibited from unfair discrimination by the Due Process Clause of the Fifth Amendment. Grossly unreasonable discrimination by the federal government could be held to violate due process, but a $25 tax on flights to popular vacation destinations probably does not. The tax is rationally related to the legitimate government interest of revenue production and so will likely be upheld.
A law student was sued by a student loan provider in federal court for failing to pay back her student loans. At the close of a federal civil trial, the court held in favor of the student loan provider for the full amount plus post-verdict interest at 8%. When the court clerk typed up the final order to send to all parties, she accidentally typed in 9% as the post-verdict interest rate. The law student did not notice the error until 14 months after the judgment was entered. She immediately filed a motion for relief from judgment based on a clerical mistake to correct the order to reflect the accurate interest rate. Will the court likely grant this motion? A No, because motions for relief from judgment to correct clerical mistakes must be made within a reasonable time, not to exceed one year. B No, because motions for relief from judgment to correct clerical mistakes must be made within 28 days of the order. C Yes, because motions for relief from judgment to correct clerical mistakes must be made within a reasonable time. D Yes, because motions for relief from judgment to correct clerical mistakes can be made at any time.
D Yes, because motions for relief from judgment to correct clerical mistakes can be made at any time.
The owner of a semi-pro baseball team offered a former player a position as the team's manager. During negotiations, the owner agreed to pay the manager $1,000 per week, but insisted that if the manager quit during the season, he would have to pay a "penalty" of $500 per week for each week that he did not manage the team, because it would cost the owner several thousand dollars to replace the manager during the season. The parties agreed in writing to those terms. The manager managed the team for 14 weeks with only mixed success. Nevertheless, with 10 weeks left in the season, he was offered and accepted a job as manager of a professional minor league baseball team. Fortunately for the owner of the semi-pro team, the manager's replacement had great success with the team, causing attendance to skyrocket. If the owner of the semi-pro team brings suit against the manager to recover $5,000, the amount due under the "penalty" provision, is the owner likely to prevail? A No, because "penalty" clauses in contracts are not enforceable. B No, because the owner of the semi-pro team was not harmed by his breach. C Yes, because the manager can be penalized for a willful breach. D Yes, because the "penalty" provision is enforceable.
D Yes, because the "penalty" provision is enforceable.
An underworld informer advised a police investigator that his neighbor was running an illegal bookmaking operation in his apartment, and that the informer had placed bets with the neighbor at this location. The officer obtained a search warrant, based on his affidavit reciting the foregoing facts, and further stating that the underworld informer was a person who had given him accurate information in previous cases, but whose identity could not be revealed because it might jeopardize other criminal investigations being carried on by the police. Armed with the search warrant, police officers went to the neighbor's apartment. They entered when the neighbor opened the door and searched the apartment. They seized various wagering slips and bookmaking apparatus (described in the search warrant) and placed the neighbor under arrest for illegal gambling. Prior to trial, the neighbor challenges the validity of the search warrant. Was the search warrant valid? A No, because it was based on hearsay information. B No, because the officer failed to disclose the identity of the informer, so that the accuracy of his information could not be verified. C Yes, because the identity of the informer is never required. D Yes, because the affidavit accompanying it is sufficiently detailed to allow a determination of probable cause.
D Yes, because the affidavit accompanying it is sufficiently detailed to allow a determination of probable cause.
A single man with a life insurance policy that pays his designated beneficiary $70,000 upon his death was killed in a car accident. His former girlfriend, a resident of State A, was named as beneficiary, but his mother, a resident of State B, also filed a claim for the life insurance proceeds. The insurance company, a State C corporation having its principal place of business in State B, filed an interpleader action in federal court to protect itself from potentially multiple and inconsistent claims. May the insurance company bring the interpleader action in federal court? A No, because the amount in controversy does not exceed $75,000, and there is no diversity between the insurance company and one of the claimants. B No, because there is no federal court where the insurance company will be able to establish personal jurisdiction over both claimants. C No, because the insurance company is not diverse from one of the claimants. D Yes, because the amount in controversy is $500 or more, and both claimants are diverse from one another.
D Yes, because the amount in controversy is $500 or more, and both claimants are diverse from one another. The insurance company can bring the interpleader action in federal court. The Federal Interpleader Act provides special rules for dealing with subject matter jurisdiction, personal jurisdiction, and venue. Under the act, interpleader can be brought if any two claimants are citizens of different states and the amount in controversy is $500 or more. Complete diversity is not required. Here, the mother is a State B resident, and the former girlfriend is a State A resident. Thus, both claimants are diverse from one another, and the $70,000 amount in controversy exceeds the $500 minimum. (A) is wrong because it states the general diversity of citizenship subject matter jurisdiction requirements that would be applicable to Rule interpleader. Statutory interpleader has different requirements. (B) is wrong because the Federal Interpleader Act permits the federal court to serve process anywhere in the United States. Therefore, as long as all the claimants reside or can be found anywhere in the country, the federal court can exercise personal jurisdiction over them. (C) is wrong because the Federal Interpleader Statute does not require complete diversity. It only requires diversity between the two contending claimants, which exists in this case because the mother is a State B resident and the former girlfriend is a State A resident.
The jurisdiction divides murder into degrees, with all murders being second degree murder unless the prosecution can prove premeditation and deliberation, in which case the killing would be first degree murder. The jurisdiction also uses the M'Naghten rule for insanity. A physician prescribed an experimental drug for the defendant's severe allergies. The physician told the defendant that the medication was experimental, but failed to inform the defendant that the manufacturer had reported a small risk that the medicine caused severe delusions. After taking the medicine the first day, the defendant began to believe that his next door neighbor was spreading false rumors about him to a few of his other neighbors. While still under the influence of the medication, the defendant grabbed a knife from the kitchen, went to his neighbor's house, and rang the doorbell. When the neighbor answered the door, the defendant plunged the knife into his neighbor's chest, killing him instantly. May the defendant be convicted of first degree murder? A No, because the defendant was temporarily insane under the M'Naghten rule. B No, because the defendant's intoxication was involuntary. C Yes, because the defendant's intoxication was voluntary. D Yes, because the defendant murdered his neighbor without justification and with premeditation.
D Yes, because the defendant murdered his neighbor without justification and with premeditation. The defendant may be convicted of first degree murder because he was not justified in killing the neighbor and it appears that he did have time to premeditate and deliberate on the murder. The facts here point to involuntary intoxication as a possible defense. Intoxication is involuntary if it results from the taking of an intoxicating substance without knowledge of its nature, under direct duress imposed by another, or pursuant to medical advice. Such intoxication is treated as mental illness, in which case the defendant is entitled to acquittal if, because of the intoxication, the defendant meets the applicable test for insanity. Here, the defendant took the medicine without knowing of its hallucinatory properties and pursuant to the advice of his doctor. Thus, the defendant's resulting state of hallucination will be considered to be involuntary intoxication. We are told that the applicable test for insanity is the M'Naghten rule. Pursuant to this rule, a defendant is entitled to acquittal if a disease of the mind caused a defect of reason such that the defendant lacked the ability at the time of his actions to either: (i) know the wrongfulness of his actions; or (ii) understand the nature and quality of his actions. If a defendant suffers from delusions (false beliefs), it must be determined whether his actions would have been criminal if the facts had been as he believed them to be. Here, the defendant falsely believed that his neighbor was spreading false rumors about him. Had this delusion been accurate, the defendant obviously would not have been legally entitled to murder the neighbor. Nothing in the facts suggests that the medication prevented him from knowing what he was doing or knowing that it was wrong. As a result, the M'Naghten rule would not allow for acquittal based on an insanity defense, which in turn would also negate the viability of an involuntary intoxication defense. Thus, (D) is correct, and (A) and (B) are incorrect. (C) is incorrect because, as explained above, the intoxication was involuntary even though the defendant voluntarily ingested the drugs.
A landowner owned a large tract of land in an area zoned for medium residential use. Permitted uses in this zone are single-family dwellings, condominium and townhouse developments, and moderate density apartment complexes. The landowner subdivided her land into 10 lots and conveyed each lot by a deed restricting the land to single-family use. All deeds were duly recorded and all lots were developed as single-family homes. The owner of lot 1 died and his property passed by will to his niece. Some time later, the owner of lot 3 sold his property to a buyer by a deed that did not contain the covenant limiting use to single-family dwellings. The buyer subsequently sold lot 3 to her friend, and did not include the covenant in the deed. Both deeds were duly recorded. A storm destroyed the friend's home, and in its place he began to build a three-unit townhouse. May the niece sue to enforce the covenant against the friend to prevent him from building the townhouse on lot 3? A No, because there is no privity with the friend. B No, because the zoning laws have not been violated. C No, because there was no restriction in the friend's deed. D Yes, because the friend's townhouse would alter the landowner's common scheme.
D Yes, because the friend's townhouse would alter the landowner's common scheme.
A plaintiff who was a citizen of State A was traveling to adjoining State B to visit his relatives. While still in State A, the plaintiff's auto was struck in the rear by a vehicle driven by the defendant, a citizen of State B. The plaintiff suffered personal injuries and damage to his vehicle amounting to approximately $90,000. The plaintiff filed suit in the federal district court for State A and obtained proper service of process on the defendant. Under the laws of State A, the driver of a vehicle that strikes another vehicle in the rear is presumed to have acted negligently, regardless of the surrounding circumstances. Neither the law of State B nor the federal statutes or case law has adopted such a rule. Should the court apply the presumption in question? A No, because federal law does not recognize such a presumption. B No, because the law of State B does not recognize such a presumption. C Yes, because in a diversity case a federal court applies the substantive and procedural laws of the state in which it sits. D Yes, because the presumption at issue operates upon elements of the prima facie case.
D Yes, because the presumption at issue operates upon elements of the prima facie case.
A township located in a farming community was composed mostly of persons belonging to a specific religious sect. To help instill proper respect for authority in children, which was a central tenet of the sect, and to maintain order in the classroom, the local school board allowed teachers to inflict corporal punishment. Such punishment was inflicted on a fourth grader in a township school immediately after his teacher saw him pulling a girl's hair. Neither he nor his parents belonged to the religious sect. When the boy's parents learned of the incident, they hired an attorney. Rather than suing the teacher for battery as permitted under state law, the attorney brought an action against the teacher under a federal statute providing a cause of action for damages against any government employee who deprives a person of his constitutional rights. Should the court find the policy allowing corporal punishment to be constitutional? A No, because the punishment policy violates the First Amendment Establishment Clause. B No, because the boy was denied any kind of hearing, in violation of his right to procedural due process under the Fourteenth Amendment. C Yes, because under the doctrine of parens patriae states may impose any punishment they see fit. D Yes, because the punishment was not grossly disproportionate under the Eighth and Fourteenth Amendments.
D Yes, because the punishment was not grossly disproportionate under the Eighth and Fourteenth Amendments.
A trainer of homing pigeons brought several of them to a park that he often used for training. He had trained this group of pigeons carefully and was confident that they would readily find their way home. When they were released, one of the pigeons inexplicably turned in the opposite direction from home. Several blocks away at the other end of the park, it collided with a radio-controlled model airplane that its owner had just purchased and was trying out for the first time. The collision sent the airplane out of control; it dipped low across a highway and was struck and run over by a truck. The airplane owner sued the pigeon trainer for the destruction of his airplane. The parties stipulated to the above facts and the airplane owner presented evidence of his damages. The trainer then moved for a directed verdict. Should it be granted? A No, because the trainer's pigeon caused the destruction of the airplane. B No, because the jury could find negligence on the trainer's part under the doctrine of res ipsa loquitur. C Yes, because the truck, rather than the pigeon, was the direct cause of the airplane's destruction. D Yes, because the trainer took reasonable care in training his pigeons.
D Yes, because the trainer took reasonable care in training his pigeons.
A plaintiff sued a defendant and his employer for personal injuries. The plaintiff claimed that she was struck on the head by a wrench dropped by the defendant from a high scaffold, on which the defendant was working in the course of a construction project. To prove that it was the defendant who dropped the wrench, the plaintiff offers the wrench itself as evidence: The wrench bears the brand name "Craftsman" on the handle, and other evidence shows that the wrenches used by the defendant on the job are "Craftsman" brand wrenches. Is the wrench admissible? A No, because but for the word "Craftsman" the wrench would be irrelevant, and the word "Craftsman" is inadmissible hearsay. B No, because the wrench is irrelevant as it fails to show that it is more likely than not that the defendant was the person who dropped it. C Yes, because the wrench is relevant direct evidence that it was the defendant who dropped the wrench and is not hearsay. D Yes, because the wrench is relevant circumstantial evidence that it was the defendant who dropped the wrench and is not hearsay.
D Yes, because the wrench is relevant circumstantial evidence that it was the defendant who dropped the wrench and is not hearsay.
A mother and her son were walking through a park where dog owners frequently exercised their dogs and sometimes unleashed them. Suddenly, a large black dog attacked the mother, seriously injuring her. The son suffered minor injuries when he tried to drive the dog away from his mother. The mother and the son filed a personal injury suit against the dog owner. The dog owner denies that the attacking animal was his dog. At trial, the son is called as a witness to testify that he was looking directly at the dog owner when the dog owner unleashed the dog, which immediately ran up to them and attacked the mother. Should this testimony be admitted over the dog owner's objection that the son is not competent to testify? A No, because the son is a close relative of a party, and therefore may not testify on that party's behalf. B No, because the son is a party-plaintiff, and therefore may not testify as to the facts of the accident. C Yes, because the son appears to be over the legal age to testify. D Yes, because there is nothing to indicate that the son is incompetent to testify.
D Yes, because there is nothing to indicate that the son is incompetent to testify.
After a power outage, the stoplights at a busy intersection were blinking red for traffic going in every direction. By statute, motorists must come to a full stop at a blinking red traffic signal before proceeding through the intersection. Cars driven by the plaintiff and by the defendant arrived at the intersection at the same time. Due to inattention, neither one stopped for the signal and the cars collided in the intersection. The plaintiff sued the defendant for his injuries. The trier of fact determined that the plaintiff was more at fault than the defendant. Will the plaintiff likely recover damages? A No, because the plaintiff did not stop at the blinking red light. B No, because the plaintiff's fault was greater than the defendant's. C Yes, if the defendant had the last clear chance to avoid the accident. D Yes, even though the plaintiff's fault was greater than the defendant's.
D Yes, even though the plaintiff's fault was greater than the defendant's.
Two neighbors owned summer homes adjacent to each other on the lake. After a week-long stay by the son of one of the property owners, the neighbor called the owner and said that his boat dock had been badly damaged and was told by another resident that the owner's son and some friends had gotten drunk and accidentally crashed their boat into his dock. The owner was surprised at the accusation because he was sure that if his son had caused the damages, he would have told him. However, he did not want to get into a dispute with his neighbor, so he told his neighbor that he would have the dock repaired and pay for the repairs if the neighbor agreed not to bring a claim against his son for the damage to the dock. The neighbor agreed, and the owner hired a local carpenter to do the work. Later, however, the owner discovered that his son did not damage the dock because the damages occurred after his son had returned to college. Is the owner obligated to pay for the repairs? A No, because the owner never really believed that his son caused the damage. B No, because his son in fact did not cause the damage. C No, because the neighbor was wrong when he accused his son of causing the damage and it would be unfair to enforce an agreement when there was a mutual mistake of fact. D Yes.
D Yes.
The owner of an animal is strictly liable for damage caused by the animal if it __________. A attacks a trespasser B is a farm animal with dangerous propensities typical for that species C commits an unforeseeable trespass D is a dog with known dangerous propensities
D is a dog with known dangerous propensities The owner of a dog or other family pet with known dangerous propensities will be held strictly liable for damage caused by the animal. Normally, the owner of a domestic animal is not strictly liable for injuries it causes. Strict liability will attach, however, if the owner knows of the domestic animal's dangerous propensities, i.e., propensities more dangerous than normal for that species. This rule applies even if the animal has never injured anyone. If an animal commits an unforeseeable trespass, the owner of the animal will not be held strictly liable for damage caused by the animal. The owner is strictly liable for damage caused by the trespass of her animal only if it was reasonably foreseeable. The owner of a farm animal with dangerous propensities typical for that species (such as bulls or bees) will not be held strictly liable for damage caused by the animal. Strict liability would apply only if the animal had propensities more dangerous than normal for that species. If an animal attacks a trespasser, the owner will not be held strictly liable for damage caused by the animal. A trespasser cannot recover for injuries caused unless there was negligence on the part of the landowner, e.g., the landowner knew of the trespasser and failed to warn of a wild animal or abnormally dangerous domestic animal.
A plaintiff sued a defendant for trespass. The defendant claimed the property in question belongs to her, not to the plaintiff. After a lengthy search through old family records stored in the defendant's attic, her attorney found an old deed that appears to provide a missing link in the plaintiff's title. The attorney immediately placed the deed in his office safe as part of his working file on the defendant's case. The plaintiff learned of the existence of the deed through other sources and seeks to inspect it and copy it. Under modern discovery rules, may the plaintiff do so? A Yes, because the deed was not prepared in anticipation of litigation. B Yes, because the discovery rules trump the attorney work product doctrine. C No, because the efforts of the defendant's attorney constitute attorney work product. D No, because the deed was obviously never officially recorded and is therefore irrelevant to the case.
A Yes, because the deed was not prepared in anticipation of litigation. The plaintiff may inspect and copy the deed because it was not work product. There is a qualified privilege for an attorney's work product. Work product is defined as material prepared in anticipation of litigation. Since the deed was not prepared in anticipation of litigation, it is unlikely to be considered work product. (B) is incorrect because the work product doctrine may be asserted in some cases. (C) is incorrect because, as stated above, the deed is not "work product," and it does not become work product merely because it took considerable effort to find. (D) is incorrect. The deed is the subject matter of the dispute, and the discovery of the deed, at the very least, may lead to other discoverable evidence.
The governor of an arid western state owned a vacation home and permitted his son to have a party there. At the end of the night the son failed to properly extinguish a bonfire that he and his friends had built, and within a few hours, wind-blown cinders had spread the fire to the trees east of the lodge. At the same time several miles away, a worker at a lumber mill was making emergency repairs to a pipe running between two mill buildings. He did not notice some of the sparks from his welding torch land in a pile of dried lumber and catch fire, and he failed to check the area after he was finished. By the time the fire was noticed by another employee, it was out of control. The wind blew both fires toward a landowner's hunting lodge. They merged a mile away and shortly thereafter totally consumed the lodge. For political reasons, the landowner did not bring a lawsuit against the governor or his son. He did, however, file a lawsuit against the lumber mill, alleging that its employee's negligence caused the destruction of his lodge. Evidence at trial established that either fire alone would have destroyed the lodge as well. Can the landowner recover from the lumber mill? A No, because the landowner's lodge would have been destroyed regardless of the conduct of the lumber mill's employee. B No, because the damage is indivisible and cannot be apportioned unless the landowner adds the other tortfeasor to the lawsuit. C Yes, because the negligence of the lumber mill's employee was a cause of the landowner's injury. D Yes, but the landowner can recover only 50% of his damages from the lumber mill.
C Yes, because the negligence of the lumber mill's employee was a cause of the landowner's injury. The landowner can recover the full amount of his damages from the lumber mill because the negligence of its employee caused the destruction of the lodge. Before a defendant will be liable for a breach of duty to the plaintiff, it must be shown that the breach was the actual and proximate cause of the injury. The general test for determining whether an act or omission is the actual cause of the injury is the "but for" test, i.e., whether the injury would not have occurred but for the act or omission. Under certain circumstances, however, the "but for" test is inadequate to determine actual cause. Where several causes combine to bring about an injury—and any one alone would have been sufficient to cause the injury—the actual cause requirement is satisfied if defendant's conduct was a substantial factor in causing the injury. Under this analysis, the fire started by the lumber mill employee was an actual cause of the destruction of the landowner's lodge because it was a substantial factor in causing the harm. It was also a proximate cause of the harm because no intervening forces broke the causal connection between the act and the harm. Because its employee was acting within the scope of his employment when he caused the fire to start, the lumber mill is vicariously liable for the injury that resulted. (A) is incorrect because the "but for" test is not applicable to these facts. Under that test, neither fire would be the actual cause of the harm because, looking at either fire alone, the harm would have occurred even without it. However, under the substantial factor test, both fires are actual causes of the injury. (B) is incorrect even though it is true that the damage is indivisible. The landowner can still recover from the lumber mill even if he does not sue the other tortfeasor. (D) is incorrect because joint and several liability rules allow the landowner to recover his full damages from the lumber mill. Where two or more tortious acts combine to proximately cause an indivisible injury to plaintiff, each tortfeasor will be jointly and severally liable for that injury, even though each defendant acted entirely independently. The effect of joint and several liability is that the plaintiff may recover the entire amount of the damages from any tortfeasor, who then may have a right of contribution from the other tortfeasor. Hence, even though the negligence of another tortfeasor was also an actual cause of the destruction of the landowner's lodge, the landowner is entitled to recover all of his damages from the lumber mill.
A homeowner entered into a written agreement with a contractor whereby the contractor agreed to completely remodel the homeowner's bathroom "to her specifications" at a cost of $10,000. The homeowner's specifications were highly detailed and required custom-made fixtures that would not be usable in other bathroom remodeling jobs. The contractor ordered the custom-made fixtures and paid $4,000 for them when they were delivered to his place of business. Figuring up the cost of the fixtures and labor, the contractor estimated that he would make a total profit of $2,000 on the job after payment for materials and workers. Before the contractor began work on the project, but after he had paid for the fixtures, the homeowner told the contractor that she had had a change of heart and would probably be selling the house the following year, and so would not need a custom bathroom. The contractor made no attempt to sell the fixtures to another contractor and filed suit against the homeowner for damages. What is the contractor likely to recover? A Nothing, because he failed to mitigate damages. B His expectation damages of $2,000. C $4,000, the cost of materials as restitution. D $2,000 as expectation damages, plus $4,000 in reliance damages.
D $2,000 as expectation damages, plus $4,000 in reliance damages. The contractor can recover $2,000 as lost profits plus the $4,000 in costs he incurred before the homeowner breached the contract. The purpose of a damages remedy based on an affirmance of the contract is to give compensation for the breach; i.e., to put the nonbreaching party where he would have been had the promise been performed. In most cases, the plaintiff's standard measure of damages will be based solely on an "expectation" measure, i.e., sufficient damages for him to buy a substitute performance. A reliance measure of damages, on the other hand, awards the plaintiff the cost of his performance, i.e., his expenditures in performing his duties under the contract. In certain situations, an award of compensatory damages will contain both an expectation and a reliance component. In a construction contract, if the owner breaches the contract after the builder has already begun his performance, the builder will be entitled to any profit he would have derived from the contract plus any costs he has incurred to date. This formula contains an expectation component (the profit the builder would have made) and a reliance component (the cost incurred prior to the breach). This formula is applicable to the facts in this case. The contractor has begun performance by ordering and purchasing the custom-made fixtures at a cost of $4,000. Because they are usable only for the homeowner's purposes, their cost, which is treated just like any other expenditure of labor and material in a partially completed construction contract, can be recovered as reliance damages. The other element of his recovery is the $2,000 profit that he would have derived from the contract—his expectation damages. His total recovery will therefore be $6,000. (A) is incorrect because the contractor can do nothing further to mitigate his damages. The nonbreaching party is always under a duty to mitigate damages after learning of the other party's breach. In construction contracts, the builder's duty to mitigate generally dictates only that he not continue work after the breach and not incur further expenditures. While the builder would also have a duty to apply any usable materials that he purchased to other jobs or to attempt to resell them to another contractor, the facts specify that the custom-made fixtures here were not usable in other remodeling jobs. Hence, the contractor's failure to attempt to sell the fixtures did not amount to a failure to mitigate damages. (B) is incorrect because an award of $2,000 does not put the contractor in the position he would have been in had the contract been performed—the $4,000 that he spent on the fixtures would have been covered by part of the $10,000 that he was to receive as the contract price. Had the homeowner not breached, the contractor would have received the contract price of $10,000, and he would have spent an additional $4,000 in labor and materials to complete the job. The difference, $6,000, consists of the $4,000 that he already spent on materials and the $2,000 profit that he expected to make. (C) is incorrect. Instead of seeking a damages remedy based on an affirmance of the contract, the nonbreaching party may rescind and sue for restitution for any "benefit" that he has transferred to the breacher in an attempt to perform the contract. The restitution recovery is generally based on the fair market value of the benefit transferred. Here, the contractor can provide the homeowner with the fixtures and seek restitution, but there is nothing to indicate that their fair market value is $4,000. Even if that is the case, however, the contractor has a provable compensatory damages remedy on the contract of $6,000; he will elect that remedy rather than the lesser restitution remedy.
A pedestrian walking on the sidewalk was struck by a car backing out of a driveway. The driver did not see the pedestrian because her neighbor's bushes obscured her view of the sidewalk. The pedestrian was seriously injured and brought suit against the driver and the neighbor. The pedestrian also included the city in his lawsuit, alleging that the city failed to enforce its ordinance requiring homeowners to provide a clear view of sidewalks where they intersect with driveways. The trier of fact determined that the driver was 60% at fault, the neighbor was 30% at fault, and the city was 10% at fault. The jurisdiction has adopted comparative contribution in cases applying joint and several liability. Which of the following is a correct statement regarding liability? A The city is liable to the pedestrian for the full amount of the damage award. B Both the driver and the neighbor are liable to the pedestrian for 90% of the damage award. C Each of the three defendants are liable to the pedestrian for one-third of the damage award. D The driver is liable to the pedestrian for 60% of the damage award, the neighbor is liable for 30% of the damage award, and the city is liable for 10% of the damage award.
The city is liable to the pedestrian for the full amount of the damage award.
To secure a loan of $100,000 from a bank, the owner in fee simple of a parcel of land conveyed a deed of trust for the land to the bank. The deed of trust contained a "power of sale" clause, permitted by the jurisdiction, which allowed the bank to sell the property in the event of default without the necessity of a judicial foreclosure action. After several years, the owner defaulted on his loan payments to the bank. The bank informed the owner that it was exercising its power of sale. After appropriate notices, the bank conducted a public sale of the land. The bank was the sole bidder and obtained the property for $80,000, which was $10,000 less than the outstanding balance on the loan plus the expenses of the sale. One month later, the owner notified the bank that he wanted to pay off the loan and extinguish the deed of trust, and was prepared to tender $80,000 to do so. The bank insisted that the owner must tender $90,000 to pay off the loan. If a court in the jurisdiction will require the bank to accept only $80,000 under the circumstances above, what is the likely reason? A The owner had the power to revoke the trust as long as he was alive. B The bank did not have the authority to bid on the property at other than a judicial foreclosure sale. C The owner was exercising a statutory power rather than an equitable power. D The bank does not have the power to clog the equity of redemption.
The owner was exercising a statutory power rather than an equitable power. If the owner can compel the bank to accept his offer, it will be because he has a statutory power to redeem the property after the foreclosure sale has occurred. In all states, the equity of redemption provides the borrower with an equitable right, at any time prior to the foreclosure sale, to redeem the land or free it of the mortgage or lien by paying off the amount due or, if an acceleration clause applies, the full balance due. Only about half the states, however, give the borrower a statutory right to redeem for some fixed period after the foreclosure sale has occurred; the amount to be paid is generally the foreclosure sale price, rather than the amount of the original debt. Thus, if the owner can redeem the land for $80,000, it will be based on the jurisdiction's statutory power of redemption. (A) is wrong because the deed of trust is a security interest (similar to a mortgage) to which the revocation rules for trusts do not apply. The deed of trust was created in part to allow the lender to foreclose on the property without going through a judicial foreclosure proceeding. (B) is wrong because, in states that permit a nonjudicial sale with deeds of trust containing a power of sale, the lender may bid at the sale, and in many cases the lender is the sole bidder. (D) is wrong because the prohibition against "clogging the equity of redemption" refers to the rule that a borrower's right to redeem his own mortgage cannot be waived in the instrument itself. Here, there is nothing to indicate that the owner's deed of trust prohibited him from redeeming the property prior to foreclosure. However, it is only through a statutory right of redemption that the owner would be able to redeem the property for $80,000 after the foreclosure sale had occurred.
A backgammon player was upset after losing a match against the club champion. Rushing out of the club, he inadvertently grabbed the champion's board, which looked very much like his own but which was much more expensive. The player left the backgammon board in the trunk of his car, as was his usual practice. During the night, the car was stolen and along with it, the champion's expensive backgammon board. In an action by the champion against the player to recover the value of the backgammon board, is the champion likely to recover? A Yes, because when the player took the backgammon board he committed a trespass to the champion's chattel. B Yes, because when the backgammon board was stolen along with the car, the player became liable for conversion of the champion's chattel. C No, because the player believed in good faith that the board was his when he took it from the backgammon club. D No, because the backgammon board was lost through no fault of the player's.
Yes, because when the backgammon board was stolen along with the car, the player became liable for conversion of the champion's chattel.
An environmentalist divided her 25-acre property into 100 quarter-acre residential lots. At the time the environmentalist sold her lots, there was a recycling center about one mile from the western boundary of the development. She included in the deed of all 100 grantees the following provision: "Grantee covenants for herself and her heirs and assigns that all aluminum cans, glass bottles, and grass clippings of Grantee and her heirs and assigns shall be recycled. This covenant runs with the land and shall remain in effect as long as there is a recycling center within five statute miles of the development." A buyer purchased a lot in the development. Her deed, which contained the recycling clause, was duly recorded. Two years later, the buyer decided to give the property to her niece as a gift. The niece's deed to the property contained the recycling covenant, and she too recorded her deed. Shortly after the niece took possession of the house, the recycling center moved its location to a new site about four and a half miles from the development. When the niece put the house up for sale, she said nothing to prospective buyers about recycling. The house was purchased by a veteran who had lost the use of his legs. The veteran's deed did not contain the recycling clause, and he hired a local disposal service to carry away his garbage and a landscaper to maintain the yard. The landscaper bagged the grass clippings and they were removed by the disposal service, which put all the trash and clippings in a landfill. When the veteran's neighbors informed him of his duty to recycle, he told them that he knew nothing of the covenant and that it would be difficult for a person in his physical condition to haul cans, bottles, and clippings to the recycling center. Unfazed, the neighbors filed suit to require the veteran to comply with the covenant or pay damages. The veteran's best defense is which of the following? A The veteran's deed did not contain the covenant. B The covenant does not touch and concern the land. C An intelligent inspection of the neighborhood would raise no inference that the covenant existed. D The veteran's physical condition requires a balancing of hardships by the court.
B The covenant does not touch and concern the land. The veteran's best defense is that the covenant does not clearly "touch and concern" the land. While recycling may benefit the community at large, "touch and concern" involves the relationship between landowners at law. Recycling by the veteran does not directly benefit the other landowners in the use and enjoyment of their land. Thus, (B) is correct. (A) is wrong because even though the veteran's deed does not contain the covenant, he has record notice because the restriction is in his chain of title. (C) is wrong because servitudes implied from a common scheme apply only to negative covenants, and the recycling requirement is an affirmative covenant. Thus, this defense does not go to the point. (D) is wrong because it goes only to issues in equity. The suit includes a claim for damages at law. In any case, balancing of hardships is not generally applied in such cases (although some courts might elect to do so).
A plaintiff filed a civil action based on negligence against a defendant in federal district court, alleging that the defendant negligently ran a red light at an intersection and collided with the plaintiff's vehicle, causing the plaintiff's injuries. A week after the close of discovery, the plaintiff filed a motion for summary judgment on the issue of whether the defendant was negligent. With the motion, the plaintiff filed (i) his own sworn affidavit, which stated that the traffic signal was green as he entered the intersection, (ii) an affidavit of a witness who was driving the car behind him, which stated that the witness saw the entire incident and that the plaintiff's traffic signal was green as he approached and entered the intersection; and (iii) an affidavit of another witness, which stated that she saw the entire incident and that the defendant's signal had been red for several seconds before the defendant entered the intersection and was still red when the defendant entered the intersection. In response to the motion, the defendant filed his own affidavit which stated that he does not recall seeing the traffic signal before entering the intersection, but believes that it was not red. He also filed the affidavit of a pedestrian on the scene. The pedestrian stated that she did not see the traffic signal prior to the accident, but that another passerby told her that the traffic signal was yellow for both drivers. How should the court rule on the plaintiff's motion? A Grant the motion, because the plaintiff has supported his motion with substantial evidence and the defendant has failed to produce admissible evidence that contradicts the plaintiff's evidence. B Grant the motion, because the defendant's conduct is still negligent even if the defendant's traffic signal was yellow. C Deny the motion, because the defendant has presented evidence that contradicts the evidence presented by the plaintiff. D Deny the motion, because a party may not obtain summary judgment on an issue on which that party has the burden of proof.
A Grant the motion, because the plaintiff has supported his motion with substantial evidence and the defendant has failed to produce admissible evidence that contradicts the plaintiff's evidence. The court should grant the motion. Summary judgment may be granted if, from the pleadings, affidavits, and discovery materials, it appears that there is no genuine dispute of material fact and the moving party is entitled to judgment as a matter of law. The court may not decide disputed fact issues on a motion for summary judgment; if there is a genuinely disputed material fact (meaning a dispute backed by evidence on both sides of the issue), the case must go to trial. Here, the plaintiff's evidence that the light was green when he entered the intersection has not been contradicted by admissible evidence. The defendant's affidavit essentially does not dispute the color of the light, as the defendant cannot remember what color it was. The pedestrian's testimony constitutes inadmissible hearsay. Thus, there is no material fact in dispute, and the motion should be granted. (Alternatively, the court could delay in ruling on the motion to give the defendant an opportunity to find the unavailable witness who spoke with the pedestrian. However, that is not an answer choice, and it is unclear from the facts whether that witness could even be found.) (B) is not correct, as it would become a matter for the jury to decide if running a yellow light is negligent. (C) is incorrect because the defendant has not produced admissible evidence that contradicts the plaintiff's position. (D) is an incorrect statement of the law. Summary judgment may be granted on an issue as to which the party has the burden of proof.
A landowner included in his will a provision giving "all of my property, both real and personal, wherever situated, to my widow for life, and after her death to any of our children who may survive her." What is the gift to the children? A A contingent remainder. B A vested remainder. C A shifting executory interest. D Void, as violating the Rule Against Perpetuities.
A A contingent remainder. The children have a contingent remainder. A remainder is a future interest created in a transferee that is capable of taking in present possession on the natural termination of the preceding estate created in the same disposition. Note that, as a rule of thumb, remainders always follow life estates. A remainder will be classified as contingent if its taking is subject to a condition precedent, or it is created in favor of unborn or unascertained persons. Here, the interest in the children follows a life estate and is a remainder because it is capable of taking in possession on the natural termination of the preceding estate. It is subject to the condition precedent of surviving the landowner's widow and, additionally, is in favor of unascertained persons (the children who survive the landowner's widow will not be ascertained until her death). Thus, the interest is a contingent remainder. (B) is incorrect because a vested remainder can be created in and held only by ascertained persons in being, and cannot be subject to a condition precedent. As discussed above, the will provision clearly does not satisfy these requirements because the takers are not ascertained and their interest is subject to a condition of survival. (C) is incorrect because a shifting executory interest is one that divests the interest of another transferee; i.e., it cuts short a prior estate created by the same conveyance. The gift to the children does not divest the interest of the widow; she retains a life estate in the property. The children's interest takes in possession only on the natural termination of the widow's estate (i.e., at her death). (D) is incorrect because the interest does not violate the Rule Against Perpetuities. The children's interest will vest, if at all, not later than 21 years after the lives in being. The landowner's widow and the children themselves are lives in being. There is no unborn widow problem because the instrument takes effect on the landowner's death and the gift is to his own widow. She must be in being at his death. Likewise, his children would be in being at his death. Thus, the vesting will be within the period of the Rule.
Which of the following suits would not fall within the United States Supreme Court's original jurisdiction under Article III, Section 2? A A suit seeking to assert the interest of state citizens in retaining diplomatic relations with a foreign nation. B A suit seeking to protect a state's timber from allegedly illegal cutting by residents of another state. C A suit seeking to enjoin enforcement of an allegedly unconstitutional executive order that will greatly limit the state's authority to make policy decisions regarding admission to state universities. D A suit by the United States Government seeking to enjoin state construction of a bridge over a navigable waterway.
A A suit seeking to assert the interest of state citizens in retaining diplomatic relations with a foreign nation. The suit to assert state citizens' rights is not within the Supreme Court's original jurisdiction. Under Article III, Section 2, the United States Supreme Court has original jurisdiction in all cases affecting ambassadors, other public ministers, and consuls, and in which a state is a party. In (A), the state is not really seeking to advance or protect any interest of its own. Rather, the state is attempting to act in parens patriae (i.e., to act as a representative of its citizens, thereby asserting their interests). Thus, the state is not an actual party in this case in the sense that the Supreme Court has traditionally required to justify exercise of original jurisdiction. (B) would be a proper case for institution under the Supreme Court's original jurisdiction because it involves an attempt by a state to protect its own economic interest rather than to assert the interests of its citizens in a representative capacity. Similarly, (C) sets forth a situation in which a state is attempting to defend its asserted right to render decisions affecting admissions policies relative to its own state universities. Thus, in (C) the state is an actual party to the case. Finally, (D) describes an attempt by the federal government to prevent state construction of a bridge (presumably pursuant to the admiralty power). Clearly, this case involves an alleged grievance that will be directly committed by a state. Therefore, the state is an actual party.
A union filed suit against a corporation, known for its antiunion management, asserting that its members were being discharged in retaliation for membership in the union rather than for any failure to perform their jobs properly. Under the pretrial discovery orders, a union employee was allowed to examine all of the records held in the corporation's files concerning discharge of employees for a seven-year period prior to the instigation of suit by the union. The employee sorted through this large volume of material and discovered that persons who were union activists usually had "lack of corporate spirit" listed as their reason for discharge, while other fired workers tended to have more specific grounds for discharge listed, e.g., persistent lateness. The employee developed a chart showing grounds for dismissal of union members versus nonmembers based on the data in the files. At the trial, the union placed the employee on the stand. She testified in some detail regarding how she had conducted her research. The employee brought out the chart and the union's lawyer asked that the chart be admitted into evidence. The corporation's attorney objected. How should the court rule on the admissibility of the chart? A Admissible, because copies of the original documents upon which the chart was based were available to the corporation prior to trial. B Admissible, because the chart is helpful to the trier of fact. C Inadmissible, because it is hearsay not within any exception. D Inadmissible, in the absence of the underlying records having been first introduced into evidence.
A Admissible, because copies of the original documents upon which the chart was based were available to the corporation prior to trial. The chart is admissible because the original documents are in the corporation's files. The original document or best evidence rule generally requires the original writing to be produced when the terms of the writing are sought to be proved and are material to the case. [Fed. R. Evid. 1002] However, under Federal Rule 1006, the contents of voluminous writings that are otherwise admissible may be presented in the form of a chart as long as the original documents are available to the other party for examination and copying. Here, the underlying documents belonged to the adverse party, and thus the corporation had unlimited access to them. (B) is incorrect because the chart could be helpful to the trier of fact and still be inadmissible, such as if the underlying material were not available to the corporation or the chart were based on inadmissible hearsay. Furthermore, (B) is not as good a choice as (A) because (B) states a generality (it basically states the relevance requirement) whereas (A) applies the law to the specific facts of this case. (C) is incorrect because the chart is admissible provided the underlying documents are admissible. Even if the documents in this case would be hearsay, they would be admissible under the business records exception to the hearsay rule because they are records of events made in the regular course of business. [Fed. R. Evid. 803(6)] (D) is incorrect because Rule 1006 is an exception to the best evidence rule designed to avoid the introduction of voluminous writings into evidence; therefore, it does not require their introduction as a prerequisite to introduction of a chart.
The defendant, who worked as a gardener for the victim, decided to break into the victim's home to steal some valuables one evening when he knew the victim would not be at home. The defendant, taking a key that the victim hid under a rock for emergencies, unlocked the front door and stepped into the doorway. At that moment, however, a security alarm sounded. On hearing the alarm, the defendant immediately left the premises. The crimes below are listed in descending order of seriousness. What is the most serious crime for which the defendant may be convicted? A Burglary. B Attempted burglary. C Attempted larceny. D No crime.
A Burglary. The defendant may be convicted of burglary. Common law burglary was the breaking and entering of the dwelling house of another in the nighttime with the intent to commit a larceny or other felony inside. In the instant case, opening the door by use of a key would be sufficient to constitute a breaking, and the facts indicate that the defendant actually entered the house. Furthermore, the defendant intended to steal valuables when he entered the victim's home; thus, the intent to commit a theft inside the home may be established. As a result, the defendant has committed burglary. (B) is incorrect because, although the defendant was unable to complete the theft of the valuables, the crime of burglary is complete on entry. (C) is also incorrect. An attempt is an act that, although done with the intention of committing a crime, falls short of completing the crime. An attempt consists of two elements: (i) the intent to commit the crime, and (ii) an overt act in furtherance of the crime. Here the defendant has committed an attempted larceny, as he had the intent to steal valuables from the victim, and, by breaking into the home, he committed an overt act in furtherance of the crime. However, burglary is a more serious offense, making choice (C) an incorrect answer. (D) is incorrect because the defendant may be convicted of burglary, as explained above.
An automotive engineer announced that he had developed a carburetor that will enable cars to achieve 100 miles per gallon of fuel, and that he will allow the carburetor to be inspected next month. Soon after, a former employer of the engineer brought an action to prohibit the engineer from displaying the carburetor, claiming that the engineer probably had stolen the carburetor's design from the employer. The court granted the employer a temporary restraining order prohibiting the engineer from disclosing any mechanical details of his carburetor, and ordered a hearing to be held in one week to determine whether a preliminary injunction should be issued. Because each party would have to reveal the mechanical details of his designs at the hearing, the employer requested that the hearing be closed to the public and that the record be sealed to avoid revelation of his designs. The court granted the request. A reporter for a monthly automobile magazine heard about the case and wanted to attend the hearing. When he was told that the hearing would be closed, he filed an action to have it opened. What is the reporter's best argument for opening the hearing? A Closure is not necessary to preserve an overriding interest here. B The right of freedom of the press is extensive and allows the press to attend all hearings of interest to the public. C Closure here amounts to a prior restraint. D Under the fairness doctrine, the magazine will be required to give each litigant an opportunity to present his side of the case.
A Closure is not necessary to preserve an overriding interest here. The reporter's best argument is that the closure here is not necessary to preserve an overriding interest because trials and pretrial hearings generally must be open to the public. The Supreme Court has held, at least in the context of criminal cases, that trials and pretrial proceedings can be closed only if closure is necessary to preserve an overriding interest and the closure order is narrowly tailored to serve the overriding interest. While the Court has not yet established the standard for civil matters such as the case here, several Justices and commentators have suggested that the same standard will be applied in civil cases since they too have historically been open to the public. (B) is a false statement of the law—freedom of the press is not absolute and does not allow the press unlimited access to any hearing of interest to the public; the hearing may be closed where an overriding interest in protecting the privacy of the parties is established. (C) is not as good an argument as (A) because, while closure here would amount to a prior restraint (a court order or administrative system that keeps speech from occurring), the prior restraint would be justified if the government proves that it was narrowly tailored to achieve a compelling interest. The argument in (A) negates this possibility and so is a better argument. (D) is incorrect because the fairness doctrine is irrelevant to the issue of whether a hearing should be open to the public. It was a rule of the Federal Communications Commission that required, among other things, that the media give political candidates an opportunity to oppose candidates or views endorsed by the media.
A federal statute just signed into law by the President provided that school districts no longer needed to recognize the tenure of elementary school teachers—all tenured teachers would lose their status and would be treated the same as nontenured teachers. The effect of the law would be to allow all tenured teachers to be fired more easily if their performance was not adequate. The law also allowed the salaries of tenured teachers to be lowered, at least until a new contract with the teachers could be negotiated. The law had a two-year grace period before it was to take effect, to give schools and teachers time to adjust to the law; however, it specifically provided that once it is in effect, school board actions under the law supersede any existing contract terms. A public elementary school district is in the first year of a three-year union contract with its teachers. The school board has stated that it plans to abolish tenured positions as soon as the law takes effect. The union, believing that numerous terms of the contract will be invalidated when the law takes effect, filed an action in federal court on behalf of the teachers, asking for an injunction to prevent the school board from abolishing tenured positions and for a declaratory judgment stating that the law is invalid. Should the federal court hear the case? A No, because a ruling on the law at this point is premature. B No, because the union does not have standing to sue on behalf of the teachers. C Yes, because the federal law encourages improper interference with a contract in violation of the Contract Clause of the Constitution. D Yes, because the teachers' rights and benefits are threatened by the law and the school board's stated plans.
A No, because a ruling on the law at this point is premature. The federal court should not hear the case because it is not yet ripe for review. A federal court will not hear a case unless there exists a "case and controversy." This has been interpreted to mean, among other things, that a plaintiff generally is not entitled to review of his claim unless he has been harmed or there is an immediate threat of harm. This is to prevent the federal courts from hearing unnecessary actions. There is no immediate threat of harm to the union here because the law does not take effect for another two years. Before that happens, Congress might change the law or repeal it altogether, or the school board may decide to keep the old contract system after all. Thus, (A) is correct and (D) is incorrect. (B) is incorrect because the union would have standing. An association has standing if (i) there is an injury in fact to its members that would give them standing, (ii) the injury is related to the organization's purpose, and (iii) neither the nature of the claim nor the relief requested requires participation of the individual members in the lawsuit. All three of the conditions are met here; thus, the union would have standing. (C) is incorrect because the Contract Clause does not limit federal power, only state power, and because the state would be acting pursuant to a federal law here, there would be no Contract Clause violation. Moreover, even if the Contract Clause limited the state's actions here, it still is not clear that there would be a constitutional violation. The Clause bans only substantial interference with existing contracts (i.e., destruction of almost all of a party's rights under a contract), and it is not clear here that the impairments would be sufficiently substantial.
A landlord leased office space to a business owner for five years, ending on November 1, reserving a yearly rent of $24,000, payable monthly. On October 1 of the fifth year, the business owner notified the landlord that he was preparing to move, but would greatly appreciate if the landlord could extend the lease for a month or two. On October 10, the landlord wrote to the business owner that she thought they could reach a satisfactory arrangement, but did not hear back from the business owner. The business owner did not vacate the office until November 20. On November 30, the landlord received a check from the business owner in the amount of $1,333 for "November's rent" and a note that he had vacated the premises. If the landlord brings an action against the business owner for additional rent, how will the court rule? A The business owner is bound to a year-to-year tenancy, because he did not vacate the premises until November 20. B The business owner is bound to a tenancy through December because one month's advance notice was required to terminate. C The business owner is not bound, because the $1,333 check discharged him from his obligations. D The business owner is not bound if the court admits parol evidence of the October 10 letter from the landlord.
A The business owner is bound to a year-to-year tenancy, because he did not vacate the premises until November 20. The court will rule that the business owner is bound to a year-to-year tenancy because he is a hold-over tenant. When a tenant fails to vacate the premises after the termination of his right to possession, the landlord may: (i) treat the hold-over tenant as a trespasser and evict him; or (ii) bind the tenant to a new periodic tenancy. The terms and conditions of the expired tenancy apply to the new tenancy. At least in commercial leases, the new tenancy will be year-to-year if the original lease term was for one year or more. Here, the businessman was a tenant for years because his lease was for a five-year fixed period of time. A tenancy for years ends automatically on its termination date. Therefore, as of November 1, the business owner became a hold-over tenant and the landlord had a right to bind him to a new periodic tenancy. Because the original lease was for more than one year, the business owner may be held to a year-to-year tenancy, at the stipulated rent of $24,000 per year. (B) is incorrect because even though the rent is payable monthly, the majority view is that reservation of an annual rent results in a year-to-year periodic tenancy. Hence, his notice of termination on November 30 would not take effect until the end of the new tenancy. (C) is incorrect because the business owner's mere continuance in possession after November 1 gave the landlord the right to bind him to another year's term. This right was not affected by the fact that the business owner paid 20 days' worth of rent. Moreover, although a tenancy for years may terminate on surrender, surrender requires the landlord's acceptance, which is not evident here. (D) is incorrect because even if the court admits the October 10 letter, it merely indicates the landlord's willingness to consider an extension. Because the business owner did not respond and no agreement was reached by the parties, the letter is not enough to allow the business owner to avoid the additional tenancy.
A plumber working for a company providing plumbing services to commercial and industrial establishments was required to be "on call" for emergency plumbing services 24 hours a day, and was required to drive his company van home each night so he would have all of his tools and equipment at hand for any calls. However, he was not permitted to use the company van for personal errands. On his way home one afternoon, he took a detour toward a supermarket a few blocks away to pick up some items for dinner. While entering the supermarket parking lot, he drove negligently and struck a pedestrian, seriously injuring him. The pedestrian filed suit against the plumber's company in a jurisdiction that maintains traditional common law rules regarding contribution and indemnity, and the jury awarded him $100,000 in damages, which the company paid. If the company sues the plumber to recoup its loss in the lawsuit, which party will prevail? A The company can recover 100% of the judgment as an indemnity, because the plumber was negligent, not the company. B The company will prevail, because the company had a rule against using company vehicles for personal errands. C The company will not prevail, because the company has already been found liable under principles of vicarious liability in the lawsuit by the pedestrian. D The company will not prevail, because the company required the plumber to be "on call" 24 hours a day.
A The company can recover 100% of the judgment as an indemnity, because the plumber was negligent, not the company. The company can recover 100% of the judgment under common law indemnity rules. The principle of indemnity permits a shifting between the tortfeasors of the entire loss (i.e., the payment made to satisfy plaintiff's judgment). This is in contrast to contribution, which apportions the loss among those who are at fault. Indemnity is available in vicarious liability situations, where one party is held liable for damages caused by another simply because of his relationship to that person. Hence, an employer such as the plumber's company that has been held vicariously liable under the doctrine of respondeat superior can obtain indemnification from the employee (the plumber) whose conduct actually caused the damage. (B) is incorrect because the company need not show that the plumber breached a company rule before it can obtain indemnity. The fact that the plumber's negligence caused the injury and that the company was liable for the judgment solely because of its relationship to the plumber permits indemnification here. (C) is incorrect because vicarious liability is one of the most common areas where indemnity is available. (D) is incorrect because the company's requirement that the plumber be on call 24 hours a day merely establishes that the company will be vicariously liable for the plumber's negligence; it does not bar the company from recovering from the plumber because the plumber's negligence actually caused the damage.
A mining company closed down operations at an isolated mine it owned and informed the electric company that electricity in the power poles that led to the mine should be cut off. However, the electric company, following its standard policy, left the power running in the line to deter thieves from stealing valuable transformers and cables. The mining company was unaware that the power was left on. A hitchhiker who was passing by the entrance to the mine saw that it was closed, so he went onto the property and climbed up a power pole to steal a transformer. He received an electric shock and fell from the pole, suffering serious injuries. If the hitchhiker sues the mining company, which of the following is the mining company's strongest defense? A The hitchhiker was a trespasser. B The hitchhiker was a thief. C The mining company asked the utility company to turn off the power. D The mining company was unaware that the utility company had not turned off the power.
A The hitchhiker was a trespasser. The hitchhiker's status as a trespasser is the mining company's strongest defense because it means that the mining company owed no duty to the hitchhiker, thereby completely relieving the mining company of any liability for his injuries. An owner or occupier of land owes no duty to an undiscovered trespasser. However, with regard to a discovered trespasser, the owner or occupier must warn of or make safe artificial conditions known to the landowner that involve a risk of death or serious bodily harm and that the trespasser is unlikely to discover. The hitchhiker, having come onto the land owned by the mining company without permission or privilege, is a trespasser. Because the mining company had no notice of the hitchhiker's presence on the property, the hitchhiker is deemed to be an undiscovered trespasser. Consequently, the mining company owes no duty to the hitchhiker with regard to the injuries incurred on its property. (C) and (D) each present factors that would be helpful to the mining company, but they are not as strong as (A). The fact that the mining company asked the power company to turn off the power, as well as the mining company's being unaware that it had not turned off the power, would be indicative of the exercise of due care on the part of the mining company (i.e., it took every reasonable step to see that the power was not left running in the abandoned mine, and could not reasonably have known that in fact the power was still on). However, if it is shown that no duty of care extended from the mining company to the hitchhiker, then the first element of a prima facie case for negligence is absent, thus eliminating any need for the mining company to attempt to show that it acted with ordinary, reasonable care. As a result, (A) is a much stronger defense than (C) or (D). (B) is incorrect because the fact that the hitchhiker tried to commit theft of the power company's transformers is not relevant to any duty that may have been owed to him by the mining company. It is relevant that the hitchhiker was a trespasser, because this means that the mining company owed him no duty. However, the hitchhiker's status as a thief is of no significance.
A man and a woman lived together for many years but never got married. Although the state in which they reside does not recognize common law marriage, it has statutes that prohibit discrimination on the basis of marital status. The man and the woman purchased a large property, taking title as joint tenants. Subsequently, the woman accumulated a $20,000 debt. She was too embarrassed to tell the man but was able to convince a bank to hold a mortgage on the property in exchange for the money. The bank was also willing to accept the woman's signature alone, and the man never learned about the mortgage. Two years later, the woman died without having paid off the mortgage. She left no will, and her only heir at law is her sister. The state in which the property is located is a "lien theory" mortgage state. Who has title to the property? A The man. B The man and the bank. C The man and the sister. D The man, the sister, and the bank.
A The man. The man takes sole title to the property under his right of survivorship. A joint tenancy carries the right of survivorship. Thus, when one joint tenant dies, the property is freed of her interest and the surviving joint tenant holds the entire property. Therefore, the man owns the property. (B) is wrong because the bank has no interest. Most states, like the one in this question, regard a mortgage as a lien on title. In these states, a mortgage of the property by one joint tenant does not, by itself, sever a joint tenancy until default and foreclosure proceedings have been completed. The bank's rights were lost when the woman died prior to foreclosure. When the woman died, her interest in the property evaporated, and with it the bank's security interest. On the other hand, in a title theory state, a mortgage is considered to be an actual transfer of title to the property, rather than just a lien on the property. Thus, a mortgage by a joint tenant transfers the legal title of the joint tenant to the mortgagee (the money lender). This action destroys the unity of title and thus severs the joint tenancy. (C) is wrong because the sister has no interest in the property. Surviving joint tenants, rather than heirs at law, succeed to a deceased joint tenant's interest. Even if the woman had left a will naming the sister as devisee of the property, the joint tenancy between the man and the woman would not have been terminated. A will is a testamentary conveyance (effective only at death) and hence is inoperative as to joint tenancy property, because at the instant of death the decedent's rights in the property evaporate. (D) is wrong because, as discussed above, only the man has title to the property.
A landowner was the owner in fee simple of a tract of land. The landowner conveyed the tract to her neighbor, "for life and then to the first child of my neighbor's only sister who shall reach the age of 21." The sister was unmarried and childless at the time of the conveyance. Five years later, the sister married, and she gave birth to a son the following year. When the son reached the age of 21, he brought an action in ejectment against the neighbor, who is still alive and living on the tract of land. Which of these is the neighbor's best defense against the son's attempt to eject her? A The neighbor's life estate was not subject to termination during her lifetime. B The son has no claim to the tract, because he was not living at the time of the landowner's original conveyance. C A conveyance of the tract to the son violates the Rule Against Perpetuities. D The son's interest, if any, would be that of a contingent remainder, and the contingency is the neighbor's death.
A The neighbor's life estate was not subject to termination during her lifetime. The neighbor's best defense against the son's attempt to eject her is that her life estate was not subject to termination during her lifetime. The usual life estate is measured by the life of the grantee. Although a life estate may be made defeasible (e.g., determinable, subject to a condition subsequent, or subject to an executory interest), the conveyance here created an indefeasible life estate (i.e., one that will end only when the life tenant dies) in the neighbor. (B) is incorrect because the son did not have to be alive at the time of the original conveyance in order to now claim an interest in the tract. (C) is incorrect because the son's interest would vest within 21 years of a life in being (his mother, the sister). (D) is incorrect because the son's interest would be a vested rather than contingent remainder. A remainder is contingent if it is created in favor of unborn or unascertained persons. Because the sister was childless at the time of the conveyance, the remainder to the sister's "first child . . . who shall reach the age of 21" was a contingent remainder. It remained contingent at the son's birth because he had to reach age 21 to take. However, the son has reached age 21, and thus his remainder is vested. It is indefeasibly vested because it is not subject to being defeated, divested, or diminished in size.
A cattle-producing state adopted a statute requiring any food service business operating in the state to serve beef raised in the United States. A licensed hot dog vendor who worked at a football field within the state and who had been buying hot dogs made with foreign beef for the past several years estimated that switching to an all-beef hot dog made from United States beef would reduce his profits by 10%. An attorney hired by the vendor to challenge the statute discovered during research into the case that most of the footballs used at the football field at which the vendor worked were made of foreign leather. Which of the following grounds is the vendor's best argument against the constitutionality of the state statute? A The statute burdens foreign commerce. B The statute violates equal protection guarantees because it is not rational to prohibit the sale of foreign beef but not foreign leather. C The statute substantially interferes with the vendor's right to earn a living under the Privileges or Immunities Clause of the Fourteenth Amendment. D The statute constitutes a taking without due process of law.
A The statute burdens foreign commerce. The best argument against the constitutionality of the state statute is that it burdens foreign commerce. For all practical purposes, the power to regulate foreign commerce lies exclusively with Congress. Therefore, a state that adopts legislation requiring private vendors to favor United States products over foreign products, as the state did here, may be acting outside the scope of its powers. (B) is incorrect because the statute is a rational method of protecting local beef interests. The rational basis standard applies when an economic law, such as the one here, is challenged on equal protection grounds. Under the standard, the Supreme Court will usually defer to a legislature's decision that the law is rational notwithstanding the fact that the statute is underinclusive. In other words, the law need not address all of the problems that prompted its passage; it will be upheld even if it is only a "first step" toward a legitimate goal. Here, prohibiting the use of foreign beef appears to be a rational method of protecting state beef raisers. Thus, it is irrelevant that the statute is underinclusive in that it allows the use of both United States beef and foreign leather. (C) is incorrect because the right to earn a living is not a privilege under the Fourteenth Amendment, which protects against infringement of rights of national citizenship, such as the right to petition Congress for redress. (Neither would the statute violate the Privileges and Immunities Clause of Article IV, because that clause only prohibits discrimination against citizens of other states and the statute here treats citizens of all states the same.) (D) is incorrect because the vendor had all of the process that was due him. Because the government action here was a general act and not an individualized adjudication, the vendor had no right to an individual hearing; the normal procedure for adopting a statute is all the process that is due.
A state Occupational Health and Safety Board recently issued regulations valid under its statutory mandate requiring that all employers in the state provide ionizing air purification systems for all employee work areas. These regulations replaced previous guidelines for employee air quality that were generally not mandatory and did not specify the method of air purification used. The requirements regarding air purification systems are likely to be unconstitutional as applied to which of the following employers? A A wholly owned subsidiary of a Japanese corporation with seven retail outlets within the state. B The state supreme court, which recently completed construction of its new courthouse with a non-ionizing air purification system which the builder is contractually bound to maintain for the next three years. C A United States Armed Forces Recruiting Center located adjacent to the state capitol building. D A privately operated community service center funded by donations and constructed through use of a loan provided by the United States Veterans Administration and repayable to that agency.
A United States Armed Forces Recruiting Center located adjacent to the state capitol building. The armed forces recruiting center is least likely to be required to comply with the new state law. A state has no power to regulate activities of the federal government unless Congress consents to the regulation. Accordingly, agents and instrumentalities of the federal government, such as the armed forces recruiting center, are immune from state regulations relating to performance of their federal functions. (D) is incorrect because, although the recreation center's construction was funded by a loan from the Veterans Administration, the center itself is privately operated and funded by donations. As a result, the center has only a tenuous connection with the federal government, so that it cannot claim the immunity afforded to a federal agency or instrumentality. Accordingly, in the same sense as is employed in the federal tax immunity cases, the agency does not "stand in the shoes" of the federal government. Thus, the application of the state regulations to the recreation center would not present constitutional problems. (A) apparently refers to the principle that the power to regulate foreign commerce lies exclusively with Congress. However, the mere fact that the regulated outlets are part of a wholly owned subsidiary of a Japanese corporation does not mean that the state regulations affect foreign commerce. The subsidiary's activities are conducted entirely within the state, and do not touch upon foreign commerce in any way. Therefore, application of the regulations so as to require the subsidiary to provide an ionizing air purification system for its employee work areas will not constitute a proscribed state regulation of foreign commerce. Thus, (A) is incorrect. (B) is more troubling, but does not offer as compelling an argument as (C). The Contract Clause limits the ability of states to enact laws that substantially impair contract rights (i.e., destroy most or all of a party's rights under an existing contract). Under the Clause, the Supreme Court will subject state actions that impair their own contracts to strict scrutiny. In any case, even if state action substantially impairs rights under an existing contract, the action still may be upheld if it: (i) serves an important and legitimate public interest; and (ii) is a reasonable and narrowly tailored means of promoting that interest. Here, the state supreme court, as an instrumentality of the state, would probably not have grounds for complaining that its rights under the contract have been impaired, but the builder might have grounds (e.g., the builder might have future economic interests during the three-year service period that will be substantially impaired if the court is required to install an ionizing system). Nevertheless, the regulation still may be valid if the state can prove that it truly serves the important public interest of protecting the health and safety of workers in the state and is narrowly tailored to promoting that interest. In any case, because it is uncertain whether the vendor's rights have been substantially impaired and, if so, whether the state can prove the worth of the regulation, (C) is a better choice.
The defendant is charged with the battery of a bouncer at a local tavern. At the trial, the prosecutor introduces evidence that while the bouncer was attempting to question the defendant about her intoxicated demeanor, the defendant committed a battery on the bouncer. The defendant attempts to defend against the charge on the basis of self-defense, insisting that the bouncer used excessive force in stopping her from entering the tavern. The defendant attempts to introduce into evidence an authenticated copy of the tavern records that show that three patrons had written complaints against the bouncer within the past six months for the use of excessive force. The prosecutor objects on the grounds that the records are inadmissible character evidence. Should the court sustain the objection? A Yes, because the character of a victim can be established only by reputation or opinion evidence. B Yes, because there is no evidence that the incidents involving the three patrons were based on the same facts as the defendant's claim. C No, because the records were authenticated. D No, because the character trait of a victim may be established by opinion evidence, reputation evidence, or specific acts of misconduct.
A Yes, because the character of a victim can be established only by reputation or opinion evidence. The court should sustain the objection because the records are evidence of specific bad acts. The Federal Rules permit a defendant to introduce evidence of a bad character trait of the alleged victim if it is relevant to the charge or the defense, but limit it to reputation and opinion evidence. Evidence of specific acts of the person in question that demonstrates that person's character is permitted only in a few instances, such as if the acts are relevant to some issue other than disposition to commit the crime charged. Here, no issue is raised by this evidence other than the bouncer's propensity to use excessive force. (A) is therefore correct and (D) is wrong. (B) is wrong because the facts do not have to be identical if evidence of bad acts were otherwise admissible. (C) is wrong; documentary evidence, even if fully authenticated and relevant, may be excluded if it violates a rule of competency, such as the rule for character evidence. Here, the objection should be sustained because the document is improper evidence of a specific bad act.
A skier broke his leg when he was knocked down by the chair lift as he tried to avoid other skiers who had fallen off while disembarking. The ski resort employee operating the lift had not been paying attention and had failed to stop the lift. Ski patrol personnel placed the skier on a stretcher, which they then hooked up to a snowmobile to bring him down the mountain. The route down ran along the edge of a ski trail. Midway down, a novice snowboarder tried to see how close he could come to the stretcher without hitting it, but he lost control and landed on top of the skier's leg, damaging it further. The skier filed a lawsuit against the snowboarder and the resort in a jurisdiction that has adopted a comparative contribution system in joint and several liability cases. At trial, the skier's physician testified that the skier's leg was permanently disabled, but that neither injury, by itself, would have caused the permanent disability and it was impossible to quantify how much each injury contributed to the disability. The jury determined that the damages from the permanent disability equaled $2 million, and that the snowboarder and the resort were each 50% at fault. What amount of damages can the skier recover from the snowboarder for his permanent disability? A $1 million, because the jurisdiction follows comparative contribution rules. B $2 million, because it was not possible to identify the portion of the injury that the snowboarder caused. C Nothing for his permanent disability, because the skier has not met his burden of proof as to the amount of damages that the snowboarder caused. D Nothing for his permanent disability, because the injury inflicted by the snowboarder, by itself, would not have caused the disability.
B $2 million, because it was not possible to identify the portion of the injury that the snowboarder caused. The skier can recover $2 million from the snowboarder because the snowboarder is jointly and severally liable for the injury. The doctrine of joint and several liability provides that when two or more tortious acts combine to proximately cause an indivisible injury to plaintiff, each tortfeasor will be jointly and severally liable for that injury. This means that plaintiff can recover the entire amount of his damages from any one defendant. The doctrine applies even though each tortfeasor acted entirely independently and at different times. Here, both the snowboarder and the employee of the ski resort breached their duty to the skier to act with reasonable care. Each tortfeasor's act was the actual cause of the skier's disability because but for either one of the acts, his leg would not have been permanently disabled. The snowboarder's act was the proximate cause of the skier's disability because the disability was the direct result of the snowboarder's act. The fact that the extent of the harm was unforeseeable is irrelevant; i.e., the tortfeasor takes the victim as he finds him. Thus, the skier can recover the entire $2 million from the snowboarder. (A) is incorrect because the contribution rules govern only whether a defendant required to pay more than his share of damages has a claim against the other jointly liable parties for the excess. Contribution does not involve the amount of damages that the plaintiff can collect in the first place. (C) is incorrect because the skier has met his burden of proof by establishing that the snowboarder was an actual and proximate cause of his permanent disability. Because the injury caused by the tortfeasors was not divisible, under joint and several liability rules, the snowboarder is liable for the full amount of the damages, including that attributable to the permanent disability. (D) is incorrect because but for the snowboarder's collision with the skier, the skier would not have been disabled. The "but for" test applies in concurrent cause situations—cases where several acts combine to cause the injury, but none of them standing alone would have been sufficient. The fact that the snowboarder's act standing alone would not have caused the disability is irrelevant to the snowboarder's liability.
The owner of a boat took two friends out on a lake near his home. One of his friends was driving the boat when it struck a partially submerged rock that the owner of the boat had forgotten to tell him about. The owner of the boat and the other passenger were injured; the driver of the boat was not hurt. In a jurisdiction that applies joint and several liability with comparative contribution, the passenger brought suit against both the boat owner and the driver, and the boat owner also sued the driver. The jury determined that the boat owner was 55% at fault and suffered $10,000 in damages, the driver of the boat was 45% at fault, and the injured passenger suffered $100,000 in damages. After entry of judgment, the boat owner paid the passenger her total damages of $100,000, while the driver of the boat has paid nothing. How much, if anything, can the boat owner recover from the driver? A $45,000, because the driver was 45% at fault. B $49,500, because the driver was 45% at fault and the boat owner suffered $10,000 in damages. C $50,000, because the boat owner and the driver are jointly liable. D Nothing, because the boat owner was more at fault than the driver.
B $49,500, because the driver was 45% at fault and the boat owner suffered $10,000 in damages. The boat owner can recover $45,000 through comparative contribution for the passenger's claim and $4,500 on his own claim against the driver of the boat. Most comparative negligence states have adopted a comparative contribution system based on the relative fault of the various tortfeasors. Nonpaying tortfeasors who are jointly and severally liable are required to contribute only in proportion to their relative fault. Here, because the jurisdiction retained joint and several liability, the boat owner had to pay the passenger all of her damages. Under comparative contribution rules, the boat owner can obtain contribution from the driver for 45% of that amount, because the driver was 45% at fault. In addition, the boat owner has a direct claim against the driver for his own damages of $10,000, reduced by 55%, the amount of his fault. Thus, the total amount that the boat owner can recover from the driver is $49,500, making (B) correct and (A) incorrect. (C) is incorrect because it reflects traditional contribution rules, in which all tortfeasors were required to pay equal shares regardless of their respective degrees of fault. (D) is incorrect because a tortfeasor who was jointly and severally liable is not precluded from recovering contribution merely because he was more at fault than the other tortfeasors.
A retailer entered into a written contract with a wholesaler whereby the wholesaler agreed to sell, and the retailer agreed to buy, 100 boxes of sunglasses manufactured by a large corporation located in a neighboring city. The agreed-upon price was $75 per box. Two weeks before the specified delivery date, the wholesaler told the retailer that it would not be able to fill its order, because of unexpected high demand for sunglasses this season. Although the retailer learned that the needed quantity of the same brand of sunglasses could be shipped within two days for $83 per box from a supplier in another area, the retailer instead purchased 100 boxes of the sunglasses locally at a cost of $90 per box. These sunglasses were of a slightly higher quality than the sunglasses that were originally contracted for. A few days before the original delivery date, the wholesaler notified the retailer that it would fill the order, and tendered 100 boxes of the sunglasses on the date of delivery. However, the retailer refused to accept them. At that time, the wholesale market price of the sunglasses had declined to $80 per box. If the retailer sues the wholesaler for damages based on the wholesaler's alleged breach, what is the retailer likely to recover? A $1,500, the difference between the cost of cover and the contract price. B $800, the difference between the contract price and the nonlocal supplier's price. C $500, representing the difference between the contract price and the wholesale market price at the time of performance. D Nothing, because the retailer obtained cover without waiting a commercially reasonable time for the wholesaler to retract the repudiation.
B $800, the difference between the contract price and the nonlocal supplier's price. The retailer is entitled to recover $800. The wholesaler's notice that it would be unable to fill the retailer's order constituted an anticipatory repudiation, which the retailer was entitled to treat as a total breach. Under the UCC, the buyer's basic remedy where the seller breaches by refusing to deliver is the difference between the contract price and either the market price or the cost of buying replacement goods ("cover"). If the buyer intends to fix damages based on the latter measure, the buyer must make a reasonable contract for substitute goods in good faith and without unreasonable delay. Here, the retailer chose to make a contract for a higher quality of sunglasses at a higher price, even though the model that he had originally ordered was available from a supplier outside the area. While the retailer need not find the lowest available price in the country or make a contract for substitute goods with an unreliable supplier, he was aware that he could have obtained the sunglasses in plenty of time from the nonlocal supplier. Absent additional facts that would justify the retailer's decision, he can recover only the difference between the contract cost and a reasonable contract for substitute goods. Hence, (A) is wrong because the retailer's contract for cover probably would not be deemed to be commercially reasonable. (C) is wrong because the retailer's remedy based on market price would be determined at the time the retailer learned of the breach, not necessarily the time of performance. In the case of an anticipatory repudiation such as this, the buyer may either treat the anticipatory repudiation as a total breach and pursue his breach of contract remedies, or suspend his performance and await the seller's performance for a commercially reasonable time. The retailer chose to treat the wholesaler's notice as a total repudiation and breach of contract. Hence, the market price remedy would be measured at that time because that is when the retailer "learned of the breach," rather than at the time of performance. (D) is wrong because the nonrepudiating party need not wait for the repudiating party to retract its repudiation. The retailer exercised its option to treat the repudiation as a total breach and buy substitute goods. Once that occurred, the wholesaler was not entitled to retract its repudiation and force the retailer to accept the sunglasses.
A father executed a deed to his art gallery "to my daughter for her life, and on my daughter's death to her children; provided, however, that if my daughter stops painting, to my brother." The daughter has two children and is still painting. At the time of the grant, what is the best description of the interest of the daughter's two children? A A contingent remainder. B A vested remainder subject to open and to total divestment. C A vested remainder subject to open. D An executory interest.
B A vested remainder subject to open and to total divestment. The daughter's two children have a vested remainder subject to open and subject to complete divestment. A remainder is a future interest created in a transferee that is capable of taking in possession on the natural termination of the preceding estate. A remainder is vested if the beneficiaries are ascertainable and their taking in possession is not subject to a condition precedent. A vested remainder created in a class of persons that is certain to take but is subject to diminution by reason of others becoming entitled to take is a vested remainder subject to open. Vested remainders may be subject to total divestment if possession is subject to being defeated by the happening of a condition subsequent. Here, the daughter's two children have a remainder because, on the expiration of the daughter's life estate, they will be entitled to possession of the property. The remainder is not subject to a condition precedent and the beneficiaries are in existence and ascertained, so the remainder is vested, not contingent. The remainder is subject to open because the daughter may have more children. Finally, the remainder is subject to total divestment because the daughter's children's right to possession is subject to being defeated by the daughter's ceasing to paint. (A) is wrong because the remainder is vested, not contingent; i.e., it is not subject to a condition precedent, and the beneficiaries are ascertainable. (C) is not the best answer because it is incomplete. The vested remainder here is also subject to total divestment. (D) is wrong because the children's interest does not divest the daughter's estate, which would indicate an executory interest. Rather, their interest is capable of taking in possession on the natural termination of the daughter's estate, and thus is a remainder.
Several members of a small terrorist group are on trial in federal court for conspiring to bomb a military installation. The prosecution would like to introduce the testimony of a military guard at one of the installation's gates. The guard had been present when a bomb that was being planted by a member of the group had exploded prematurely. The guard will testify that she ran over to administer first aid to the member, who in great pain told her that his group was in the process of planting three other bombs in other areas of the military installation and was going to detonate them all at the same time to get publicity for their cause. The guard will also testify that the member disclosed the locations of the other bombs and the names of two other members of the group. The authorities were able to prevent the other bombings and arrest the other members of the group. The member died from his injuries. What is the best basis for allowing the guard to testify as to the member's statements? A As a vicarious admission of a co-conspirator. B As a statement against interest. C As a statement of present state of mind. D As a dying declaration.
B As a statement against interest. The member's statements are admissible as a statement against interest. Under the Federal Rules, statements of a person, now unavailable as a witness, against that person's pecuniary, proprietary, or penal interest when made are admissible as an exception to the hearsay rule. Here, the member's statements implicating himself in the bombing conspiracy were against his penal interest when he made them; hence, they are probably admissible under that exception. (A) is wrong because for a statement to qualify under the Federal Rules as a vicarious admission of another member of the conspiracy, the admission must have been in furtherance of the conspiracy by a participant in it. Here, the member's statements were not made in furtherance of the conspiracy but instead served to thwart its success. (C) is wrong because the member's statements are not being used to show his then-existing state of mind but rather the scope of the conspiracy and the defendants' participation in it. (D) is wrong because, even assuming that the member made the statements while believing his death was imminent (which the facts do not clearly establish), dying declarations are admissible under the Federal Rules only in a prosecution for homicide or in a civil action, and this case was neither of those.
A state bans the use of disposable diapers to reduce the volume of nonbiodegradable material in its landfills. The ban was a boon for diaper services within the state, but many parents of young children were displeased with the use of conventional diapers. With support from retail establishments that lost business from the disposable diaper ban, a grass roots coalition formed to fight the ban funded a study showing that the trucks and cleaning supplies used by diaper services within the state harmed the environment more than disposable diapers. The coalition and retailers then filed suit seeking to have the ban on disposable diapers declared unconstitutional. If the court strikes down the statute, on which of the following constitutional provisions would its decision most likely be based? A The Equal Protection Clause of the Fourteenth Amendment. B The Due Process Clause. C The Impairment of Contracts Clause. D The Privileges or Immunities Clause of the Fourteenth Amendment.
B The Due Process Clause. Of the choices presented, the only likely basis to strike down the statute is under the Due Process Clause as a violation of substantive due process. Substantive due process tests the reasonableness of a statute; it prohibits arbitrary governmental action. Under substantive due process, when government action limits a fundamental right, the government must prove that the action is necessary to promote a compelling interest. If a fundamental right is not involved, the challenging party must prove that the act is not rationally related to any legitimate government interest. The retail sale of diapers is not a fundamental right, and so a challenger must prove that there is no rational basis for the statute. Almost any law can be justified under the rational basis standard. The law need not be the best law for accomplishing the government's goal. Thus, even if it is true that the disposable diaper ban causes more pollution than it prevents, because the ban is rationally related to reducing the volume of trash in landfills, the challenge is unlikely to succeed. Nevertheless, none of the other choices states a viable ground for invalidating the statute, and so (B) is the best choice. (A) is wrong because equal protection applies where a statute or governmental action treats similar people in a dissimilar manner (i.e., classifies people), and here there is no classification—under the statute no one can sell disposable diapers for use within the state. Thus, an equal protection argument is not applicable. (C) is wrong because the Impairment of Contracts Clause prohibits only the substantial impairment of existing contracts (and there are exceptions even where there is substantial impairment), and nothing in the facts indicates that forbidding the retail sale of disposable diapers would substantially impair any existing contract. (D) is wrong because the privileges and immunities covered by the Fourteenth Amendment are those attributes peculiar to United States citizenship (e.g., the right to petition Congress for redress or the right to vote for federal officers). The statute here does not affect such rights.
As part of legislation enacted for the stated purpose of improving science skills of schoolchildren, Congress appropriated funds to permit public school teachers who had been certified by state school districts as science lab instructors to provide supplemental science instruction to any students in either public or private schools who did not have access to science lab resources. To help ensure content neutrality, the statute required the instructors coming to the private schools to use portable science labs supplied by the public school districts, which contained the equipment and experiments that the instructors used for the same purpose in the public schools. A citizens' group filed suit in federal district court to challenge the constitutionality of funding the science teachers for private schools, alleging that most of the private schools covered by the statute were religiously affiliated schools. No members of the group have any children in either public schools or private schools affected by the statute. How is the court likely to rule? A Dismiss the case on the pleadings, because the citizens' group does not have a sufficient stake in the controversy to have standing to challenge Congress's expenditure, which was authorized under its power to spend for the general welfare. B Decide the case on the merits in favor of the government, because the legislation defines the context in which instruction can be provided in private schools so as to avoid excessive government entanglement with religion. C Decide the case on the merits in favor of the citizens' group, because the appropriation's primary effect advances religion in violation of the Establishment Clause of the First Amendment. D Decide the case on the merits in favor of the citizens' group, because the court will presume that any instruction provided on the premises of a religiously affiliated school will be influenced by religion.
B Decide the case on the merits in favor of the government, because the legislation defines the context in which instruction can be provided in private schools so as to avoid excessive government entanglement with religion. The court will probably decide in favor of the government on the merits. Programs of aid to religiously affiliated grade schools and high schools are subject to the same three-part test as are other laws under the Establishment Clause: The program must (i) have a secular purpose, (ii) have a primary effect that neither advances nor inhibits religion, and (iii) not produce excessive government entanglement with religion. With respect to the first prong of the test, most of the time such programs (including this one) will have a secular purpose—to aid education. With respect to the second prong, the program may be deemed to have a primary effect that advances religion if it results in governmental indoctrination of religion or defines its recipients by reference to religion. Here, the statute establishes a religiously neutral program that funds a supplemental service for the schools, and offers the instruction to all disadvantaged students regardless of whether they choose to attend public or private schools. [See Agostini v. Felton (1997)—government program providing remedial education services to all disadvantaged children at their schools, including children at parochial schools, held not to violate the Establishment Clause] Thus, (C) is incorrect. (D) is incorrect because the courts will not presume that the instruction provided by this program will be influenced by religion. Furthermore, with respect to the "excessive entanglement" prong of the test, there is no indication that the program requires detailed monitoring of the government employees to prevent them from incorporating religion in their instruction—the equipment and experiments that they use in the private schools are the same as they use in the public schools. (A) is incorrect because the citizens' group has standing to challenge the expenditure on behalf of its members, who have a right to sue based on their status as federal taxpayers. The one recognized exception to the rule that people do not have standing as taxpayers to challenge the way tax dollars are spent by the federal government is if the expenditure was enacted under Congress's taxing and spending power and allegedly exceeds the specific limitation on that power found in the Establishment Clause. That exception applies here because the citizens' group is alleging that the federal appropriation is an unconstitutional attempt to provide government funds to religiously affiliated schools.
A plaintiff filed a civil action based on negligence against a defendant in federal district court, alleging that the defendant negligently ran a red light at an intersection and collided with the plaintiff's vehicle, causing the plaintiff's injuries. A week after the close of discovery, the plaintiff filed a motion for summary judgment on the issue of whether the defendant was negligent. With the motion, the plaintiff filed (i) his own sworn affidavit, which stated that the traffic signal was green as he entered the intersection, (ii) an affidavit of a witness who was driving the car behind him, which stated that the witness saw the entire incident and that the plaintiff's traffic signal was green as he approached and entered the intersection; and (iii) an affidavit of another witness, which stated that she saw the entire incident and that the defendant's signal had been red for several seconds before the defendant entered the intersection and was still red when the defendant entered the intersection. In response to the motion, the defendant filed her own affidavit, which stated that her traffic signal was green when she entered the intersection. She also filed the sworn deposition transcript of a pedestrian on the scene, in which the pedestrian said that, although he was intoxicated, he saw the entire accident and that the defendant's traffic signal was green as the defendant approached and entered the intersection. How should the court rule on the plaintiff's motion for summary judgment? A Deny the motion, because the plaintiff cannot obtain summary judgment on issues on which he has the burden of proof. B Deny the motion, because both parties have come forward with evidence from which a jury could find for either party if the jury believes that party's evidence. C Grant the motion, because valid reasons exist to question the credibility of the evidence that the defendant presented. D Grant the motion, because the evidence presented by the plaintiff is substantially more persuasive than the evidence presented by the defendant.
B Deny the motion, because both parties have come forward with evidence from which a jury could find for either party if the jury believes that party's evidence. The court should deny the plaintiff's motion. Summary judgment may be granted if, from the pleadings, affidavits, and discovery materials, it appears that there is no genuine dispute of material fact and the moving party is entitled to judgment as a matter of law. The court may not decide disputed fact issues on a motion for summary judgment; if there is a genuinely disputed material fact (meaning a dispute backed by evidence on both sides of the issue), the case must go to trial. Here, although the plaintiff's case may seem stronger, the defendant has presented some evidence showing that she was not negligent. Thus, the case must proceed to trial. (A) is incorrect because summary judgment may be granted to the plaintiff on issues on which he has the burden of proof. (Here, had the defendant not responded with evidence, summary judgment could have been granted.) (C) and (D) are incorrect because, in a motion for summary judgment, the credibility of the witnesses and the strength of the evidence is not weighed.
The plaintiff is suing the defendant for personal injuries suffered when she was struck by the defendant's car. The plaintiff testified regarding the extent of her injuries. The defendant seeks to testify that when she visited the plaintiff in the hospital, the plaintiff said that if the defendant paid all of her medical bills and gave her $25,000, she would not institute legal proceedings. The plaintiff's attorney objects to the offered testimony. How should the judge rule on the admissibility of the testimony? A Inadmissible, because it is hearsay. B Inadmissible, because it was an offer of compromise. C Admissible, because it is a statement against interest and, thus, an exception to the hearsay rule. D Admissible, because it is relevant to show that the plaintiff tried to "blackmail" the defendant, and does not really have a claim.
B Inadmissible, because it was an offer of compromise. The judge should rule the testimony inadmissible. Evidence of compromises or offers to compromise is inadmissible to prove or disprove the validity or amount of a disputed claim. (A) is incorrect; the evidence is not hearsay because it is a statement made by and offered against a party. (C) is incorrect because the statement against interest exception requires that the witness be unavailable. Here, the plaintiff is available to testify. (D) may be relevant, but the facts show that it was merely an offer to compromise a disputed claim, and therefore public policy encouraging settlements precludes the introduction of such evidence. Thus, (B) is correct.
A state statute defines all murders as second degree murders unless deliberation and premeditation can be shown, in which case the crime is elevated to first degree murder. Manslaughter is defined as at common law. The defendant, just having been served with divorce papers, decided to drown his sorrows at the local pub. After drinking heavily and becoming very intoxicated, the defendant became enraged when another patron spilled a drink on him. He took a nearby ashtray and smashed it over the patron's head, killing him instantly. The crimes below are listed in descending order of seriousness. What is the most serious crime of which the defendant could be convicted? A Murder in the first degree. B Murder in the second degree. C Voluntary manslaughter. D Involuntary manslaughter.
B Murder in the second degree. The most serious crime that the defendant would be convicted of is murder in the second degree. The jurisdiction defines murder in the first degree as deliberate premeditated murder, whereas all other types of killings are defined as at common law. Deliberation and premeditation requires some time of cool reflection on the idea of killing. In the instant case, there are no facts indicating that the defendant coolly reflected on the idea of killing. The facts indicate an impulsive killing rather than any type of deliberate, premeditated killing. Furthermore, the facts indicate that the defendant was very intoxicated, which would serve as a basis for reducing the crime to second degree murder. Thus, (A) is incorrect. The state defines murder in the second degree just like common law murder. At common law, murder required malice; i.e., (i) the intent to kill, (ii) the intent to inflict great bodily injury, (iii) reckless indifference to an unjustifiably high risk to human life, or (iv) the intent to commit a felony. Here, the reckless indifference element arguably could be satisfied. By smashing a heavy ashtray over the other patron's head, the defendant unjustifiably disregarded that the blow could be a killing blow. Neither would intoxication be a defense, as there would be no specific intent-to-kill requirement under this type of analysis. (D) is incorrect. Involuntary manslaughter is a killing committed with criminal negligence or during the perpetration of some unlawful act not encompassing a felony for felony murder. Certainly, striking another with a heavy object would constitute criminal negligence sufficient for conviction. However, involuntary manslaughter is a lesser crime than murder in the second degree, and the call of the question asks for the most serious crime of which the defendant could be convicted, making (B) a better choice than (D). (C) is also incorrect. Voluntary manslaughter is a killing committed under the duress of an adequate provocation, and it requires (i) a provocation sufficient to arouse the sudden and intense passion in the mind of an ordinary person such as to cause him to lose self-control; (ii) the defendant to be in fact provoked; (iii) an insufficient time to cool off; and (iv) the defendant did not in fact cool off. It is unlikely that having a drink spilled on him would cause an ordinary person to commit murder. As a result, such a provocation is not adequate to reduce the killing to voluntary manslaughter.
After drinking at his favorite bar all day long and becoming very intoxicated, the defendant pulled out a gun to reenact a scene from a movie he had seen the other day. Pointing his gun at a bystander, he slurred a line from the movie and pulled the trigger. He was shocked to see the bystander fall down dead with a bullet in him. Due to his intoxicated state, the defendant cannot remember pulling out or firing the gun. A state statute defines all murders as second degree murders unless deliberation and premeditation can be shown, in which case the crime is elevated to first degree murder. The crimes below are listed in descending order of seriousness. What is the most serious crime of which the defendant may be convicted? A First degree murder. B Second degree murder. C Manslaughter. D No crime.
B Second degree murder. The defendant may be convicted of second degree murder. In states that divide murder into degrees, evidence that the defendant was intoxicated may be used to show that the defendant was unable to premeditate and deliberate on the idea of killing, making (A) incorrect. In such jurisdictions, however, the fact that the defendant was intoxicated may not further reduce the killing from second degree murder to manslaughter, given that second degree murder encompasses common law murder and thus includes depraved heart murder. Carrying and firing a gun while intoxicated certainly shows a reckless indifference to an unjustifiably high risk to human life, thus meeting the definition of second degree murder. This makes (B) correct and (C) and (D) incorrect.
To satisfy a debt owed to a creditor, a son executed and delivered to the creditor a warranty deed to a large tract of undeveloped land. The creditor promptly recorded the deed. Shortly thereafter, she built a house on the property and has lived there ever since. The son never actually owned the land. It belonged to his father, but the father had promised to leave the property to the son. Later, the father died and his will devised the property to the son. Pressed for money, the son then sold the land to an investor by warranty deed, which the investor promptly recorded. Although the investor paid full value for the property, he purchased it strictly for investment and never visited the site. He therefore did not realize that the creditor was living there, and knew nothing of the son's earlier deed to the creditor. The jurisdiction in which the land is located has the following statute: "A conveyance of an estate in land (other than a lease for less than one year) shall not be valid against any subsequent purchaser for value without notice thereof unless the conveyance is recorded." Which of the following is the most likely outcome of a quiet title action brought by the creditor against the investor? A The creditor prevails, because the son had no title to convey to the investor. B The creditor prevails, because the investor was not a purchaser for value without notice of the creditor's interest. C The investor prevails, because under the doctrine of estoppel by deed, title inures to the benefit of the original grantee only as against the grantor. D The investor prevails, because under the recording acts, the deed from the son to the creditor was not in the chain of title and hence did not constitute notice to the investor.
B The creditor prevails, because the investor was not a purchaser for value without notice of the creditor's interest. The creditor will prevail in a suit to quiet title because the investor had notice of the creditor's interest in the property and, thus, is not a bona fide purchaser for value. When a grantor purports to convey property that he does not own, his subsequent acquisition of title to that property vests in the grantee under the doctrine of estoppel by deed. Most courts, however, hold that this is personal estoppel, which means that title inures to the grantee's benefit only as against the grantor, not a subsequent bona fide purchaser. If the grantor transfers his after-acquired title to an innocent purchaser for value, the bona fide purchaser gets good title. There is a split of authority as to whether the original grantee's recordation of the deed imparts sufficient notice to prevent a subsequent purchaser from being a bona fide purchaser, but the majority view is that it does not because it is not in his chain of title. Thus, it is not the fact that the creditor recorded that prevents the investor from being a bona fide purchaser. The fact that the creditor built a home and was living on the property gave the investor constructive notice of her interest. A title search is not complete without an examination of possession. If the possession is unexplained by the record, the subsequent purchaser is charged with knowledge of whatever an inspection of the property would have disclosed and anything that would have been disclosed by inquiring of the possessor. Therefore, the investor is charged with knowledge of the creditor's possession and with what the creditor would have told him about her possession; i.e., that the property was conveyed to her by the son prior to his conveyance to the investor. Consequently, the investor does not qualify as a bona fide purchaser, and (C) is an incorrect choice. (A) is incorrect because, although the son is estopped to deny that he acquired title for the benefit of the creditor, he could have conveyed valid title to a subsequent purchaser for value who had no notice of the creditor's interest. Therefore, it is not exactly correct to say that the son had no title to convey. (D) is incorrect because the investor will not prevail. It is true that under the recording acts the creditor's deed was not in the chain of title, but the investor still does not qualify as a bona fide purchaser. The investor is on inquiry notice arising from the creditor's possession of the property.
The state has the following homicide statutes: Murder is the unlawful killing of a human being with malice aforethought. Such malice may be express or implied. It is express when there is manifested a deliberate intention to unlawfully take away the life of a fellow creature. It is implied when no considerable provocation appears or when the circumstances attending the killing show an abandoned and malignant heart. All murder that is perpetrated by willful, deliberate, or premeditated killing or committed in the perpetration of or attempt to perpetrate arson, rape, robbery, or burglary is murder of the first degree. All other kinds of murders are of the second degree. The defendant and her associate entered a jewelry store to shoplift a diamond bracelet. Just as the defendant put the bracelet into her pocket, a sales clerk saw her and grabbed her by the wrist. The associate grabbed a knife from one of the silver displays and lunged at the sales clerk, but then a store guard shot and killed her. The defendant is charged with the first degree murder of her associate. Which of the following is the defendant's strongest argument? A The defendant cannot be convicted of murder because when they went into the store they were not carrying any weapons; therefore, there was no felony on which the felony murder rule may arise. B The defendant cannot be convicted of murder because the associate's death was not murder but justifiable homicide. C The defendant cannot be convicted of murder because she and her associate had an agreement never to use violence when they stole anything. D The associate did not intend to hurt the sales clerk, but just wanted to scare him so that the defendant could run.
B The defendant cannot be convicted of murder because the associate's death was not murder but justifiable homicide. The defendant's strongest argument is that her associate's death was justifiable homicide. Most courts today would not allow the defendant to be convicted on a felony murder theory when a co-felon is killed by a third party during the crime. Some courts base this result on the fact that the person who did the killing was justified in doing so. (A) is wrong. Aside from the fact that the attempt to steal from the jewelry store is probably statutory burglary, the fact that the associate attempted to aid the defendant in stealing the bracelet by attacking the clerk with a knife is probably robbery. When or how the associate came by the dangerous weapon is immaterial. (C) is wrong because the circumstances of one co-felon breaking an agreement not to commit violence would not prevent the application of the felony murder rule if it were otherwise applicable. (D) is wrong because if the felony murder rule is otherwise applicable, the fact that the person who killed the co-felon may have mistaken the co-felon's intentions does not prevent the operation of this rule.
A driver drove into an intersection and struck a pedestrian. The driver immediately left his car and ran to the pedestrian. Before the ambulance arrived, the driver said to the pedestrian, "It was all my fault; I'm sorry I ran a red light." The driver then said, "I'll pay for all your medical expenses." The pedestrian sued the driver for his injuries and, at the resulting trial, the pedestrian wished to testify to the two statements made by the driver. The defense objected. How should the court rule? A Both of the driver's statements are admissible. B The driver's statement acknowledging that he ran a red light is admissible, but his promise to pay the pedestrian's medical expenses is inadmissible. C The driver's statement acknowledging that he ran a red light is inadmissible, but his promise to pay the pedestrian's medical expenses is admissible. D Neither of the driver's statements is admissible.
B The driver's statement acknowledging that he ran a red light is admissible, but his promise to pay the pedestrian's medical expenses is inadmissible. The court should rule the driver's statement acknowledging that he ran a red light is admissible, but his promise to pay the pedestrian's medical expenses inadmissible. The driver's statement acknowledging that he ran a red light is admissible as a statement by an opposing party (commonly called an admission), while his promise to the pedestrian is inadmissible as an offer to pay medical expenses. An admission is a statement made by a party and offered against that party. The Federal Rules of Evidence treat such statements as nonhearsay. In his first statement, the driver acknowledges that he ran the red light, and that the accident resulting in the pedestrian's injuries was his fault. This statement qualifies as an admission by a party-opponent. Evidence that a party paid (or offered to pay) an injured party's medical bills is not admissible to prove liability for the injuries. Such payment (or offer to pay) might be prompted solely by humanitarian motives. The driver's second statement is clearly an offer to pay the medical bills of the pedestrian, an injured party. Thus, the statement is not admissible to prove liability for the pedestrian's injuries (and proving liability appears to be the only reason the pedestrian has for attempting to introduce the statement into evidence). Note that had this been a single statement, the outcome would have been the same, because an admission of fact accompanying offers to pay medical expenses is admissible. (A) is wrong because, as explained above, the driver's second statement is inadmissible. (C) is wrong as to both statements because the admission is admissible and the offer to pay is inadmissible. (D) is wrong because the admission is admissible evidence.
A landlord owned a prestigious downtown office building. A law firm leased the entire building from the landlord for a term of 20 years. The lease included a provision that taxes on the building would be paid by "the lessee, his successors, and assigns." The law firm occupied the building and paid the rent and taxes for eight years. At the end of the eight-year period, the law firm assigned the balance of the lease to an accounting firm and vacated the premises. The assignment was written, but there was no provision concerning the accounting firm's assumption of the duties under the lease. The accounting firm occupied the building and paid the rent and taxes for five years. At the end of the five-year period, the accounting firm subleased the building for five years to an investment company and vacated the premises. The sublease was written, but there was no provision concerning the investment company's assumption of the duties under the lease. The investment company now occupies the building and has paid the rent but not the taxes. The landlord has sued all three (i.e., the law firm, the accounting firm, and the investment company) for failure to pay the taxes. The landlord should prevail against whom? A The law firm only. B The law firm and the accounting firm, but not the investment company. C The accounting firm and the investment company, but not the law firm. D The law firm, the accounting firm, and the investment company.
B The law firm and the accounting firm, but not the investment company. The law firm and the accounting firm are liable. After an assignment, the original tenant is no longer in privity of estate with the landlord. However, a tenant may still be held liable on its original contractual obligations to the landlord on privity of contract grounds. Here, the law firm is liable because it made the original deal with the landlord, which included the obligation to pay taxes on the building. The law firm remains in privity of contract with the landlord throughout the term of the lease unless it is otherwise discharged. In an assignment, the assignee stands in the shoes of the original tenant in a direct relationship with the landlord. Each is liable to the other on all covenants in the lease that run with the land, which would include the obligation of the lessee to pay taxes on the property. Here, the accounting firm is liable because as an assignee it is in privity of estate with the landlord. The accounting firm remains in privity of estate until it assigns to someone else. The sublease to the investment company is not an assignment. A sublessee is not personally liable to the landlord for rent or for the performance of any other covenants made by the original lessee in the main lease (unless the covenants are expressly assumed) because the sublessee does not hold the tenant's full estate in the land (so no privity of estate). Here, the investment company is not liable because, as a nonassuming sublessee, it is not in privity of contract or estate with the landlord. Therefore, (B) is the correct choice, and (A), (C), and (D) are wrong.
A statute requires that any pilot who flies passengers for hire must have a commercial pilot's license. An experienced pilot who had only a private pilot's license and not the commercial license required by statute was asked by an attorney to fly her to another city to close a deal. The attorney knew that the pilot did not have a commercial license but the only commercial flight to the city was at an inconvenient time. The pilot flew the attorney through bad weather and landed safely, but because of a minor navigational error he landed at an airport a few miles away from the airport he was heading for. As he was going to start taxiing toward the hangar, another plane struck the aircraft. The student pilot of that plane had ignored the control tower's instructions and gone onto the landing runway instead of the takeoff runway. The attorney was injured in the collision. If the attorney sues the pilot for her injuries, who will prevail? A The pilot, because the attorney knew he lacked a commercial license and voluntarily assumed the risk of flying with him. B The pilot, because the injuries to the attorney were caused by the negligence of the student pilot of the other plane. C The attorney, because the pilot violated a statute designed to prevent persons without commercial licenses from flying passengers for a fee, and such violation imposes liability per se. D The attorney, because the pilot landed at the wrong airport, and but for this mistake the attorney could not have been injured by the other aircraft.
B The pilot, because the injuries to the attorney were caused by the negligence of the student pilot of the other plane. The pilot will prevail because the conduct of the other plane's student pilot constituted a superseding intervening force that relieves the pilot from liability. To establish a prima facie case for negligence, the attorney must show that the pilot's breach of his duty to her was the actual and proximate cause of her injury. The attorney can establish actual cause because but for the pilot's error, she would not have been injured. However, not all injuries "actually" caused by a defendant will be deemed to have been proximately caused by his acts. The general rule of proximate cause is that the defendant is liable for all harmful results that are the normal incidents of and within the increased risk caused by his acts. This rule applies to cases such as this, where an intervening force comes into motion after the defendant's negligent act and combines with it to cause plaintiff's injury (indirect cause cases). Here, the pilot's navigational error did create a greater risk of collision with other planes in the process of landing, but it did not increase the risk of a plane using the landing runway to take off in disregard of the control tower's instructions once the pilot was safely on the ground. Hence, the student pilot's unforeseeable conduct was not within the increased risk created by the pilot's negligence and constitutes a superseding force that breaks the causal connection between the pilot's conduct and the attorney's injury, enabling the pilot to avoid liability to the attorney. (A) is incorrect because assumption of the risk requires knowledge of the specific risk and the voluntary assumption of that risk. Although the attorney knew that the pilot lacked a commercial license, she also was under the impression that he was a very good pilot. There is no indication that she knew of or voluntarily assumed any risk. Certainly, she did not assume the risk of the type of harm she suffered. (C) is incorrect because even though the pilot may be negligent per se, he would not be liable per se. A clearly stated specific duty imposed by a statute may replace the more general common law duty of due care when: (i) the plaintiff is within the class to be protected by the statute, and (ii) the statute was designed to prevent the type of harm suffered. The statutory duty is clearly stated and arguably applies here because the attorney, as the pilot's paying passenger, is within the protected class, and runway collisions and other pilot errors are what the license requirement is officially designed to prevent. There are no grounds for excusing the pilot's violation of the statute, so the pilot's conduct could be seen as "negligence per se." This means that plaintiff will have established a conclusive presumption of duty and breach of duty. However, for the attorney to prevail, she must also establish actual and proximate causation. As explained above, the attorney will not be able to show that the pilot's negligence was the proximate cause of her injuries. Thus, while she may be able to establish "negligence per se," she has not made a case for "liability per se." (D) is true as far as it goes. An act or omission to act is the cause in fact of an injury when the injury would not have occurred but for the act, and this injury would not have occurred but for the pilot's landing at the wrong airport. However, the attorney must also be able to establish that the pilot's conduct was a proximate cause of her injury. As noted above, the pilot's conduct was not a proximate cause of her injury because the student pilot's actions acted as a superseding intervening force.
A town adopted an ordinance providing that a person must have been a resident of the town for at least one year to be eligible to vote in school board elections. A resident who moved to the town seven months ago attempted to register to vote in the school board elections scheduled for the next month. However, the town clerk refused to register the resident because he will not have resided in the town for a full year prior to the election. The resident filed a class action suit on behalf of all of the new residents of the town, challenging the validity of the one-year residency requirement. Which of the following statements is correct? A If the resident's suit is not heard before the election, it will be dismissed as moot, because the resident will have met the residency requirement by the time of the next annual election. B The resident will prevail even if the matter is not decided until after next month's election. C As long as there is some legitimate purpose for the one-year residency requirement, such as the need to prepare voting lists, the residency requirement will be upheld. D The resident will lose because one-year residency requirements have been held permissible restrictions on the right to vote in local elections.
B The resident will prevail even if the matter is not decided until after next month's election. The resident will prevail even if the matter is not decided until after the election, because the suit is not moot and the residency requirement is unconstitutional. The resident's suit is not moot even if the matter will not be decided until after the election because other members of the class might have a live controversy. Under the case and controversy requirement of the Constitution, there must be a real, live controversy at all stages of the suit. If through the passage of time, the controversy between the parties is resolved, the case is said to be moot. However, there are exceptions to the mootness doctrine. In a class action, it is not necessary that the suit by the named plaintiff be viable at all stages, as long as the claim is viable by some member of the class. Thus, the suit here would not be moot. Moreover, the residency requirement here violates the resident's fundamental rights to vote and to interstate travel. A restriction on the right to vote is subject to strict scrutiny and is valid only if it is necessary to achieve a compelling state interest (otherwise the restriction violates the Equal Protection Clause by treating new residents differently from old residents). Relatively short residency requirements (e.g., 30 days) have been upheld as being necessary to promote the compelling interest of assuring that only bona fide residents vote. However, the Supreme Court has struck down longer durational requirements for lack of a compelling justification. Thus, the one-year requirement here probably unconstitutionally impinges on the right to vote. The residency requirement also impinges on the fundamental right to travel in the same manner (i.e., it discourages people from migrating by denying them the right to vote without a compelling reason). Thus, the requirement is invalid. (A) is incorrect because, as indicated above, the case will not be moot since other members of the class might have a live controversy. (C) is incorrect because it applies the wrong standard. Because fundamental rights are affected by the residency requirement here, the government must show a compelling justification; a mere rational or legitimate basis is not enough. (D) is incorrect because, as stated above, the Supreme Court has found that there was no compelling interest for a one-year residency requirement in order to vote.
Adherents of a particular religion whose tenets focused mostly on business practices forbade women from studying their sacred texts. A group of college students who were adherents of that religion applied to use an empty room at their state college to study sacred texts. The school permitted numerous student groups to use its facilities for extracurricular activities during times when classes were not in session. However, the school administration denied the requests from the group in question, claiming that it would be in violation of a state statute forbidding any group using public facilities to discriminate on the basis of race or gender. The students brought an action in federal court challenging application of the statute to them by the school administration. If the court finds the actions of the school valid, what is the most likely reason? A Permitting the religious group to hold the meeting in a public school facility would violate the Establishment Clause, applicable to the state under the Fourteenth Amendment. B The statute is the least restrictive means of advancing the state's compelling interest in ending discrimination by groups using public facilities. C Allowing student groups to use classroom facilities when classes are not in session does not constitute state action for purposes of the Fourteenth Amendment. D The right of freedom of association does not apply to groups involved in business and commercial activities.
B The statute is the least restrictive means of advancing the state's compelling interest in ending discrimination by groups using public facilities. If the school's action is valid, it will be because the state statute is the least restrictive means of advancing the state's compelling interest in ending discrimination by groups using public facilities. While schools are generally not public forums, they may become a designated public forum by being held open to student groups for meetings. In that case, the First Amendment may be violated if a college restricts use of its classrooms based on the content of a student group's speech. To justify content-based regulation of otherwise protected speech, the government must show that the regulation is necessary to achieve a compelling state interest that cannot be satisfied by less restrictive means. Similarly, the right to associate for expressive purposes is not absolute. At the very least, the right may be infringed to serve a compelling government interest, unrelated to the suppression of ideas, that cannot be achieved through means significantly less restrictive of associational freedoms. Here, the state's interest in not allowing its facilities to be used by groups practicing discrimination of various types is compelling. [See Roberts v. United States Jaycees (1984)] The denial of access to the student group based on the students' religious principles, while it may be viewed as content-based discrimination, is the most narrowly drawn means of advancing the state's interest. [See Bob Jones University v. United States (1983)] (A) is incorrect because a school does not violate the Establishment Clause by permitting a religious student group the same after-class access to its facilities that other student groups have. [Good News Club v. Milford Central School (2001)] (C) is incorrect because the actions of administrators of a state college in allowing or denying access to its facilities is clearly state action that brings the Fourteenth Amendment into play. (D) is incorrect. While the right to join together for expressive or political activity, which is protected by the First Amendment, may be less strong for large organizations that engage in both commercial and expressive activity than for smaller and more selective groups, it is still a recognized right. [See Roberts v. United States Jaycees, supra] Furthermore, the student group's discussion of business activity in this case is tied to its religion. Hence, the association rights of the student group are based on freedom of religion as well as freedom of expression. The state would probably have to show a compelling interest to support a restriction on the group's association rights.
A seller conveyed her residential city property to a buyer by a general warranty deed. On taking possession of the property, the buyer discovered that the garage of his neighbor encroached six inches onto his property. If the buyer wishes to compel the seller to assist him in a suit against the neighbor, which of the following covenants may he rely on to do so? A Seisin and encumbrances. B Warranty and further assurances. C Seisin and warranty. D Encumbrances and further assurances.
B Warranty and further assurances. The buyer would rely on the covenants of warranty and further assurances to compel the seller to assist him in a suit against his encroaching neighbor. Under the covenant of warranty, the grantor agrees to defend, on behalf of the grantee, any lawful or reasonable claims of title by a third party, and to compensate the grantee for any loss sustained by the claim of superior title. The covenant for further assurances is a covenant to perform whatever acts are reasonably necessary to perfect the title conveyed if it turns out to be imperfect. These covenants are "continuous" (run with the land) and require the grantor to assist the grantee in establishing title. The covenants of seisin and encumbrances do not require such assistance. A covenant of seisin is a covenant that the grantor has the estate or interest that she purports to convey. Both title and possession at the time of the grant are necessary to satisfy this covenant. The covenant against encumbrances is a covenant assuring that there are neither visible encumbrances (easements, profits, etc.) nor invisible encumbrances (mortgages, etc.) against the title or interest conveyed. While the seller may have violated these two covenants because of the garage encroachment, they do not provide the basis to compel her to assist the buyer in a title suit. Instead, the buyer merely has a cause of action against the seller for their breach. Therefore, (A), (C), and (D) are wrong.
A dog owner lived next door to a day care center. Because he had a large yard and there were no applicable zoning restrictions, he installed a kennel and began training attack dogs to sell to businesses. As soon as he opened the business and posted signs in front advertising the exceptional ferocity of the dogs, some parents who had children enrolled in the day care center became alarmed at the prospect of the dogs right next to the yard where the children played, especially because the children could see and hear the dogs being taught to attack people. Within a few months of the dogs' arrival next door, the owner of the day care lost 10% of her enrollment. If the day care owner brings a nuisance action against the dog owner, what will be the most critical factual issue that the trier of fact must resolve to determine who should prevail? A Whether the day care owner suffered other damages in addition to her economic losses. B Whether the day care owner's use of her property makes her business abnormally sensitive to the presence of the dogs. C Whether the dog owner conducted his business with reasonable care. D Whether the dog owner was apprised of the day care owner's concerns and did nothing to alleviate them.
B Whether the day care owner's use of her property makes her business abnormally sensitive to the presence of the dogs. The determining factor for the day care owner in prevailing will be whether her use of the property is abnormally sensitive to the presence of the dogs. Nuisance is an invasion of private property rights by conduct that is either intentional, negligent, or subject to strict liability. Strict liability will be the basis for a nuisance action (sometimes called an "absolute" nuisance or a "nuisance per se") when wild animals or abnormally dangerous domestic animals are involved, or when defendant is engaged in an abnormally dangerous activity. Thus, dogs known by their owner to be vicious may create a private nuisance when they interfere with the use and enjoyment of the land next door, and the owner may be subject to strict liability because of his knowledge of the dogs' dangerous propensities. [See Restatement (Second) of Torts §822, comment j] For the presence of the dogs to be an actionable nuisance, however, they must result in a substantial interference with the day care owner's use of her land. The interference will not be characterized as substantial if it is merely the result of plaintiff's specialized use of her own property. [See Foster v. Preston Mill Co., 268 P.2d 645 (1954)—D not strictly liable for blasting operations that caused female mink on P's ranch to kill their young in reaction to the vibrations] Hence, (B) states the most critical factual issue. (A) is incorrect because the day care owner does not need to establish other types of damages to recover once she has established that the dog owner's activity is an actionable interference with the use and enjoyment of her land. (C) is incorrect because the exercise of reasonable care by the dog owner is irrelevant; the day care owner's nuisance action arises from an activity for which the dog owner is strictly liable. (D) is incorrect because the dog owner's knowledge of his interference with the day care owner's use of her property would only establish that his conduct might also be an intentional nuisance, which would require the day care owner to show unreasonableness, i.e., that her injury outweighs the utility of his conduct. She does not need to make that showing for a nuisance action based on strict liability.
A defendant was charged with arson (a felony) of an antique shop. Only one corner of the shop was damaged before the fire was extinguished. Under a plea agreement, the defendant pled guilty and received a suspended sentence. Because the owner of the shop had not yet insured a recently acquired 400-year-old refectory table that was destroyed by the fire, he sued the defendant for damages. At trial, the owner offers the properly authenticated record of the defendant's conviction for arson. Should the record be admitted into evidence? A Yes, as proof of the defendant's character in order to infer liability. B Yes, as proof that the defendant set the fire. C No, because the conviction was not the result of a trial. D No, because it is hearsay not within any exception.
B Yes, as proof that the defendant set the fire. The record of the defendant's conviction should be admitted to prove that the defendant set the fire. The record of the conviction is hearsay; i.e., it is a statement, other than one made by the declarant while testifying at the trial or hearing, offered to prove the truth of the matter asserted. Under the Federal Rules, however, such judgments fall within the hearsay exception for records of felony convictions. Under the Federal Rules, judgments of felony convictions are admissible in both criminal and civil actions to prove any fact essential to the judgment, whether the judgment arose after trial or upon a plea of guilty. [Fed. R. Evid. 803(22)] For purposes of this Rule, a felony is any crime punishable by death or imprisonment in excess of one year. Arson is a felony. Consequently, a properly authenticated copy of the defendant's conviction of this crime is admissible to prove the fact that the fire that destroyed the table was set by the defendant, a fact essential to the judgment of conviction. Note that the actual plea of guilty is also admissible as a statement of a party-opponent (commonly called an admission). This type of judicial admission is not conclusive, and the defendant may explain the circumstances of the plea. The plea, being an admission, is nonhearsay under the Federal Rules. (A) is incorrect because, in a civil case, evidence of character to prove the conduct of a person in the litigated event is generally not admissible. Circumstantial use of prior behavior patterns for the purpose of inferring that, at the time and place in question, a person probably acted in accord with such patterns creates a danger of prejudice and distraction from the main issues. Therefore, the record of the conviction cannot be used to infer liability by showing the defendant's character. (C) is incorrect because, as noted above, a judgment of a felony conviction is admissible under Federal Rule 803(22) regardless of whether the conviction resulted from a trial or a guilty plea. (D) is incorrect because, as discussed above, the judgment is within the exception to the hearsay rule for records of felony convictions.
A recent law school graduate was offered a job as an aide by a state legislator. The legislator told the graduate that before she could begin working, she had to take the following loyalty oath: "I swear to uphold our state and federal Constitutions; to show respect for the state and federal flags; and to oppose the overthrow of the government by violent, illegal, or unconstitutional means." The graduate told the legislator that the oath is unconstitutional and refused to take the oath. Is the graduate correct? A Yes, as to the promise to uphold the state and federal Constitutions. B Yes, as to the promise to respect the flag. C Yes, as to the promise to oppose the overthrow of the government. D No, as to all three provisions.
B Yes, as to the promise to respect the flag. The graduate is correct as to the promise to respect the flag, but incorrect as to the other two promises. The Supreme Court has upheld oaths requiring government employees to oppose the violent overthrow of the government and to support the Constitution; hence, (A) and (C) are incorrect. However, it has held that government employees cannot be required to show respect for the flag, as a person might refuse to salute the flag on religious grounds. Thus, (B) is correct and (D) is incorrect.
While working on a construction project, a plaintiff was injured when a heavy object struck his knee. Although the plaintiff was fully compensated for his injuries at the time of the incident, he now seeks disability payments from the construction company because he has developed arthritis in the same knee. The construction company claims that the arthritis has nothing to do with the plaintiff's on-the-job injury and refuses to pay him disability money. The plaintiff sues. A doctor takes the stand to testify for the plaintiff. He is qualified as an expert witness and during direct examination states that in his opinion the blow to the plaintiff's knee caused his arthritis. On cross-examination, the construction company's attorney produces a treatise on arthritis and asks the doctor if the treatise is considered to be authoritative. The doctor responds that the treatise is a standard authority in the field, but that he did not rely on it in forming his professional opinion regarding the plaintiff's condition. The attorney then seeks to introduce into evidence a statement in the treatise that "the idea that arthritis can be caused by a single traumatic event is purely folklore, although it is widely believed by the ignorant who have no scientific basis for their beliefs." The plaintiff's attorney objects. How should the court rule on the admissibility of the statement from the treatise? A Admissible, but only for the purpose of impeaching the doctor's testimony. B Admissible, but only as substantive evidence. C Admissible, both as substantive evidence and for purposes of impeaching the doctor. D Inadmissible.
C Admissible, both as substantive evidence and for purposes of impeaching the doctor. The statement from the treatise is admissible to impeach and as substantive evidence. Under the Federal Rules, learned treatises can be used either for impeachment or as substantive evidence. One way the credibility of an expert witness may be attacked is by cross-examining him as to his general knowledge of the field in which he is claiming to be an expert. This can be done by cross-examining the expert on statements contained in any scientific publication that is established as reliable authority. Reliability of a publication may be established by: (i) the direct testimony or cross-examination admission of the expert, (ii) the testimony of another expert, or (iii) judicial notice. The Federal Rules recognize an exception to the hearsay rule for learned treatises and admit them as substantive evidence if: (i) the expert is on the stand and it is called to his attention, and (ii) it is established as reliable authority (see above). The doctor has admitted on cross-examination that the treatise is authoritative in the field. Thus, the attorney may use the statement in the treatise to attack the doctor's general knowledge of the field of arthritis by showing that the doctor's opinion that the blow to the plaintiff's knee caused his arthritis is considered to be ignorant and unfounded in the text of the treatise. As noted above, such an attack on the doctor's general knowledge of the field is a proper means of impeaching his credibility. In addition, pursuant to the Federal Rules, the statement may be read into the record as substantive evidence (i.e., as a means of proving that the plaintiff's arthritis could not have been caused by a single traumatic event, such as the blow to his knee). The statement may be used as substantive evidence because it has been brought to the attention of the doctor during cross-examination and he established it as a reliable authority, and it will be read into evidence while he is on the stand. (A), which reflects the traditional view, is incorrect because the Federal Rules permit the use of the statement in the treatise as substantive evidence. (B) is incorrect because it precludes use of the statement for impeachment purposes. (D) is incorrect because it would not allow introduction of the statement for either impeachment or substantive evidentiary purposes, and thus it is an incorrect statement of the law.
The owner of a corner lot allowed a hedge on his property to become overgrown, obstructing the view of motorists at that corner. Two motorists were driving inattentively and each ran a stop sign at the intersection bordering the lot. Their cars collided in the intersection and one of the motorists was injured. She sued the owner of the lot. The jury determined that the lot owner was 10% at fault and each of the motorists was 45% at fault. Will the injured motorist recover damages from the lot owner? A Yes, because she was not more than 50% at fault. B Yes, because it was foreseeable that motorists could be injured if the hedge was not cut back. C No, because the other motorist's negligence was a superseding cause of her injuries. D No, because the lot owner's fault was slight compared with the motorist's fault.
B Yes, because it was foreseeable that motorists could be injured if the hedge was not cut back. The injured motorist may recover damages from the lot owner because the jury found that the lot owner should have foreseen that motorists could be injured if the hedge was not cut back. The lot owner owes the duty of an owner and occupier of land to those off the premises for unreasonably dangerous artificial conditions. In contrast to overgrown weeds, which are a natural condition for which no duty is owed absent a statute, a hedge is considered an artificial condition, analogous to a fence. Hence, by letting the hedge become so large that it created a foreseeable danger to motorists by obstructing their vision, the lot owner has breached his duty to the motorist. The other elements of the motorist's negligence action (besides a duty and a breach of the duty) are actual and proximate cause, and damages. The motorist can establish actual cause by showing that, although she failed to notice the stop sign, she would have noticed another car traveling on a collision course with hers; i.e., but for the overgrown hedge, the motorist would have been able to avoid the accident. Proximate cause in an indirect case such as this can be established by showing that any intervening forces were foreseeable and not superseding. The other motorist's negligent failure to stop may also have been caused in part by the overgrown hedge and is a foreseeable intervening force that does not break the chain of causation. Hence, (C) is incorrect. Under pure comparative negligence, the injured motorist can recover 10% of her damages even though her fault was greater than that of the lot owner. (A) is incorrect because a pure comparative negligence jurisdiction allows a plaintiff to recover no matter how great her negligence is. Thus, even though the motorists are both 45% at fault and the lot owner is only 10% at fault, the motorist can recover 10% of her damages from the lot owner. (D) is incorrect because, as discussed above, the lot owner is still liable for some damages in a pure comparative negligence jurisdiction even if his fault was only slight.
A motorist was driving to a luncheon in a car that he knew did not have operating headlights. On the way there he was rear-ended by another driver who had been driving 20 m.p.h. over the speed limit posted on that stretch of road. He suffered personal injuries and his car was extensively damaged. The jurisdiction makes it a misdemeanor to drive a vehicle that does not have operating headlights. If the motorist brings an action against the other driver and the above facts are established, will he prevail? A Yes, because the other driver violated the speeding statute, but the motorist's damages will be reduced because of his violation of the headlight statute. B Yes, because the other driver violated the speeding statute, and the motorist's damages will not be reduced despite his violation of the headlight statute. C No, because the motorist's violation of the headlight statute constitutes negligence per se. D No, because the motorist has not established that driving 20 m.p.h. over the speed limit created an unreasonable risk of injury to others.
B Yes, because the other driver violated the speeding statute, and the motorist's damages will not be reduced despite his violation of the headlight statute. The motorist will recover all of his damages because the other driver's violation of the statute constituted negligence per se. A clearly stated duty created by a criminal statute may replace the more general duty of care if the proponent of the statutory standard shows that (i) he is in a class intended to be protected by the statute, and (ii) the statute was designed to prevent the type of harm that was suffered. Here, the motorist can establish that the statutory standard regarding speeding should be applied against the other driver because the speed limit was posted, the motorist, as a fellow driver, is in the class intended to be protected by the statute, and it was designed to prevent accidents such as that which occurred. No excuse for violating the statute is present in the facts; thus, violation of the statute establishes negligence per se (i.e., duty and breach of duty). The motorist has established causation and damages, completing the prima facie case. (A) is incorrect because, while an applicable statute may establish plaintiff's contributory negligence, the headlight statute does not apply here. Even though the statute was intended to protect drivers against cars being driven without headlights, it would be very difficult to show that it was designed to prevent rear-end collisions during the day, or that violation of the statute was a cause of the motorist's injury. (C) is similarly incorrect; the motorist's violation of the headlight statute does not apply in this action. (D) is incorrect because the motorist does not need to establish a breach of the general duty of care by the other driver. Here, the speed limit statute's specific duty replaces the more general common law duty of due care, and the other driver's violation of that statute constituted negligence per se.
A defendant is on trial for manslaughter after he hit a victim in a bar, causing the victim to fall and hit his head on the marble bar top. The defendant claims that he hit the victim in self-defense after the victim lunged at him with a knife. During the prosecution's case, a witness testifies that she heard the victim's friend shout at the defendant, "You just killed a helpless man!" A defense witness is called to testify that he was there and does not remember hearing the victim's friend say anything. Should the defense witness's testimony be admitted? A No, it is irrelevant to any issue in the case. B Yes, it is proper impeachment of the prosecution's witness. C No, it is improper impeachment of the prosecution's witness because it relates to a collateral matter. D No, it is improper impeachment because it does not positively controvert the prosecution witness's testimony, as the defense witness merely says he does not remember.
B Yes, it is proper impeachment of the prosecution's witness. The defense witness's testimony should be admitted as proper impeachment of the prosecution's witness. Impeachment is the casting of an adverse reflection on the veracity of a witness. A witness may be impeached by either cross-examination or extrinsic evidence, such as by putting other witnesses on the stand who contradict the witness's testimony. Here, the defense is using the testimony of its witness to impeach the prosecution witness's testimony as to what the victim's friend said. This is proper. (A) is wrong because a witness's credibility is always relevant. Furthermore, the defense witness's testimony relates to a crucial issue in the case; i.e., whether the victim was armed with a knife or was "helpless." Thus, the testimony is relevant. (C) is wrong because this is not a collateral matter. Impeachment on a collateral matter is prohibited, but a collateral matter is one that arises when a witness makes a statement not directly relevant to the issues in the case. The prosecution witness's statement about the victim being "helpless" is directly relevant to the issue of the defendant's self-defense claim and, thus, is a proper subject of impeachment. (D) is wrong because impeachment evidence need not positively controvert the prior testimony; it need only tend to discredit the credibility of the prior witness.
The defendant planned to break into a home, steal any valuables that he could easily pawn, and then burn down the home using gasoline from his lawnmower. When the defendant got to the home that night, he realized that he had forgotten the gas at home. Nonetheless, the defendant broke into the home through a basement window. Unbeknownst to him, the police were alerted by a silent alarm and arrested the defendant just as he was leaving the home with a sack filled with valuables. At common law, what crimes has the defendant committed? A Burglary and attempted larceny. B Burglary, attempted larceny, and attempted arson. C Burglary and larceny. D Burglary, larceny, and attempted arson
C Burglary and larceny. The defendant has committed burglary and larceny. Burglary consists of a breaking and entry of the dwelling of another at nighttime, with the intent of committing a felony therein. The felony need not be carried out—all that is required is that the person committing the crime have the intent to commit a felony at the time of entry. At night, the defendant broke into and entered the house with the intent to commit the felony of larceny. Regardless of whether the defendant took any property or committed a burning, the burglary was complete on his breaking and entering the dwelling at nighttime with the requisite intent. The defendant has also committed common law larceny. Larceny is the taking and carrying away of the personal property of another, by trespass, with the intent to permanently deprive the owner of his interest in the property. The element of carrying away, or asportation, is satisfied as long as there is some movement of the property as a step in carrying it away. The movement need only be slight as long as it was part of the carrying away process. Here, the defendant placed valuables in a sack and started to leave the home. This movement was sufficient to constitute a carrying away. Having acted with the requisite intent to permanently deprive the true owner of his property, the defendant has committed larceny. (A) is incorrect because, as discussed above, the defendant is liable for the completed crime of larceny rather than attempted larceny. (B) is incorrect for the same reason, and also because the defendant probably has not committed an act sufficiently close to success to be liable for attempted arson. An attempt requires the intent to commit the completed offense and an act beyond mere preparation for the offense. Traditionally, courts used the proximity test, requiring an act that is dangerously close to success. Here, although the defendant was in the house and had at one time intended to burn it, he has done nothing else toward committing the arson. In fact, he left the gas at home. It is also important to note that possession of the gas has another lawful purpose for this defendant; as such, the possession of the gas, by itself, probably would not be sufficient in this case. Hence, the defendant's acts probably are not so dangerously close to success as to make him liable for attempted arson, also making (D) an incorrect answer.
A beneficiary has filed a petition in the probate court to contest the validity of a testator's will. The beneficiary contends that when the testator executed the will eight years before, he was an alcoholic and was incapable of forming a valid testamentary intent. In support of this contention, the beneficiary seeks to offer an affidavit prepared by the testator's former attorney, which states that she was asked to prepare a will for the testator just four months before this will was made. The attorney had refused to do so because it was her opinion that the testator suffered from severe mental deficiency as an apparent result of his chronic alcoholism. How should the judge rule on the admissibility of this affidavit? A Admissible. B Inadmissible, as being violative of the attorney-client privilege. C Inadmissible, because it is hearsay not within any exception. D Inadmissible, because it is improper opinion evidence.
C Inadmissible, because it is hearsay not within any exception The judge should rule this affidavit to be inadmissible hearsay. This affidavit is clearly hearsay, and there is nothing in the facts that shows that it is admissible under any of the exceptions to this rule. Hence, (A) is wrong. (B) is wrong because the observations of the attorney would not be deemed a "communication received from the client." Also, while the attorney-client privilege generally survives the client's death, it does not apply to communications relevant to an issue between parties who are claiming through the same deceased client, such as in the probate proceedings here. (D) is incorrect because a lay person could probably testify to her opinion in this situation since it is rationally based on her own perception, it is helpful to a determination of a fact in issue, and it is not based on scientific, technical, or other specialized knowledge.
A witness is called in a contract action between a plaintiff and a defendant. The witness takes his oath and testifies. During cross-examination, the defendant's attorney asked the witness this question: "Isn't it true that even though you took an oath to tell the truth so help you God, you are an atheist and don't even believe in God?" Upon the proper objection, will the judge require that the witness answer this question? A Yes, because the question is relevant to the witness's character for truthfulness. B Yes, because instead of taking the oath, the witness could have requested to testify by affirmation without any reference to God. C No, because evidence of the beliefs or opinions of a witness on matters of religion is not admissible to impair credibility. D No, because an attack on the competency of a witness must be made at the time the witness is sworn.
C No, because evidence of the beliefs or opinions of a witness on matters of religion is not admissible to impair credibility. The judge should not require that the witness answer the question because evidence of the religious beliefs of a witness is not admissible to challenge credibility. Lack of religious belief is no longer a basis for excluding a witness. Not only are a person's religious convictions irrelevant in determining the competence of a witness, Federal Rule 610 provides that a witness's religious beliefs or opinions are not admissible to show that the witness's credibility is thereby impaired or enhanced. Thus, (C) is correct and (A) is incorrect. (B) is incorrect. While it is true that the witness could have requested a different type of oath, Rule 610 prohibits this type of question because it would have shown his lack of religious beliefs. (D) is incorrect because, as discussed above, lack of religious belief is no longer a basis for disqualification; thus, this would not constitute an attack on the witness's competency.
Congress passed legislation banning the hunting of snipe birds within the United States. The range of the snipe is quite limited; they are found primarily in only one state, although they migrate annually to several nearby states. Hunters from throughout the United States have traditionally traveled to the snipe's home state during snipe hunting season, bringing considerable revenue into the state. A state statute allows hunting of snipe during a two-week period in November and charges a $50 license fee for state residents and a $250 fee for hunters from other states. The bag limit is one snipe bird per licensed hunter. Is the state statute allowing snipe hunting valid? A Yes, because states have the right to control their own natural resources and wildlife. B Yes, because the power exercised is reserved to the states by the Tenth Amendment. C No, because of the Supremacy Clause. D No, because of the Commerce Clause.
C No, because of the Supremacy Clause. The state statute is invalid because of the Supremacy Clause. Under the Clause, if the federal government adopts legislation that it has the power to adopt, the federal legislation is supreme, and a conflicting state law is rendered invalid. The federal law here, banning the hunting of snipe, is within the federal government's power under the Commerce Clause, which gives the government power to regulate anything that might affect interstate commerce. Because the birds themselves are found in a few states, they probably cross state lines. Also, hunters come from out of state and generate revenue in the state, so interstate commerce is involved. The state law directly conflicts with the federal law because it allows hunting of snipe. Therefore, the state law will be held invalid under the Supremacy Clause. (A) is incorrect because, while states do have a limited right to control their natural resources, the right is concurrent with the federal government's power, and cannot be exercised to conflict with federal regulation in the area. Note further that a state's power to control its natural resources is also limited even if Congress does not act: A state may not adopt a law discriminating against interstate commerce or excessively burdening interstate commerce, even absent federal legislation. Regarding (B), the Tenth Amendment reserves all powers not granted to the federal government to the states. (B) is incorrect because the Court will not likely strike down on Tenth Amendment grounds a federal regulation that subjects state governments to the same regulations as apply to the private sector. In such cases, the states' interests are best protected by the states' representation in Congress. (D) is incorrect because the Commerce Clause does not render the state's action invalid; it merely gives Congress the power to act. It is the Supremacy Clause that makes the interfering state law invalid.
A patient properly sued her doctor in federal court for medical malpractice. At trial, the doctor's attorney called four well-known experts in the field who uniformly agreed that the doctor acted within the normal standard of care in treating the patient. The patient's attorney called only one young and inexperienced expert who opined that the doctor did not act within the normal standard of care; however, he contradicted himself on the stand and could not answer certain simple questions. During the trial, no motions were made by the doctor's attorney. Surprisingly, the jury returned a verdict in favor of the patient, and the doctor's attorney filed a renewed motion for judgment as a matter of law 22 days after the judgment was entered. Will the court grant the motion? A Yes, because the judgment was based on a verdict that a reasonable jury would not have had a legally sufficient basis to reach. B Yes, because there was no genuine dispute of material fact and the moving party was entitled to judgment as a matter of law. C No, because the doctor did not move for a judgment as a matter of law during the trial. D No, because a renewed motion for judgment as a matter of law must be filed within 21 days after the judgment is entered.
C No, because the doctor did not move for a judgment as a matter of law during the trial. The court should not grant the motion. A judgment notwithstanding the verdict ("JNOV") is now called a renewed motion for a judgment as a matter of law. To be valid, the party making the renewed motion must have moved for judgment as a matter of law at some time during the trial. Here, the doctor never moved for a judgment as a matter of law during the trial. Therefore, his motion for a renewed judgment as a matter of law will be denied. (A) is wrong. Although it states the correct standard of review for a renewed motion for judgment as a matter of law (the judgment was based upon a verdict that a reasonable jury would not have had a legally sufficient basis to reach a verdict), the problem in this question is that the doctor failed to move for a judgment as a matter of law at trial. Therefore, this motion cannot even be considered. (B) is wrong because this is the standard used to review a motion for summary judgment. (D) is wrong because it is a misstatement of law. A renewed motion for judgment as a matter of law must be filed no later than 28 days after the judgment is entered, not 21 days.
An attorney came to work on a Saturday. When he signed in, he was advised by the morning security guard employed by the building management that he must be out of the building by 5 p.m., when it closes. However, he stayed past 5 p.m. to complete a brief that had to be filed on Monday morning. At 5:15 p.m., the afternoon security guard set the locks on all the doors of the building and left. Because she was in a hurry, she did not check the sign-in sheet to make sure that everyone had signed out, contrary to mandatory procedures. When the attorney tried to exit 15 minutes later, he discovered that the doors were all locked and could not be opened from the inside. He used his cell phone to call for help, and a supervisor from the building arrived and let him out shortly thereafter. If the attorney sues the building management for false imprisonment, is he likely to win? A Yes, because the guard acted recklessly by locking the doors and leaving without checking that everyone was out of the building. B No, because the attorney became a trespasser by staying in the building past 5 p.m. C No, because the guard did not know that the attorney was locked in the building. D No, because the attorney suffered no harm from the confinement.
C No, because the guard did not know that the attorney was locked in the building. The attorney will lose because the guard did not know that he was still in the building. For false imprisonment, the plaintiff must show (i) an act or omission on the part of the defendant that confines or restrains the plaintiff to a bounded area, (ii) intent on the part of the defendant to confine or restrain the plaintiff, and (iii) causation. Here, because the guard apparently did not know that the attorney was still in the building, she had no intent to confine him when she locked the doors. (A) is incorrect because recklessness is not enough; while the attorney likely has a cause of action for negligence against the guard, and through respondeat superior, the building, his claim is for false imprisonment. For liability for false imprisonment, there must be an intent to confine. (B) is incorrect because his status as a trespasser, while it may otherwise make him liable to the building for trespass, does not preclude him from recovering for false imprisonment. (D) is incorrect because the attorney need not show harm from the confinement to recover for false imprisonment, as long as he was aware of the confinement.
A motorist driving home one night on a desolate two-lane road stopped when he saw a person lying on the road next to a bicycle. The cyclist had slipped and fell off his bicycle, and was knocked unconscious when he hit his head on the pavement. Not wishing to get involved and seeing that no one else was around, the motorist got back into his car and drove away without making any effort to help the cyclist, even though he had a cell phone with which he could have summoned aid. The cyclist remained lying in the same place and was later struck by another car. If the cyclist brings suit against the motorist for injuries suffered when he was struck by the other car, will the cyclist prevail? A Yes, because a reasonable person under the same circumstances would have come to the cyclist's aid. B Yes, because by stopping and examining the cyclist, the motorist, as a matter of law, assumed a duty to aid him. C No, because the motorist was not responsible for causing the cyclist to be lying by the side of the road. D No, because the cyclist's injuries were caused by the negligence of another driver.
C No, because the motorist was not responsible for causing the cyclist to be lying by the side of the road. The cyclist will not prevail because the motorist was not responsible for putting the cyclist in a position of danger. While one whose conduct places another in a position of peril has a duty to assist the person, there is no general affirmative duty to rescue, except by a professional rescuer. Also, once any person decides to assist in a rescue, he must act as a reasonable person in an emergency situation. Here, the motorist was under no duty to assist the cyclist, and his actions when he stopped and got out of his car did nothing to make the cyclist's situation worse (such as causing others not to stop because they believed the cyclist was receiving assistance). Hence, the motorist will not be liable. (A) is wrong because even though it appears that the motorist acted unreasonably, he is not liable because he had no duty to rescue. (B) is similarly wrong; his stopping did not by itself create a duty to rescue. (D) is wrong because the fact that another driver negligently injured the cyclist would not preclude the motorist from also being liable if he had been responsible for putting the cyclist in a position of danger.
A defendant visited her doctor to seek treatment for a bullet wound. While he was treating the wound, the doctor asked the defendant how she was shot. The defendant replied that she was struck by a police officer's bullet while running away from a jewelry store she had robbed, but she implored the doctor not to tell this to anyone. The doctor promised that he would not. Although the defendant was never charged by the police, the owner of the jewelry store brought suit against her seeking the value of the stolen goods. The defendant denied robbing the store. At the trial, the owner calls the doctor to testify to the statement made to him by the defendant. The defense attorney objects on the ground that such testimony is barred by the jurisdiction's physician-patient privilege. Should the objection be sustained? A Yes, because the doctor acquired this information while attending the defendant in the course of treatment. B Yes, because the doctor agreed to the defendant's specific request that this information be kept confidential. C No, because the physician-patient privilege is inapplicable to the defendant's statement. D No, because the doctor is the one who is entitled to either claim this privilege or waive it.
C No, because the physician-patient privilege is inapplicable to the defendant's statement. The court should overrule the objection because the physician-patient privilege cannot be invoked for information dealing with a nonmedical matter. Under the physician-patient privilege, a physician is foreclosed from divulging in judicial proceedings information that he acquired while attending a patient in a professional capacity, which information was necessary to enable the physician to act in his professional capacity. Information given by a patient that deals with a nonmedical matter is not protected by the privilege. Hence, the defendant's admission that she was shot while running from a jewelry store that she robbed is not barred by the privilege. (A) is incorrect because, although it is true that the doctor acquired the information while attending the defendant in the course of treatment, the privilege is inapplicable because, as discussed above, the statement deals with a nonmedical matter. (B) is incorrect because a promise to comply with a request by the patient that information be kept confidential will not by itself render the information protectable under the physician-patient privilege. To qualify for such protection, the information must have been necessary for treatment, and there must be no applicable exceptions to the privilege. (D) is incorrect because this privilege belongs to the patient. Thus, the defendant is the one who is entitled to claim or waive the privilege, not the doctor.
A defendant is on trial for murder. The only evidence linking the defendant to the crime is some blood found at the scene. The lead detective testifies that an officer took a vial containing a blood sample that had been retrieved by a crime scene technician and drove off with it. The officer is now dead. Next, the prosecution presents as a witness a crime lab chemist. The chemist will testify that he took a vial of blood that contained a label identifying it as having been retrieved from the subject crime scene, and that he performed tests that established a match between that blood and a blood sample taken from the defendant. Is the testimony of the chemist admissible? A Yes, because there has been proper authentication. B Yes, because the chemist qualifies as an expert witness. C No, because there is insufficient evidence of chain of custody. D No, because he did not take the original blood sample at the scene of the crime.
C No, because there is insufficient evidence of chain of custody. The testimony is inadmissible because it has not been shown what happened to the blood between the time the officer took it and the time the chemist examined it. Real evidence presents an object in issue directly to the trier of fact. One of the general requirements for admissibility of real evidence is that it be authenticated; i.e., that it be identified as being what its proponent claims it is. If the evidence is of a type that is likely to be confused or can be easily tampered with, the proponent of the object must present evidence of chain of custody. The proponent must show that the object has been held in a substantially unbroken chain of possession. It is not necessary to negate all possibilities of substitution or tampering; rather, what is required is to show adherence to some system of identification and custody. Here, the proponent of the blood sample (the prosecution) has not shown what the officer did with it after leaving the crime scene. There is no showing that the vial was placed directly in a properly secured area so as to diminish the possibility of tampering. In short, it has not been demonstrated that there was adherence to some defined system of identification and custody. In the absence of a substantially unbroken chain of custody, the evidence is inadmissible for lack of proper authentication, and (A) is incorrect. (B) is incorrect. Although it may be true that the chemist qualifies as an expert witness (i.e., he has special knowledge, skill, experience, training, or education sufficient to qualify him as an expert on the subject to which his testimony relates), the fact remains that his testimony is inadmissible, as explained above. (D) is incorrect because the chemist would be permitted to testify to the results of the blood comparisons if there were proper authentication of the blood taken from the crime scene. There is no need for the chemist to have taken the original blood sample himself.
A woman was struck by a brick with her name scrawled on it that was thrown through her bedroom window. The victim believes that her ex-boyfriend, who is a gang member, threw the brick because she has become active in anti-gang groups, but she did not actually see him throw it. If the ex-boyfriend is arrested and put on trial for battery, which of the following items of the victim's proposed testimony is LEAST likely to be admitted? A The victim recently moved to a new apartment and only her ex-boyfriend and a few family members knew its location. B The victim had testified against a member of her ex-boyfriend's gang last month in a drug case. C On another occasion, the victim had seen her ex-boyfriend throw a rock through the window of a rival street gang member. D Immediately after the brick went through her window, the victim heard a voice she recognized as her ex-boyfriend's yell, "If you don't start minding your own business, you'll get a lot worse than this next time!"
C On another occasion, the victim had seen her ex-boyfriend throw a rock through the window of a rival street gang member. Evidence of the defendant's other crimes or misconduct is admissible only if relevant to some issue other than the defendant's character or propensity to commit the crime charged. Such acts would be admissible to show motive, intent, absence of mistake, identity, or a common plan or scheme. Of these, the only one possibly relevant to these facts is identity. Evidence that the accused committed prior criminal acts that are so distinctive as to operate as a "signature" may be introduced to prove that the accused committed the act in question. Merely throwing an object, such as a brick, through a window could not be considered so distinctive as to operate as a signature. Thus, this evidence would not show identity. The only possible reason for offering the evidence is to show the ex-boyfriend's propensity to commit the crime charged, in which case the testimony will be inadmissible. (A) is wrong because it is circumstantial evidence that the ex-boyfriend threw the brick. It is relevant because it tends to make it more probable that he threw the brick than it would be without the evidence. (B) is wrong because it is relevant and goes to motive. It too makes it more probable that the ex-boyfriend threw the brick than it would be if the victim had not testified against a member of his gang. (D) is wrong because the victim's identification of the ex-boyfriend's voice places him at the scene and is thus relevant. It is more probable that he threw the brick than it would be in the absence of this testimony. The identification of a voice is properly authenticated by the opinion of a person familiar with the alleged speaker's voice. As his ex-girlfriend, the victim would be sufficiently familiar with the ex-boyfriend's voice to make a proper identification.
A salvage company offered for sale Confederate dollars that had been recovered when the company recently raised a shipwreck off the coast of South Carolina. A purchasing agent for a private west coast museum purchased the bills, but he had represented that he was buying them for himself in hopes of obtaining a lower price. After purchasing the bills, the agent carefully packaged them and had them shipped to his museum. While the bills were in transit, the museum burned to the ground and its owner decided that she would not rebuild because most of her collections had been destroyed. When the bills arrived after the fire, the owner opened the package only to discover that the bills were too brittle for shipping by this method—three bills had disintegrated in transit. Undaunted, the owner took the remaining nine bills and had them mounted behind a glass frame so she could display them in her study. While the bills were being framed, the owner read on the Internet that a large cache of similar bills had just been discovered, and the market price for such bills had just been cut in half. Frustrated but still undaunted, the owner hung the framed bills in her study. Unfortunately, the salt water had reacted with the pigments in the bills in such a way that shortly after they had been exposed to indirect sunlight, all of the color in the bills faded almost completely away. No other Confederate bills raised from the ocean before had similar reactions; these bills appear to have been printed using substandard dyes. Which of the following facts would give the museum owner the best basis for rescinding the contract with the salvage company? A The bills were too brittle for transport. B The discovery of a large cache of similar bills a few days after the sale. C The bills' unusual reaction to indirect sunlight. D The destruction of the museum before the bills arrived.
C The bills' unusual reaction to indirect sunlight. The circumstances of (C) offer the best grounds for rescinding the contract based on mutual mistake. When both parties entering into a contract are mistaken about existing facts relating to the agreement, the contract may be voidable by the adversely affected party if (i) the mistake concerns a basic assumption on which the contract is made; (ii) the mistake has a material effect on the agreed-upon exchange; and (iii) the party seeking avoidance did not assume the risk of the mistake. Here, both parties probably believed that the bills would be suitable for display, like other bills that had been raised from the ocean. They had no reason to suspect that the bills would discolor when exposed to indirect sunlight. This occurrence probably rendered the bills nearly worthless, creating a material imbalance in the exchange. Finally, there is nothing to indicate that the museum owner/purchasing agent assumed the risk of what occurred. (A) is incorrect. Since the museum's agent was responsible for transporting the bills, the museum clearly assumed the risk that the bills were too brittle for the type of transport. (B) is incorrect because this is a fact occurring after the contract was made. When both parties entering into a contract are mistaken about existing facts (not future happenings) relating to the agreement, the contract may be voidable by the adversely affected party. Here, the subsequent change in price cannot be considered a mistake that was made at the time the parties entered into their contract. (D) is incorrect because the circumstances do not satisfy the requirements for discharge by frustration. Frustration will exist where the purpose of the contract has become valueless by virtue of some supervening event not the fault of the party seeking discharge. To establish frustration, the following must be shown: (i) there is some supervening act or event leading to the frustration; (ii) at the time of entering into the contract, the parties did not reasonably foresee the act or event occurring; (iii) the purpose of the contract has been completely or almost completely destroyed by this act or event; and (iv) the purpose of the contract was realized by both parties at the time of making the contract. Here, the salvage company thought that the purchasing agent was purchasing the bills for himself; thus, it did not realize at the time the contract was made that the purpose of the contract was to procure the bills for the museum that was subsequently destroyed. Therefore, frustration will not be available as a ground for rescission here.
A dog whistle manufacturer's factory was located near a residential area. The manufacturer used the most effective methods for testing its whistles, but it was impossible to completely soundproof the testing area. A breeder of champion show dogs bought some property near the factory and raised and trained her dogs there. Although the whistles were too high-pitched to be perceived by human ears, they could be heard by the breeder's dogs. Consequently, the dogs often were in a constant state of agitation. In a suit by the breeder against the manufacturer, what is the likely outcome? A The breeder will prevail on a trespass theory, because the sound waves are entering onto the breeder's property. B The breeder will prevail on a nuisance theory, because the sound of the whistles is a substantial interference with the breeder's use of her land. C The breeder will not prevail, because the sound of the whistles is not a substantial interference with the breeder's use of her land. D The breeder will not prevail, because the manufacturer has acted reasonably in testing its whistles.
C The breeder will not prevail, because the sound of the whistles is not a substantial interference with the breeder's use of her land. The breeder will not recover because there has been no substantial interference with her use or enjoyment of her land, nor has there been a trespass. A private nuisance is a substantial, unreasonable interference with another person's use or enjoyment of her property. The interference must be offensive, inconvenient, or annoying to the average person in the community. It is not a substantial interference if it merely interferes with a specialized use of the land. Here, the testing of the dog whistles did not bother humans, and so it did not disturb the average person in the community. It is disturbing to the breeder's dogs, but this affects only her specialized use of her land. Thus, the manufacturer's actions do not constitute a private nuisance. (Nor do they constitute a public nuisance—an act that unreasonably interferes with the health, safety, or property rights of the community.) Therefore, (C) is correct, and (B) is incorrect. (A) is incorrect because the sounds reaching the breeder's property do not constitute a trespass. A trespass is an intentional physical invasion of another's land. Sound waves do not produce a physical invasion. Thus, the facts here do not support a basis for trespass. (D) is incorrect because the manufacturer could be found liable to the breeder even if it acted reasonably. In determining whether there is a nuisance, a court would consider the manufacturer's care in testing its whistles, but that factor alone would not be determinative. If the activities were offensive to the average person, the court might still find there is a nuisance—even if it is impossible to do a better job of soundproofing. The court would have to consider the "reasonableness" of the interference, i.e., balance the injury against the utility of the manufacturer's conduct.
A businesswoman entered into a written contract with a general contractor to build a studio and broadcast transmitter for $3 million by July 1. Among his tasks, the contractor was to install underground cables and fiberoptic lines necessary to broadcast. When digging the deep trench necessary to lay the conduit containing the fiberoptic lines, the contractor encountered a stretch of extremely soggy soil. This was an indication that an offshoot of the nearby city's aquifer underlay the property. This was not indicated on any of the geological survey maps available in the office of the county recorder of deeds. The contractor told the businesswoman that it would cost an additional $50,000 to lay the conduit through that stretch of soil. The businesswoman had already launched an advertising campaign indicating that the station would begin broadcasting on July 4, which was rapidly approaching. Therefore, when the contractor threatened to quit the job without the additional $50,000, the businesswoman reluctantly agreed orally to the contractor's demand as long as he promised that all of the work would be completed by the middle of June. The contractor agreed, proceeded to lay the conduit, and completed building the studio and transmitter by June 15. The businesswoman paid the contractor $3 million, but when the contractor demanded $50,000 more, she refused to pay it. The contractor sues the businesswoman for the $50,000. Who will prevail? A The businesswoman, because the oral modification was not effective to alter the prior written agreement. B The businesswoman, because no valid consideration was provided for the agreement to pay the additional $50,000. C The contractor, because the modification was supported by consideration. D The contractor, because he detrimentally relied on the businesswoman's promise to pay the additional $50,000.
C The contractor, because the modification was supported by consideration. The original contract was modified by the parties, and this modification discharged the payment term of the original contract ($3 million) and replaced it with a new payment term ($3,050,000). If a contract is subsequently modified by the parties, this will serve to discharge those terms of the original contract that are the subject of the modification. Generally, a modifying agreement must be mutually assented to and supported by consideration. In most cases, consideration is found to be present in that each party has limited her right to enforce the original contract as is. Generally, if a modification will benefit only one of the parties, it may be unenforceable without some consideration being given to the other party. If, however, a promisee has given something in addition to what he already owes in return for the promise he now seeks to enforce, or has in some way agreed to vary his preexisting duty, there is consideration. Here, the businesswoman agreed to pay the contractor an additional $50,000 and the contractor agreed to complete the work early—by mid-June instead of July 1. Thus, there was sufficient consideration to support the modification. Note that this modification may have been enforceable without consideration. Under the modern view, a modification is enforceable without consideration if the modification is fair and equitable in view of the unanticipated circumstances. Here, the aquifer did not appear on city surveys, so the soggy soil was not anticipated. The $50,000 represented the electrician's additional cost, so the modification seems fair. (A) is incorrect because neither the Statute of Frauds nor the parol evidence rule affects the validity of the oral modification. The Statute of Frauds does not require a writing for the modification of a construction contract, and parol evidence can be offered to show subsequent modifications of a written contract. (B) is incorrect because, as explained above, the contractor's agreement to vary his contractual duty by promising to perform all of the work by a date earlier than that originally agreed to constitutes consideration sufficient to support the businesswoman's promise to pay the additional $50,000. (D) is incorrect because the electrician was already obligated to perform; i.e., he was already obligated to do the job he did in reliance on the businesswoman's promise. Therefore, fulfilling his duty cannot be detrimental reliance.
A woman purchased a tract of land from a man by warranty deed. Unbeknownst to the woman, the man was not the actual owner of the tract. The woman built a home on the tract and moved into it. Two years later, the actual owner learned of the man's transaction with the woman and prevented the woman from entering the tract from that point forward. This led to a costly court battle. When the woman notified the man and told him that she thought it was his duty to straighten this out, he ignored her. The woman would succeed in a suit for damages against the man for breach of which of the following covenants of title? A The covenant of quiet enjoyment only. B The covenants of seisin, right to convey, quiet enjoyment, warranty, further assurances, and the covenant against encumbrances. C The covenants of seisin, right to convey, quiet enjoyment, warranty, and further assurances. D The covenants of seisin and right to convey only.
C The covenants of seisin, right to convey, quiet enjoyment, warranty, and further assurances. The woman would succeed in a suit for damages against the man for breach of the covenants of seisin, right to convey, quiet enjoyment, warranty, and further assurances, but not on the covenant against encumbrances. A general warranty deed gives the grantee six covenants of title: the right to seisin, the right to convey, a covenant against encumbrances, the covenant of quiet enjoyment, the covenant of further assurances, and a general warranty. Under the covenants of quiet enjoyment, warranty, and further assurances, the man promised that (i) the woman would not be disturbed in her possession of the tract; (ii) he would defend the woman's title against lawful claims; and (iii) he would perform whatever acts are necessary to perfect the woman's title. Because the man neither owned the tract of land nor was acting as the actual owner's agent, he breached the covenants of seisin and right to convey at the time of the conveyance to the woman. When the actual owner prevented the woman from re-entering the property, this interfered with the woman's quiet enjoyment, and the man's refusal to "straighten this out" was a breach of the covenant of further assurances. Thus, (C) is the correct answer. Hence, (A) is incorrect because quiet enjoyment was not the only covenant breached. There is nothing in the facts to suggest the property is encumbered; thus, the man did not breach the covenant against encumbrances, and (B) is therefore incorrect. (D) is incorrect because seisin and right to convey were not the only covenants that the man breached.
A blacksmith ran a small forge in a tourist attraction depicting village life in the 1800s, and produced small metal trinkets for sale as souvenirs. A tourist came into the forge and started ridiculing the blacksmith, telling him that he was foolish for practicing such an out-of-date trade when modern equipment could produce the same trinkets faster and far more cheaply. Although he maintained a calm demeanor, the blacksmith was enraged by the time the customer finished and headed back out the door. He picked up an anvil and hurled it in the general direction of the customer. The anvil fell harmlessly to the ground after traveling maybe a foot. If the blacksmith is charged with assault, which of the following statements would be most helpful for his defense? A The blacksmith did not succeed in hitting the customer with the anvil, and he knew that it was impossible to do so. B The blacksmith knew that it was impossible to hit the customer with the anvil. C The customer did not see the blacksmith throw the anvil, and the blacksmith knew that it was impossible to hit the customer with the anvil. D The customer did not see the blacksmith throw the anvil.
C The customer did not see the blacksmith throw the anvil, and the blacksmith knew that it was impossible to hit the customer with the anvil. That the customer did not see the blacksmith throw the anvil, and that the blacksmith knew it was impossible to hit the customer with the anvil, would be most helpful to the blacksmith's defense. For purposes of the MBE, an assault is either (i) an attempt to commit a battery, or (ii) the intentional creation, other than by mere words, of a reasonable apprehension in the mind of the victim of imminent bodily harm. The fact that the blacksmith knew that it was impossible to hit the customer with the anvil negates the specific intent to commit a battery that is required for the first type of assault. (If the blacksmith knew that, when he threw the anvil, it was impossible to hit the customer, the blacksmith's conduct was not motivated by the intent to commit a battery against the customer.) The fact that the customer did not see the blacksmith throw the anvil negates the second type of assault because no apprehension of harm would have been created in the customer if he did not see the blacksmith throw the weight. Because the type of assault is not specified here, (C) is a better choice than (B) or (D) because both types of assaults are negated. Choice (A) is not correct because the fact that the blacksmith failed in his attempt to hit the customer with the anvil establishes only that there was a failure to commit a battery. It does nothing to negate the blacksmith's potential liability for assault/attempted battery.
A vintner divided his vineyard into two parcels, drawing the boundaries so that the single well that had irrigated the entire vineyard fell on the border of the two properties. The vintner then conveyed the eastern parcel to his friend by a deed that contained the following covenant: If the well located on the boundary of the eastern and western parcels continues to be used for irrigation purposes and becomes in need of repair or replacement, the grantee, his heirs, and assigns and the grantor, his heirs, and assigns each promise to pay one-half of the cost of such repair or replacement. This covenant shall run with the land. The deed from the vintner to the friend was not recorded, and the vintner did not record a copy of the deed with the records for the western parcel. The friend later sold the eastern parcel to a farmer. The farmer's deed did not contain the covenant about the well. After 15 years of use by the owners of both the eastern and western parcels, the well began to fail. The farmer took it upon himself to have the well repaired at a cost of $30,000. About two weeks later, the farmer discovered the deed from the vintner to the friend in some old files. By this time, the western parcel had passed to the vintner's son by inheritance and again to the son's daughter by inheritance from the now-deceased son. The daughter knew nothing of the covenant concerning the well. The farmer presented the daughter with the bill for the well repair with a copy of the vintner/friend deed and a note that said he expected to be reimbursed for $15,000. The daughter refuses to pay, and the farmer sues. The jurisdiction has a 10-year statute of limitations for acquiring property by adverse possession, and the following recording statute: "Any conveyance of an interest in land shall not be valid against any subsequent purchaser for value, without notice thereof, unless the conveyance is recorded." For whom is the court most likely to rule? A The daughter, because the deed from the vintner to the friend was never recorded. B The daughter, because the farmer has acquired the well by adverse possession. C The farmer, because the covenant runs with the land. D The farmer, because he is a bona fide purchaser.
C The farmer, because the covenant runs with the land. The farmer will most likely prevail in his suit for one-half the cost of the well repairs because the covenant runs with the land. When a covenant runs with the land, subsequent owners of the land may enforce or be burdened by the covenant. If all of the requirements for the burden to run are met, the successor in interest to the burdened estate will be bound by the arrangement as effectively as if he had himself expressly agreed to be bound. To be bound: (i) the parties must have intended that the covenant run with the land; (ii) the original parties must have been in horizontal privity; (iii) the succeeding party must be in vertical privity with the original promisor; (iv) the covenant must touch and concern the land; and (v) generally, the burdened party must have actual or constructive notice of the covenant. Here, the intent is shown by the express language of the covenant, which says that it is intended to run with the land. Even without that language, the use of the words "heirs" and "assigns" would show the intent for the covenant to run. The original parties were in horizontal privity because at the time the vintner entered into the covenant, he and the friend shared an interest in the land independent of the covenant—as grantor and grantee. The daughter is in vertical privity with the vintner because she holds the entire interest in the western parcel held by the vintner. The covenant touches and concerns the land because promises to pay money to be used in a way connected with the land are held to touch and concern the property. Because the daughter was unaware of the covenant, the required notice seems to be missing. While it is generally true that the owner of the burdened land must have notice, it should be remembered that the requirement is a function of the recording statute. (At common law, the covenant was enforceable in an action for damages regardless of notice; this was changed by the recording statutes.) However, because the daughter is a donee (an heir) and not a bona fide purchaser, she is not protected by the recording statute and thus is subject to the covenant even without notice. For that reason, (A) is wrong. (B) is wrong because the farmer's possession does not satisfy several of the requirements for adverse possession. Because the farmer had a legal right to use the well, his use was not adverse or hostile to the rights of the vintner's son and the son's daughter, but was rather permissive. The farmer's possession also fails the exclusivity requirement because the facts state that the well was used to irrigate both parcels for most of the statutory period. (D) is wrong because the farmer's status as a bona fide purchaser has no effect on his ability to enforce the covenant. A successor in interest to the original promisee may enforce the covenant (enjoy the benefit) if there was intent and vertical privity, and the covenant touches and concerns the land. Notice is not required for the benefit to run. Thus, because the above requirements are met here, the farmer may enforce the covenant regardless of his status as a bona fide purchaser. Had the farmer taken the property as a donee, the above analysis would be the same.
A homeowner, a citizen of State A, hired an electrician, a citizen of State B, to fix the wiring in her basement and hired a gas worker, also a citizen of State B, to install a new gas stove in her kitchen. Unfortunately, the home caught fire and burned down while they were both working on their separate jobs. The homeowner sued the gas worker for negligence in federal court in State A, seeking $100,000. The homeowner promptly served the gas worker, and the gas worker timely filed an answer with the court. One month after filing the answer, the gas worker moved to file and serve a third-party complaint against the electrician, alleging that the electrician was the sole cause of the accident. Which of the following arguments is most likely to achieve the electrician's goal of dismissal of the third-party complaint? A The gas worker's motion for leave to file a third-party complaint is untimely and thus should be denied as a matter of law. B The court does not have subject matter jurisdiction over the third-party complaint because the electrician's claim and the gas worker's claim do not arise from a common nucleus of operative fact. C The gas worker's claim against the electrician is not a proper third-party claim. D Dismissing the gas worker's claim will not impede his ability to protect his rights in a separate action.
C The gas worker's claim against the electrician is not a proper third-party claim. (C) The electrician's best argument is that the gas worker's claim against the electrician is not a proper third-party claim. Under Rule 14, a defendant may assert a third-party claim against "a nonparty who is or may be liable to it for all or part of the claim against it." In other words, a third-party claim must be a derivative claim; the third-party plaintiff must be seeking indemnification or contribution from the third-party defendant. Here, the gas worker's claim is not that the electrician must indemnify him or that the electrician is a joint tortfeasor who may be jointly liable under principles of contribution. Rather, the gas worker is alleging that he (the gas worker) is not liable and that the electrician is. Because the claim is not derivative, it is not properly asserted as a third-party claim under Rule 14. (A) is incorrect. A defendant may serve a third-party complaint as of right within 14 days of serving his original answer. Thereafter, he must make a motion to serve the complaint, and it is within the trial court's discretion whether to grant or deny the motion. Here, it is unlikely that a court would deny a defendant's motion to serve a third-party complaint at such an early stage of the proceeding. (B) is incorrect because both claims arose from a single occurrence: the fire that burned down the homeowner's house. Thus, there would be supplemental jurisdiction for the gas worker's claim because it arises from the same set of facts as the homeowner's claim against the gas worker, which is based on diversity of citizenship. (D) is incorrect because the gas worker's ability to bring a separate action against the electrician is not a compelling reason for dismissing a properly asserted third-party claim.QUESTION ID: MJ154
To increase tourism, a city began sponsoring laser light shows, which proved to be very popular. Several charitable organizations received permission from the council to sponsor a show and charge admission to raise money to help support their causes. One of them hired a famous laser light artist to give their show. When the artist arrived, he began setting up his lasers for the show. A city official soon stopped him, informing him that he could use only the city's lasers because the city feared that outsiders might use powerful lasers that could cause eye damage to viewers. The artist told the charitable organization that had hired him that the success of his art depends on the power of his lasers and that he could not produce desirable effects using the city's lasers. The charitable organization appealed to the city, but the city held fast to its rule requiring all laser light artists to use the city's lasers. If the charitable organization files an action against the city, how will the court most likely rule? A Find for the charitable organization, because art is protected by the First Amendment and the city rule interferes with the artist's freedom of expression. B Find for the charitable organization, because the city rule is not the least restrictive method for achieving the city's goals. C Find for the city, because the laser light show is not speech and therefore is not protected by the First Amendment. D Find for the city, because the rule is a reasonable time, place, and manner restriction.
D Find for the city, because the rule is a reasonable time, place, and manner restriction. The city will prevail because its rule is a reasonable time, place, and manner restriction. Speech protected by the First Amendment includes not only verbal communication, but also conduct that is undertaken to communicate an idea. The laser light show, like other art, probably is protected speech. While the content of speech generally cannot be limited, the conduct associated with speech in public forums can be regulated by reasonable time, place, and manner restrictions. To be valid, the regulation must be content neutral, narrowly tailored to serve an important government interest, and leave open alternative channels of communication. The city's rule meets these requirements: The types of images displayed are not controlled, just the means of showing them; the rule is narrowly tailored because it does not regulate substantially more speech than is necessary to further an important government interest (here, preventing eye damage); and alternative channels of communication are available because the artist can use the city's equipment, albeit with less spectacular results. (A) is incorrect because while the artist's art is protected by the First Amendment, it may still be regulated by reasonable time, place, and manner regulations, as indicated above. (B) is incorrect because it states the wrong standard. To be valid, a time, place, and manner regulation need not be the least restrictive means for achieving the desired result, but rather only narrowly tailored to the result. [See Ward v. Rock Against Racism (1989)] (C) is incorrect because art, including performance art such as the laser light show, is protected by the First Amendment. As discussed above, the First Amendment guarantee of freedom of speech protects more than merely spoken or written words; it includes conduct and other forms of expression undertaken to communicate an idea.
A landowner possessed a 40-acre tract of land. He had inherited 30 acres and had possessed the other 10 acres for longer than the statutory period necessary to acquire title by adverse possession from a rancher. The landowner entered into a land sale contract promising to convey the 40 acres to a developer. The contract provided that the landowner would convey marketable title. The developer paid the landowner the purchase price and accepted a deed from him. The developer promptly recorded the deed. The rancher, having learned of the sale, brought a successful action against the developer to quiet title. The developer realized for the first time that there were no covenants for title in his deed. The developer brings an action against the landowner. What is the most likely outcome of the suit? A The developer will win, because the landowner breached the terms of the contract. B The developer will win, because the landowner misrepresented the size of the tract. C The landowner will win, because the terms of the deed control his liability. D The landowner will win, because the developer was negligent in not checking the covenants of title at the time of closing.
C The landowner will win, because the terms of the deed control his liability. The landowner will win because the terms of the deed, not of the contract, control his liability. There is an implied covenant in every land sale contract that at closing the seller will provide the buyer with a title that is "marketable." Marketable title is title reasonably free from doubt, i.e., title that a reasonably prudent buyer would be willing to accept. It need not be a "perfect" title, but the title must be free from questions that might present an unreasonable risk of litigation. Generally, this means an unencumbered fee simple with good record title. Generally, a title acquired by adverse possession is not considered marketable because the purchaser might be later forced to defend in court the facts that gave rise to the adverse possession against the record owner. Here, the marketability requirement did not have to be implied, it was an express term of the contract. Under the doctrine of merger, the contract merges into the deed, and the terms of the contract are meaningless. Even though the contract specified a "good and marketable title," it is the deed that controls, and the deed contained no covenants of title. A deed does not incorporate the title terms of a contract. Thus, (A) is wrong. (B) is wrong; it is not supported by the facts. (D) is wrong because the developer's negligence is irrelevant.
The mayor of a town received several complaints from residents regarding the growing number of adult theaters and nude dancing establishments in a nearby town. To allay fears, the mayor asked the town's attorney what could be done to prevent or at least limit such establishments from setting up business in their town, which currently follows a zoning plan that provides for residential, commercial, and light industrial uses. Which of the following most correctly describes the town's constitutional options? A The town may revise its zoning ordinance to prohibit adult theaters and nude dancing establishments because erotica is unprotected speech. B The town may revise its zoning ordinance to limit the location of adult theaters and nude dancing establishments only if this serves a compelling interest. C The town may revise its zoning ordinance to limit the location of adult theaters and nude dancing establishments to control the secondary effects of such businesses. D The town may not limit the location of either adult theaters or nude dancing establishments in any manner different from limitations on other commercial establishments.
C The town may revise its zoning ordinance to limit the location of adult theaters and nude dancing establishments to control the secondary effects of such businesses. The town may constitutionally limit the location of these establishments to control the secondary effects of these businesses. A municipality may use zoning to limit the location of adult entertainment establishments to combat the secondary effects of such businesses, such as lowering of property values, increased traffic, etc. (A) is incorrect because nude dancing (and by implication other erotica) is marginally protected speech, as it is symbolic content that conveys an erotic message. However, like other symbolic conduct, it can be regulated to serve important government interests unrelated to the suppression of speech. [See Barnes v. Glen Theatre, Inc. (1991)—allowing state to prohibit public nudity, including nude dancing] (B) reaches a correct result—municipalities can limit the location of such establishments—but uses the wrong standard. As discussed above, erotica is only marginally protected speech and can be regulated to serve important, rather than compelling, interests unrelated to the supression of speech. Protecting community morals and property values are important enough to justify regulation. (D) is incorrect because the Supreme Court has allowed municipalities to treat adult theaters differently from other theaters, despite the fact that this appears to be regulation based on the content of speech. For example, the Court has allowed a municipality to prohibit adult theaters from being located within 1,000 feet of residential zones, schools, and parks.
A landowner and her neighbor owned adjoining tracts of land. No public road abutted the neighbor's land, so the landowner granted the neighbor an express easement over the north 25 feet of the landowner's land. However, the following month the county extended the public road to the neighbor's land, and he ceased using the easement for ingress and egress. Twenty years later, the neighbor conveyed the easement to his friend, who owned the land adjoining the other side of the landowner. The following year, the neighbor conveyed his land to the landowner. None of the parties has used the easement since the public road was extended. The jurisdiction has a 15-year statute of limitations for acquiring property interests by adverse possession. At what point was the easement extinguished? A When the neighbor attempted to convey the easement to the friend without conveying the dominant tenement itself. B Fifteen years after the neighbor ceased using the easement. C When the neighbor conveyed his land to the landowner. D The easement was not extinguished.
C When the neighbor conveyed his land to the landowner. The easement was extinguished when the neighbor conveyed his land to the landowner. An easement is extinguished when the easement is conveyed to the owner of the servient tenement. For an easement to exist, the ownership of the easement and the servient tenement must be in different persons. (By definition, an easement is the right to use the land of another for a special purpose.) If ownership of the two property interests comes together in one person, the easement is extinguished. Thus, (D) is wrong. (A) is wrong because, although an attempt to convey an easement appurtenant apart from the dominant tenement is ineffective, it does not extinguish the easement. The easement continues despite the attempted conveyance and will pass with the ownership of the dominant tenement. (B) is wrong because mere nonuse does not extinguish an easement. An easement may be extinguished by abandonment, but to constitute abandonment sufficient to extinguish an easement, the easement holder must demonstrate by physical action an intent to permanently abandon the easement. Nonuse of the easement is not enough to show the intent never to make use of the easement again.
A husband was on his way to meet his wife for lunch at the restaurant in the lobby of a bank building where she worked. He had just entered the building, which was owned and operated by the bank, when he heard screams and the sound of breaking glass. A large piece of artwork made of stained glass had fallen onto the seating area of the restaurant. In the seating area he saw several injured persons, including his wife, lying in the wreckage of the artwork. He fainted and hit his head on the marble floor, fracturing his skull. The artwork had collapsed because the pedestal that the bank had provided for the artwork was not properly constructed. If the husband sues the bank for his injury, is he likely to prevail? A No, because he was not personally in the zone of danger of physical injury. B No, because he did not actually see the artwork collapse onto the diners. C Yes, because his wife was one of the persons he saw lying in the wreckage. D Yes, because the bank had provided the pedestal for the artwork.
C Yes, because his wife was one of the persons he saw lying in the wreckage. The husband will recover for his injuries because his wife was among those injured by the collapse of the artwork. The duty to avoid negligent infliction of emotional distress may be breached when the defendant creates a foreseeable risk of physical injury to the plaintiff. In most jurisdictions, a bystander who sees the defendant negligently injuring another can recover for his own distress if (i) the plaintiff and the person injured by the defendant's negligence are closely related, (ii) the plaintiff was present at the scene of the injury, and (iii) the plaintiff personally observed or perceived the event. Observation may be by sight, hearing, or other senses. Here, the husband heard the screams and the sound of breaking glass when the artwork collapsed as he entered the lobby. Even though he evidently did not see the artwork collapse on the diners, he perceived it and was present at the scene. Because his wife was one of the persons injured by the collapse of the artwork, he can recover damages for the injuries caused by his distress. (A) is incorrect because, as stated above, the majority rule allows a bystander to recover based on the factors stated above even if he is outside the zone of danger of physical injury. (B) is incorrect because, as discussed above, a plaintiff who is present at the scene of the injury may perceive the event by hearing or other senses; it is not essential that he observe it with his eyes. (D) is incorrect because it does not matter that the bank had provided the pedestal. Even if the negligent construction of the pedestal had been done by a third party, the bank remains liable to invitees on its premises because a business has a nondelegable duty to keep its premises safe for customers.
A hockey player who was playing in the final game of the season before a hostile crowd in the opponent's packed stadium had an opportunity to get his team into the playoffs, but he missed a shot into an open net as the horn sounded, ending the game. As the crowd cheered and jeered, the puck bounced back to him and he shot it in anger toward the stands. A fan who had been looking the other way turned back toward the rink just in time to be struck in the face by the puck. He suffered a broken nose and a severe gash under his eye. After the game, the league commissioner fined the player for violating league rules by intentionally directing the puck out of the playing area. If the fan sues the player for battery, will the fan likely prevail? A No, because by attending a hockey game, the fan assumed the risk of pucks being shot into the stands. B No, because the player did not have the intent to strike the fan with the puck. C Yes, because the player knew that it was substantially certain that a fan would be hit by the puck. D Yes, because the player violated league rules by intentionally shooting the puck out of the playing area.
C Yes, because the player knew that it was substantially certain that a fan would be hit by the puck. The fan will prevail in his battery action because the player had the requisite intent for battery. A prima facie case for battery requires plaintiff to prove (i) an act by defendant that brings about a harmful or offensive contact to the plaintiff's person, (ii) intent on defendant's part to bring about harmful or offensive contact, and (iii) causation. The intent element is satisfied as long as the defendant knew with substantial certainty that the harmful or offensive contact would result. Here, the player's conduct caused a harmful contact to the fan, because the player set into motion the force that caused injury to the fan. His intentionally shooting the puck into the crowded stands is enough to establish that he knew with substantial certainty that the puck would strike a spectator. (Note that even if he only intended to cause apprehension of contact, which is the intent for assault, this intent would suffice for liability for battery under the doctrine of transferred intent.) (A) is wrong because assumption of risk is not a defense to intentional torts. The fan may have assumed a risk of injury from a hockey puck's being accidentally or even negligently shot into the stands, but he did not assume the risk of a player's intentionally shooting the puck at a spectator. (B) is wrong because the player need not have intended to strike that fan to be liable. As long as he knew with substantial certainty that a fan would be struck, he is liable even if he did not single out the fan as the target. (D) is wrong. The fact that the player violated league rules when he shot the puck into the stands tends to establish only that a spectator does not impliedly consent to a puck's intentionally being shot at him, thus negating the defense of consent in a battery action. It does nothing to establish that the player did have the intent to commit a battery.
A developer subdivided a 25-acre tract of land into 100 quarter-acre lots. On each lot she built a two-unit townhouse. The deeds to each of the purchasers contained a covenant that "the grantee, his heirs and assigns" would use the property only for single-family use. All deeds were promptly and properly recorded. Subsequently, the zoning laws were amended to allow multifamily use within the subdivision. Six months later, a social worker offered to purchase an original owner's unit that was for sale. The social worker informed the owner that she planned to operate a halfway house out of the unit, an activity in conformity with the applicable zoning regulations. Therefore, the owner did not include the single-family restriction in the deed to the social worker. If a neighbor, who purchased his lot from the developer, seeks to enjoin the operation of the halfway house, will he succeed? A No, because the deed from the owner to the social worker did not refer to the covenant. B No, because the social worker relied on the zoning regulations when purchasing the unit. C Yes, because the social worker had notice of the restrictive covenant. D Yes, but only if the neighbor can establish a common scheme for development.
C Yes, because the social worker had notice of the restrictive covenant. The neighbor will succeed in enjoining the operation of the halfway house because the social worker had notice of the restrictive covenant. A covenant runs with the land to a subsequent purchaser with notice of the covenant if it touches and concerns the land and is intended to run. Notice may be actual or constructive. Here, the social worker was on record notice of the covenant because the original owner's deed was recorded. Restricting land to single-family use touches and concerns the land, and it is evident that the developer and the original owners, including the neighbor, intended it to run with the land by use of the language "grantee, his heirs and assigns." The social worker thus will be bound even though her deed did not refer to the covenant. Thus, (A) is incorrect. (B) is incorrect because compliance with zoning regulations does not excuse noncompliance with an enforceable covenant; both must be complied with. (D) is incorrect because the neighbor can prevail without needing to show a servitude implied from a common scheme, which comes into play when a developer subdivides land into several parcels and some of the deeds contain negative covenants and some do not. Here, the covenant relating to single-family use was in all of the original deeds and, as discussed above, it runs with the land. A covenant that runs with the land may be enforced as an equitable servitude if the assignees of the burdened land have notice of the covenant; the usual remedy is an injunction. Here, the social worker had record notice of the covenant and it runs with the land, so the neighbor can enforce the covenant as an equitable servitude without resort to implying a reciprocal negative servitude.
A city ordinance that gave churches veto power over the grant of liquor licenses for businesses within a quarter mile of the church was struck down by the state's supreme court. In its decision, the court noted that even if the ordinance were not a violation of the federal Establishment Clause, it clearly violated a provision in the state constitution requiring the separation of church and state because it delegated governmental authority to the churches. The supreme court opinion also harshly criticized the city council for passing an ordinance that was clearly a violation of state law. To comply with the court's decision and avoid future criticism, the city council adopted an ordinance prohibiting any cleric from sitting on any public board within the city. A cleric who is currently on a school board within the city brings a suit in federal district court to have the ordinance declared unconstitutional. How should the court rule? A Uphold the ordinance, because it has a secular purpose and it does not promote excessive entanglement between government and religion. B Uphold the ordinance, because the previous court decision was based on adequate and independent state grounds. C Hold the ordinance unconstitutional, because its passage violated the cleric's right to procedural due process. D Hold the ordinance unconstitutional, because it violates the Free Exercise Clause.
D Hold the ordinance unconstitutional, because it violates the Free Exercise Clause. The ordinance should be held unconstitutional because it violates the Free Exercise Clause. The Free Exercise Clause generally prohibits government from acting in a way that interferes with the free exercise of religion. The Supreme Court has found the Clause to provide almost absolute protection for religious belief. Here, the city is prohibiting clerics from serving on public boards. Essentially, the ordinance takes away a right of citizenship merely because a person has chosen to become a religious cleric. Such punishment of religious belief is prohibited, either absolutely or at least unless the discrimination is necessary to achieve a compelling government interest. No such compelling interest is present. Although the state supreme court found that the city could not delegate its authority regarding liquor licenses to churches, and complying with the decision might be a compelling government interest, the replacement ordinance here is not necessary to achieving that interest. (A) is incorrect, at the very least because it is incomplete. It is based on the Lemon test for determining whether government action violates the Establishment Clause, but the choice is missing a crucial element: The government action cannot substantially advance nor inhibit religion. The ordinance here—prohibiting clerics from serving on public boards—would have a tendency to inhibit religion and so would run afoul of the Lemon test. (B) is incorrect because it is irrelevant. Even if the previous decision was based on state law, the current ordinance was not at issue and clearly violates the federal Constitution. (C) is incorrect because when a government action does not focus on an individual, due process does not require an individualized hearing. The process of passing the ordinance at the city council meeting was all of the process that was due the cleric.
An antique lover spotted a beautiful Early American bedroom ensemble at her favorite antique store. The ensemble included a bed, a mirror, and two dressers. Over a period of several weeks, the shop owner and the antique lover negotiated over a price, but they were unable to come to an agreement. On April 3, the shop owner and the antique lover signed a statement whereby the shop owner offered to sell to the antique lover an Early American bedroom ensemble, recorded as items 20465, 20466, 20467, and 20468 in the shop's registry, if the parties agree upon a price on or before April 12. On April 6, the shop owner sent a letter to the antique lover, telling her that she could have the bedroom ensemble for $22,000. Also on April 6, the antique lover sent a letter to the shop owner telling him that she was willing to pay him $22,000 for the bedroom ensemble. Both parties received their letters on April 7. Without assuming any additional facts, which of the following statements is most correct as of April 8? A The shop owner and the antique lover had a valid contract from the moment the letters of April 6 were mailed. B A contract exists between the shop owner and the antique lover, because the shop owner, a merchant, sent the antique lover an offer in writing. C A contract exists between the shop owner and the antique lover, because the crossing offers were identical and received before April 12. D No contract exists between the shop owner and the antique lover, because of a lack of mutual assent.
D No contract exists between the shop owner and the antique lover, because of a lack of mutual assent. Although the crossing offers as to price were identical, there is no requisite mutual assent absent an acceptance. If offers stating precisely the same terms cross in the mail, they do not give rise to a contract despite the apparent meeting of the minds. An offer cannot be accepted if there is no knowledge of it. Here, the shop owner and the antique lover each sent offers setting the price of the ensemble at $22,000. Despite the fact that these offers were identical, there is no mutual assent without at least one of the parties manifesting acceptance of the terms of the offer, and communicating that acceptance to the other. We are told that this has not yet happened even though the shop owner and the antique lover both have received the letters. Consequently, although there is an apparent meeting of the minds as to price, there has not been a sufficient objective manifestation of this agreement as to denote a mutual assent. (C) fails to account for the principle discussed above, that identical crossing offers do not give rise to a contract. Despite their receipt of identical offers before April 12, there is no agreement between the parties. (A) is incorrect because it misstates the mailbox rule. Acceptance by mail or similar means creates a contract at the moment of posting, properly addressed and stamped, unless the offer stipulates that acceptance is not effective until received, or unless an option contract is involved. This rule does not operate to create a contract from the moment an offer is mailed (or in this case, two identical offers are mailed). Thus, (A) is incorrect. Regarding (B), the fact that a merchant sends an offer in writing is significant because it will limit the offeror's power to revoke if it gives assurances that it will be held open for a stated time. Here, the written offer by the shop owner is irrevocable at least until April 12, but the issue in the question is whether it has been accepted rather than whether it has been revoked.
A new federal law prohibited the use of various pesticides in areas with a certain population density near navigable waters. A city located in the southeastern United States was plagued by a sharp increase in disease-carrying mosquitoes. The city's board of health recommended that all residential areas be sprayed with a pesticide proven to be highly effective against mosquitoes. Despite the fact that the federal law would prohibit use of that pesticide in these areas, the city council passed an ordinance adopting the board of health plan, relying on the opinions of several independent experts that the health benefits of reducing the mosquito population outweighed the risks of spraying. An environmentally minded citizen of the city brought an action in federal court challenging the ordinance. Assuming that the citizen has standing, is the court likely to find the ordinance valid? A Yes, because pursuant to the police power, cities have a compelling interest in laws designed to protect the health, safety, and welfare of their citizens. B Yes, because controlling health hazards is an integral governmental function. C No, because it is superseded by the power of Congress to adopt laws to protect the health, safety, and welfare of citizens. D No, because it conflicts with a federal law that Congress had the power to make under the Commerce Clause.
D No, because it conflicts with a federal law that Congress had the power to make under the Commerce Clause. Congress's power to regulate commerce has been construed broadly, so that it may regulate any activity, local or interstate, that either in itself or in combination with other activities has a substantial economic effect on interstate commerce. If Congress has determined that the use of chemical pesticides and their runoff into waterways (which are channels of interstate commerce) will have an overall detrimental impact on the environment, this determination will be sufficient in this case to satisfy the standards established by the Supreme Court. Therefore, the law probably is a valid exercise of the commerce power. Any state or local action that conflicts with a valid act of Congress is invalid under the Supremacy Clause. (A) is incorrect because while the police power (the power to adopt regulations for the health, safety, and welfare of citizens) belongs to the states, a police power regulation that conflicts with a federal law is invalid under the Supremacy Clause. (B) is incorrect because state and local government activities may be regulated by a general law that applies to both the public and private sectors, even if the regulation affects integral governmental functions, as long as there is a constitutional basis for the law. (C) is incorrect because Congress does not have a general "police power" to adopt laws on health and safety. The laws that Congress has passed banning activities that it has deemed harmful to public health have been based on its power to regulate interstate commerce.
The defendant was on trial for murdering his mother, who was found dead in her bathtub. At trial, the prosecutor called the nurse of the defendant's aunt to testify to what the aunt told the nurse just before the aunt died of cancer. The nurse is prepared to testify that, shortly before she died, the aunt stated, "I know I don't have much longer to live, so I must tell someone what my nephew said to me yesterday. He told me that he was very angry with his mother and that he wanted to kill her and make it look like an accident!" Should this testimony be admitted? A Yes, because it is a statement by an opposing party. B Yes, because it falls within the hearsay exception for dying declarations. C Yes, because it is a statement by an opposing party and falls within the hearsay exception for dying declarations. D No, because it is inadmissible hearsay.
D No, because it is inadmissible hearsay. The testimony is inadmissible. Hearsay is an out-of-court statement offered in evidence to prove the truth of the matter asserted. An out-of-court statement that incorporates other hearsay is "hearsay within hearsay." Hearsay within hearsay is admissible only if both the outer hearsay statement and the inner hearsay statement fall within an exception to the hearsay rule. Here, the aunt's statement incorporating the defendant's statement constitutes hearsay within hearsay. Therefore, both statements must fall within an exception to the hearsay rule to be admissible. The defendant's statement to his aunt ordinarily would be hearsay because it is being offered to prove the truth of the matter asserted—that he intended to kill his mother. However, statements by a party-opponent (commonly called admissions) are admissible nonhearsay under the Federal Rules. Thus, the defendant's statement is admissible as a statement by a party-opponent. However, the aunt's statement made to the nurse regarding the defendant's admission must also fall within an exception to the hearsay rule. Because no exception applies to that statement, the entire testimony is inadmissible. (A) is incorrect because, while it correctly states that the defendant's statement is a statement by an opposing party, it incorrectly deems the statement admissible because it fails to address the hearsay within hearsay issue discussed above. (B) is incorrect. A declaration made by the now unavailable declarant, while believing that her death was imminent, is admissible if it concerns the cause or circumstances of what she believes to be her impending death. Here, the statement was made while the aunt believed that her death was imminent; however, the statement did not concern the cause or circumstances of her death and therefore does not qualify as a dying declaration. (C) is incorrect for the same reason. While the defendant's statement is an admission, the aunt's statement is not a dying declaration, as discussed above.
A dentist filling a child's cavities used a newly developed local anesthetic that was more effective than Novocain. However, it carried a 1% risk of causing a serious seizure when administered to children, which the dentist did not mention to the child's mother or note in the consent forms, which stated only that a local anesthetic would be used. The child's dental work was completed without any problem, but the mother looked up the anesthetic on the Internet and learned about the risk. She complained to the dentist that she would not have consented to use of the anesthetic had she known of the risk, but the dentist argued that using the new anesthetic was justified in the child's case because otherwise he would not have been willing to sit still for the dental work. Does the mother have a cause of action on behalf of the child against the dentist? A Yes, because a reasonable person would have considered information about the risk important. B Yes, because the mother would not have consented to the use of the anesthetic if she had known of the risk of seizure. C No, because the dentist used his best judgment in deciding that the benefits of using the anesthetic outweighed the risk. D No, because the child suffered no harm from use of the anesthetic.
D No, because the child suffered no harm from use of the anesthetic. The mother has no cause of action because the child suffered no damages from the dentist's breach of duty. One of the duties that doctors, dentists, and other health professionals owe their patients is the duty to provide a patient with enough information about the risks of a proposed course of treatment or surgical procedure to enable the patient to make an "informed consent" to the treatment. If an undisclosed risk was serious enough that a reasonable person in the patient's position would have withheld consent to the treatment, the health care professional has breached this duty. However, breach of duty is only one element of a cause of action for negligence. The plaintiff must also establish actual and proximate cause and some damage to plaintiff's person or property. Damage means actual harm or injury. Unlike for some intentional torts, damage will not be presumed and nominal damages are not available. Note that complete absence of consent to a medical or surgical procedure may often constitute battery, which does not require damage as an element. However, a nondisclosure of the risks of the procedure is characterized instead as a breach of the duty of care. Here, the mother consented to the surgery and use of a local anesthetic, so battery is not applicable. Further, the child's dental work was completed without any problem and no other injury is apparent from the facts; the mother's possible distress at not being informed of the risk is not, standing alone, a compensable injury. [Restatement (Second) of Torts §436A] Hence, the mother does not have a cause of action against the dentist. (A) is incorrect even though it is the key factor for establishing that the dentist breached his duty by not disclosing the risk of seizure. As discussed above, breach of duty is just one element of the prima facie case. (B) is incorrect. If the child had suffered harm from the anesthetic, the element of actual cause would be established because the mother could show that she would not have consented to the use of the anesthetic had she known of the risk (i.e., but for the dentist's nondisclosure, the child's injury would not have occurred). However, in the absence of the injury element, the prima facie case is not complete. (C) is incorrect because the fact that the dentist used his best judgment in deciding not to disclose the risk would not be a defense if the child had been harmed by the anesthetic.
A newspaper printed in a news article that a successful businessperson running for the state legislature had attempted suicide and had just been released from the hospital, where he had undergone intensive psychotherapy. Actually, the businessperson had been hospitalized because he had contracted hepatitis. The businessperson's opponent, the incumbent legislator, read the story into the legislative record the next day. The businessperson sued the incumbent for defamation. At trial, the businessperson established that the incumbent had serious doubts as to the accuracy of the story when she read it into the record. Is the businessperson likely to recover? A Yes, because he established that the incumbent acted with actual malice. B Yes, because the statement was slander per se. C No, because the incumbent was relying on the veracity of the newspaper article. D No, because the incumbent was privileged to make the defamatory statements.
D No, because the incumbent was privileged to make the defamatory statements. The businessperson will not recover against the incumbent because, as a state legislator, she was absolutely privileged to read the story into the record on the floor of the legislature. Under certain circumstances, a speaker will not be liable for defamatory statements because she is afforded an absolute privilege. Such a privilege is not affected by a showing of malice, abuse, or excessive provocation. Remarks made by either federal or state legislators in their official capacity during legislative proceedings are absolutely privileged. There is no requirement of a reasonable relationship to any matter at hand. The incumbent is a state legislator. When she read the newspaper article into the legislative record, she was speaking in her official capacity as a legislator, on the floor of the legislature. Thus, her reading of the article is cloaked with absolute privilege, and she will be shielded from liability for defamation. (A) is incorrect because, even though the businessperson appears to have established actual malice by the incumbent, he will not recover. The businessperson, as a candidate for public office, is a public figure, and information about his health is probably a matter of public concern. Thus, to recover, the businessperson must show actual malice (i.e., knowledge of falsity or reckless disregard for truth or falsity). However, this showing of malice still will not provide the businessperson a recovery because the incumbent has an absolute privilege. If she had only a qualified privilege, a showing of malice would defeat the privilege. (B) is incorrect because slander per se will not provide the businessperson a recovery. Slander per se is a characterization applied to certain categories of spoken defamation. If defamation falls within one of these categories, injury to reputation is presumed without proof of special damages. One such category is a defamatory statement that adversely reflects on the plaintiff's abilities in his business, trade, or profession. Although statements to the effect that the businessperson suffered from severe psychological problems might adversely reflect on his fitness for public office, this would be significant only in terms of establishing that the businessperson need not prove special damages. However, proof of damages is of no importance here, because the incumbent is not liable due to her absolute privilege. If the incumbent is shielded from liability, the possible existence of slander per se is of no use to the businessperson. Note also that, in any event, any defamation here would be characterized as libel, rather than slander. Libel is a defamatory statement recorded in writing or some other permanent form. Where the original defamation is libel, any repetition, even if oral, is also libel. Here, the original defamation was in a newspaper article and thus was libel. Consequently, the oral repetition of the article would also be libel, if the incumbent were subject to defamation liability. (C) is incorrect because, if the incumbent were not protected by the absolute privilege, the mere fact that she relied on the article would not afford her a defense. A republisher (one who repeats a defamatory statement) is liable on the same general basis as a primary publisher.
A homeowner and a local builder entered into a written contract that called for the builder to build a second story onto the top of the homeowner's one-story residence. When scheduling conflicts arose, the builder asked the homeowner if they could substitute his buddy, an out-of-town builder who had comparable experience and skills, to perform the local builder's part of the contract. All of the parties agreed to the substitution. Unfortunately, the out-of-town builder made a major blunder that will be quite expensive to correct. Is the local builder liable to the homeowner for the cost of correcting the defect? A Yes, because the substitution in and of itself does not relieve the local builder of liability on the underlying contract. B Yes, because the local builder did not give any consideration on which to base a release. C No, because the local builder transferred his duties to the out-of-town builder. D No, because the local builder was discharged through a novation.
D No, because the local builder was discharged through a novation. The agreement among all of the parties to substitute the out-of-town builder for the local builder operates as a novation which immediately discharged the local builder from any duties he had under the original contract. A novation arises when the parties enter into an agreement to substitute a third party for one of the parties in a contract, releasing the party who was substituted. All parties must agree to the substitution. Here, the facts say that all of the parties agreed that the out-of-town builder would substitute for the local builder. Thus, there was a novation and the local builder was released immediately and is not liable for the out-of-town builder's blunder. (A) states incorrectly that the substitution does not relieve the local builder of liability. If the parties had not all agreed to substitute the out-of-town builder for the local builder, or the facts said that there was merely an assignment of rights and delegation of duties, (A) would reflect the correct result. However, where the parties agree to substitute a new party for an old party, there is a novation that does release the old party. (B) is incorrect because there was consideration to support the release—the local builder implicitly agreed to give up his rights under the original contract, the homeowner implicitly agreed to give up his right to look to the local builder for performance, and the out-of-town builder agreed to perform. (C) is not as good an answer as (D). The mere fact that a contractual duty was transferred does not release the transferor from a duty under the contract. It is only the agreement among the parties to substitute the new party for the old that released the local builder here.
In a criminal battery case brought against the defendant, the prosecutor asked the court to take judicial notice of the fact that a car driven from Chicago to Detroit has to cross state lines. The defense attorney raised no objection, and the judge declared that she was taking judicial notice of the fact as requested by the prosecution. What is the effect of such judicial notice? A To raise an irrebuttable presumption. B To satisfy the prosecutor's burden of persuasion on that issue. C To shift the burden of persuasion on that issue to the defendant. D That the judge should instruct the jury that it may, but is not required to, accept the noticed fact as conclusively proven.
D That the judge should instruct the jury that it may, but is not required to, accept the noticed fact as conclusively proven. The effect of the judge's noticing that a car driven from Chicago to Detroit must cross state lines is that the judge will now instruct the jury that it may, but is not required to, accept that fact as conclusively proven. Under the Federal Rules, in a civil case, the court must instruct the jury to accept the judicially noticed fact as conclusive. [Fed. R. Evid. 201(f)] Because this question deals with a prosecution for criminal battery, the applicable rule is that the jury be instructed that the fact that has been judicially noticed may be accepted by it as conclusive, but that the jury is not required to do so. (A) would be correct if this were a civil case. In such an instance, the jury would be instructed to accept as conclusive the judicially noticed fact. This would have the effect of raising an irrebuttable presumption. (B) is incorrect because, in a criminal case, the prosecution has the burden of proving every element of the crime beyond a reasonable doubt. Only the jury can decide, after all of the evidence is in, whether the burden of persuasion is satisfied. (C) is incorrect because the burden of persuasion does not shift from party to party during the course of the trial. The burden of persuasion is never on a criminal defendant.
A town in a rural state facing financial difficulties passed a variety of "sin taxes," including one aimed at electronic game arcades frequented by local juveniles. The tax is a one cent per game tax imposed on the manufacturers of the games based on the estimated number of plays over a machine's lifetime. There are no electronic game manufacturers in the state. Which of the following constitutional provisions would support the best argument against enforcement of the tax? A The Equal Protection Clause. B Substantive due process. C The Privileges and Immunities Clause of Article IV. D The Commerce Clause.
D The Commerce Clause. The best argument against enforcement of the tax is that it violates the Commerce Clause. If Congress has not adopted laws regarding a subject, local governments are free to tax or regulate local aspects of the subject area as long as the tax or regulation does not discriminate against interstate commerce or unduly burden it. Here, the tax does not discriminate against interstate commerce, since it does not single out interstate commerce for taxation in order to benefit the local economy. However, it could be argued that the tax unduly burdens interstate commerce. A local tax will be held to unduly burden interstate commerce if the locality's need for the revenue does not outweigh the burden on interstate commerce. The Supreme Court will consider whether there is a substantial nexus between the activity or property taxed and the taxing state, whether the tax is fairly apportioned, and whether there is a fair relationship between the tax and the benefit the taxed party receives from the state. Here, there is little nexus between the manufacturer and the town. The facts indicate that out-of-state manufacturers' machines are used in the town, but do not indicate whether the manufacturers conduct any selling activity in the town. Similarly, nothing indicates that there is a relationship between the tax and any benefit that the manufacturers derive from the town. Thus, the tax would probably be unconstitutional under the Commerce Clause. (A) is not as good an argument as (D) because the Equal Protection Clause prohibits the states from treating similarly situated persons differently without sufficient justification. Where a classification does not involve a suspect or quasi-suspect class or a fundamental right, the classification will be upheld as long as it is rationally related to a legitimate government interest. While the tax here singles out arcade game manufacturers for special tax treatment, no suspect or quasi-suspect class is involved, nor is a fundamental right affected. Thus, the tax will be valid under the Equal Protection Clause because it is rationally related to the legitimate government interest of raising revenue. (B) is not a good argument because substantive due process requires that laws not be arbitrary. When laws do not involve a fundamental right, they will be held valid under the Due Process Clause as long as they are rationally related to a legitimate government interest. As established above, no fundamental right is involved and the tax is rationally related to a legitimate government interest. Thus, under the Due Process Clause the tax may be enforced. (C) is not a good argument because the Privileges and Immunities Clause of Article IV prohibits states from discriminating against out-of-state residents when a fundamental right is involved, and the tax here does not differentiate between residents and nonresidents.
A seller owned a large parcel of land. The western half was undeveloped, and the eastern half contained a grove of apple trees. The seller gave a buyer a deed conveying "the western half of the parcel from the western boundary to the grove of apple trees, comprising 220 acres." It was subsequently determined by survey that the land conveyed to the buyer was in fact 229 acres. In a dispute between the seller and the buyer as to the mistake, which of the following is most accurate? A The deed is invalid because of the mutual mistake of the parties. B The deed is invalid unless the court admits parol evidence as to the amount of acreage conveyed. C The deed is valid, and the buyer is the owner of 220 acres. D The deed is valid, and the buyer is the owner of 229 acres.
D The deed is valid, and the buyer is the owner of 229 acres. The deed is valid and the buyer owns 229 acres. When there is a mistake or inconsistency in the description of property in the deed, one of the rules of construction is that the physical description takes precedence over the quantity description unless there are grounds for reformation of the deed. Reformation is an equitable action in which the court rewrites the deed to make it conform to the intention of the parties. It is granted when the deed does not express the agreement of the parties due to mutual mistake or a scrivener's error, and may also be granted when there is a unilateral mistake if misrepresentation is involved. Here, the facts indicate that the seller and the buyer were bargaining for a specific physical location ("the western half of the parcel from the western boundary to the grove of apple trees") and not for a specific number of acres. Thus, there appear to be no grounds for reformation. A conflict in description does not invalidate a deed, so (A) and (B) are incorrect. While parol evidence may be admissible to ascertain the parties' intent, the absence of parol evidence will not invalidate the deed as long as rules of construction may be applied to resolve any inconsistency. (C) is incorrect because, as discussed above, physical descriptions prevail over quantity.
A critically acclaimed movie that had received a number of awards opened in a small town. The film had portrayals of nudity and scenes involving sexuality, but its advertising was very tasteful and concentrated on its critical acclaim and its receipt of seven Academy Award nominations. Nevertheless, when the movie opened in the small town, there was a public outcry against it, including picketing. The town, which had been founded in the late nineteenth century by a fundamentalist religious group, remained very conservative and highly religious, and was the only community in the state where a consensus of the community would find the movie to be obscene. The town prosecutor went to the local court seeking an injunction to halt the showing of the movie. The theater owner refused to voluntarily stop showing the film and appeared in court to defend against the proposed injunction. What is the owner's best defense? A The proper "community standards" should be those of the entire state rather than of the town. B The film has some redeeming social value. C The Establishment Clause of the First Amendment prevents the state from enforcing a particular set of religious beliefs. D The film has proven artistic merit.
D The film has proven artistic merit. The theater owner's best defense is that the film has proven artistic merit. The First Amendment generally protects the right of freedom of speech, and this freedom includes the right to show movies. Thus, to enjoin the showing of the movie here, the city will have to prove that the speech involved is unprotected speech. Obscenity is the category of unprotected speech most relevant here. The Court has defined obscenity as a depiction of sexual conduct that, taken as a whole, by the average person, using contemporary community standards: (i) appeals to the prurient interest in sex; (ii) portrays sex in a patently offensive way; and (iii) using a rational, reasonable person standard, does not have serious literary, artistic, political, or scientific value. If the theater owner shows that the film has proven artistic merit, it cannot be held to be obscene because the third element of the above definition will have failed. Thus, (D) is his best argument. (A) is not as good an argument as (D) because the Supreme Court has held that while a statewide community standard may be used, it is not mandatory—a local community standard is sufficient to evaluate whether the film is "patently offensive." Thus, the town's community standards would be sufficient. (B) is not as good an argument as (D) because it is not sufficient that there is some redeeming social value; it must have serious redeeming value, as indicated above. For example, it would not be sufficient that an otherwise obscene movie included short tips on the importance of brushing teeth. (C) is not a good argument. The Establishment Clause forbids the government from adopting a law or program that establishes religion. It is inapplicable here because the town is not trying to enforce a particular set of religious views; rather, it is trying to prohibit obscenity. The Supreme Court has held that the government has a legitimate interest in prohibiting obscenity, and the fact that this happens to coincide with the beliefs of a particular religious group does not render such bans void.
A city's water board election laws provide that, although members of the board are elected at large, one member of the board is required to live within each of the five designated water districts within the city. The city's population was more or less evenly distributed among the districts when this election law was enacted. A resident and registered voter of the city investigated the district residency requirement and discovered that most of the city's newer residents had moved into the same two water districts, so that the city's population was no longer evenly distributed among the five water districts. Instead, 80% of the city's residents lived within its central and eastern water districts, while the other 20% of the city's residents were scattered among its three other, more rural, districts. If the resident files suit in federal court challenging the constitutionality of the residency requirement, how will the court most likely rule? A The residency requirement is unconstitutional because it impairs the voters' equal protection rights, in that it gives the voters in the less populous districts more effective representation on the water board. B The residency requirement is unconstitutional because it violates the candidates' equal protection rights. C The residency requirement is constitutionally permissible because the water board members do not exercise legislative power. D The residency requirement is constitutionally permissible because the water board members are elected at large.
D The residency requirement is constitutionally permissible because the water board members are elected at large. The residency requirement is permissible because the water board is elected at large. The Equal Protection Clause prohibits state dilution of the right to vote, so that when a governmental body establishes voting districts for the election of representatives, the number of persons in each district may not vary significantly. This is known as the principle of "one person, one vote." This principle applies to almost every election where a person is being elected to perform normal governmental functions (e.g., an election for trustees for a junior college district). However, the principle of one person, one vote generally is inapplicable where there is an at-large system of election (except where the system is adopted for discriminatory purposes). Here, the water board members are elected by all of the qualified voters in the city in an at-large system (rather than having the voters of each individual district select one board member apiece), and no discriminatory intent is evident. Thus, the statutory provision requiring board members to reside in each of the five districts does not result in an imbalance or a dilution of the voting rights of the citizens of the city. Consequently, (A) is incorrect, and (D) presents an accurate statement of the constitutionality of the residency requirement. (Note that the answer might be different under federal statute because the city would have to prove a valid, nondiscriminatory purpose.) (C) is incorrect even though it reaches the correct result. While the Supreme Court has exempted special purpose water storage districts from the one person, one vote requirement, the basis of the decision was the specialized nature of the entity. The constitutional requirements apply not only to legislators, but also to elected administrative and executive officials. (B) is incorrect because, even assuming that the residency requirement violates the candidates' equal protection rights, the resident would not have standing to raise the issue. Generally, a claimant must assert his own constitutional rights and cannot assert the rights of third parties.
Depending on the circumstances, strict liability may be imposed on the owners of what type of animals? A Wild animals only B Wild animals and domestic animals, but not trespassing animals C Wild animals and trespassing animals, but not domestic animals D Wild animals, domestic animals, and trespassing animals
D Wild animals, domestic animals, and trespassing animals Depending on the circumstances, strict liability may be imposed on the owners of wild animals, domestic animals, and trespassing animals. Unless an owner of wild animals can rely on a public duty exception (e.g., a zookeeper), the owner is strictly liable for injuries caused by the wild animals, even those kept as pets. An owner is strictly liable for the damage done by the trespass of his animals (other than household pets) as long as the damage was reasonably foreseeable. It does not matter that the owner acted with reasonable care to keep them from trespassing. Normally, the owner of a domestic animal is not strictly liable for injuries it causes. Strict liability will attach, however, if the owner knows of the domestic animal's dangerous propensities (i.e., propensities more dangerous than normal for that species). This rule applies even if the animal has never injured anyone.
A mother was in a nursing home and asked her attorney to draft a deed that would give her farm to her son. The attorney drew up the deed, had the mother properly execute it, and thereafter properly recorded the deed. The attorney then told the son what she had done. The son immediately went to the nursing home and told the mother that he did not want the farm so she should take back the deed. A week later, the mother returned home to the farm. Shortly thereafter, the son died without a will, leaving his wife as his only heir. The mother has brought an action against the wife to quiet her title to the farm. If the mother is successful in this action, what is the likely reason? A The son's statement to the mother was a constructive reconveyance of the farm. B The attorney's recording of the deed had no effect because the son was unaware of what was happening. C The wife is subject to a constructive trust to carry out the son's intent. D The son never effectively accepted delivery of the deed.
D The son never effectively accepted delivery of the deed. If the mother prevails, it will be because the son never effectively accepted delivery. A deed is not effective to transfer an interest in realty unless it has been delivered, and there must be acceptance by the grantee to complete the conveyance. (D) is the best answer because even though most states presume acceptance, the presumption is rebutted when the grantee expressly refuses to accept the conveyance. (A) is wrong because there is no such thing as a "constructive reconveyance" of land. (B) may look good at first, but it is a minority rule. In most states, acceptance is presumed if the conveyance is beneficial to the grantee, regardless of whether the grantee has knowledge of the conveyance. (C) is wrong because neither the son nor the wife is guilty of any wrongdoing and there is no ground to impose a constructive trust.
A fee simple owner of a restaurant provided in his will that the property should go on his death "in fee simple to my friend, but if during my friend's lifetime my son has children and those children are alive when my friend dies, then to said living children." When the owner died, the friend took over the restaurant. If the son has children and one or more of them are alive when the friend dies, who will take title to the restaurant at that time? A The friend's heirs, because the attempted gift to the son's children is invalid under the Rule Against Perpetuities. B The son's children, because their interest is not contingent, being a possibility of reverter. C The son's children, because their interest is vested, subject to defeasance. D The son's children, because their interest will vest, if at all, within a life in being plus 21 years.
D The son's children, because their interest will vest, if at all, within a life in being plus 21 years. The interest given to the son's children does not violate the Rule Against Perpetuities because the interest will vest, if at all, within 21 years after the life of the friend. Pursuant to the Rule Against Perpetuities, no interest in property is valid unless it must vest, if at all, not later than 21 years after one or more lives in being at the creation of the interest. In the case of a will, the perpetuities period begins to run on the date of the testator's death, and measuring lives used to show the validity of an interest must be in existence at that time. Here, the interest given to any of the son's children who are born during the friend's lifetime and who survive the friend must vest, if at all, on the death of the friend (who is a life in being at the time of the owner's death). Thus, this interest will vest, if it does vest, within 21 years after the friend's life, and is therefore not in violation of the Rule Against Perpetuities. (A) is therefore incorrect; if one or more of the son's children is alive at the time of the friend's death, the friend's heirs will get nothing because their fee simple will be divested. (B) incorrectly characterizes the interest of the son's children as a possibility of reverter. A possibility of reverter is the future interest left in a grantor who conveys a fee simple determinable estate. Although under different circumstances the son's children could acquire a possibility of reverter as heirs of the grantor (the owner), their interest in this case was conveyed directly to them in the owner's will. (C) is incorrect because the interest of the son's children is not vested. Their interest is a shifting executory interest rather than a remainder because it divests the fee simple estate of the friend and his heirs. The friend has a fee simple subject to an executory interest because the estate will remain with his heirs if none of the son's children are alive when the friend dies. The friend's death while the son's children are alive divests the interest of the friend's heirs; it is therefore a shifting executory interest rather than a remainder.
A landowner validly conveyed a small office building to the Green Party "as long as they use it for operating quarters until the next presidential election." After the next presidential election, which was in three years, the building would go to a private organization that monitors and prepares comprehensive listings of gas prices throughout the country. A year after the conveyance, the landowner died, validly devising all of her property to her son. Although this jurisdiction is a common law jurisdiction with respect to all real property considerations, the state's probate laws provide that future interests or estates in real property may be passed by will or descent in the same manner as present or possessory interests. Last week, the Green Party and the gas monitoring organization joined together to sell the office building in fee simple absolute to a developer. The son filed suit to prevent the sale of the property to the developer. In this action, who should prevail? A The Green Party and the gas monitoring organization, because together they own a fee simple absolute in the building. B The Green Party and the gas monitoring organization, because the attempted restrictions on the use of the property violate the Rule Against Perpetuities. C The Green Party and the gas monitoring organization, because the deed restriction was an unlawful restraint on alienation. D The son, because he did not sign the contract of sale.
D The son, because he did not sign the contract of sale. The son may enjoin the sale because he has an interest in the property. A fee simple determinable is an estate that automatically terminates on the happening of a stated event. The Green Party's interest in the office building is a fee simple determinable because it lasts as long as the Party is using the building for operating quarters. However, the grant does not provide for the contingency of the Green Party ceasing to use the building as operating quarters before the next presidential election. This gap would be filled by a possibility of reverter retained by the landowner. Because the landowner passed that interest to her son in her will, there can be no contract to sell the property without his signature. Note: Although the gas monitoring organization appears to have an indefeasibly vested remainder (i.e., it is created in an ascertained company, is certain to become possessory, and is not subject to being defeated, divested, or diminished in size), its interest is not capable of taking on the natural termination of the preceding estate and so is characterized as a springing executory interest. (A) is wrong because the son also has an interest in the land. (B) is wrong because the interest in the office building will pass to the gas monitoring organization, if at all, within 21 years. (C) is wrong because the Green Party is not prohibited from transferring any interest; it could pass a defeasible fee.
Which of the following is correct regarding strict liability for abnormally dangerous activities? A To be characterized as an abnormally dangerous activity, the activity must be considered abnormally dangerous in every community B Strict liability will arise for any type of harm caused by engaging in the abnormally dangerous activity C Whether an activity is abnormally dangerous is a question of fact for the jury to decide D To be abnormally dangerous, the activity must create a foreseeable risk of serious harm even when reasonable care is exercised by all actors
D To be abnormally dangerous, the activity must create a foreseeable risk of serious harm even when reasonable care is exercised by all actors An activity may be characterized as abnormally dangerous if it creates a foreseeable risk of serious harm even when reasonable care is exercised by all actors. Determining whether an activity is abnormally dangerous is NOT a question of fact for the jury to decide; rather, it is a question of law that the court can decide on a motion for a directed verdict. It is incorrect that strict liability will arise from any type of harm caused by the activity. The harm must result from the kind of danger to be anticipated from the abnormally dangerous activity; i.e., it must flow from the "normally dangerous propensity" of the condition or thing involved. It is also incorrect that the activity must be considered abnormally dangerous in every community. Courts generally impose a requirement that the activity must not be a matter of common usage in the community where it takes place. An activity may be considered abnormally dangerous in some areas but not in others.
Small, prolific mussels called zebra mussels were first introduced into the Great Lakes by a foreign cargo ship. They became a serious problem because they attached themselves to smooth, hard surfaces, and often clogged water intake pipes. Congress determined that zebra mussels posed a great threat to the economic welfare of the Great Lakes region and passed a statute requiring all Great Lakes water intakes to be coated with a special chemical compound that repels zebra mussels. Studies by biologists at a major state university showed that while the special chemical compound that the federal government has required was effective, it also was toxic to other aquatic life. The biologists recommended that Great Lakes intake pipes be coated with a less toxic and less expensive copper-based paint. On the basis of those studies and the recommendation, three Great Lakes states adopted laws permitting municipal water districts to coat their intake pipes with copper paint. Can municipalities using copper-based paint on their intake pipes successfully be prosecuted for violating the federal law? A No, because the Tenth Amendment prevents Congress from interfering with integral government functions. B No, because the municipalities are taking effective steps to combat zebra mussels in compliance with the spirit and purpose of the federal law. C Yes, because Congress is in a better position to regulate the entire Great Lakes region than the individual states. D Yes, because Congress may adopt laws regulating navigable waters.
D Yes, because Congress may adopt laws regulating navigable waters. The cities can be prosecuted because state or local government action that conflicts with valid federal laws is invalid under the Supremacy Clause. The federal law here could be found valid as an exercise of the commerce power (Congress can regulate any activity that either in itself or in combination with other activities has a substantial economic effect on interstate commerce) or under the admiralty power (Congress can regulate all navigable waterways). The action of the municipalities directly conflicts with the directives of the federal law and can therefore be stopped. (B) is incorrect because the fact that the copper paint may be as effective as the special compound does not change the result. The action by the municipalities can be prohibited under the Supremacy Clause. (A) is incorrect because for regulations that apply to both the public sector and the private sector, the Supreme Court has held that states' Tenth Amendment rights are best protected by the states' representation in Congress; hence, the Tenth Amendment is not a likely ground for striking this federal legislation because it is not directed only at state or local governments. (C) is incorrect because it is irrelevant; the federal law is superior to the states' laws because it is within Congress's power, not because Congress is in a better position than the states to adopt the legislation involved.
While cross-examining a defendant on trial for robbery and assault with a deadly weapon, the prosecutor asks him whether he was convicted of fraud within the previous year. Is this question proper? A No, because fraud is not probative of a tendency to commit violence. B No, unless the proper foundation was laid. C Yes, because fraud is a form of stealing, and so it will tend to show that the defendant could commit robbery. D Yes, because it tends to show that the defendant would lie.
D Yes, because it tends to show that the defendant would lie. The question is proper. The defendant has taken the stand in his own defense, and therefore the prosecutor can attack his credibility as a witness. Under Federal Rule 609, evidence of conviction of a crime requiring proof of an act of dishonesty or false statement can always be used to attack a witness's character for truthfulness. (A) is incorrect because even if fraud were probative of the tendency to commit violence, evidence of other crimes is not admissible to prove that a person has a propensity to commit criminal acts. (C) is incorrect for the same reason. (B) is wrong because no foundation is needed to show a prior conviction for impeachment purposes.
A plaintiff filed a negligence action against a defendant in federal district court after a two-car accident. The plaintiff's attorney created a list of everyone he could identify who observed the accident or otherwise had information relevant to the accident. The list includes one eyewitness whom the plaintiff's attorney was able to identify only through the expenditure of several thousand dollars in investigation costs. The defendant served the following interrogatory on the plaintiff: "Please state the name of each person of whom you are aware who may know or have information relevant to this action." Must the plaintiff provide the defendant with the names of all of the people on the plaintiff's attorney's list? A No, because the names on the list are protected from discovery under the work product doctrine. B No as to the name of the eyewitness found through the plaintiff's investigation efforts, but yes as to the other names on the list. C Yes, because, while the names are subject to qualified immunity from discovery under the work product doctrine, the defendant will be able to show sufficient need to obtain a court order requiring the names' disclosure. D Yes, because the names are relevant to the claims and defenses of the parties, and they do not constitute work product.
D Yes, because the names are relevant to the claims and defenses of the parties, and they do not constitute work product. The plaintiff must provide the defendant with the names of the people on the plaintiff's attorney's list. In general, discovery may be had of any nonprivileged matter that is relevant to any party's claim or defense and proportional to the needs of the case, including the identity of individuals with knowledge of any discoverable matter. Because the Federal Rules of Civil Procedure state that the identities of people with knowledge of discoverable matters are discoverable, the plaintiff's attorney's list is not considered work product. Thus, (A) and (C) are incorrect. (B) is incorrect because the identity of the eyewitness is discoverable regardless of the extent of the plaintiff's investigation efforts.
A local news station broadcast a live interview with a bystander about his views concerning the state of local education. The bystander responded by saying that the principal of his daughter's high school had been embezzling school funds for years. The principal saw the telecast and also recorded it. He sued the owner of the station for defamation. At trial, the principal sought to testify to the defamatory statement made in the interview. Will the principal's testimony likely be held to be admissible? A No, because the testimony would be hearsay not within any exception. B No, because a recording of the interview exists. C Yes, because the statement is being offered to show its effect on the principal. D Yes, because the principal personally saw the interview on television.
D Yes, because the principal personally saw the interview on television. Because the principal had firsthand knowledge that the statement was made, his testimony will be admissible unless there is a specific rule excluding the evidence. Witnesses are generally presumed competent to testify until the contrary is demonstrated. While a witness may not testify to a matter unless evidence is introduced to support a finding that the witness has personal knowledge of the matter, this evidence may consist of the witness's own testimony. (A) is incorrect. Hearsay is a statement, other than one made by the declarant while testifying at the trial or hearing, offered in evidence to prove the truth of the matter asserted. In a defamation action, evidence of the statement alleged to be defamatory is not hearsay because the evidence is by definition not offered to prove the truth of the matter asserted. It is offered only to show that the actionable statement was made. (B) is incorrect. Because the principal had firsthand knowledge of the event he can testify about the event, even though there might exist a recording that would be better proof of the event. The "best evidence rule" does not apply because the recording is not an essential repository of the facts recorded. (C) is incorrect. Although the statement is not hearsay, it is not being offered to show its effect on the hearer (e.g., knowledge, motive), but rather to show that the statement was made, as explained above.
A company operated a small amusement park on property it owned near a residential neighborhood. On a day when the park was closed, a 10-year-old girl snuck into the park with some friends by climbing over a chain link fence. While climbing on one of the carnival rides, the girl slipped and cut her leg on an exposed gear assembly, sustaining serious injuries. Through her guardian ad litem, the girl brought suit against the company to recover damages for her injuries. At trial, she presented evidence of the accident and her injuries. In defense, the company established that the girl read and understood the "No Trespassing" signs that were attached to the fence. The company also established that it had not had any previous reports of children sneaking into the park when it was closed. Before submission of the case to the jury, the company moved for summary judgment. Is the court likely to grant the company's motion? A Yes, because the girl was a trespasser who the company had no reason to anticipate would be on the property. B Yes, because the girl knew she was trespassing and was old enough to recognize the danger. C No, because the jury could find that the company should have foreseen that children would sneak into the park. D No, because the appeal of the carnival rides attracted the girl into the park.
The court is not likely to grant the company's motion because the jury must determine whether the attractive nuisance doctrine applies. Under this doctrine, a landowner has a duty to exercise ordinary care to avoid reasonably foreseeable risk of harm to children, including trespassing children, caused by artificial conditions on his property. Here, while the company has presented some evidence against application of the attractive nuisance doctrine, it is ultimately the trier of fact's role to determine whether the doctrine applies and whether the company exercised ordinary care. Hence, the court will likely deny the company's motion and allow the jury to make that determination. (A) is incorrect because the jury could find that even though the company was not aware of children trespassing, it should have anticipated that they might try to sneak onto the property because it was an amusement park operation. (B) is incorrect because even though the girl knew she was trespassing, the jury could find that she did not appreciate the risk of playing on the rides. (D) is incorrect because the fact that the girl was attracted onto the land by the artificial condition is just one factor for determining whether the attractive nuisance doctrine applies. Foreseeability of harm to a child is the true basis of liability.
While at a party, the defendant ran into an acquaintance. The acquaintance proceeded to ridicule the defendant about his looks. After an hour of verbal abuse by the acquaintance, the defendant suddenly took a champagne bottle that was on a nearby table and struck the acquaintance over the head, killing him instantly. At his arrest, the defendant told the police that voices inside his head told him to shut the acquaintance up, permanently. The defendant was tried in a jurisdiction that follows the Model Penal Code test for insanity. At trial, the defendant's lawyer introduced psychiatric testimony indicating that the defendant suffered from a mental illness. Which of the following, if proved by the defense, would most likely relieve the defendant of criminal responsibility? A The defendant's actions were a product of his mental illness. B The defendant could not appreciate the criminality of killing the acquaintance, or he could not conform his conduct to the requirements of the law. C The defendant did not know that killing the acquaintance was wrong, or he could not understand the nature and quality of his actions. D The defendant was unable to control himself or conform his conduct to the law.
The defendant could not appreciate the criminality of killing the acquaintance, or he could not conform his conduct to the requirements of the law.
A plaintiff read of the success of a box-office hit movie about aardvarks in various entertainment journals. The movie was enormously popular among young children, and cartoon figures from the movie began appearing on T-shirts, soft drink mugs, and other novelties. The plaintiff filed suit against the studio alleging that the production company unlawfully used his ideas for the movie. The studio admitted that it had received a clay model of a cartoon animal from the plaintiff, but denied that the model had any substantial similarity to the now-famous aardvarks. The studio had returned the model to the plaintiff, but he had destroyed it. For the plaintiff to testify at trial as to the appearance of the model, which of the following is true? A The plaintiff can testify as to the appearance of the model because he has personal knowledge of it. B The plaintiff must show that the destruction of the model was not committed in bad faith. C The plaintiff must introduce a photograph of the model, if one exists. D The plaintiff must give advance notice to the opposing party that he plans to use such oral testimony in his case.
The plaintiff can testify as to the appearance of the model because he has personal knowledge of it. The plaintiff can testify as to the appearance of the model because he has personal knowledge of it. A witness must be competent to testify, which includes the requirement that he have personal knowledge of the matter he is to testify about. Here, the plaintiff has personal knowledge of the model, as he is the person that had submitted it to the studio. Thus, he is competent to testify as to the model's appearance. (B) is wrong because it states the foundation requirement for the admissibility of secondary evidence under the best evidence rule (also called the original document rule), which does not apply under these circumstances. The best evidence rule covers writings and recordings, which are defined as "letters, words, numbers, or their equivalent, set down in any form." A clay model clearly does not fit within that definition. Similarly, (C) states an acceptable form of secondary evidence under the best evidence rule, which does not apply here. Note, however, that under the Federal Rules (unlike most states), there are no degrees of secondary evidence. Therefore, this choice would be wrong even if the best evidence rule were applicable, because the plaintiff would not be limited to photographic evidence. (D) is wrong because it incorrectly assumes that notice must be given. This type of notice is not a prerequisite for the plaintiff's testimony even had the best evidence rule been applicable.
The defendant was on trial for murder. The defendant called a witness to testify to an alibi. On cross-examination of the witness, the prosecutor asked, "Weren't you on the jury that acquitted the defendant of another criminal charge?" What is the best reason for sustaining an objection to this question? A The question goes beyond the scope of direct examination. B The probative value of the answer would be substantially outweighed by its tendency to mislead. C The question is a leading question. D Prior jury service in a case involving a party renders the witness incompetent.
The probative value of the answer would be substantially outweighed by its tendency to mislead. This question raises several different issues: competency of witnesses, use of leading questions on cross-examination, the proper scope of cross-examination, and the probative value/prejudicial impact balancing test. Through a process of elimination, (B) emerges as the correct answer. (D) is incorrect. Under the Federal Rules, virtually all witnesses with personal knowledge are competent to testify. [Fed. R. Evid. 601] A witness is not rendered incompetent simply by having served on a jury in a prior case involving a party to the current suit. Such prior jury service might render the witness's testimony unpersuasive, but it would not make it inadmissible. (C) is incorrect because ordinarily, leading questions are permitted on cross-examination. [Fed. R. Evid. 611(c)] The prosecutor's question is a leading question, but that is perfectly permissible, especially in a case like this, where the alibi witness is not "friendly" toward the prosecution. (A) is incorrect because cross-examination is generally limited in scope to the subject matter of the direct examination and matters affecting the credibility of the witness [Fed. R. Evid. 611(b)], and the prosecutor's question is, in a roundabout way, an attempt to impeach the witness's credibility. The implication behind the question is that if the witness had served on a jury that acquitted the defendant of another criminal charge, the witness would be inclined to think the defendant innocent of the pending charge. Alternatively, the implication behind the question could be that the witness is the kind of person who is "soft on crime" and for that reason is not a credible witness. In either event, because the question is an attempt to impeach the witness's testimony, it is within the proper scope of cross-examination. This leaves (B) as the remaining correct answer. (B) is not unquestionably correct, because the probative value/prejudicial impact balancing test found in Rule 403 is weighted heavily toward admission of evidence. For evidence to be excluded under this balancing test, its probative value must be substantially outweighed by its prejudicial impact. Nevertheless, in this case, a plausible reason for sustaining an objection to the prosecutor's question is that the probative value of the answer would be substantially outweighed by its tendency to mislead. The question and answer would inevitably let the jury know that the defendant had been previously charged with a crime. This information could be highly prejudicial to his defense. Because the question and answer have little probative value (the negative inferences pertaining to the witness's credibility being very weak), it is reasonable to sustain an objection to the question on the basis that its probative value is substantially outweighed by its prejudicial impact.